Download as pdf or txt
Download as pdf or txt
You are on page 1of 333

A.

30 seconds
B. 1 minute
PNLE I for Foundation of C. 2 minute
D. 3 minutes

Answer: A. Depending on the degree of


Nursing exposure to pathogens, hand washing may last
1. Which element in the circular chain of infection from 10 seconds to 4 minutes. After routine
can be eliminated by preserving skin integrity? patient contact, hand washing for 30 seconds
effectively minimizes the risk of
A. Host pathogen transmission.
B. Reservoir
C. Mode of transmission 6. Which of the following procedures always
D. Portal of entry requires surgical asepsis?

Answer: D. In the circular chain of infection, A. Vaginal instillation of conjugated estrogen


pathogens must be able to leave their reservoir B. Urinary catheterization
and be transmitted to a susceptible host through C. Nasogastric tube insertion
a portal of entry, such as broken skin. D. Colostomy irrigation

2. Which of the following will probably result in a Answer: B. The urinary system is normally free
break in sterile technique for respiratory of microorganisms except at the urinary meatus.
isolation? Any procedure that involves entering this system
must use surgically aseptic measures to
A. Opening the patient’s window to the maintain a bacteria-free state
outside environment
B. Turning on the patient’s room ventilator 7. Sterile technique is used whenever:
C. Opening the door of the patient’s
room leading into the hospital corridor A. Strict isolation is required
D. Failing to wear gloves when administering B. Terminal disinfection is performed
a bed bath C. Invasive procedures are performed
D. Protective isolation is necessary
Answer: C. Respiratory isolation, like strict
isolation, requires that the door to the door Answer: C. All invasive procedures, including
patient’s room remain closed. However, the surgery, catheter insertion, and administration
patient’s room should be well ventilated, so of parenteral therapy, require sterile technique
opening the window or turning on the ventricular to maintain a sterile environment. All equipment
is desirable. The nurse does not need to wear must be sterile, and the nurse and the physician
gloves for respiratory isolation, but good hand must wear sterile gloves and maintain surgical
washing is important for all types of isolation asepsis. In the operating room, the nurse and
physician are required to wear sterile gowns,
3. Which of the following patients is at greater gloves, masks, hair covers, and shoe covers for
risk for contracting an infection? all invasive procedures. Strict isolation requires
the use of clean gloves, masks, gowns and
A. A patient with leukopenia equipment to prevent the transmission of highly
B. A patient receiving broad-spectrum communicable diseases by contact or by
antibiotics airborne routes. Terminal disinfection is
C. A postoperative patient who has the disinfection of all contaminated supplies and
undergone orthopedic surgery equipment after a patient has been discharged
D. A newly diagnosed diabetic patient to prepare them for reuse by another patient.
The purpose of protective (reverse) isolation is
Answer: A. Leukopenia is a decreased number of to prevent a person with seriously impaired
leukocytes (white blood cells), which are resistance from coming into contact who
important in resisting infection. None of the potentially pathogenic organisms.
other situations would put the patient at risk for
contracting an infection; taking 8. Which of the following constitutes a break in
broadspectrum antibiotics might actually reduce sterile technique while preparing a sterile field for
the infection risk. a dressing change?

4. Effective hand washing requires the use of: A. Using sterile forceps, rather than sterile
gloves, to handle a sterile item
A. Soap or detergent to promote B. Touching the outside wrapper of sterilized
emulsification material without sterile gloves
B. Hot water to destroy bacteria C. Placing a sterile object on the edge of
C. A disinfectant to increase surface tension the sterile field
D. All of the above D. Pouring out a small amount of solution (15
to 30 ml) before pouring the solution into a
Answer: A. Soaps and detergents are used to sterile container
help remove bacteria because of their ability to
lower the surface tension of water and act as Answer: C. The edges of a sterile field are
emulsifying agents. Hot water may lead to skin considered contaminated. When sterile items are
irritation or burns allowed to come in contact with the edges of the
5. After routine patient contact, hand washing field, the sterile items also become
should last at least: contaminated
when administering an I.M. injection. Enteric
9. A natural body defense that plays an active precautions prevent the transfer of pathogens
role in preventing infection is: via feces.

A. Yawning 13.All of the following measures are


B. Body hair recommended to prevent pressure ulcers except:
C. Hiccupping
D. Rapid eye movements A. Massaging the reddened are with
lotion
Answer: B. Hair on or within body areas, such as B. Using a water or air mattress
the nose, traps and holds particles that contain C. Adhering to a schedule for positioning and
microorganisms. Yawning and hiccupping do turning
not prevent microorganisms from entering or D. Providing meticulous skin care
leaving the body. Rapid eye movement marks
the stage of sleep during which dreaming Answer: A. Nurses and other health care
occurs. professionals previously believed that massaging
a reddened area with lotion would promote
10. All of the following statement are true about venous return and reduce edema to the area.
donning sterile gloves except: However, research has shown that massage only
increases the likelihood of cellular ischemia and
A. The first glove should be picked up by necrosis to the area
grasping the inside of the cuff. 14.Which of the following blood tests should be
B. The second glove should be picked up by performed before a blood transfusion?
inserting the gloved fingers under the cuff
outside the glove. A. Prothrombin and coagulation time
C. The gloves should be adjusted by sliding B. Blood typing and cross-matching
the gloved fingers under the sterile cuff and C. Bleeding and clotting time
pulling the glove over the wrist D. Complete blood count (CBC) and
D. The inside of the glove is considered electrolyte levels.
sterile Answer: B. Before a blood transfusion is
Answer: D. The inside of the glove is always performed, the blood of the donor and recipient
considered to be clean, but not sterile. must be checked for compatibility. This is done
by blood typing (a test that determines a
11.When removing a contaminated gown, the person’s blood type) and cross-matching
nurse should be careful that the first thing she (a procedure that determines the compatibility
touches is the: of the donor’s and recipient’s blood after the
blood types has been matched). If the blood
A. Waist tie and neck tie at the back of specimens are incompatible, hemolysis and
the gown antigen-antibody reactions will occur
B. Waist tie in front of the gown
C. Cuffs of the gown 15.The primary purpose of a platelet count is to
D. Inside of the gown evaluate the:

Answer: A. The back of the gown is considered A. Potential for clot formation
clean, the front is contaminated. So, after B. Potential for bleeding
removing gloves and washing hands, the nurse C. Presence of an antigen-antibody response
should untie the back of the gown; slowly move D. Presence of cardiac enzymes
backward away from the gown, holding
the inside of the gown and keeping the edges off Answer: A. Platelets are disk-shaped cells that
the floor; turn and fold the gown inside out; are essential for blood coagulation. A platelet
discard it in a contaminated linen container; count determines the number of thrombocytes in
then wash her hands again. blood available for promoting hemostasis and
assisting with blood coagulation after injury. It
12.Which of the following nursing interventions is also is used to evaluate the patient’s potential for
considered the most effective form or universal bleeding; however, this is not its primary
precautions? purpose. The normal count ranges from 150,000
to 350,000/mm3. A count of 100,000/mm3 or less
A. Cap all used needles before removing indicates a potential for bleeding; count of less
them from their syringes than 20,000/mm3 is associated with
B. Discard all used uncapped needles spontaneous bleeding.
and syringes in an impenetrable
protective container 16.Which of the following white blood cell (WBC)
C. Wear gloves when administering IM counts clearly indicates leukocytosis?
injections
D. Follow enteric precautions A. 4,500/mm³
B. 7,000/mm³
Answer: B. According to the Centers for Disease C. 10,000/mm³
Control (CDC), blood-to-blood contact occurs D. 25,000/mm³
most commonly when a health care worker
attempts to cap a used needle. Therefore, used Answer: D. Leukocytosis is any transient increase
needles should never be recapped; instead they in the number of white blood cells (leukocytes) in
should be inserted in a specially designed the blood. Normal WBC counts range from 5,000
puncture resistant, labeled container. Wearing to 100,000/mm3. Thus, a count of 25,000/mm3
gloves is not always necessary indicates leukocytosis
Answer: A. Initial sensitivity to penicillin is
17. After 5 days of diuretic therapy with 20mg of commonly manifested by a skin rash, even in
furosemide (Lasix) daily, a patient begins to individuals who have not been allergic to it
exhibit fatigue, muscle cramping and muscle previously. Because of the danger of anaphylactic
weakness. These symptoms probably indicate shock, he nurse should withhold the drug
that the patient is experiencing: and notify the physician, who may choose to
substitute another drug. Administering an
A. Hypokalemia antihistamine is a dependent nursing intervention
B. Hyperkalemia that requires a written physician’s order. Although
C. Anorexia applying corn starch to the rash may relieve
D. Dysphagia discomfort, it is not the nurse’s top priority in
such a potentially life-threatening situation.
Answer: A. Fatigue, muscle cramping, and muscle
weaknesses are symptoms of hypokalemia (an 21.All of the following nursing interventions are
inadequate potassium level), which is a potential correct when using the Ztrack method of drug
side effect of diuretic therapy. The physician injection except:
usually orders supplemental potassium to
prevent hypokalemia in patients receiving A. Prepare the injection site with alcohol
diuretics. Anorexia is another symptom of B. Use a needle that’s a least 1” long
hypokalemia. Dysphagia means C. Aspirate for blood before injection
difficulty swallowing. D. Rub the site vigorously after the
injection to promote absorption
18.Which of the following statements about chest
X-ray is false? Answer: D. The Z-track method is an I.M.
injection technique in which the patient’s skin is
A. No contradictions exist for this test pulled in such a way that the needle track is
B. Before the procedure, the patient should sealed off after the injection. This procedure
remove all jewelry, metallic objects, and seals medication deep into the muscle,
buttons above the waist thereby minimizing skin staining and irritation.
C. A signed consent is not required Rubbing the injection site is contraindicated
D. Eating, drinking, and medications are because it may cause the medication to
allowed before this test extravasate into the skin

Answer: A. Pregnancy or suspected pregnancy is 22.The correct method for determining the vastus
the only contraindication for a chest X-ray. lateralis site for I.M. injection is to:
However, if a chest X-ray is necessary, the
patient can wear a lead apron to protect the A. Locate the upper aspect of the upper
pelvic region from radiation. Jewelry, outer quadrant of the buttock about 5 to 8
metallic objects, and buttons would interfere with cm below the iliac crest
the X-ray and thus should not be worn above the B. Palpate the lower edge of the acromion
waist. A signed consent is not required because a process and the midpoint lateral aspect of
chest X-ray is not an invasive examination. the arm
Eating, drinking and medications are allowed C. Palpate a 1” circular area anterior to the
because the X-ray is of the chest, not the umbilicus
abdominal region. D. Divide the area between the greater
femoral trochanter and the lateral femoral
19.The most appropriate time for the nurse to condyle into thirds, and select the middle
obtain a sputum specimen for culture is: third on the anterior of the thigh

A. Early in the morning Answer: D. The vastus lateralis, a long, thick


B. After the patient eats a light breakfast muscle that extends the full length of the thigh, is
C. After aerosol therapy viewed by many clinicians as the site of choice
D. After chest physiotherapy for I.M. injections because it has relatively few
major nerves and blood vessels. The middle third
Answer: A. Obtaining a sputum specimen early in of the muscle is recommended as the injection
this morning ensures an adequate supply of site. The patient can be in a supine or sitting
bacteria for culturing and decreases the risk position for an injection into this site.
of contamination from food or medication.

20.A patient with no known allergies is to receive 23.The mid-deltoid injection site is seldom used
penicillin every 6 hours. When administering the for I.M. injections because it:
medication, the nurse observes a fine rash on the
A. Can accommodate only 1 ml or less of
patient’s skin. The most appropriate nursing medication
action would be to: B. Bruises too easily
C. Can be used only when the patient is lying
A. Withhold the moderation and notify down
the physician D. Does not readily parenteral medication
B. Administer the medication and notify the
physician Answer: A. The mid-deltoid injection site can
C. Administer the medication with an accommodate only 1 ml or less of medication
antihistamine because of its size and location (on the deltoid
D. Apply corn starch soaks to the rash muscle of the arm, close to the brachial artery
and radial nerve
29.Which of the following is a sign or symptom of
24.The appropriate needle size for insulin a hemolytic reaction to blood transfusion?
injection is:
A. Hemoglobinuria
A. 18G, 1 ½” long B. Chest pain
B. 22G, 1” long C. Urticaria
C. 22G, 1 ½” long D. Distended neck veins
D. 25G, 5/8” long
Answer: A. Hemoglobinuria, the abnormal
Answer: D. A 25G, 5/8” needle is the presence of hemoglobin in the urine, indicates a
recommended size for insulin injection because hemolytic reaction (incompatibility of the donor’s
insulin is administered by the subcutaneous and recipient’s blood). In this reaction, antibodies
route. An 18G, 1 ½” needle is usually used for in the recipient’s plasma combine rapidly with
I.M. injections in children, typically in the donor RBC’s; the cells are hemolyzed in
vastus lateralis. A 22G, 1 ½” needle is usually either circulatory or reticuloendothelial system.
used for adult I.M. injections, which are typically Hemolysis occurs more rapidly in ABO
administered in the vastus lateralis or incompatibilities than in Rh incompatibilities.
ventrogluteal site. Chest pain and urticaria may be symptoms of
impending anaphylaxis. Distended neck veins are
25.The appropriate needle gauge for intradermal an indication of hypervolemia.
injection is:
30.Which of the following conditions may require
A. 20G fluid restriction?
B. 22G
C. 25G A. Fever
D. 26G B. Chronic Obstructive Pulmonary Disease
C. Renal Failure
Answer: D. Because an intradermal injection does D. Dehydration
not penetrate deeply into the skin, a small-bore
25G needle is recommended. This type of Answer: C. In real failure, the kidney loses their
injection is used primarily to administer antigens ability to effectively eliminate wastes and fluids.
to evaluate reactions for allergy or sensitivity Because of this, limiting the patient’s intake of
studies. A 20G needle is usually used for I.M. oral and I.V. fluids may be necessary. Fever,
injections of oilbased medications; a 22G needle chronic obstructive pulmonary disease, and
for I.M. injections; and a 25G needle, for I.M. dehydration are conditions for which fluids should
injections; and a 25G needle, for subcutaneous be encouraged.
insulin injections.
31.All of the following are common signs and
26.Parenteral penicillin can be administered as symptoms of phlebitis except:
an:
A. Pain or discomfort at the IV insertion site
A. IM injection or an IV solution B. Edema and warmth at the IV insertion site
B. IV or an intradermal injection C. A red streak exiting the IV insertion site
C. Intradermal or subcutaneous injection D. Frank bleeding at the insertion site
D. IM or a subcutaneous injection
Answer: D. Phlebitis, the inflammation of a vein,
Answer: A. Parenteral penicillin can be can be caused by chemical irritants (I.V. solutions
administered I.M. or added to a solution and or medications), mechanical irritants (the needle
given I.V. It cannot be administered or catheter used during venipuncture or
subcutaneously or intradermally. cannulation), or a localized allergic reaction to the
needle or catheter. Signs and symptoms of
27.The physician orders gr 10 of aspirin for a phlebitis include pain or discomfort, edema and
patient. The equivalent dose in milligrams is: heat at the I.V. insertion site, and a red streak
going up the arm or leg from the I.V. insertion
A. 0.6 mg site.
B. 10 mg
C. 60 mg 32.The best way of determining whether a patient
D. 600 mg has learned to instill ear medication properly is
for the nurse to:
Answer: D. gr 10 x 60mg/gr 1 = 600 mg
A. Ask the patient if he/she has used ear
28.The physician orders an IV solution of dextrose drops before
5% in water at 100ml/hour. What would the flow B. Have the patient repeat the nurse’s
rate be if the drop factor is 15 gtt = 1 ml? instructions using her own words
C. Demonstrate the procedure to the patient
A. 5 gtt/minute and encourage to ask questions
B. 13 gtt/minute D. Ask the patient to demonstrate the
C. 25 gtt/minute procedure
D. 50 gtt/minute
Answer: D. Return demonstration provides the
Answer: C. 100ml/60 min X 15 gtt/ 1 ml = 25 most certain evidence for evaluating the
gtt/minute effectiveness of patient teaching.
33.Which of the following types of medications Answer: A. Coughing, a protective response that
can be administered via gastrostomy tube? clears the respiratory tract of irritants, usually is
involuntary; however it can be voluntary, as
A. Any oral medications when a patient is taught to perform coughing
B. Capsules whole contents are dissolve in exercises. An antitussive drug inhibits coughing.
water Splinting the abdomen supports the abdominal
C. Enteric-coated tablets that are thoroughly muscles when a patient coughs
dissolved in water
D. Most tablets designed for oral use,
except for extended- 37.An infected patient has chills and begins
duration compounds shivering. The best nursing intervention is to:

Answer: D. Capsules, enteric-coated tablets, and A. Apply iced alcohol sponges


most extended duration or sustained release B. Provide increased cool liquids
products should not be dissolved for use in C. Provide additional bedclothes
a gastrostomy tube. They are pharmaceutically D. Provide increased ventilation
manufactured in these forms for valid reasons,
and altering them destroys their purpose. The Answer: C. In an infected patient, shivering
nurse should seek an alternate physician’s order results from the body’s attempt to increase heat
when an ordered medication is inappropriate for production and the production of neutrophils
delivery by tube. and phagocytotic action through increased
skeletal muscle tension and contractions. Initial
34.A patient who develops hives after receiving vasoconstriction may cause skin to feel cold to
an antibiotic is exhibiting drug: the touch. Applying additional bed clothes helps
to equalize the body temperature and stop the
A. Tolerance chills. Attempts to cool the body result in
B. Idiosyncrasy further shivering, increased metabloism, and thus
C. Synergism increased heat production.
D. Allergy
38.A clinical nurse specialist is a nurse who has:
Answer: D. A drug-allergy is an adverse reaction
resulting from an immunologic response following A. Been certified by the National League for
a previous sensitizing exposure to the drug. Nursing
The reaction can range from a rash or hives to B. Received credentials from the Philippine
anaphylactic shock. Tolerance to a drug means Nurses’ Association
that the patient experiences a decreasing C. Graduated from an associate degree
physiologic response to repeated administration program and is a registered professional
of the drug in the same dosage. Idiosyncrasy is nurse
an individual’s unique hypersensitivity to a drug, D. Completed a master’s degree in the
food, or other substance; it appears to be prescribed clinical area and is a
genetically determined. Synergism, is a drug registered professional nurse.
interaction in which the sum of the drug’s
combined effects is greater than that of their Answer: D. A clinical nurse specialist must have
separate effects. completed a master’s degree in a clinical
specialty and be a registered professional nurse.
35.A patient has returned to his room after The National League of Nursing accredits
femoral arteriography. All of the following are educational programs in nursing and provides a
appropriate nursing interventions except: testing service to evaluate student nursing
competence but it does not certify nurses. The
A. Assess femoral, popliteal, and pedal American Nurses Association identifies
pulses every 15 minutes for 2 hours requirements for certification and offers
B. Check the pressure dressing for examinations for certification in many areas
sanguineous drainage of nursing., such as medical surgical nursing.
C. Assess a vital signs every 15 minutes for 2 These certification (credentialing) demonstrates
hours that the nurse has the knowledge and the ability
D. Order a hemoglobin and hematocrit to provide high quality nursing care in the area of
count 1 hour after the arteriography her certification. A graduate of an associate
degree program is not a clinical nurse
Answer: D. A hemoglobin and hematocrit count specialist: however, she is prepared to provide
would be ordered by the physician if bleeding bed side nursing with a high degree of knowledge
were suspected. The other answers are and skill. She must successfully complete the
appropriate nursing interventions for a patient licensing examination to become a registered
who has undergone femoral arteriography. professional nurse.

39.The purpose of increasing urine acidity


36.The nurse explains to a patient that a cough: through dietary means is to:
A. Is a protective response to clear the A. Decrease burning sensations
respiratory tract of irritants B. Change the urine’s color
B. Is primarily a voluntary action C. Change the urine’s concentration
C. Is induced by the administration of an D. Inhibit the growth of microorganisms
antitussive drug
D. Can be inhibited by “splinting” the Answer: D. Microorganisms usually do not grow in
abdomen an acidic environment.
C. Aid in diagnosing a patient with AIDS
40.Clay colored stools indicate: D. All of the above

A. Upper GI bleeding Answer: D. The ELISA test of venous blood is


B. Impending constipation used to assess blood and potential blood donors
C. An effect of medication to human immunodeficiency virus (HIV). A
D. Bile obstruction positive ELISA test combined with various signs
and symptoms helps to diagnose acquired
Answer: D. Bile colors the stool brown. Any immunodeficiency syndrome (AIDS)
inflammation or obstruction that impairs bile flow
will affect the stool pigment, yielding light, clay- 45.The two blood vessels most commonly used
colored stool. Upper GI bleeding results in black for TPN infusion are the:
or tarry stool. Constipation is characterized by
small, hard masses. Many medications and foods A. Subclavian and jugular veins
will discolor stool – for example, drugs containing B. Brachial and subclavian veins
iron turn stool black.; beets turn stool red. C. Femoral and subclavian veins
D. Brachial and femoral veins
41.In which step of the nursing process would the
nurse ask a patient if the medication she Answer: D. Tachypnea (an abnormally rapid rate
administered relieved his pain? of breathing) would indicate that the patient was
still hypoxic (deficient in oxygen).The partial
A. Assessment pressures of arterial oxygen and carbon dioxide
B. Analysis listed are within the normal range. Eupnea refers
C. Planning to normal respiration.
D. Evaluation

Answer: D. In the evaluation step of the nursing 46.Effective skin disinfection before a surgical
process, the nurse must decide whether the procedure includes which of the following
patient has achieved the expected outcome that methods?
was identified in the planning phase.
A. Shaving the site on the day before surgery
42.All of the following are good sources of vitamin B. Applying a topical antiseptic to the skin on
A except: the evening before surgery
C. Having the patient take a tub bath on the
A. White potatoes morning of surgery
B. Carrots D. Having the patient shower with an
C. Apricots antiseptic soap on the
D. Egg yolks evening v=before and the morning of
surgery
Answer: A. The main sources of vitamin A are
yellow and green vegetables (such as carrots, Answer: D. Studies have shown that showering
sweet potatoes, squash, spinach, collard greens, with an antiseptic soap before surgery is the most
broccoli, and cabbage) and yellow fruits (such as effective method of removing microorganisms
apricots, and cantaloupe). Animal sources include from the skin. Shaving the site of the intended
liver, kidneys, cream, butter, and egg yolks. surgery might cause breaks in the skin, thereby
increasing the risk of infection; however, if
43.Which of the following is a primary nursing indicated, shaving, should be done immediately
intervention necessary for all patients with a before surgery, not the day before. A topical
Foley Catheter in place? antiseptic would not remove microorganisms and
would be beneficial only after proper cleaning and
A. Maintain the drainage tubing and rinsing. Tub bathing might transfer organisms to
collection bag level with the patient’s another body site rather than rinse them away.
bladder
B. Irrigate the patient with 1% Neosporin 47.When transferring a patient from a bed to a
solution three times a daily chair, the nurse should use which muscles to
C. Clamp the catheter for 1 hour every 4
avoid back injury?
hours to maintain the bladder’s elasticity
D. Maintain the drainage tubing and A. Abdominal muscles
collection bag below bladder level to B. Back muscles
facilitate drainage by gravity C. Leg muscles
D. Upper arm muscles
Answer: D. Maintaing the drainage tubing and
collection bag level with the patient’s bladder Answer: C. The leg muscles are the strongest
could result in reflux of urine into the kidney. muscles in the body and should bear the greatest
Irrigating the bladder with Neosporin and stress when lifting. Muscles of the abdomen,
clamping the catheter for 1 hour every 4 hours back, and upper arms may be easily injured.
must be prescribed by a physician
48.Thrombophlebitis typically develops in
44.The ELISA test is used to: patients with which of the following conditions?
A. Screen blood donors for antibodies to A. Increases partial thromboplastin time
human immunodeficiency virus (HIV) B. Acute pulsus paradoxus
B. Test blood to be used for transfusion for C. An impaired or traumatized blood
HIV antibodies vessel wall
D. Chronic Obstructive Pulmonary Disease
(COPD)

Answer: C. The factors, known as Virchow’s triad,


collectively predispose a patient to
thromboplebitis; impaired venous return to the
heart, blood hypercoagulability, and injury to a
blood vessel wall. Increased
partial thromboplastin time indicates a prolonged
bleeding time during fibrin clot formation,
commonly the result of anticoagulant (heparin)
therapy. Arterial blood disorders (such as pulsus
paradoxus) and lung diseases (such as COPD) do
not necessarily impede venous return of injure
vessel walls.

49.In a recumbent, immobilized patient, lung


ventilation can become altered, leading to such
respiratory complications as:

A. Respiratory acidosis, ateclectasis, and


hypostatic pneumonia
B. Appneustic breathing, atypical pneumonia
and respiratory alkalosis
C. Cheyne-Strokes respirations and
spontaneous pneumothorax
D. Kussmail’s respirations and
hypoventilation

Answer: A. Because of restricted respiratory


PNLE II for Community Health
movement, a recumbent, immobilize patient is
at particular risk for respiratory acidosis from
Nursing and Care of the
poor gas exchange; atelectasis from reduced
surfactant and accumulated mucus in the
Mother and Child
bronchioles, and hypostatic pneumonia from 1. May arrives at the health care clinic and tells the nurse that
bacterial growth caused by stasis of mucus her last menstrual period was 9 weeks ago. She also tells the
secretions nurse that a home pregnancy test was positive but she began
to have mild cramps and is now having moderate vaginal
50.Immobility impairs bladder elimination, bleeding. During the physical examination of the client, the
resulting in such disorders as nurse notes that May has a dilated cervix. The nurse
determines that May is experiencing which type of abortion?
A. Increased urine acidity and relaxation of A. Inevitable
the perineal muscles, causing incontinence B. Incomplete
B. Urine retention, bladder distention, C. Threatened
and infection D. Septic
C. Diuresis, natriuresis, and decreased urine
specific gravity Answer: Answer: (A) Inevitable. An inevitable abortion is
D. Decreased calcium and phosphate levels termination of pregnancy that cannot be prevented.
in the urine
Moderate to severe bleeding with mild cramping
and cervical dilation would be noted in this type of abortion
Answer: B. The immobilized patient commonly
suffers from urine retention caused by
decreased muscle tone in the perineum. This 2. Nurse Reese is reviewing the record of a pregnant client for
leads to bladder distention and urine stagnation, her first prenatal visit. Which of the following data, if noted
which provide an excellent medium for bacterial on the client’s record, would alert the nurse that the client is
growth leading to infection. Immobility also at risk for a spontaneous abortion?
results in more alkaline urine with excessive A. Age 36 years
amounts of calcium, sodium and phosphate, a B. History of syphilis
gradual decrease in urine production, and an C. History of genital herpes
increased specific gravity. D. History of diabetes mellitus

Answer: Answer: (B) History of syphilis. Maternal infections


such as syphilis, toxoplasmosis, and rubella are causes of
spontaneous abortion.

3. Nurse Hazel is preparing to care for a client who is newly


admitted to the hospital with a possible diagnosis of ectopic
pregnancy. Nurse Hazel develops a plan of care for the client
and determines that which of the following nursing actions is
the priority?
A. Monitoring weight
B. Assessing for edema
C. Monitoring apical pulse
D. Monitoring temperature 8. A pregnant client is receiving oxytocin (Pitocin) for
induction of labor. A condition that warrant the nurse in-
Answer: Answer: (C) Monitoring apical pulse. Nursing care charge to discontinue I.V. infusion of Pitocin is:
for the client with a possible ectopic pregnancy is focused on A. Contractions every 1 ½ minutes lasting 70-80 seconds.
preventing or identifying hypovolemic shock and B. Maternal temperature 101.2
controlling pain. An elevated pulse rate is an indicator of C. Early decelerations in the fetal heart rate.
shock D. Fetal heart rate baseline 140-160 bpm.

4. Nurse Oliver is teaching a diabetic pregnant client about Answer: Answer: (A) Contractions every 1 ½ minutes lasting
nutrition and insulin needs during pregnancy. The nurse 70-80 seconds. Contractions every 1 ½ minutes lasting 70-80
determines that the client understands dietary and insulin seconds, is indicative of hyperstimulation of the uterus, which
needs if the client states that the second half of pregnancy could result in injury to the mother and the fetus if Pitocin is
require: not discontinued.
A. Decreased caloric intake
B. Increased caloric intake 9. Calcium gluconate is being administered to a client with
C. Decreased Insulin pregnancy induced hypertension (PIH). A nursing action that
D. Increase Insulin must be initiated as the plan of care throughout injection of
the drug is:
Answer: (B) Increased caloric intake. Glucose crosses the A. Ventilator assistance
placenta, but insulin does not. High fetal demands for B. CVP readings
glucose, combined with the insulin resistance caused C. EKG tracings
by hormonal changes in the last half of pregnancy can result D. Continuous CPR
in elevation of maternal blood glucose levels. This increases
the mother’s demand for insulin and is referred to as the Answer: Answer: (C) EKG tracings. A potential side effect of
diabetogenic effect of pregnancy. calcium gluconate administration is cardiac arrest. Continuous
monitoring of cardiac activity (EKG) throught administration
5. Nurse Michelle is assessing a 24 year old client with a of calcium gluconate is an essential part of care.
diagnosis of hydatidiform mole. She is aware that one of the
following is unassociated with this condition? 10. A trial for vaginal delivery after an earlier caesareans,
A. Excessive fetal activity. would likely to be given to a gravida, who had:
B. Larger than normal uterus for gestational age. A. First low transverse cesarean was for active herpes type
C. Vaginal bleeding 2 infections; vaginal culture at 39 weeks pregnancy was
D. Elevated levels of human chorionic gonadotropin. positive.
B. First and second caesareans were for cephalopelvic
Answer: Answer: (A) Excessive fetal activity. The most disproportion.
common signs and symptoms of hydatidiform mole includes C. First caesarean through a classic incision as a result of
elevated levels of human chorionic gonadotropin, severe fetal distress.
vaginal bleeding, larger than normal uterus for gestational D. First low transverse caesarean was for breech position.
age, failure to detect fetal heart activity even with sensitive Fetus in this pregnancy is in a vertex presentation.
instruments, excessive nausea and vomiting, and early
development of pregnancy-induced hypertension. Fetal Answer: Answer: (D) First low transverse caesarean was for
activity would not be noted. breech position. Fetus in this pregnancy is in a vertex
presentation. This type of client has no obstetrical indication
6. A pregnant client is receiving magnesium sulfate for severe for a caesarean section as she did with her first caesarean
pregnancy induced hypertension (PIH). The clinical findings delivery.
that would warrant use of the antidote , calcium gluconate is:
A. Urinary output 90 cc in 2 hours. 11.Nurse Ryan is aware that the best initial approach when
B. Absent patellar reflexes. trying to take a crying toddler’s temperature is:
C. Rapid respiratory rate above 40/min. A. Talk to the mother first and then to the toddler.
D. Rapid rise in blood pressure. B. Bring extra help so it can be done quickly.
C. Encourage the mother to hold the child.
Answer: Answer: (B) Absent patellar reflexes. Absence of D. Ignore the crying and screaming.
patellar reflexes is an indicator of hypermagnesemia, which
requires administration of calcium gluconate Answer: Answer: (A) Talk to the mother first and then to the
toddler. When dealing with a crying toddler, the best
7. During vaginal examination of Janah who is in labor, the approach is to talk to the mother and ignore the toddler first.
presenting part is at station plus two. Nurse, correctly This approach helps the toddler get used to the nurse before
interprets it as: she attempts any procedures. It also gives the toddler an
A. Presenting part is 2 cm above the plane of the ischial opportunity to see that the mother trusts the nurse.
spines.
B. Biparietal diameter is at the level of the ischial spines. 12.Baby Tina a 3 month old infant just had a cleft lip and
C. Presenting part in 2 cm below the plane of the ischial palate repair. What should the nurse do to prevent trauma to
spines. operative site?
D. Biparietal diameter is 2 cm above the ischial spines. A. Avoid touching the suture line, even when cleaning.
B. Place the baby in prone position.
Answer: Answer: (C) Presenting part in 2 cm below the plane C. Give the baby a pacifier.
of the ischial spines. Fetus at station plus two indicates that D. Place the infant’s arms in soft elbow restraints.
the presenting part is 2 cm below the plane of the ischial
spines. Answer: Answer: (D) Place the infant’s arms in soft elbow
restraints. Soft restraints from the upper arm to the wrist
prevent the infant from touching her lip but allow him to hold B. Efficiency
a favorite item such as a blanket. Because they could damage C. Adequacy
the operative site, such as objects as pacifiers, suction D. Appropriateness
catheters, and small spoons shouldn’t be placed in a baby’s
mouth after cleft repair. A baby in a prone position may rub Answer: Answer: (B) Efficiency. Efficiency is determining
her face on the sheets and traumatize the operative site. The whether the goals were attained at the least possible cost.
suture line should be cleaned gently to prevent infection,
which could interfere with healing and damage the cosmetic 18.Vangie is a new B.S.N. graduate. She wants to become a
appearance of the repair. Public Health Nurse. Where should she apply?
A. Department of Health
13. Which action should nurse Marian include in the care plan B. Provincial Health Office
for a 2 month old with heart failure? C. Regional Health Office
A. Feed the infant when he cries. D. Rural Health Unit
B. Allow the infant to rest before feeding.
C. Bathe the infant and administer medications before Answer: Answer: (D) Rural Health Unit. R.A. 7160 devolved
feeding. basic health services to local government units (LGU’s ). The
D. Weigh and bathe the infant before feeding. public health nurse is an employee of the LGU.

Answer: Answer: (B) Allow the infant to rest before 19.Tony is aware the Chairman of the Municipal Health Board
feeding. Because feeding requires so much energy, an infant is:
with heart failure should rest before feeding A. Mayor
B. Municipal Health Officer
14.Nurse Hazel is teaching a mother who plans to discontinue C. Public Health Nurse
breast feeding after 5 months. The nurse should advise her to D. Any qualified physician
include which foods in her infant’s diet?
A. Skim milk and baby food. Answer: Answer: (A) Mayor. The local executive serves as the
B. Whole milk and baby food. chairman of the Municipal Health Board
C. Iron-rich formula only.
D. Iron-rich formula and baby food. 20.Myra is the public health nurse in a municipality with a
total population of about 20,000. There are 3 rural health
Answer: Answer: (C) Iron-rich formula only. The infants at age midwives among the RHU personnel. How many more
5 months should receive iron-rich formula and that they midwife items will the RHU need?
shouldn’t receive solid food, even baby food until age 6 A. 1
months. B. 2
C. 3
15.Mommy Linda is playing with her infant, who is sitting D. The RHU does not need any more midwife item.
securely alone on the floor of the clinic. The mother hides a
toy behind her back and the infant looks for it. The nurse is Answer: Answer: (A) 1. Each rural health midwife is given a
aware that estimated age of the infant would be: population assignment of about 5,000.
A. 6 months
B. 4 months 21.According to Freeman and Heinrich, community health
C. 8 months nursing is a developmental service. Which of the following
D. 10 months best illustrates this statement?
A. The community health nurse continuously develops
Answer: Answer: (D) 10 months. A 10 month old infant can himself personally and professionally.
sit alone and understands object permanence, so he would B. Health education and community organizing are
look for the hidden toy. At age 4 to 6 months, infants can’t sit necessary in providing community health services.
securely alone. At age 8 months, infants can sit securely alone C. Community health nursing is intended primarily for
but cannot understand the permanence of objects. health promotion and prevention and treatment of
disease.
16.Which of the following is the most prominent feature of D. The goal of community health nursing is to provide
public health nursing? nursing services to people in their own places of
A. It involves providing home care to sick people who are residence.
not confined in the hospital.
B. Services are provided free of charge to people within Answer: Answer: (B) Health education and community
the catchments area. organizing are necessary in providing community health
C. The public health nurse functions as part of a team services. The community health nurse develops the health
providing a public health nursing services. capability of people through health education and
D. Public health nursing focuses on preventive, not community organizing activities
curative, services.
22.Nurse Tina is aware that the disease declared through
Answer: Answer: (D) Public health nursing focuses on Presidential Proclamation No. 4 as a target for eradication in
preventive, not curative, services. The catchments area in the Philippines is?
PHN consists of a residential community, many of whom are A. Poliomyelitis
well individuals who have greater need for preventive rather B. Measles
than curative services. C. Rabies
D. Neonatal tetanus
17.When the nurse determines whether resources were
maximized in implementing Ligtas Tigdas, she is evaluating Answer: Answer: (B) Measles. Presidential Proclamation No.
A. Effectiveness 4 is on the Ligtas Tigdas Program.
23.May knows that the step in community organizing that 28.The skin in the diaper area of a 7 month old infant is
involves training of potential leaders in the community is: excoriated and red. Nurse Hazel should instruct the mother
A. Integration to:
B. Community organization A. Change the diaper more often.
C. Community study B. Apply talc powder with diaper changes.
D. Core group formation C. Wash the area vigorously with each diaper change.
D. Decrease the infant’s fluid intake to decrease saturating
Answer: Answer: (D) Core group formation. In core group diapers.
formation, the nurse is able to transfer the technology of
community organizing to the potential or informal community Answer: Answer: (A) Change the diaper more
leaders through a training program. often. Decreasing the amount of time the skin comes contact
with wet soiled diapers will help heal the irritation.
24.Beth a public health nurse takes an active role in
community participation. What is the primary goal of 29.Nurse Carla knows that the common cardiac anomalies in
community organizing? children with Down Syndrome (tri-somy 21) is:
A. To educate the people regarding community health A. Atrial septal defect
problems B. Pulmonic stenosis
B. To mobilize the people to resolve community health C. Ventricular septal defect
problems D. Endocardial cushion defect
C. To maximize the community’s resources in dealing with
health problems. Answer: (D) Endocardial cushion defect. Endocardial cushion
D. To maximize the community’s resources in dealing with defects are seen most in children with Down syndrome,
health problems. asplenia, or polysplenia
Answer: (D) To maximize the community’s resources in
dealing with health problems. Community organizing is a 30.Malou was diagnosed with severe preeclampsia is now
developmental service, with the goal of developing the receiving I.V. magnesium sulfate. The adverse effects
people’s self-reliance in dealing with community health associated with magnesium sulfate is:
problems. A, B and C are objectives of contributory A. Anemia
objectives to this goal. B. Decreased urine output
C. Hyperreflexia
D. Increased respiratory rate
25.Tertiary prevention is needed in which stage of the natural
history of disease? Answer: Answer: (B) Decreased urine output. Decreased urine
A. Pre-pathogenesis output may occur in clients receiving I.V. magnesium and
B. Pathogenesis should be monitored closely to keep urine output at greater
C. Prodromal than 30 ml/hour, because magnesium is excreted through the
D. Terminal kidneys and can easily accumulate to toxic levels.

Answeer: Answer: Answer: (D) Terminal. Tertiary prevention 31.A 23 year old client is having her menstrual period every 2
involves rehabilitation, prevention of permanent disability weeks that last for 1 week. This type of menstrual pattern is
and disability limitation appropriate for convalescents, the bets defined by:
disabled, complicated cases and the terminally ill (those in A. Menorrhagia
the terminal stage of a disease B. Metrorrhagia
C. Dyspareunia
26.The nurse is caring for a primigravid client in the labor and D. Amenorrhea
delivery area. Which condition would place the client at risk
for disseminated intravascular coagulation (DIC)? Answer: Answer: (A) Menorrhagia. Menorrhagia is an
A. Intrauterine fetal death. excessive menstrual period.
B. Placenta accreta.
C. Dysfunctional labor. 32. Jannah is admitted to the labor and delivery unit. The
D. Premature rupture of the membranes. critical laboratory result for this client would be:
A. Oxygen saturation
Answer: Answer: (A) Intrauterine fetal death. Intrauterine B. Iron binding capacity
fetal death, abruptio placentae, septic shock, and amniotic C. Blood typing
fluid embolism may trigger normal clotting mechanisms; if D. Serum Calcium
clotting factors are depleted, DIC may occur. Placenta accreta,
dysfunctional labor, and premature rupture of the Answer: Answer: (C) Blood typing. Blood type would be a
membranes aren’t associated with DIC. critical value to have because the risk of blood loss is always
a potential complication during the labor and delivery
27.A fullterm client is in labor. Nurse Betty is aware that the process. Approximately 40% of a woman’s cardiac output is
fetal heart rate would be: delivered to the uterus, therefore, blood loss can occur quite
A. 80 to 100 beats/minute rapidly in the event of uncontrolled bleeding
B. 100 to 120 beats/minute
C. 120 to 160 beats/minute 33.Nurse Gina is aware that the most common condition
D. 160 to 180 beats/minute found during the second-trimester of pregnancy is:
A. Metabolic alkalosis
Answer: Answer: (C) 120 to 160 beats/minute. A rate of 120 B. Respiratory acidosis
to 160 beats/minute in the fetal heart appropriate for filling C. Mastitis
the heart with blood and pumping it out to the system. D. Physiologic anemia
A. “I should check the diaphragm carefully for holes every
Answer: Answer: (D) Physiologic anemia. Hemoglobin values time I use it”
and hematocrit decrease during pregnancy as the increase in B. “I may need a different size of diaphragm if I gain or lose
plasma volume exceeds the increase in red blood cell weight more than 20 pounds”
production. C. “The diaphragm must be left in place for atleast 6 hours
after intercourse”
34.Nurse Lynette is working in the triage area of an D. “I really need to use the diaphragm and jelly most
emergency department. She sees that several pediatric clients during the middle of my menstrual cycle”.
arrive simultaneously. The client who needs to be treated first
is: Answer: (D) “I really need to use the diaphragm and jelly most
A. A crying 5 year old child with a laceration on his scalp. during the middle of my menstrual cycle”. The woman must
B. A 4 year old child with a barking coughs and flushed understand that, although the “fertile” period is
appearance. approximately mid-cycle, hormonal variations do occur and
C. A 3 year old child with Down syndrome who is pale and can result in early or late ovulation. To be effective, the
asleep in his mother’s arms. diaphragm should be inserted before every intercourse.
D. A 2 year old infant with stridorous breath sounds,
sitting up in his mother’s arms and drooling. 39.Hypoxia is a common complication of
laryngotracheobronchitis. Nurse Oliver should frequently
Answer: Answer: (D) A 2 year old infant with stridorous assess a child with laryngotracheobronchitis for:
breath sounds, sitting up in his mother’s arms and A. Drooling
drooling. The infant with the airway emergency should be B. Muffled voice
treated first, because of the risk of epiglottitis. C. Restlessness
D. Low-grade fever
35.Maureen in her third trimester arrives at the emergency Answer: (C) Restlessness. In a child, restlessness is the earliest
room with painless vaginal bleeding. Which of the following sign of hypoxia. Late signs of hypoxia in a child are associated
conditions is suspected? with a change in color, such as pallor or cyanosis.
A. Placenta previa
B. Abruptio placentae 40.How should Nurse Michelle guide a child who is blind to
C. Premature labor walk to the playroom?
D. Sexually transmitted disease A. Without touching the child, talk continuously as the
child walks down the hall.
Answer: Answer: (A) Placenta previa. Placenta previa with B. Walk one step ahead, with the child’s hand on the
painless vaginal bleeding. nurse’s elbow.
C. Walk slightly behind, gently guiding the child forward.
36.A young child named Richard is suspected of having D. Walk next to the child, holding the child’s hand.
pinworms. The community nurse collects a stool specimen to
confirm the diagnosis. The nurse should schedule the Answer: (B) Walk one step ahead, with the child’s hand on the
collection of this specimen for: nurse’s elbow. This procedure is generally recommended to
A. Just before bedtime follow in guiding a person who is blind.
B. After the child has been bathe
C. Any time during the day 41.When assessing a newborn diagnosed with ductus
D. Early in the morning arteriosus, Nurse Olivia should expect that the child most
likely would have an:
Answer: Answer: (D) Early in the morning. Based on the A. Loud, machinery-like murmur.
nurse’s knowledge of microbiology, the specimen should be B. Bluish color to the lips.
collected early in the morning. The rationale for this timing is C. Decreased BP reading in the upper extremities
that, because the female worm lays eggs at night around the D. Increased BP reading in the upper extremities.
perineal area, the first bowel movement of the day will yield
the best results. The specific type of stool specimen used in Answer: (A) Loud, machinery-like murmur. A loud,
the diagnosis of pinworms is called the tape test machinery-like murmur is a characteristic finding associated
with patent ductus arteriosus
37.In doing a child’s admission assessment, Nurse Betty
should be alert to note which signs or symptoms of chronic 42.The reason nurse May keeps the neonate in a neutral
lead poisoning? thermal environment is that when a newborn becomes too
A. Irritability and seizures cool, the neonate requires:
B. Dehydration and diarrhea A. Less oxygen, and the newborn’s metabolic rate
C. Bradycardia and hypotension increases.
D. Petechiae and hematuria B. More oxygen, and the newborn’s metabolic rate
decreases.
:Answer: (A) Irritability and seizures. Lead poisoning primarily C. More oxygen, and the newborn’s metabolic rate
affects the CNS, causing increased intracranial pressure. This increases.
condition results in irritability and changes in level of D. Less oxygen, and the newborn’s metabolic rate
consciousness, as well as seizure disorders, hyperactivity, and decreases.
learning disabilities.
Answer: (C) More oxygen, and the newborn’s metabolic rate
38.To evaluate a woman’s understanding about the use of increases. When cold, the infant requires more oxygen and
diaphragm for family planning, Nurse Trish asks her to explain there is an increase in metabolic rate. Non-shievering
how she will use the appliance. Which response indicates a thermogenesis is a complex process that increases the
need for further health teaching? metabolic rate and rate of oxygen consumption, therefore,
the newborn increase heat production.
43.Before adding potassium to an infant’s I.V. line, Nurse Ron 48.Myrna a public health nurse knows that to determine
must be sure to assess whether this infant has: possible sources of sexually transmitted infections, the BEST
A. Stable blood pressure method that may be undertaken is:
B. Patant fontanelles A. Contact tracing
C. Moro’s reflex B. Community survey
D. Voided C. Mass screening tests
D. Interview of suspects
Answer: (D) Voided. Before administering potassium I.V. to
any client, the nurse must first check that the client’s kidneys Answer: (A) Contact tracing. Contact tracing is the most
are functioning and that the client is voiding. If the client is practical and reliable method of finding possible sources of
not voiding, the nurse should withhold the potassium and person-to-person transmitted infections, such as sexually
notify the physician. transmitted diseases

44.Nurse Carla should know that the most common causative 49.A 33-year old female client came for consultation at the
factor of dermatitis in infants and younger children is: health center with the chief complaint of fever for a week.
A. Baby oil Accompanying symptoms were muscle pains and body
B. Baby lotion malaise. A week after the start of fever, the client noted
C. Laundry detergent yellowish discoloration of his sclera. History showed that he
D. Powder with cornstarch waded in flood waters about 2 weeks before the onset of
symptoms. Based on her history, which disease condition will
Answer: Answer: (C) Laundry detergent. Eczema or you suspect?
dermatitis is an allergic skin reaction caused by an offending A. Hepatitis A
allergen. The topical allergen that is the most common B. Hepatitis B
causative factor is laundry detergent C. Tetanus
45.During tube feeding, how far above an infant’s stomach D. Leptospirosis
should the nurse hold the syringe with formula?
A. 6 inches Answer: (D) Leptospirosis. Leptospirosis is transmitted
B. 12 inches through contact with the skin or mucous membrane with
C. 18 inches water or moist soil contaminated with urine of infected
D. 24 inches animals, like rats

Answer: (A) 6 inches. This distance allows for easy flow of 50.Mickey a 3-year old client was brought to the health center
the formula by gravity, but the flow will be slow enough not with the chief complaint of severe diarrhea and the passage
to overload the stomach too rapidly of “rice water” stools. The client is most probably suffering
from which condition?
46. In a mothers’ class, Nurse Lhynnete discussed childhood A. Giardiasis
diseases such as chicken pox. Which of the following B. Cholera
statements about chicken pox is correct? C. Amebiasis
A. The older one gets, the more susceptible he becomes D. Dysentery
to the complications of chicken pox.
B. A single attack of chicken pox will prevent future Answer: (B) Cholera. Passage of profuse watery stools is the
episodes, including conditions such as shingles. major symptom of cholera. Both amebic and bacillary
C. To prevent an outbreak in the community, quarantine dysentery are characterized by the presence of blood and/or
may be imposed by health authorities. mucus in the stools. Giardiasis is characterized by fat
D. Chicken pox vaccine is best given when there is an malabsorption and, therefore, steatorrhea.
impending outbreak in the community.
51.The most prevalent form of meningitis among children
Answer: (A) The older one gets, the more susceptible he aged 2 months to 3 years is caused by which microorganism?
becomes to the complications of chicken pox. Chicken pox is A. Hemophilus influenzae
usually more severe in adults than in children. Complications, B. Morbillivirus
such as pneumonia, are higher in incidence in adults. C. Steptococcus pneumoniae
D. Neisseria meningitides
47.Barangay Pinoy had an outbreak of German measles. To
prevent congenital rubella, what is the BEST advice that you Answer: (A) Hemophilus influenzae. Hemophilus meningitis is
can give to women in the first trimester of pregnancy in the unusual over the age of 5 years. In developing countries, the
barangay Pinoy? peak incidence is in children less than 6 months of age.
A. Advice them on the signs of German measles. Morbillivirus is the etiology of measles. Streptococcus
B. Avoid crowded places, such as markets and movie pneumoniae and Neisseria meningitidis may cause meningitis,
houses. but age distribution is not specific in young children.
C. Consult at the health center where rubella vaccine may
be given. 52.The student nurse is aware that the pathognomonic sign of
D. Consult a physician who may give them rubella measles is Koplik’s spot and you may see Koplik’s spot by
immunoglobulin. inspecting the:
A. Nasal mucosa
Answer: (D) Consult a physician who may give them rubella B. Buccal mucosa
immunoglobulin. Rubella vaccine is made up of attenuated C. Skin on the abdomen
German measles viruses. This is contraindicated in pregnancy. D. Skin on neck
Immune globulin, a specific prophylactic against German
measles, may be given to pregnant women.
Answer: (B) Buccal mucosa. Koplik’s spot may be seen on the 58.Several clients is newly admitted and diagnosed with
mucosa of the mouth or the throat. leprosy. Which of the following clients should be classified as
a case of multibacillary leprosy?
53.Angel was diagnosed as having Dengue fever. You will say A. 3 skin lesions, negative slit skin smear
that there is slow capillary refill when the color of the nailbed B. 3 skin lesions, positive slit skin smear
that you pressed does not return within how many seconds? C. 5 skin lesions, negative slit skin smear
A. 3 seconds D. 5 skin lesions, positive slit skin smear
B. 6 seconds
C. 9 seconds Answer: (D) 5 skin lesions, positive slit skin smear. A
D. 10 seconds multibacillary leprosy case is one who has a positive slit skin
smear and at least 5 skin lesions.
Answer: (A) 3 seconds. Adequate blood supply to the area
allows the return of the color of the nailbed within 3 59.Nurses are aware that diagnosis of leprosy is highly
seconds dependent on recognition of symptoms. Which of the
following is an early sign of leprosy?
54.In Integrated Management of Childhood Illness, the nurse A. Macular lesions
is aware that the severe conditions generally require urgent B. Inability to close eyelids
referral to a hospital. Which of the following severe C. Thickened painful nerves
conditions DOES NOT always require urgent referral to a D. Sinking of the nosebridge
hospital?
A. Mastoiditis Answer: Answer: (C) Thickened painful nerves. The lesion of
B. Severe dehydration leprosy is not macular. It is characterized by a change in skin
C. Severe pneumonia color (either reddish or whitish) and loss of sensation,
D. Severe febrile disease sweating and hair growth over the lesion. Inability to close
Answer: (B) Severe dehydration. The order of priority in the the eyelids (lagophthalmos) and sinking of the nosebridge are
management of severe dehydration is as follows: late symptoms.
intravenous fluid therapy, referral to a facility where IV fluids
can be initiated within 30 minutes, Oresol or nasogastric 60.Marie brought her 10 month old infant for consultation
tube. When the foregoing measures are not possible or because of fever, started 4 days prior to consultation. In
effective, then urgent referral to the hospital is done. determining malaria risk, what will you do?
A. Perform a tourniquet test.
55.Myrna a public health nurse will conduct outreach B. Ask where the family resides.
immunization in a barangay Masay with a population of about C. Get a specimen for blood smear.
1500. The estimated number of infants in the barangay would D. Ask if the fever is present everyday.
be:
A. 45 infants Answer: (B) Ask where the family resides. Because malaria is
B. 50 infants endemic, the first question to determine malaria risk is where
C. 55 infants the client’s family resides. If the area of residence is not a
D. 65 infants known endemic area, ask if the child had traveled within the
past 6 months, where she was brought and whether she
Answer: (A) 45 infants. To estimate the number of infants, stayed overnight in that area.
multiply total population by 3%.
61.Susie brought her 4 years old daughter to the RHU because
56.The community nurse is aware that the biological used in of cough and colds. Following the IMCI assessment guide,
Expanded Program on Immunization (EPI) should NOT be which of the following is a danger sign that indicates the need
stored in the freezer? for urgent referral to a hospital?
A. DPT A. Inability to drink
B. Oral polio vaccine B. High grade fever
C. Measles vaccine C. Signs of severe dehydration
D. MMR D. Cough for more than 30 days

Answer: (A) DPT. DPT is sensitive to freezing. The appropriate Answer: (A) Inability to drink. A sick child aged 2 months to 5
storage temperature of DPT is 2 to 8° C only. OPV and measles years must be referred urgently to a hospital if he/she has
vaccine are highly sensitive to heat and require freezing. one or more of the following signs: not able to feed or drink,
MMR is not an immunization in the Expanded Program on vomits everything, convulsions, abnormally sleepy or
Immunization. difficult to awaken

57.It is the most effective way of controlling schistosomiasis in 62.Jimmy a 2-year old child revealed “baggy pants”. As a
an endemic area? nurse, using the IMCI guidelines, how will you manage
A. Use of molluscicides Jimmy?
B. Building of foot bridges A. Refer the child urgently to a hospital for confinement.
C. Proper use of sanitary toilets B. Coordinate with the social worker to enroll the child in a
D. Use of protective footwear, such as rubber boots feeding program.
C. Make a teaching plan for the mother, focusing on menu
Answer: (C) Proper use of sanitary toilets. The ova of the planning for her child.
parasite get out of the human body together with feces. D. Assess and treat the child for health problems like
Cutting the cycle at this stage is the most effective way of infections and intestinal parasitism.
preventing the spread of the disease to susceptible hosts.
Answer: (A) Refer the child urgently to a hospital for C. 8 hours
confinement. “Baggy pants” is a sign of severe marasmus. The D. At the end of the day
best management is urgent referral to a hospital.
Answer: (B) 4 hours. While the unused portion of other
63.Gina is using Oresol in the management of diarrhea of her biologicals in EPI may be given until the end of the day, only
3-year old child. She asked you what to do if her child vomits. BCG is discarded 4 hours after reconstitution. This is why BCG
As a nurse you will tell her to: immunization is scheduled only in the morning.
A. Bring the child to the nearest hospital for further
assessment.
B. Bring the child to the health center for intravenous fluid 68.The nurse explains to a breastfeeding mother that breast
therapy. milk is sufficient for all of the baby’s nutrient needs only up
C. Bring the child to the health center for assessment by to:
the physician. A. 5 months
D. Let the child rest for 10 minutes then continue giving B. 6 months
Oresol more slowly. C. 1 year
D. 2 years
Answer: (D) Let the child rest for 10 minutes then continue
giving Oresol more slowly. If the child vomits persistently, that Answer: (B) 6 months. After 6 months, the baby’s nutrient
is, he vomits everything that he takes in, he has to be referred needs, especially the baby’s iron requirement, can no longer
urgently to a hospital. Otherwise, vomiting is managed by be provided by mother’s milk alone
letting the child rest for 10 minutes and then continuing with
Oresol administration. Teach the mother to give Oresol more 69.Nurse Ron is aware that the gestational age of a conceptus
slowly. that is considered viable (able to live outside the womb) is:
A. 8 weeks
64.Nikki a 5-month old infant was brought by his mother to B. 12 weeks
the health center because of diarrhea for 4 to 5 times a day. C. 24 weeks
Her skin goes back slowly after a skin pinch and her eyes are D. 32 weeks
sunken. Using the IMCI guidelines, you will classify this infant Answer: (C) 24 weeks. At approximately 23 to 24 weeks’
in which category? gestation, the lungs are developed enough to sometimes
A. No signs of dehydration maintain extrauterine life. The lungs are the most immature
B. Some dehydration system during the gestation period. Medical care for
C. Severe dehydration premature labor begins much earlier (aggressively at 21
D. The data is insufficient. weeks’ gestation

Answer: (B) Some dehydration. Using the assessment 70.When teaching parents of a neonate the proper position
guidelines of IMCI, a child (2 months to 5 years old) with for the neonate’s sleep, the nurse Patricia stresses the
diarrhea is classified as having SOME DEHYDRATION if he importance of placing the neonate on his back to reduce the
shows 2 or more of the following signs: restless or irritable, risk of which of the following?
sunken eyes, the skin goes back slow after a skin pinch. A. Aspiration
B. Sudden infant death syndrome (SIDS)
65.Chris a 4-month old infant was brought by her mother to C. Suffocation
the health center because of cough. His respiratory rate is D. Gastroesophageal reflux (GER)
42/minute. Using the Integrated Management of Child Illness
(IMCI) guidelines of assessment, his breathing is considered Answer: (B) Sudden infant death syndrome (SIDS). Supine
as: positioning is recommended to reduce the risk of SIDS in
A. Fast infancy. The risk of aspiration is slightly increased with the
B. Slow supine position. Suffocation would be less likely with an infant
C. Normal supine than prone and the position for GER requires the head
D. Insignificant of the bed to be elevated.

Answer: (C) Normal. In IMCI, a respiratory rate of 50/minute 71.Which finding might be seen in baby James a neonate
or more is fast breathing for an infant aged 2 to 12 months. suspected of having an infection?
A. Flushed cheeks
66.Maylene had just received her 4th dose of tetanus toxoid. B. Increased temperature
She is aware that her baby will have protection against C. Decreased temperature
tetanus for D. Increased activity level
A. 1 year
B. 3 years Answer: (C) Decreased temperature. Temperature instability,
C. 5 years especially when it results in a low temperature in the
D. Lifetime neonate, may be a sign of infection. The neonate’s color often
changes with an infection process but generally becomes
Answer: (A) 1 year. The baby will have passive natural ashen or mottled. The neonate with an infection will usually
immunity by placental transfer of antibodies. The mother will show a decrease in activity level or lethargy.
have active artificial immunity lasting for about 10 years. 5
doses will give the mother lifetime protection. 72.Baby Jenny who is small-for-gestation is at increased risk
during the transitional period for which complication?
67.Nurse Ron is aware that unused BCG should be discarded A. Anemia probably due to chronic fetal hyposia
after how many hours of reconstitution? B. Hyperthermia due to decreased glycogen stores
A. 2 hours C. Hyperglycemia due to decreased glycogen stores
B. 4 hours D. Polycythemia probably due to chronic fetal hypoxia
Answer: (B) Conjunctival hemorrhage. Conjunctival
Answer: (D) Polycythemia probably due to chronic fetal hemorrhages are commonly seen in neonates secondary to
hypoxia. The small-for-gestation neonate is at risk for the cranial pressure applied during the birth process. Bulging
developing polycythemia during the transitional period in an fontanelles are a sign of intracranial pressure. Simian creases
attempt to decreasehypoxia. The neonates are also at are present in 40% of the neonates with trisomy 21. Cystic
increased risk for developing hypoglycemia and hypothermia hygroma is a neck mass that can affect the airway.
due to decreased glycogen stores.
78.Dr. Esteves decides to artificially rupture the membranes
73.Marjorie has just given birth at 42 weeks’ gestation. When of a mother who is on labor. Following this procedure, the
the nurse assessing the neonate, which physical finding is nurse Hazel checks the fetal heart tones for which the
expected? following reasons?
A. A sleepy, lethargic baby A. To determine fetal well-being.
B. Lanugo covering the body B. To assess for prolapsed cord
C. Desquamation of the epidermis C. To assess fetal position
D. Vernix caseosa covering the body D. To prepare for an imminent delivery.

Answer: (C) Desquamation of the epidermis. Postdate fetuses Answer: (B) To assess for prolapsed cord. After a client has an
lose the vernix caseosa, and the epidermis may become amniotomy, the nurse should assure that the cord isn’t
desquamated. These neonates are usually very alert. Lanugo prolapsed and that the baby tolerated the procedure well.
is missing in the postdate neonate. The most effective way to do this is to check the fetal heart
rate. Fetal well-being is assessed via a nonstress test. Fetal
74.After reviewing the Myrna’s maternal history of position is determined by vaginal examination. Artificial
magnesium sulfate during labor, which condition would nurse rupture of membranes doesn’t indicate an imminent delivery
Richard anticipate as a potential problem in the neonate?
A. Hypoglycemia 79.Which of the following would be least likely to indicate
B. Jitteriness anticipated bonding behaviors by new parents?
C. Respiratory depression A. The parents’ willingness to touch and hold the new
D. Tachycardia born.
Answer: (C) Respiratory depression. Magnesium sulfate B. The parent’s expression of interest about the size of the
crosses the placenta and adverse neonatal effects are new born.
respiratory depression, hypotonia, and bradycardia. The C. The parents’ indication that they want to see the
serum blood sugar isn’t affected by magnesium sulfate. The newborn.
neonate would be floppy, not jittery. D. The parents’ interactions with each other.

75.Which symptom would indicate the Baby Alexandra was Answer: (D) The parents’ interactions with each
adapting appropriately to extra-uterine life without difficulty? other. Parental interaction will provide the nurse with a good
A. Nasal flaring assessment of the stability of the family’s home life but it has
B. Light audible grunting no indication for parental bonding. Willingness to touch and
C. Respiratory rate 40 to 60 breaths/minute hold the newborn, expressing interest about the newborn’s
D. Respiratory rate 60 to 80 breaths/minute size, and indicating a desire to see the newborn are behaviors
indicating parental bonding.
Answer: (C) Respiratory rate 40 to 60 breaths/minute. A
respiratory rate 40 to 60 breaths/minute is normal for a 80.Following a precipitous delivery, examination of the
neonate during the transitional period. Nasal flaring, client’s vagina reveals
respiratory rate more than 60 breaths/minute, and audible a fourth-degree laceration. Which of the following would be
grunting are signs of respiratory distress. contraindicated when caring for this client?
A. Applying cold to limit edema during the first 12 to 24
76. When teaching umbilical cord care for Jennifer a new hours.
mother, the nurse Jenny would include which information? B. Instructing the client to use two or more peripads to
A. Apply peroxide to the cord with each diaper change cushion the area.
B. Cover the cord with petroleum jelly after bathing C. Instructing the client on the use of sitz baths if ordered.
C. Keep the cord dry and open to air D. Instructing the client about the importance of perineal
D. Wash the cord with soap and water each day during a (kegel) exercises.
tub bath.
Answer: (B) Instructing the client to use two or more peripads
Answer: (C) Keep the cord dry and open to air. Keeping the to cushion the area. Using two or more peripads would do
cord dry and open to air helps reduce infection and hastens little to reduce the pain or promote perineal healing. Cold
drying. Infants aren’t given tub bath but are sponged off until applications, sitz baths, and Kegel exercises are important
the cord falls off. Petroleum jelly prevents the cord from measures when the client has a fourth-degree laceration.
drying and encourages infection. Peroxide could be painful
and isn’t recommended. 81. A pregnant woman accompanied by her husband, seeks
admission to the labor and delivery area. She states that she’s
77.Nurse John is performing an assessment on a neonate. in labor and says she attended the facility clinic for prenatal
Which of the following findings is considered common in the care. Which question should the nurse Oliver ask her first?
healthy neonate? A. “Do you have any chronic illnesses?”
A. Simian crease B. “Do you have any allergies?”
B. Conjunctival hemorrhage C. “What is your expected due date?”
C. Cystic hygroma D. “Who will be with you during labor?”
D. Bulging fontanelle
Answer: (C) “What is your expected due date?” When C. At least 2 ml per feeding
obtaining the history of a client who may be in labor, the D. 90 to 100 calories per kg
nurse’s highest priority is to determine her current status,
particularly her due date, gravidity, and parity. Gravidity and Answer: (A) 110 to 130 calories per kg. Calories per kg is the
parity affect the duration of labor and the potential for labor accepted way of determined appropriate nutritional intake for
complications. Later, the nurse should ask about chronic a newborn. The recommended calorie requirement is 110 to
illnesses, allergies, and support persons. 130 calories per kg of newborn body weight. This level will
maintain a consistent blood glucose level and provide enough
82.A neonate begins to gag and turns a dusky color. What calories for continued growth and development.
should the nurse do first?
A. Calm the neonate. 86. Nurse John is knowledgeable that usually individual twins
B. Notify the physician. will grow appropriately and at the same rate as singletons
C. Provide oxygen via face mask as ordered until how many weeks?
D. Aspirate the neonate’s nose and mouth with a bulb A. 16 to 18 weeks
syringe. B. 18 to 22 weeks
C. 30 to 32 weeks
Answer: (D) Aspirate the neonate’s nose and mouth with a D. 38 to 40 weeks
bulb syringe. The nurse’s first action should be to clear the
neonate’s airway with a bulb syringe. After the airway is clear Answer: (C) 30 to 32 weeks. Individual twins usually grow at
and the neonate’s color improves, the nurse should comfort the same rate as singletons until 30 to 32 weeks’ gestation,
and calm the neonate. If the problem recurs or the neonate’s then twins don’t’ gain weight as rapidly as singletons of the
color doesn’t improve readily, the nurse should notify the same gestational age. The placenta can no longer keep pace
physician. Administering oxygen when the airway isn’t clear with the nutritional requirements of both fetuses after 32
would be ineffective. weeks, so there’s some growth retardation in twins if they
remain in utero at 38 to 40 weeks.

83. When a client states that her “water broke,” which of the 87. Which of the following classifications applies to
following actions would be inappropriate for the nurse to do? monozygotic twins for whom the cleavage of the fertilized
A. Observing the pooling of straw-colored fluid. ovum occurs more than 13 days after fertilization?
B. Checking vaginal discharge with nitrazine paper. A. conjoined twins
C. Conducting a bedside ultrasound for an amniotic fluid B. diamniotic dichorionic twins
index. C. diamniotic monochorionic twin
D. Observing for flakes of vernix in the vaginal discharge. D. monoamniotic monochorionic twins

Answer: (C) Conducting a bedside ultrasound for an amniotic Answer: (A) conjoined twins. The type of placenta that
fluid index. It isn’t within a nurse’s scope of practice to develops in monozygotic twins depends on the time at which
perform and interpret a bedside ultrasound under these cleavage of the ovum occurs. Cleavage in conjoined twins
conditions and without specialized training. Observing for occurs more than 13 days after fertilization. Cleavage that
pooling of straw-colored fluid, checking vaginal discharge with occurs less than 3 day after fertilization results in diamniotic
nitrazine paper, and observing for flakes of vernix are dicchorionic twins. Cleavage that occurs between days 3 and
appropriate assessments for determining whether a client has 8 results in diamniotic monochorionic twins. Cleavage that
ruptured membranes. occurs between days 8 to 13 result in monoamniotic
monochorionic twins.

84. A baby girl is born 8 weeks premature. At birth, she has no 88. Tyra experienced painless vaginal bleeding has just been
spontaneous respirations but is successfully resuscitated. diagnosed as having a placenta previa. Which of the following
Within several hours she develops respiratory grunting, procedures is usually performed to diagnose placenta previa?
cyanosis, tachypnea, nasal flaring, and retractions. She’s A. Amniocentesis
diagnosed with respiratory distress syndrome, intubated, and B. Digital or speculum examination
placed on a ventilator. Which nursing action should be C. External fetal monitoring
included in the baby’s plan of care to prevent retinopathy of D. Ultrasound
prematurity?
A. Cover his eyes while receiving oxygen. Answer: (D) Ultrasound. Once the mother and the fetus are
B. Keep her body temperature low. stabilized, ultrasound evaluation of the placenta should be
C. Monitor partial pressure of oxygen (Pao2) levels. done to determine the cause of the bleeding. Amniocentesis
D. Humidify the oxygen. is contraindicated in placenta previa. A digital or speculum
examination shouldn’t be done as this may lead to severe
Answer: (C) Monitor partial pressure of oxygen (Pao2) bleeding or hemorrhage. External fetal monitoring won’t
levels. Monitoring PaO2 levels and reducing the oxygen detect a placenta previa, although it will detect fetal distress,
concentration to keep PaO2 within normal limits reduces the which may result from blood loss or placenta separation.
risk of retinopathy of prematurity in a premature infant
receiving oxygen. Covering the infant’s eyes and humidifying 89. Nurse Arnold knows that the following changes in
the oxygen don’t reduce the risk of retinopathy of respiratory functioning during pregnancy is considered
prematurity. Because cooling increases the risk of acidosis, normal:
the infant should be kept warm so that his respiratory A. Increased tidal volume
distress isn’t aggravated. B. Increased expiratory volume
C. Decreased inspiratory capacity
85. Which of the following is normal newborn calorie intake? D. Decreased oxygen consumption
A. 110 to 130 calories per kg.
B. 30 to 40 calories per lb of body weight.
Answer: (A) Increased tidal volume. A pregnant client
breathes deeper, which increases the tidal volume of gas 94. Marlyn is screened for tuberculosis during her first
moved in and out of the respiratory tract with each breath. prenatal visit. An intradermal injection of purified protein
The expiratory volume and residual volume decrease as the derivative (PPD) of the tuberculin bacilli is given. She is
pregnancy progresses. The inspiratory capacity increases considered to have a positive test for which of the following
during pregnancy. The increased oxygen consumption in the results?
pregnant client is 15% to 20% greater than in the A. An indurated wheal under 10 mm in diameter appears
nonpregnant state. in 6 to 12 hours.
B. An indurated wheal over 10 mm in diameter appears in
90. Emily has gestational diabetes and it is usually managed 48 to 72 hours.
by which of the following therapy? C. A flat circumcised area under 10 mm in diameter
A. Diet appears in 6 to 12 hours.
B. Long-acting insulin D. A flat circumcised area over 10 mm in diameter appears
C. Oral hypoglycemic in 48 to 72 hours.
D. Oral hypoglycemic drug and insulin
Answer: (B) An indurated wheal over 10 mm in diameter
Answer: (A) Diet. Clients with gestational diabetes are appears in 48 to 72 hours. A positive PPD result would be an
usually managed by diet alone to control their glucose indurated wheal over 10 mm in diameter that appears in 48
intolerance. Oral hypoglycemic drugs are contraindicated in to 72 hours. The area must be a raised wheal, not a flat
pregnancy. Long-acting insulin usually isn’t needed for blood circumcised area to be considered positive.
glucose control in the client with gestational diabetes
95. Dianne, 24 year-old is 27 weeks’ pregnant arrives at her
91. Magnesium sulfate is given to Jemma with preeclampsia physician’s office with complaints of fever, nausea, vomiting,
to prevent which of the following condition? malaise, unilateral flank pain, and costovertebral angle
A. Hemorrhage tenderness. Which of the following diagnoses is most likely?
B. Hypertension A. Asymptomatic bacteriuria
C. Hypomagnesemia B. Bacterial vaginosis
D. Seizure C. Pyelonephritis
D. Urinary tract infection (UTI)
1. Answer: (D) Seizure. The anticonvulsant mechanism Answer: (C) Pyelonephritis. The symptoms indicate acute
of magnesium is believes to depress seizure foci in the pyelonephritis, a serious condition in a pregnant client. UTI
brain and peripheral neuromuscular blockade. symptoms include dysuria, urgency, frequency, and
Hypomagnesemia isn’t a complication of preeclampsia. suprapubic tenderness. Asymptomatic bacteriuria doesn’t
Antihypertensive drug other than magnesium are cause symptoms. Bacterial vaginosis causes milky white
preferred for sustained hypertension. Magnesium vaginal discharge but no systemic symptoms
doesn’t help prevent hemorrhage in preeclamptic
clients. 96. Rh isoimmunization in a pregnant client develops during
which of the following conditions?
A. Rh-positive maternal blood crosses into fetal blood,
92. Cammile with sickle cell anemia has an increased risk for stimulating fetal antibodies.
having a sickle cell crisis during pregnancy. Aggressive B. Rh-positive fetal blood crosses into maternal blood,
management of a sickle cell crisis includes which of the stimulating maternal antibodies.
following measures? C. Rh-negative fetal blood crosses into maternal blood,
A. Antihypertensive agents stimulating maternal antibodies.
B. Diuretic agents D. Rh-negative maternal blood crosses into fetal blood,
C. I.V. fluids stimulating fetal antibodies.
D. Acetaminophen (Tylenol) for pain
Answer: (B) Rh-positive fetal blood crosses into maternal
Answer: (C) I.V. fluids. A sickle cell crisis during pregnancy is blood, stimulating maternal antibodies. Rh isoimmunization
usually managed by exchange transfusion oxygen, and L.V. occurs when Rh-positive fetal blood cells cross into the
Fluids. The client usually needs a stronger analgesic than maternal circulation and stimulate maternal
acetaminophen to control the pain of a crisis. antibody production. In subsequent pregnancies with Rh-
Antihypertensive drugs usually aren’t necessary. Diuretic positive fetuses, maternal antibodies may cross back into the
wouldn’t be used unless fluid overload resulted. fetal circulation and destroy the fetal blood cells

93. Which of the following drugs is the antidote for 97. To promote comfort during labor, the nurse John advises a
magnesium toxicity? client to assume certain positions and avoid others. Which
A. Calcium gluconate (Kalcinate) position may cause maternal hypotension and fetal hypoxia?
B. Hydralazine (Apresoline) A. Lateral position
C. Naloxone (Narcan) B. Squatting position
D. Rho (D) immune globulin (RhoGAM) C. Supine position
D. Standing position
Answer: (A) Calcium gluconate (Kalcinate). Calcium gluconate
is the antidote for magnesium toxicity. Ten milliliters of 10% Answer: (C) Supine position. The supine position causes
calcium gluconate is given L.V. push over 3 to 5 minutes. compression of the client’s aorta and inferior vena cava by the
Hydralazine is given for sustained elevated blood pressure in fetus. This, in turn, inhibits maternal circulation, leading to
preeclamptic clients. Rho (D) immune globulin is given to maternal hypotension and, ultimately, fetal hypoxia. The
women with Rh-negative blood to prevent antibody other positions promote comfort and aid labor progress. For
formation from RH-positive conceptions. Naloxone is used to instance, the lateral, or side-lying, position improves maternal
correct narcotic toxicity. and fetal circulation, enhances comfort, increases maternal
relaxation, reduces muscle tension, and eliminates pressure Answer: (C) Loose, bloody. Normal bowel function and soft-
points. The squatting position promotes comfort by taking formed stool usually do not occur until around the seventh
advantage of gravity. The standing position also takes day following surgery. The stool consistency is related to how
advantage of gravity and aligns the fetus with the pelvic much water is being absorbed
angle.
2. Where would nurse Kristine place the call light for a male
98. Celeste who used heroin during her pregnancy delivers a client with a right-sided brain attack and left homonymous
neonate. When assessing the neonate, the nurse Lhynnette hemianopsia?
expects to find:
A. Lethargy 2 days after birth. A. On the client’s right side
B. Irritability and poor sucking. B. On the client’s left side
C. A flattened nose, small eyes, and thin lips. C. Directly in front of the client
D. Congenital defects such as limb anomalies. D. Where the client like

Answer: (B) Irritability and poor sucking. Neonates of heroin- Answer: (A) On the client’s right side. The client has left
addicted mothers are physically dependent on the drug and visual field blindness. The client will see only from the right
experience withdrawal when the drug is no longer supplied. side
Signs of heroin withdrawal include irritability, poor sucking,
and restlessness. Lethargy isn’t associated with neonatal 3. A male client is admitted to the emergency department
heroin addiction. A flattened nose, small eyes, and thin lips following an accident. What are the first nursing actions of
are seen in infants with fetal alcohol syndrome. Heroin use the nurse?
during pregnancy hasn’t been linked to specific congenital
anomalies. A. Check respiration, circulation, neurological response.
B. Align the spine, check pupils, and check for hemorrhage.
99. The uterus returns to the pelvic cavity in which of the C. Check respirations, stabilize spine, and check
following time frames? circulation.
A. 7th to 9th day postpartum. D. Assess level of consciousness and circulation.
B. 2 weeks postpartum.
C. End of 6th week postpartum. Answer: (C) Check respirations, stabilize spine, and check
D. When the lochia changes to alba. circulation. Checking the airway would be priority, and a
Answer: (A) 7th to 9th day postpartum. The normal neck injury should be suspected
involutional process returns the uterus to the pelvic cavity in
7 to 9 days. A significant involutional complication is the 4. In evaluating the effect of nitroglycerin, Nurse Arthur
failure of the uterus to return to the pelvic cavity within the should know that it reduces preload and relieves angina by:
prescribed time period. This is known as subinvolution
A. Increasing contractility and slowing heart rate.
B. Increasing AV conduction and heart rate.
100. Maureen, a primigravida client, age 20, has just
C. Decreasing contractility and oxygen consumption.
completed a difficult, forceps-assisted delivery of twins. Her
D. Decreasing venous return through vasodilation.
labor was unusually long and required oxytocin (Pitocin)
augmentation. The nurse who’s caring for her should stay
Answer: (D) Decreasing venous return through
alert for:
vasodilation. The significant effect of nitroglycerin is
A. Uterine inversion
vasodilation and decreased venous return, so the heart does
B. Uterine atony
not have to work hard.
C. Uterine involution
D. Uterine discomfort
5. Nurse Patricia finds a female client who is post-myocardial
Answer: (B) Uterine atony. Multiple fetuses, extended labor infarction (MI) slumped on the side rails of the bed and
stimulation with oxytocin, and traumatic delivery commonly unresponsive to shaking or shouting. Which is the nurse next
are associated with uterine atony, which may lead to action?
postpartum hemorrhage. Uterine inversion may precede or
A. Call for help and note the time.
follow delivery and commonly results from apparent
B. Clear the airway
excessive traction on the umbilical cord and attempts to
C. Give two sharp thumps to the precordium, and check
deliver the placenta manually. Uterine involution and some
the pulse.
uterine discomfort are normal after delivery.
D. Administer two quick blows.

PNLE III for Care of Clients with Answer: (A) Call for help and note the time. Having
established, by stimulating the client, that the client
Physiologic and Psychosocial is unconscious rather than sleep, the nurse should
Alterations (Part 1) immediately call for help. This may be done by dialing the
operator from the client’s phone and giving the hospital
code for cardiac arrest and the client’s room number to the
1. Nurse Michelle should know that the drainage is normal 4 operator, of if the phone is not available, by pulling the
days after a sigmoid colostomy when the stool is: emergency call button. Noting the time is important baseline
information for cardiac arrest procedure
A. Green liquid
B. Solid formed
C. Loose, bloody 6. Nurse Monett is caring for a client recovering from gastro-
D. Semiformed intestinal bleeding. The nurse should:
A. Plan care so the client can receive 8 hours of Answer: (C) The client is oriented when aroused from sleep,
uninterrupted sleep each night. and goes back to sleep immediately. This finding suggest that
B. Monitor vital signs every 2 hours. the level of consciousness is decreasing.
C. Make sure that the client takes food and medications
at prescribed intervals. 11.Mrs. Cruz, 80 years old is diagnosed with pneumonia.
D. Provide milk every 2 to 3 hours. Which of the following symptoms may appear first?

Answer: (C) Make sure that the client takes food and A. Altered mental status and dehydration
medications at prescribed intervals. Food and drug therapy B. Fever and chills
will prevent the accumulation of hydrochloric acid, or will C. Hemoptysis and Dyspnea
neutralize and buffer the acid that does accumulate. D. Pleuritic chest pain and cough

7. A male client was on warfarin (Coumadin) before Answer: (A) Altered mental status and dehydration. Fever,
admission, and has been receiving heparin I.V. for 2 days. The chills, hemortysis, dyspnea, cough, and pleuritic chest pain
partial thromboplastin time (PTT) is 68 seconds. What should are the common symptoms of pneumonia, but elderly clients
may first appear with only an altered lentil status and
Nurse Carla do?
dehydration due to a blunted immune response.
A. Stop the I.V. infusion of heparin and notify the physician.
B. Continue treatment as ordered. 12. A male client has active tuberculosis (TB). Which of the
C. Expect the warfarin to increase the PTT. following symptoms will be exhibit?
D. Increase the dosage, because the level is lower than
normal. A. Chest and lower back pain
B. Chills, fever, night sweats, and hemoptysis
Answer: (B) Continue treatment as ordered. The effects of C. Fever of more than 104°F (40°C) and nausea
heparin are monitored by the PTT is normally 30 to 45 D. Headache and photophobia
seconds; the therapeutic level is 1.5 to 2 times the normal
level. Answer: (B) Chills, fever, night sweats, and
hemoptysis. Typical signs and symptoms are chills, fever,
8. A client undergone ileostomy, when should the drainage night sweats, and hemoptysis. Chest pain may be present
from coughing, but isn’t usual. Clients with TB typically have
appliance be applied to the stoma?
low-grade fevers, not higher than 102°F (38.9°C). Nausea,
A. 24 hours later, when edema has subsided. headache, and photophobia aren’t usual TB symptoms.
B. In the operating room.
C. After the ileostomy begin to function. 13. Mark, a 7-year-old client is brought to the emergency
D. When the client is able to begin self-care procedures. department. He’s tachypneic and afebrile and has a
respiratory rate of 36 breaths/minute and has a
Answer: (B) In the operating room. The stoma drainage bag is nonproductive cough. He recently had a cold. Form this
applied in the operating room. Drainage from the ileostomy history; the client may have which of the following
contains secretions that are rich in digestive enzymes and conditions?
highly irritating to the skin. Protection of the skin from
the effects of these enzymes is begun at once. Skin exposed A. Acute asthma
to these enzymes even for a short time becomes reddened, B. Bronchial pneumonia
painful, and excoriated. C. Chronic obstructive pulmonary disease (COPD)
D. Emphysema
9. A client undergone spinal anesthetic, it will be important
that the nurse immediately position the client in: Answer:(A) Acute asthma. Based on the client’s history and
symptoms, acute asthma is the most likely diagnosis. He’s
A. On the side, to prevent obstruction of airway by tongue. unlikely to have bronchial pneumonia without a productive
B. Flat on back. cough and fever and he’s too young to have developed
C. On the back, with knees flexed 15 degrees. (COPD) and emphysema.
D. Flat on the stomach, with the head turned to the side.
14. Marichu was given morphine sulfate for pain. She is
Answer: (B) Flat on back. To avoid the complication of a sleeping and her respiratory rate is 4 breaths/minute. If action
painful spinal headache that can last for several days, the isn’t taken quickly, she might have which of the following
client is kept in flat in a supine position for approximately 4 to reactions?
12 hours postoperatively. Headaches are believed to be
causes by the seepage of cerebral spinal fluid from the A. Asthma attack
puncture site. By keeping the client flat, cerebral spinal fluid B. Respiratory arrest
pressures are equalized, which avoids trauma to the neurons. C. Seizure
D. Wake up on his own
10.While monitoring a male client several hours after a motor
vehicle accident, which assessment data suggest increasing Answer: (B) Respiratory arrest. Narcotics can cause
intracranial pressure? respiratory arrest if given in large quantities. It’s unlikely the
client will have asthma attack or a seizure or wake up on his
A. Blood pressure is decreased from 160/90 to 110/70. own.
B. Pulse is increased from 87 to 95, with an occasional
skipped beat. 15. A 77-year-old male client is admitted for elective knee
C. The client is oriented when aroused from sleep, and surgery. Physical examination reveals shallow respirations but
goes back to sleep immediately. no sign of respiratory distress. Which of the following is a
D. The client refuses dinner because of anorexia.
normal physiologic change related to aging?
A. Increased elastic recoil of the lungs osteoporosis. Calcium and vitamin D supplements may be
B. Increased number of functional capillaries in the alveoli used to support normal bone metabolism, But a negative
C. Decreased residual volume calcium balance isn’t a complication of osteoporosis.
D. Decreased vital capacity Dowager’s hump results from bone fractures. It
develops when repeated vertebral fractures increase spinal
Answer: (D) Decreased vital capacity. Reduction in vital curvature.
capacity is a normal physiologic changes include decreased
elastic recoil of the lungs, fewer functional capillaries in the 20. Nurse Len is teaching a group of women to perform BSE.
alveoli, and an increased in residual volume. The nurse should explain that the purpose of performing the
examination is to discover:
16. Nurse John is caring for a male client receiving lidocaine
I.V. Which factor is the most relevant to administration of this A. Cancerous lumps
medication? B. Areas of thickness or fullness
C. Changes from previous examinations.
A. Decrease in arterial oxygen saturation (SaO2) when D. Fibrocystic masses
measured with a pulse oximeter.
B. Increase in systemic blood pressure. Answer: (C) Changes from previous examinations. Women are
C. Presence of premature ventricular contractions (PVCs) instructed to examine themselves to discover changes that
on a cardiac monitor. have occurred in the breast. Only a physician can
D. Increase in intracranial pressure (ICP). diagnose lumps that are cancerous, areas of thickness or
fullness that signal the presence of a malignancy, or masses
Answer: (C) Presence of premature ventricular contractions that are fibrocystic as opposed to malignant.
(PVCs) on a cardiac monitor. Lidocaine drips are commonly
used to treat clients whose arrhythmias haven’t been 21. When caring for a female client who is being treated for
controlled with oral medication and who are having PVCs that hyperthyroidism, it is important to:
are visible on the cardiac monitor. SaO2, blood pressure, and
ICP are important factors but aren’t as significant as PVCs in A. Provide extra blankets and clothing to keep the client
the situation. warm.
B. Monitor the client for signs of restlessness, sweating,
17. Nurse Ron is caring for a male client taking an and excessive weight loss during thyroid replacement
anticoagulant. The nurse should teach the client to: therapy.
C. Balance the client’s periods of activity and rest.
A. Report incidents of diarrhea. D. Encourage the client to be active to prevent constipation
B. Avoid foods high in vitamin K
C. Use a straight razor when shaving. Answer: (C) Balance the client’s periods of activity and
D. Take aspirin to pain relief. rest. A client with hyperthyroidism needs to be encouraged
to balance periods of activity and rest. Many clients with
Answer: (B) Avoid foods high in vitamin K. The client should hyperthyroidism are hyperactive and complain of feeling
avoid consuming large amounts of vitamin K because vitamin very warm
K can interfere with anticoagulation. The client may need to .
report diarrhea, but isn’t effect of taking an anticoagulant. An 22. Nurse Kris is teaching a client with history of
electric razor-not a straight razor-should be used to prevent atherosclerosis. To decrease the risk of atherosclerosis, the
cuts that cause bleeding. Aspirin may increase the risk of nurse should encourage the client to:
bleeding; acetaminophen should be used to pain relief.
A. Avoid focusing on his weight.
18. Nurse Lhynnette is preparing a site for the insertion of an B. Increase his activity level.
I.V. catheter. The nurse should treat excess hair at the site by: C. Follow a regular diet.
D. Continue leading a high-stress lifestyle.
A. Leaving the hair intact
B. Shaving the area Answer: (B) Increase his activity level. The client should be
C. Clipping the hair in the area encouraged to increase his activity level. Maintaining an ideal
D. Removing the hair with a depilatory. weight; following a low-cholesterol, low sodium diet; and
avoiding stress are all important factors in decreasing the risk
Answer: (C) Clipping the hair in the area. Hair can be a source of atherosclerosis.
of infection and should be removed by clipping. Shaving the
area can cause skin abrasions and depilatories can irritate the 23. Nurse Greta is working on a surgical floor. Nurse Greta
skin. must logroll a client following a:

19. Nurse Michelle is caring for an elderly female with A. Laminectomy


osteoporosis. When teaching the client, the nurse should B. Thoracotomy
include information about which major complication: C. Hemorrhoidectomy
D. Cystectomy.
A. Bone fracture
B. Loss of estrogen Answer: (A) Laminectomy. The client who has had spinal
C. Negative calcium balance surgery, such as laminectomy, must be log rolled to keep the
D. Dowager’s hump spinal column straight when turning. Thoracotomy and
cystectomy may turn themselves or may be assisted into a
Answer: (A) Bone fracture. Bone fracture is a major comfortable position. Under normal
complication of osteoporosis that results when loss of calcium circumstances, hemorrhoidectomy is an outpatient
and phosphate increased the fragility of bones. Estrogen procedure, and the client may resume normal activities
deficiencies result from menopause-not immediately after surgery.
breath. Kussmaul’s respirations are rapid, deep breathing
24. A 55-year old client underwent cataract removal with without pauses. Tachypnea is shallow breathing with
intraocular lens implant. Nurse Oliver is giving the client increased respiratory rate
discharge instructions. These instructions should include
which of the following? 28. Nurse Bea is assessing a male client with heart failure. The
breath sounds commonly auscultated in clients with heart
A. Avoid lifting objects weighing more than 5 lb (2.25 kg). failure are:
B. Lie on your abdomen when in bed
C. Keep rooms brightly lit. A. Tracheal
D. Avoiding straining during bowel movement or bending B. Fine crackles
at the waist. C. Coarse crackles
D. Friction rubs
Answer: (D) Avoiding straining during bowel movement or
bending at the waist. The client should avoid straining, lifting Answer: (B) Fine crackles. Fine crackles are caused by fluid in
heavy objects, and coughing harshly because these activities the alveoli and commonly occur in clients with heart failure.
increase intraocular pressure. Typically, the client is Tracheal breath sounds are auscultated over the trachea.
instructed to avoid lifting objects weighing more than 15 lb Coarse crackles are caused by secretion accumulation in the
(7kg) – not 5lb. instruct the client when lying in bed to lie on airways. Friction rubs occur with pleural inflammation.
either the side or back. The client should avoid bright light
by wearing sunglasses.
29. The nurse is caring for Kenneth experiencing an acute
25. George should be taught about testicular examinations asthma attack. The client stops wheezing and breath sounds
during: aren’t audible. The reason for this change is that:
A. when sexual activity starts A. The attack is over.
B. After age 69 B. The airways are so swollen that no air cannot get
C. After age 40 through.
D. Before age 20. C. The swelling has decreased.
D. Crackles have replaced wheezes.
Answer: (D) Before age 20. Testicular cancer commonly
occurs in men between ages 20 and 30. A male client should Answer: (B) The airways are so swollen that no air cannot get
be taught how to perform testicular selfexamination before through. During an acute attack, wheezing may stop and
age 20, preferably when he enters his teens. breath sounds become inaudible because the airways are so
swollen that air can’t get through. If the attack is over and
26. A male client undergone a colon resection. While turning swelling has decreased, there would be no more wheezing
him, wound dehiscence with evisceration occurs. Nurse Trish and less emergent concern. Crackles do not replace wheezes
first response is to: during an acute asthma attack.

A. Call the physician 30. Mike with epilepsy is having a seizure. During the active
B. Place a saline-soaked sterile dressing on the wound. seizure phase, the nurse should:
C. Take a blood pressure and pulse.
D. Pull the dehiscence closed. A. Place the client on his back remove dangerous objects,
and insert a bite block.
Answer: (B) Place a saline-soaked sterile dressing on the B. Place the client on his side, remove dangerous objects,
wound. The nurse should first place saline-soaked sterile and insert a bite block.
dressings on the open wound to prevent tissue drying and C. Place the client o his back, remove dangerous objects,
possible infection. Then the nurse should call the physician and hold down his arms.
and take the client’s vital signs. The dehiscence needs to be D. Place the client on his side, remove dangerous objects,
surgically closed, so the nurse should never try to close it. and protect his head.

Answer: (D) Place the client on his side, remove dangerous


27. Nurse Audrey is caring for a client who has suffered a objects, and protect his head. During the active seizure
severe cerebrovascular accident. During routine assessment, phase, initiate precautions by placing the client on his side,
the nurse notices Cheyne- Strokes respirations. Cheyne- removing dangerous objects, and protecting his head from
strokes respirations are: injury. A bite block should never be inserted during the
active seizure phase. Insertion can break the teeth and lead
A. A progressively deeper breaths followed by shallower to aspiration
breaths with apneic periods. 31. After insertion of a cheat tube for a pneumothorax, a
B. Rapid, deep breathing with abrupt pauses between client becomes hypotensive with neck vein distention,
each breath. tracheal shift, absent breath sounds, and diaphoresis. Nurse
C. Rapid, deep breathing and irregular breathing without Amanda suspects a tension pneumothorax has
pauses. occurred. What cause of tension pneumothorax should the
D. Shallow breathing with an increased respiratory rate.
nurse check for?

A. Infection of the lung.


Answer: (A) A progressively deeper breaths followed by B. Kinked or obstructed chest tube
shallower breaths with apneic periods. Cheyne-Strokes C. Excessive water in the water-seal chamber
respirations are breaths that become progressively deeper D. Excessive chest tube drainage
fallowed by shallower respirations with apneas periods. Biot’s
respirations are rapid, deep breathing with abrupt Answer: (B) Kinked or obstructed chest tube. Kinking and
pauses between each breath, and equal depth between each blockage of the chest tube is a common cause of a tension
pneumothorax. Infection and excessive drainage won’t cause C. Pneumonia
a tension pneumothorax. Excessive water won’t affect the D. Tuberculosis
chest tube drainage.
Answer: (C) Pneumonia. Fever productive cough and pleuritic
32. Nurse Maureen is talking to a male client, the client chest pain are common signs and symptoms of pneumonia.
begins choking on his lunch. He’s coughing forcefully. The The client with ARDS has dyspnea and hypoxia with
nurse should: worsening hypoxia over time, if not treated
aggressively. Pleuritic chest pain varies with respiration, unlike
A. Stand him up and perform the abdominal thrust the constant chest pain during an MI; so this client most likely
maneuver from behind. isn’t having an MI. the client with TB typically has a cough
B. Lay him down, straddle him, and perform the abdominal producing blood-tinged sputum. A sputum culture should be
thrust maneuver. obtained to confirm the nurse’s suspicions.
C. Leave him to get assistance
D. Stay with him but not intervene at this time. 36. Nurse Oliver is working in a out patient clinic. He has been
alerted that there is an outbreak of tuberculosis (TB). Which
Answer: (D) Stay with him but not intervene at this time. If of the following clients entering the clinic today most likely to
the client is coughing, he should be able to dislodge have TB?
the object or cause a complete obstruction. If complete
obstruction occurs, the nurse should perform the abdominal A. A 16-year-old female high school student
thrust maneuver with the client standing. If the client is B. A 33-year-old day-care worker
unconscious, she should lay him down. A nurse should never C. A 43-yesr-old homeless man with a history of
leave a choking client alone. alcoholism
D. A 54-year-old businessman
33. Nurse Ron is taking a health history of an 84 year old
client. Which information will be most useful to the nurse for Answer: (C) A 43-yesr-old homeless man with a history of
planning care? alcoholism. Clients who are economically disadvantaged,
malnourished, and have reduced immunity, such as a client
A. General health for the last 10 years. with a history of alcoholism, are at extremely high risk for
B. Current health promotion activities. developing TB. A high school student, daycare worker, and
C. Family history of diseases. businessman probably have a much low risk of contracting TB.
D. Marital status.
37. Virgie with a positive Mantoux test result will be sent for a
Answer: (B) Current health promotion activities. Recognizing chest X-ray. The nurse is aware that which of the following
an individual’s positive health measures is very useful. reasons this is done?
General health in the previous 10 years is important,
however, the current activities of an 84 year old client are A. To confirm the diagnosis
most significant in planning care. Family history of disease for B. To determine if a repeat skin test is needed
a client in later years is of minor significance. Marital status C. To determine the extent of lesions
information may be important for discharge planning but is D. To determine if this is a primary or secondary infection
not as significant for addressing the immediate
medical problem. Answer: (C ) To determine the extent of lesions. If the lesions
are large enough, the chest X-ray will show their presence in
34. When performing oral care on a comatose client, Nurse the lungs. Sputum culture confirms the diagnosis. There
Krina should: can be false-positive and false-negative skin test results. A
chest X-ray can’t determine if this is a primary or secondary
A. Apply lemon glycerin to the client’s lips at least every 2 infection.
hours.
B. Brush the teeth with client lying supine. 38. Kennedy with acute asthma showing inspiratory and
C. Place the client in a side lying position, with the head expiratory wheezes and a decreased forced expiratory volume
of the bed lowered.
should be treated with which of the following classes of
D. Clean the client’s mouth with hydrogen peroxide.
medication right away?
Answer: (C) Place the client in a side lying position, with the A. Beta-adrenergic blockers
head of the bed lowered. The client should be positioned in a B. Bronchodilators
side-lying position with the head of the bed lowered to C. Inhaled steroids
prevent aspiration. A small amount of toothpaste should be D. Oral steroids
used and the mouth swabbed or suctioned to remove pooled Answer: (B) Bronchodilators. Bronchodilators are the first line
secretions. Lemon glycerin can be drying if used for extended of treatment for asthma because broncho-constriction is the
periods. Brushing the teeth with the client lying supine may cause of reduced airflow. Beta adrenergic blockers aren’t used
lead to aspiration. Hydrogen peroxide is caustic to tissues and to treat asthma and can cause bronchoconstriction. Inhaled
should not be used. oral steroids may be given to reduce the inflammation but
aren’t used for emergency relief.
35. A 77-year-old male client is admitted with a diagnosis of
dehydration and change in mental status. He’s being hydrated 39. Mr. Vasquez 56-year-old client with a 40-year history of
with L.V. fluids. When the nurse takes his vital signs, she notes smoking one to two packs of cigarettes per day has a chronic
he has a fever of 103°F (39.4°C) a cough producing yellow cough producing thick sputum, peripheral edema and
sputum and pleuritic chest pain. The nurse suspects this client cyanotic nail beds. Based on this information, he most likely
may have which of the following conditions? has which of the following conditions?
A. Adult respiratory distress syndrome (ARDS) A. Adult respiratory distress syndrome (ARDS)
B. Myocardial infarction (MI)
B. Asthma lymphocytes resulting in leukocytosis, and proliferation of
C. Chronic obstructive bronchitis these cells within the bone marrow, spleen and liver.
D. Emphysema
44. Robert, a 57-year-old client with acute arterial occlusion
Answer: (C) Chronic obstructive bronchitis. Because of this of the left leg undergoes an emergency embolectomy. Six
extensive smoking history and symptoms the client most hours later, the nurse isn’t able to obtain pulses in his left foot
likely has chronic obstructive bronchitis. Client with using Doppler ultrasound. The nurse immediately notifies the
ARDS have acute symptoms of hypoxia and typically need physician, and asks her to prepare the client for surgery. As
large amounts of oxygen. Clients with asthma and the nurse enters the client’s room to prepare him, he states
emphysema tend not to have chronic cough or peripheral
that he won’t have any more surgery. Which of the following
edema
is the best initial response by the nurse?
Situation: Francis, age 46 is admitted to the hospital with A. Explain the risks of not having the surgery
diagnosis of Chronic Lymphocytic Leukemia. B. Notifying the physician immediately
C. Notifying the nursing supervisor
40. The treatment for patients with leukemia is bone marrow
D. Recording the client’s refusal in the nurses’ notes
transplantation. Which statement about bone marrow
transplantation is not correct? Answer: (A) Explain the risks of not having the surgery. The
best initial response is to explain the risks of not having the
A. The patient is under local anesthesia during the
surgery. If the client understands the risks but still refuses the
procedure
nurse should notify the physician and the nurse supervisor
B. The aspirated bone marrow is mixed with heparin.
and then record the client’s refusal in the nurses’ notes.
C. The aspiration site is the posterior or anterior iliac crest.
D. The recipient receives cyclophosphamide (Cytoxan) for 4
45. During the endorsement, which of the following clients
consecutive days before the procedure.
should the on-duty nurse assess first?
Answer: (A) The patient is under local anesthesia during the A. The 58-year-old client who was admitted 2 days ago
procedure. Before the procedure, the patient is administered with heart failure, blood pressure of 126/76 mm Hg, and
with drugs that would help to prevent infection and rejection a respiratory rate of 22 breaths/minute.
of the transplanted cells such as antibiotics, cytotoxic, and B. The 89-year-old client with end-stage right-sided heart
corticosteroids. During the transplant, the patient is placed failure, blood pressure of 78/50 mm Hg, and a “do not
under general anesthesia. resuscitate” order
C. The 62-year-old client who was admitted 1 day ago
41. After several days of admission, Francis becomes with thrombophlebitis and is receiving L.V. heparin
disoriented and complains of frequent headaches. The nurse D. The 75-year-old client who was admitted 1 hour ago
in-charge first action would be: with new-onset atrial fibrillation and is receiving L.V.
dilitiazem (Cardizem)
A. Call the physician
B. Document the patient’s status in his charts.
Answer: (D) The 75-year-old client who was admitted 1 hour
C. Prepare oxygen treatment
ago with new-onset atrial fibrillation and is receiving L.V.
D. Raise the side rails
dilitiazem (Cardizem). The client with atrial fibrillation has the
greatest potential to become unstable and is on L.V.
Answer: (D) Raise the side rails. A patient who is disoriented
medication that requires close monitoring. After assessing
is at risk of falling out of bed. The initial action of the nurse
this client, the nurse should assess the client
should be raising the side rails to ensure patients safety
with thrombophlebitis who is receiving a heparin infusion,
and then the 58- year-old client admitted 2 days ago with
42. During routine care, Francis asks the nurse, “How can I be
heart failure (his signs and symptoms are resolving and don’t
anemic if this disease causes increased my white blood cell
require immediate attention). The lowest priority is the 89-
production?” The nurse in-charge best response would be year-old with end stage right-sided heart failure, who requires
that the increased number of white blood cells (WBC) is: time-consuming supportive measures.
A. Crowd red blood cells
46. Honey, a 23-year old client complains of substernal chest
B. Are not responsible for the anemia.
C. Uses nutrients from other cells pain and states that her heart feels like “it’s racing out of the
D. Have an abnormally short life span of cells. chest”. She reports no history of cardiac disorders. The nurse
attaches her to a cardiac monitor and notes sinus tachycardia
Answer: (A) Crowd red blood cells. The excessive production with a rate of 136beats/minutes. Breath sounds are clear and
of white blood cells crowd out red blood cells production the respiratory rate is 26 breaths/minutes. Which of the
which causes anemia to occur. following drugs should the nurse question the client about
using?
43. Diagnostic assessment of Francis would probably not
reveal: A. Barbiturates
B. Opioids
A. Predominance of lymhoblasts C. Cocaine
B. Leukocytosis D. Benzodiazepines
C. Abnormal blast cells in the bone marrow
D. Elevated thrombocyte counts Answer: (C) Cocaine. Because of the client’s age and
negative medical history, the nurse should question her
Answer: (B) Leukocytosis. Chronic Lymphocytic leukemia (CLL) about cocaine use. Cocaine increases myocardial oxygen
is characterized by increased production of leukocytes and consumption and can cause coronary artery spasm, leading
to tachycardia, ventricular fibrillation, myocardial ischemia,
and myocardial infarction. Barbiturate overdose may trigger A. “Keep the stoma uncovered.”
respiratory depression and slow pulse. Opioids can cause B. “Keep the stoma dry.”
marked respiratory depression, while benzodiazepines can C. “Have a family member perform stoma care initially
cause drowsiness and confusion until you get used to the procedure.”
D. “Keep the stoma moist.”
47. A 51-year-old female client tells the nurse in-charge that
she has found a painless lump in her right breast during her Answer: (D) “Keep the stoma moist.” The nurse should
monthly self-examination. Which assessment finding would instruct the client to keep the stoma moist, such as by
strongly suggest that this client’s lump is cancerous? applying a thin layer of petroleum jelly around the
edges, because a dry stoma may become irritated. The nurse
A. Eversion of the right nipple and mobile mass should recommend placing a stoma bib over the stoma to
B. Nonmobile mass with irregular edges filter and warm air before it enters the stoma. The client
C. Mobile mass that is soft and easily delineated should begin performing stoma care without assistance as
D. Nonpalpable right axillary lymph nodes soon as possible to gain independence in self-care activities.

Answer: (B) Nonmobile mass with irregular edges. Breast 51. A 37-year-old client with uterine cancer asks the nurse,
cancer tumors are fixed, hard, and poorly delineated with “Which is the most common type of cancer in women?” The
irregular edges. A mobile mass that is soft and easily nurse replies that it’s breast cancer. Which type of cancer
delineated is most often a fluid-filled benign cyst. Axillary causes the most deaths in women?
lymph nodes may or may not be palpable on initial detection
of a cancerous mass. Nipple retraction — not eversion — A. Breast cancer
may be a sign of cancer B. Lung cancer
C. Brain cancer
48. A 35-year-old client with vaginal cancer asks the nurse, D. Colon and rectal cancer
“What is the usual treatment for this type of cancer?” Which
treatment should the nurse name? Answer: (B) Lung cancer. Lung cancer is the most deadly type
of cancer in both women and men. Breast cancer ranks
A. Surgery second in women, followed (in descending order) by colon
B. Chemotherapy and rectal cancer, pancreatic cancer, ovarian cancer, uterine
C. Radiation cancer, lymphoma, leukemia, liver cancer, brain cancer,
D. Immunotherapy stomach cancer, and multiple myeloma.

Answer: (C) Radiation. The usual treatment for vaginal 52. Antonio with lung cancer develops Horner’s syndrome
cancer is external or intravaginal radiation therapy. Less when the tumor invades the ribs and affects the sympathetic
often, surgery is performed. Chemotherapy typically is nerve ganglia. When assessing for signs and symptoms of this
prescribed only if vaginal cancer is diagnosed in an early syndrome, the nurse should note:
stage, which is rare. Immunotherapy isn’t used to treat
vaginal cancer. A. miosis, partial eyelid ptosis, and anhidrosis on the
affected side of the face.
49. Cristina undergoes a biopsy of a suspicious lesion. The B. chest pain, dyspnea, cough, weight loss, and fever.
biopsy report classifies the lesion according to the TNM C. arm and shoulder pain and atrophy of arm and hand
staging system as follows: TIS, N0, M0. What does this muscles, both on the affected side.
classification mean? D. hoarseness and dysphagia.

A. No evidence of primary tumor, no abnormal regional Answer: (A) miosis, partial eyelid ptosis, and anhidrosis on the
lymph nodes, and no evidence of distant metastasis affected side of the face. Horner’s syndrome, which occurs
B. Carcinoma in situ, no abnormal regional lymph nodes, when a lung tumor invades the ribs and affects the
and no evidence of distant metastasis sympathetic nerve ganglia, is characterized by miosis, partial
C. Can’t assess tumor or regional lymph nodes and no eyelid ptosis, and anhidrosis on the affected side of the face.
evidence of metastasis Chest pain, dyspnea, cough, weight loss, and fever are
D. Carcinoma in situ, no demonstrable metastasis of the associated with pleural tumors. Arm and shoulder pain and
regional lymph nodes, and ascending degrees of distant atrophy of the arm and hand muscles on the affected side
metastasis suggest Pancoast’s tumor, a lung tumor involving the first
thoracic and eighth cervical nerves within the brachial plexus.
Answer: (B) Carcinoma in situ, no abnormal regional lymph Hoarseness in a client with lung cancer suggests that
nodes, and no evidence of distant metastasis. TIS, N0, M0 the tumor has extended to the recurrent laryngeal nerve;
denotes carcinoma in situ, no abnormal regional lymph dysphagia suggests that the lung tumor is compressing the
nodes, and no evidence of distant metastasis. No evidence esophagus.
of primary tumor, no abnormal regional lymph nodes, and no
evidence of distant metastasis is classified as T0, N0, M0. If 53. Vic asks the nurse what PSA is. The nurse should reply
the tumor and regional lymph nodes can’t be assessed and no that it stands for:
evidence of metastasis exists, the lesion is classified as TX, NX,
M0. A progressive increase in tumor size, no demonstrable A. prostate-specific antigen, which is used to screen for
metastasis of the regional lymph nodes, and prostate cancer.
ascending degrees of distant metastasis is classified as T1, T2, B. protein serum antigen, which is used to determine
T3, or T4; N0; and M1, M2, or M3. protein levels.
C. pneumococcal strep antigen, which is a bacteria that
50. Lydia undergoes a laryngectomy to treat laryngeal cancer. causes pneumonia.
When teaching the client how to care for the neck stoma, the D. Papanicolaou-specific antigen, which is used to screen
nurse should include which instruction? for cervical cancer.
Answer: (A) prostate-specific antigen, which is used to
screen for prostate cancer. PSA stands for prostate-specific 58. Nurse Mandy is preparing a client for magnetic resonance
antigen, which is used to screen for prostate cancer. The imaging (MRI) to confirm or rule out a spinal cord lesion.
other answers are incorrect During the MRI scan, which of the following would pose a
threat to the client?
54. What is the most important postoperative instruction that
nurse Kate must give a client who has just returned from the A. The client lies still.
operating room after receiving a subarachnoid block? B. The client asks questions.
C. The client hears thumping sounds.
A. “Avoid drinking liquids until the gag reflex returns.” D. The client wears a watch and wedding band.
B. “Avoid eating milk products for 24 hours.”
C. “Notify a nurse if you experience blood in your urine.” Answer: (D) The client wears a watch and wedding
D. “Remain supine for the time specified by the band. During an MRI, the client should wear no metal
physician.” objects, such as jewelry, because the strong magnetic field
can pull on them, causing injury to the client and (if they fly
Answer: (D) “Remain supine for the time specified by the off) to others. The client must lie still during the MRI but can
physician.” The nurse should instruct the client to remain talk to those performing the test by way of the microphone
supine for the time specified by the physician. Local inside the scanner tunnel. The client should hear
anesthetics used in a subarachnoid block don’t alter the gag thumping sounds, which are caused by the sound waves
reflex. No interactions between local anesthetics and food thumping on the magnetic field
occur. Local anesthetics don’t cause hematuria.
59. Nurse Cecile is teaching a female client about preventing
55. A male client suspected of having colorectal cancer will osteoporosis. Which of the following teaching points is
require which diagnostic study to confirm the diagnosis? correct?
A. Stool Hematest A. Obtaining an X-ray of the bones every 3 years is
B. Carcinoembryonic antigen (CEA) recommended to detect bone loss.
C. Sigmoidoscopy B. To avoid fractures, the client should avoid strenuous
D. Abdominal computed tomography (CT) scan exercise.
C. The recommended daily allowance of calcium may be
Answer: (C) Sigmoidoscopy. Used to visualize the lower GI found in a wide variety of foods.
tract, sigmoidoscopy and proctoscopy aid in the detection of D. Obtaining the recommended daily allowance of calcium
two-thirds of all colorectal cancers. Stool Hematest detects requires taking a calcium supplement
blood, which is a sign of colorectal cancer; however, the test
doesn’t confirm the diagnosis. CEA may be elevated Answer: (C) The recommended daily allowance of calcium
in colorectal cancer but isn’t considered a confirming test. may be found in a wide variety of foods. Premenopausal
An abdominal CT scan is used to stage the presence of women require 1,000 mg of calcium per day. Postmenopausal
colorectal cancer. women require 1,500 mg per day. It’s often, though
not always, possible to get the recommended daily
56. During a breast examination, which finding most strongly requirement in the foods we eat. Supplements are available
suggests that the Luz has breast cancer? but not always necessary. Osteoporosis doesn’t show up on
ordinary X-rays until 30% of the bone loss has occurred. Bone
A. Slight asymmetry of the breasts. densitometry can detect bone loss of 3% or less. This test is
B. A fixed nodular mass with dimpling of the overlying sometimes recommended routinely for women over 35
skin who are at risk. Strenuous exercise won’t cause fractures.
C. Bloody discharge from the nipple .
D. Multiple firm, round, freely movable masses that 60. Before Jacob undergoes arthroscopy, the nurse reviews
change with the menstrual cycle
the assessment findings for contraindications for this
procedure. Which finding is a contraindication?
Answer: (B) A fixed nodular mass with dimpling of the
overlying skin. A fixed nodular mass with dimpling of the A. Joint pain
overlying skin is common during late stages of breast cancer. B. Joint deformity
Many women have slightly asymmetrical breasts. Bloody C. Joint flexion of less than 50%
nipple discharge is a sign of intraductal papilloma, a benign D. Joint stiffness
condition. Multiple firm, round, freely movable masses that
change with the menstrual cycle indicate fibrocystic breasts, a Answer: (C) Joint flexion of less than 50%. Arthroscopy is
benign condition. contraindicated in clients with joint flexion of less than 50%
because of technical problems in inserting the instrument into
57. A female client with cancer is being evaluated for possible the joint to see it clearly. Other contraindications for this
metastasis. Which of the following is one of the most procedure include skin and wound infections. Joint pain may
common metastasis sites for cancer cells? be an indication, not a contraindication, for arthroscopy. Joint
deformity and joint stiffness aren’t contraindications for this
A. Liver procedure.
B. Colon
C. Reproductive tract 61. Mr. Rodriguez is admitted with severe pain in the knees.
D. White blood cells (WBCs)
Which form of arthritis is characterized by urate deposits and
joint pain, usually in the feet and legs, and occurs primarily in
Answer: (A) Liver. The liver is one of the five most common
men over age 30?
cancer metastasis sites. The others are the lymph nodes, lung,
bone, and brain. The colon, reproductive tract, and WBCs are A. Septic arthritis
occasional metastasis sites. B. Traumatic arthritis
C. Intermittent arthritis C. Osteoarthritis is a systemic disease, rheumatoid arthritis
D. Gouty arthritis is localized
D. Osteoarthritis has dislocations and subluxations,
Answer: (D) Gouty arthritis. Gouty arthritis, a metabolic rheumatoid arthritis doesn’t
disease, is characterized by urate deposits and pain in the
joints, especially those in the feet and legs. Urate deposits Answer: (B) Osteoarthritis is a localized disease rheumatoid
don’t occur in septic or traumatic arthritis. Septic arthritis arthritis is systemic. Osteoarthritis is a localized disease,
results from bacterial invasion of a joint and leads to rheumatoid arthritis is systemic. Osteoarthritis isn’t gender-
inflammation of the synovial lining. Traumatic arthritis results specific, but rheumatoid arthritis is. Clients have dislocations
from blunt trauma to a joint or ligament. Intermittent arthritis and subluxations in both disorders.
is a rare, benign condition marked by regular, recurrent joint
effusions, especially in the knees. 66. Mrs. Cruz uses a cane for assistance in walking. Which of
the following statements is true about a cane or other
62. A heparin infusion at 1,500 unit/hour is ordered for a 64- assistive devices?
year-old client with stroke in evolution. The infusion contains
25,000 units of heparin in 500 ml of saline solution. How A. A walker is a better choice than a cane.
many milliliters per hour should be given? B. The cane should be used on the affected side
C. The cane should be used on the unaffected side
A. 15 ml/hour D. A client with osteoarthritis should be encouraged to
B. 30 ml/hour ambulate without the cane
C. 45 ml/hour
D. 50 ml/hour Answer: (C) The cane should be used on the unaffected
side. A cane should be used on the unaffected side. A client
Answer: (B) 30 ml/hour. An infusion prepared with 25,000 with osteoarthritis should be encouraged to ambulate with a
units of heparin in 500 ml of saline solution yields 50 units of cane, walker, or other assistive device as needed; their use
heparin per milliliter of solution. The equation is set up as 50 takes weight and stress off joints.
units times X (the unknown quantity) equals 1,500 units/hour,
X equals 30 ml/hour. 67. A male client with type 1 diabetes is scheduled to receive
30 U of 70/30 insulin. There is no 70/30 insulin available. As a
63. A 76-year-old male client had a thromboembolic right substitution, the nurse may give the client:
stroke; his left arm is swollen. Which of the following
conditions may cause swelling after a stroke? A. 9 U regular insulin and 21 U neutral protamine
Hagedorn (NPH).
A. Elbow contracture secondary to spasticity B. 21 U regular insulin and 9 U NPH.
B. Loss of muscle contraction decreasing venous return C. 10 U regular insulin and 20 U NPH.
C. Deep vein thrombosis (DVT) due to immobility of the D. 20 U regular insulin and 10 U NPH.
ipsilateral side
D. Hypoalbuminemia due to protein escaping from an Answer: (A) 9 U regular insulin and 21 U neutral protamine
inflamed glomerulus Hagedorn (NPH). A 70/30 insulin preparation is 70% NPH
and 30% regular insulin. Therefore, a correct substitution
Answer: (B) Loss of muscle contraction decreasing venous requires mixing 21 U of NPH and 9 U of regular insulin. The
return. In clients with hemiplegia or hemiparesis loss of other choices are incorrect dosages for the prescribed
muscle contraction decreases venous return and may cause insulin
swelling of the affected extremity. Contractures, or bony
calcifications may occur with a stroke, but don’t appear with 68. Nurse Len should expect to administer which medication
swelling. DVT may develop in clients with a stroke but is to a client with gout?
more likely to occur in the lower extremities. A stroke
isn’t linked to protein loss A. aspirin
B. furosemide (Lasix)
64. Heberden’s nodes are a common sign of osteoarthritis. C. colchicines
Which of the following statement is correct about this D. calcium gluconate (Kalcinate)
deformity?
Answer: (C) colchicines. A disease characterized by joint
A. It appears only in men inflammation (especially in the great toe), gout is
B. It appears on the distal interphalangeal joint caused by urate crystal deposits in the joints.
C. It appears on the proximal interphalangeal joint The physician prescribes colchicine to reduce these
D. It appears on the dorsolateral aspect of the deposits and thus ease joint inflammation. Although
interphalangeal joint. aspirin is used to reduce joint inflammation and pain in
clients with osteoarthritis and rheumatoid arthritis, it
Answer: (B) It appears on the distal interphalangeal isn’t indicated for gout because it has no effect on urate
joint. Heberden’s nodes appear on the distal interphalageal crystal formation. Furosemide, a diuretic, doesn’t relieve
joint on both men and women. Bouchard’s node appears on gout. Calcium gluconate is used to reverse a negative
the dorsolateral aspect of the proximal interphalangeal joint. calcium balance and relieve muscle cramps, not to treat
gout.
65. Which of the following statements explains the main
difference between rheumatoid arthritis and osteoarthritis? 69. Mr. Domingo with a history of hypertension is diagnosed
with primary hyperaldosteronism. This diagnosis indicates
A. Osteoarthritis is gender-specific, rheumatoid arthritis that the client’s hypertension is caused by excessive hormone
isn’t secretion from which of the following glands?
B. Osteoarthritis is a localized disease rheumatoid
arthritis is systemic A. Adrenal cortex
B. Pancreas mellitus with diet and exercise. To determine the
C. Adrenal medulla effectiveness of the client’s efforts, the nurse should check:
D. Parathyroid
A. urine glucose level.
Answer: (A) Adrenal cortex. Excessive secretion of B. fasting blood glucose level.
aldosterone in the adrenal cortex is responsible for the C. serum fructosamine level.
client’s hypertension. This hormone acts on the renal tubule, D. glycosylated hemoglobin level.
where it promotes reabsorption of sodium and excretion
of potassium and hydrogen ions. The pancreas mainly Answer: (D) glycosylated hemoglobin level. Because some of
secretes hormones involved in fuel metabolism. The adrenal the glucose in the bloodstream attaches to some of the
medulla secretes the catecholamines — epinephrine and hemoglobin and stays attached during the 120-day life span of
norepinephrine. The parathyroids secrete parathyroid red blood cells, glycosylated hemoglobin levels provide
hormone. information about blood glucose levels during the previous 3
months. Fasting blood glucose and urine glucose levels only
70. For a diabetic male client with a foot ulcer, the doctor give information about glucose levels at the point in time
orders bed rest, a wetto- dry dressing change every shift, and when they were obtained. Serum fructosamine levels provide
blood glucose monitoring before meals and bedtime. Why are information about blood glucose control over the past 2 to
3 weeks.
wet-to-dry dressings used for this client?

A. They contain exudate and provide a moist wound 74. Nurse Trinity administered neutral protamine Hagedorn
environment. (NPH) insulin to a diabetic client at 7 a.m. At what time would
B. They protect the wound from mechanical trauma and the nurse expect the client to be most at risk for a
promote healing. hypoglycemic reaction?
C. They debride the wound and promote healing by
secondary intention. A. 10:00 am
D. They prevent the entrance of microorganisms and B. Noon
minimize wound discomfort. C. 4:00 pm
D. 10:00 pm
Answer: (C) They debride the wound and promote healing
by secondary intention. For this client, wet-to-dry dressings Answer: (C) 4:00 pm. NPH is an intermediate-acting insulin
are most appropriate because they clean the foot ulcer by that peaks 8 to 12 hours after administration. Because the
debriding exudate and necrotic tissue, thus promoting nurse administered NPH insulin at 7 a.m., the client is at
healing by secondary intention. Moist, transparent dressings greatest risk for hypoglycemia from 3 p.m. to 7 p.m.
contain exudate and provide a moist wound
environment. Hydrocolloid dressings prevent the entrance of 75. The adrenal cortex is responsible for producing which
microorganisms and minimize wound discomfort. Dry sterile substances?
dressings protect the wound from mechanical trauma and
promote healing. A. Glucocorticoids and androgens
B. Catecholamines and epinephrine
71. Nurse Zeny is caring for a client in acute addisonian crisis. C. Mineralocorticoids and catecholamines
D. Norepinephrine and epinephrine
Which laboratory data would the nurse expect to find?

A. Hyperkalemia Answer: (A) Glucocorticoids and androgens. The adrenal


B. Reduced blood urea nitrogen (BUN) glands have two divisions, the cortex and medulla. The
C. Hypernatremia cortex produces three types of hormones:
D. Hyperglycemia glucocorticoids, mineralocorticoids, and androgens. The
medulla produces catecholamines — epinephrine and
Answer: (A) Hyperkalemia. In adrenal insufficiency, the client norepinephrine
has hyperkalemia due to reduced aldosterone secretion. BUN
increases as the glomerular filtration rate is reduced. 76. On the third day after a partial thyroidectomy, Proserfina
Hyponatremia is caused by reduced aldosterone secretion. exhibits muscle twitching and hyperirritability of the nervous
Reduced cortisol secretion leads to impaired system. When questioned, the client reports numbness and
glyconeogenesis and a reduction of glycogen in the liver and tingling of the mouth and fingertips. Suspecting a
muscle, causing hypoglycemia. lifethreatening electrolyte disturbance, the nurse notifies the
surgeon immediately. Which electrolyte disturbance most
72. A client is admitted for treatment of the syndrome of commonly follows thyroid surgery?
inappropriate antidiuretic hormone (SIADH). Which nursing
intervention is appropriate? A. Hypocalcemia
B. Hyponatremia
A. Infusing I.V. fluids rapidly as ordered C. Hyperkalemia
B. Encouraging increased oral intake D. Hypermagnesemia
C. Restricting fluids
D. Administering glucose-containing I.V. fluids as ordered Answer: (A) Hypocalcemia. Hypocalcemia may follow thyroid
surgery if the parathyroid glands were removed accidentally.
Answer: (C) Restricting fluids. To reduce water retention in a Signs and symptoms of hypocalcemia may be delayed for up
client with the SIADH, the nurse should restrict fluids. to 7 days after surgery. Thyroid surgery doesn’t directly cause
Administering fluids by any route would further increase the serum sodium, potassium, or magnesium
client’s already heightened fluid load. abnormalities. Hyponatremia may occur if the client
inadvertently received too much fluid; however, this can
73. A female client tells nurse Nikki that she has been working happen to any surgical client receiving I.V. fluid therapy, not
hard for the last 3 months to control her type 2 diabetes just one recovering from thyroid surgery. Hyperkalemia
and hypermagnesemia usually are associated with reduced Answer: (C) “Avoid sharing such articles as toothbrushes and
renal excretion of potassium and magnesium, not thyroid razors.” The human immunodeficiency virus (HIV), which
surgery. causes AIDS, is most concentrated in the blood. For this
reason, the client shouldn’t share personal articles that may
77. Which laboratory test value is elevated in clients who be blood-contaminated, such as toothbrushes and razors,
smoke and can’t be used as a general indicator of cancer? with other family members. HIV isn’t transmitted by bathing
or by eating from plates, utensils, or serving dishes used by
A. Acid phosphatase level a person with AIDS.
B. Serum calcitonin level
C. Alkaline phosphatase level 81. Nurse Marie is caring for a 32-year-old client admitted
D. Carcinoembryonic antigen level with pernicious anemia. Which set of findings should the
nurse expect when assessing the
Answer: (D) Carcinoembryonic antigen level. In clients who
smoke, the level of carcinoembryonic antigen is elevated. client?
Therefore, it can’t be used as a general indicator of
cancer. However, it is helpful in monitoring cancer treatment A. Pallor, bradycardia, and reduced pulse pressure
because the level usually falls to normal within 1 month if B. Pallor, tachycardia, and a sore tongue
treatment is successful. An elevated acid phosphatase level C. Sore tongue, dyspnea, and weight gain
may indicate prostate cancer. An elevated alkaline D. Angina, double vision, and anorexia
phosphatase level may reflect bone metastasis. An elevated
serum calcitonin level usually signals thyroid cancer Answer: (B) Pallor, tachycardia, and a sore tongue. Pallor,
tachycardia, and a sore tongue are all characteristic findings in
78. Francis with anemia has been admitted to the medical- pernicious anemia. Other clinical manifestations
surgical unit. Which assessment findings are characteristic of include anorexia; weight loss; a smooth, beefy red tongue; a
iron-deficiency anemia? wide pulse pressure; palpitations; angina; weakness; fatigue;
and paresthesia of the hands and feet. Bradycardia, reduced
A. Nights sweats, weight loss, and diarrhea pulse pressure, weight gain, and double vision aren’t
B. Dyspnea, tachycardia, and pallor characteristic findings in pernicious anemia.
C. Nausea, vomiting, and anorexia
D. Itching, rash, and jaundice 82. After receiving a dose of penicillin, a client develops
dyspnea and hypotension. Nurse Celestina suspects the client
Answer: (B) Dyspnea, tachycardia, and pallor. Signs of iron- is experiencing anaphylactic shock. What should the nurse do
deficiency anemia include dyspnea, tachycardia, and pallor as first?
well as fatigue, listlessness, irritability, and headache.
Night sweats, weight loss, and diarrhea may signal acquired A. Page an anesthesiologist immediately and prepare to
immunodeficiency syndrome (AIDS). Nausea, vomiting, and intubate the client.
anorexia may be signs of hepatitis B. Itching, rash, and B. Administer epinephrine, as prescribed, and prepare to
jaundice may result from an allergic or hemolytic reaction. intubate the client if necessary.
C. Administer the antidote for penicillin, as prescribed, and
79. In teaching a female client who is HIV-positive about continue to monitor the client’s vital signs.
pregnancy, the nurse would know more teaching is necessary D. Insert an indwelling urinary catheter and begin to infuse
when the client says: I.V. fluids as ordered.

A. The baby can get the virus from my placenta.” Answer: (B) Administer epinephrine, as prescribed, and
B. “I’m planning on starting on birth control pills.” prepare to intubate the client if necessary. To reverse
C. “Not everyone who has the virus gives birth to a baby anaphylactic shock, the nurse first should administer
who has the virus.” epinephrine, a potent bronchodilator as prescribed.
D. “I’ll need to have a C-section if I become pregnant and The physician is likely to order additional medications, such
have a baby.” as antihistamines and corticosteroids; if these medications
don’t relieve the respiratory compromise associated with
Ans wer: (D) “I’ll need to have a C-section if I become anaphylaxis, the nurse should prepare to intubate the client.
pregnant and have a baby.” The human immunodeficiency No antidote for penicillin exists; however, the nurse should
virus (HIV) is transmitted from mother to child via the continue to monitor the client’s vital signs. A client who
transplacental route, but a Cesarean section delivery isn’t remains hypotensive may need fluid resuscitation and fluid
necessary when the mother is HIV-positive. The use of intake and output monitoring; however, administering
birth control will prevent the conception of a child who might epinephrine is the first priority.
have HIV. It’s true that a mother who’s HIV positive can give
birth to a baby who’s HIV negative. 83. Mr. Marquez with rheumatoid arthritis is about to begin
aspirin therapy to reduce inflammation. When teaching the
80. When preparing Judy with acquired immunodeficiency client about aspirin, the nurse discusses adverse reactions to
syndrome (AIDS) for discharge to the home, the nurse should prolonged aspirin therapy. These include:
be sure to include which instruction?
A. weight gain.
A. “Put on disposable gloves before bathing.” B. fine motor tremors.
B. “Sterilize all plates and utensils in boiling water.” C. respiratory acidosis.
C. “Avoid sharing such articles as toothbrushes and D. bilateral hearing loss.
razors.”
D. “Avoid eating foods from serving dishes shared by other Answer: (D) bilateral hearing loss. Prolonged use of aspirin
family members.” and other salicylates sometimes causes bilateral hearing loss
of 30 to 40 decibels. Usually, this adverse effect resolves
within 2 weeks after the therapy is discontinued.
Aspirin doesn’t lead to weight gain or fine motor tremors. immunodeficiency virus (HIV), the nurse expects the physician
Large or toxic salicylate doses may cause respiratory alkalosis, to order:
not respiratory acidosis.
A. E-rosette immunofluorescence.
84. A 23-year-old client is diagnosed with human B. quantification of T-lymphocytes.
immunodeficiency virus (HIV). After recovering from the C. enzyme-linked immunosorbent assay (ELISA).
initial shock of the diagnosis, the client expresses a desire to D. Western blot test with ELISA.
learn as much as possible about HIV and acquired
Answer: (D) Western blot test with ELISA. HIV infection is
immunodeficiency syndrome (AIDS). When teaching the client
detected by analyzing blood for antibodies to HIV, which form
about the immune system, the nurse states that adaptive
approximately 2 to 12 weeks after exposure to HIV
immunity is provided by which type of white blood cell? and denote infection. The Western blot test —
A. Neutrophil electrophoresis of antibody proteins — is more than 98%
B. Basophil accurate in detecting HIV antibodies when used in
C. Monocyte conjunction with the ELISA. It isn’t specific when used alone.
D. Lymphocyte Erosette immunofluorescence is used to detect viruses in
general; it doesn’t confirm HIV infection. Quantification of T-
Answer: (D) Lymphocyte. The lymphocyte provides adaptive lymphocytes is a useful monitoring test but isn’t diagnostic for
immunity — recognition of a foreign antigen and formation of HIV. The ELISA test detects HIV antibody particles but may
memory cells against the antigen. Adaptive immunity is yield inaccurate results; a positive ELISA result must be
mediated by B and T lymphocytes and can be acquired confirmed by the Western blot test.
actively or passively. The neutrophil is crucial to
phagocytosis. The basophil plays an important role in the 88. A complete blood count is commonly performed before a
release of inflammatory mediators. The monocyte functions Joe goes into surgery. What does this test seek to identify?
in phagocytosis and monokine production.
A. Potential hepatic dysfunction indicated by decreased
blood urea nitrogen (BUN) and creatinine levels
85. In an individual with Sjögren’s syndrome, nursing care
B. Low levels of urine constituents normally excreted in the
should focus on: urine
A. moisture replacement. C. Abnormally low hematocrit (HCT) and hemoglobin (Hb)
B. electrolyte balance. levels
C. nutritional supplementation. D. Electrolyte imbalance that could affect the blood’s
D. arrhythmia management. ability to coagulate properly

Answer: (A) moisture replacement. Sjogren’s syndrome is an Answer: (C) Abnormally low hematocrit (HCT) and
autoimmune disorder leading to progressive loss of hemoglobin (Hb) levels. Low preoperative HCT and Hb levels
lubrication of the skin, GI tract, ears, nose, and indicate the client may require a blood transfusion before
vagina. Moisture replacement is the mainstay of therapy. surgery. If the HCT and Hb levels decrease during surgery
Though malnutrition and electrolyte imbalance may occur as because of blood loss, the potential need for a transfusion
a result of Sjogren’s syndrome’s effect on the GI tract, it isn’t increases. Possible renal failure is indicated by elevated
the predominant problem. Arrhythmias aren’t a problem BUN or creatinine levels. Urine constituents aren’t found in
associated with Sjogren’s syndrome. the blood. Coagulation is determined by the presence of
appropriate clotting factors, not electrolytes.
86. During chemotherapy for lymphocytic leukemia, Mathew
89. While monitoring a client for the development of
develops abdominal pain, fever, and “horse barn” smelling
disseminated intravascular coagulation (DIC), the nurse
diarrhea. It would be most important for the nurse to advise
should take note of what assessment parameters?
the physician to order:
A. Platelet count, prothrombin time, and partial
A. enzyme-linked immunosuppressant assay (ELISA) test.
thromboplastin time
B. electrolyte panel and hemogram.
B. Platelet count, blood glucose levels, and white blood cell
C. stool for Clostridium difficile test.
(WBC) count
D. flat plate X-ray of the abdomen.
C. Thrombin time, calcium levels, and potassium levels
D. Fibrinogen level, WBC, and platelet count
Answer: (C) stool for Clostridium difficile
test. Immunosuppressed clients — for example, clients
Answer: (A) Platelet count, prothrombin time, and partial
receiving chemotherapy, — are at risk for infection with C.
thromboplastin time. The diagnosis of DIC is based on the
difficile, which causes “horse barn” smelling diarrhea.
results of laboratory studies of prothrombin time, platelet
Successful treatment begins with an accurate diagnosis,
count, thrombin time, partial thromboplastin time, and
which includes a stool test. The ELISA test is diagnostic for
fibrinogen level as well as client history and
human immunodeficiency virus (HIV) and isn’t indicated
other assessment factors. Blood glucose levels, WBC count,
in this case. An electrolyte panel and hemogram may be
calcium levels, and potassium levels aren’t used to confirm a
useful in the overall evaluation of a client but aren’t
diagnosis of DIC
diagnostic for specific causes of diarrhea. A flat plate of the
abdomen may provide useful information about
90. When taking a dietary history from a newly admitted
bowel function but isn’t indicated in the case of “horse barn”
smelling diarrhea female client, Nurse Len should remember that which of the
following foods is a common allergen?
87. A male client seeks medical evaluation for fatigue, night
A. Bread
sweats, and a 20-lb weight loss in 6 weeks. To confirm that B. Carrots
the client has been infected with the human C. Orange
D. Strawberries Answer: (D) Use comfort measures and pillows to position
the client.Using comfort measures and pillows to position
Answer: (D) Strawberries. Common food allergens include the client is a non-pharmacological methods of pain relief
berries, peanuts, Brazil nuts, cashews, shellfish, and eggs.
Bread, carrots, and oranges rarely cause allergic reactions. 95. Nurse Tina prepares a client for peritoneal dialysis. Which
of the following actions should the nurse take first?
91. Nurse John is caring for clients in the outpatient clinic.
Which of the following phone calls should the nurse return A. Assess for a bruit and a thrill.
first? B. Warm the dialysate solution.
C. Position the client on the left side.
A. A client with hepatitis A who states, “My arms and legs D. Insert a Foley catheter
are itching.”
B. A client with cast on the right leg who states, “I have a Answer: (B) Warm the dialysate solution. Cold dialysate
funny feeling in my right leg.” increases discomfort. The solution should be warmed to body
C. A client with osteomyelitis of the spine who states, “I temperature in warmer or heating pad; don’t use microwave
am so nauseous that I can’t eat.” oven.
D. A client with rheumatoid arthritis who states, “I am
having trouble sleeping.” 96. Nurse Jannah teaches an elderly client with right-sided
weakness how to use cane. Which of the following behaviors,
Answer: (B) A client with cast on the right leg who states, “I if demonstrated by the client to the nurse, indicates that the
have a funny feeling in my right leg.” It may indicate teaching was effective?
neurovascular compromise, requires immediate assessment
A. The client holds the cane with his right hand, moves the
92. Nurse Sarah is caring for clients on the surgical floor and can forward followed by the right leg, and then moves
has just received report from the previous shift. Which of the the left leg.
following clients should the nurse see first? B. The client holds the cane with his right hand, moves the
cane forward followed by his left leg, and then moves
A. A 35-year-old admitted three hours ago with a gunshot the right leg.
wound; 1.5 cm area of dark drainage noted on the C. The client holds the cane with his left hand, moves the
dressing. cane forward followed by the right leg, and then
B. A 43-year-old who had a mastectomy two days ago; 23 moves the left leg.
ml of serosanguinous fluid noted in the Jackson-Pratt D. The client holds the cane with his left hand, moves the
drain. cane forward followed by his left leg, and then moves
C. A 59-year-old with a collapsed lung due to an accident; the right leg.
no drainage noted in the previous eight hours.
D. A 62-year-old who had an abdominal-perineal resection Answer: (C) The client holds the cane with his left hand,
three days ago; client complaints of chills. moves the cane forward followed by the right leg, and then
moves the left leg. The cane acts as a support and aids in
Answer: (D) A 62-year-old who had an abdominal-perineal weight bearing for the weaker right leg.
resection three days ago; client complaints of chills. The client
is at risk for peritonitis; should be assessed for further 97. An elderly client is admitted to the nursing home setting.
symptoms and infection. The client is occasionally confused and her gait is often
unsteady. Which of the following actions, if taken by the
93. Nurse Eve is caring for a client who had a thyroidectomy nurse, is most appropriate?
12 hours ago for treatment of Grave’s disease. The nurse
would be most concerned if which of the following was A. Ask the woman’s family to provide personal items such
observed? as photos or mementos.
B. Select a room with a bed by the door so the woman can
A. Blood pressure 138/82, respirations 16, oral look down the hall.
temperature 99 degrees Fahrenheit. C. Suggest the woman eat her meals in the room with her
B. The client supports his head and neck when turning his roommate.
head to the right. D. Encourage the woman to ambulate in the halls twice a
C. The client spontaneously flexes his wrist when the day.
blood pressure is obtained.
D. The client is drowsy and complains of sore throat. Answer: (A) Ask the woman’s family to provide personal items
such as photos or mementos.Photos and mementos provide
Answer: (C) The client spontaneously flexes his wrist when visual stimulation to reduce sensory deprivation.
the blood pressure is obtained. Carpal spasms indicate
hypocalcemia. 98. Nurse Evangeline teaches an elderly client how to use a
standard aluminum walker. Which of the following behaviors,
94. Julius is admitted with complaints of severe pain in the if demonstrated by the client, indicates that the nurse’s
lower right quadrant of the abdomen. To assist with pain teaching was effective?
relief, the nurse should take which of the following actions?
A. The client slowly pushes the walker forward 12 inches,
A. Encourage the client to change positions frequently in then takes small steps forward while leaning on the
bed. walker.
B. Administer Demerol 50 mg IM q 4 hours and PRN. B. The client lifts the walker, moves it forward 10 inches,
C. Apply warmth to the abdomen with a heating pad. and then takes several small steps forward.
D. Use comfort measures and pillows to position the C. The client supports his weight on the walker while
client. advancing it forward, then takes small steps while
balancing on the walker.
D. The client slides the walker 18 inches forward, then 3. Matilda, with hyperthyroidism is to receive Lugol’s iodine
takes small steps while holding onto the walker for solution before a subtotal thyroidectomy is performed. The
balance. nurse is aware that this medication is given to:
A. Decrease the total basal metabolic rate.
Answer: (B) The client lifts the walker, moves it forward 10 B. Maintain the function of the parathyroid glands.
inches, and then takes several small steps forward. A walker C. Block the formation of thyroxine by the thyroid gland.
needs to be picked up, placed down on all legs. D. Decrease the size and vascularity of the thyroid gland.

99. Nurse Deric is supervising a group of elderly clients in a Answer: (D) Decrease the size and vascularity of the thyroid
residential home setting. The nurse knows that the elderly are gland. Lugol’s solution provides iodine, which aids in
at greater risk of developing sensory deprivation for what decreasing the vascularity of the thyroid gland, which limits
reason? the risk of hemorrhage when surgery is performed.

A. Increased sensitivity to the side effects of medications. 4. Ricardo, was diagnosed with type I diabetes. The nurse is
B. Decreased visual, auditory, and gustatory abilities. aware that acute hypoglycemia also can develop in the client
C. Isolation from their families and familiar surroundings. who is diagnosed with:
D. Decrease musculoskeletal function and mobility. A. Liver disease
B. HypertensionX
Answer: (C) Isolation from their families and familiar C. Type 2 diabetes
surroundings. Gradual loss of sight, hearing, and taste D. Hyperthyroidism
interferes with normal functioning.
Answer: (A) Liver Disease. The client with liver disease has a
100. A male client with emphysema becomes restless and decreased ability to metabolize carbohydrates because of a
confused. What step should nurse Jasmine take next? decreased ability to form glycogen (glycogenesis) and to form
glucose from glycogen.
A. Encourage the client to perform pursed lip breathing.
B. Check the client’s temperature. 5. Tracy is receiving combination chemotherapy for treatment
C. Assess the client’s potassium level. of metastatic carcinoma. Nurse Ruby should monitor the
D. Increase the client’s oxygen flow rate. client for the systemic side effect of:
A. Ascites
Answer: (A) Encourage the client to perform pursed lip B. Nystagmus
breathing. Purse lip breathing prevents the collapse of lung C. Leukopenia
unit and helps client control rate and depth of breathing. D.
E.
PNLE IV for Care of F. Polycythemia

Clients with Physiologic Answer: (C) Leukopenia. Leukopenia, a reduction in WBCs, is


a systemic effect of chemotherapy as a result of

and Psychosocial myelosuppression

6. Norma, with recent colostomy expresses concern about the


Alterations (Part 2) inability to control the passage of gas. Nurse Oliver should
suggest that the client plan to:
A. Eliminate foods high in cellulose.
B. Decrease fluid intake at meal times.
1. Randy has undergone kidney transplant, what assessment C. Avoid foods that in the past caused flatus.
would prompt Nurse Katrina to suspect organ rejection? D. Adhere to a bland diet prior to social events.
A. Sudden weight loss
B. Polyuria Answer: (C) Avoid foods that in the past caused flatus. Foods
C. Hypertension that bothered a person preoperatively will continue to do so
D. Shock after a colostomy.

Answer: (C) Hypertension. Hypertension, along with fever, 7. Nurse Ron begins to teach a male client how to perform
and tenderness over the grafted kidney, reflects acute colostomy irrigations. The nurse would evaluate that the
rejection. instructions were understood when the client states, “I
should:
2. The immediate objective of nursing care for an overweight, A. Lie on my left side while instilling the irrigating solution.”
mildly hypertensive male client with ureteral colic and B. Keep the irrigating container less than 18 inches above
hematuria is to decrease: the stoma.”
A. Pain C. Instill a minimum of 1200 ml of irrigating solution to
B. Weight stimulate evacuation of the bowel.”
C. Hematuria D. Insert the irrigating catheter deeper into the stoma if
D. Hypertension cramping occurs during the procedure.”

Answer: (A) Pain. Sharp, severe pain (renal colic) radiating Answer: (B) Keep the irrigating container less than 18 inches
toward the genitalia and thigh is caused by uretheral above the stoma.” This height permits the solution to flow
distention and smooth muscle spasm; relief form pain is the slowly with little force so that excessive peristalsis is not
priority. immediately precipitated
8. Patrick is in the oliguric phase of acute tubular necrosis and Answer: (D) may engage in contact sports. The client should
is experiencing fluid and electrolyte imbalances. The client is be advised by the nurse to avoid contact sports. This will
somewhat confused and complains of nausea and muscle prevent trauma to the area of the pacemaker generator.
weakness. As part of the prescribed therapy to correct this
electrolyte imbalance, the nurse would expect to: 13.The nurse is ware that the most relevant knowledge about
A. Administer Kayexalate oxygen administration to a male client with COPD is
B. Restrict foods high in protein A. Oxygen at 1-2L/min is given to maintain the hypoxic
C. Increase oral intake of cheese and milk. stimulus for breathing.
D. Administer large amounts of normal saline via I.V. B. Hypoxia stimulates the central chemoreceptors in the
medulla that makes the client breath.
Answer: (A) Administer Kayexalate. Kayexalate,a potassium C. Oxygen is administered best using a non-rebreathing
exchange resin, permits sodium to be exchanged for mask
potassium in the intestine, reducing the serum D. Blood gases are monitored using a pulse oximeter.
potassium level.
Answer: (A) Oxygen at 1-2L/min is given to maintain the
9. Mario has burn injury. After Forty48 hours, the physician hypoxic stimulus for breathing. COPD causes a chronic CO2
orders for Mario 2 liters of IV fluid to be administered q12 h. retention that renders the medulla insensitive to the CO2
The drop factor of the tubing is 10 gtt/ml. The nurse should stimulation for breathing. The hypoxic state of the client then
set the flow to provide: becomes the stimulus for breathing. Giving the client oxygen
A. X18 gtt/min in low concentrations will maintain the client’s hypoxic drive.
B. 28 gtt/min
C. 32 gtt/min 14.Tonny has undergoes a left thoracotomy and a partial
D. 36 gtt/min pneumonectomy. Chest tubes are inserted, and one-bottle
water-seal drainage is instituted in the operating room. In the
Answer:(B) 28 gtt/min. This is the correct flow rate; multiply postanesthesia care unit Tonny is placed in Fowler’s position
the amount to be infused (2000 ml) by the drop factor (10) on either his right side or on his back. The nurse is aware that
and divide the result by the amount of time in minutes (12 this position:
hours x 60 minutes A. Reduce incisional pain.
B. Facilitate ventilation of the left lung.
10.Terence suffered form burn injury. Using the rule of nines, C. Equalize pressure in the pleural space.
which has the largest percent of burns? D. Increase venous return
A. Face and neck
B. Right upper arm and penis Answer: (B) Facilitate ventilation of the left lung. Since only a
C. Right thigh and penis partial pneumonectomy is done, there is a need to promote
D. Upper trunk expansion of this remaining Left lung by positioning the
Answer: (D) Upper trunk. The percentage designated for client on the opposite unoperated side.
each burned part of the body using the rule of nines: Head
and neck 9%; Right upper extremity 9%; Left upper extremity 15.Kristine is scheduled for a bronchoscopy. When teaching
9%; Anterior trunk 18%; Posterior trunk 18%; Right Kristine what to expect afterward, the nurse’s highest priority
lower extremity 18%; Left lower extremity 18%; Perineum of information would be:
1%. A. Food and fluids will be withheld for at least 2 hours.
B. Warm saline gargles will be done q 2h.
11. Herbert, a 45 year old construction engineer is brought to C. Coughing and deep-breathing exercises will be done
the hospital unconscious after falling from a 2-story building. q2h.
When assessing the client, the nurse would be most D. Only ice chips and cold liquids will be allowed initially.
concerned if the assessment revealed:
A. Reactive pupils Answer: (A) Food and fluids will be withheld for at least 2
B. A depressed fontanel hours. Prior to bronchoscopy, the doctors sprays the back of
C. Bleeding from ears the throat with anesthetic to minimize the gag reflex and
D. An elevated temperature thus facilitate the insertion of the bronchoscope. Giving the
client food and drink after the procedure without checking
Answer: (C) Bleeding from ears. The nurse needs to perform on the return of the gag reflex can cause the client to
a thorough assessment that could indicate alterations in aspirate. The gag reflex usually returns after two hours
cerebral function, increased intracranial pressures, fractures
and bleeding. Bleeding from the ears occurs only with basal 16.Nurse Tristan is caring for a male client in acute renal
skull fractures that can easily contribute to increased failure. The nurse should expect hypertonic glucose, insulin
intracranial pressure and brain herniation infusions, and sodium bicarbonate to be used to treat:
A. hypernatremia.
12. Nurse Sherry is teaching male client regarding his B. hypokalemia.
permanent artificial pacemaker. Which information given by C. hyperkalemia.
the nurse shows her knowledge deficit about the artificial D. hypercalcemia.
cardiac pacemaker?
A. take the pulse rate once a day, in the morning upon Answer: (C) hyperkalemia. Hyperkalemia is a common
awakening complication of acute renal failure. It’s life-threatening if
B. May be allowed to use electrical appliances immediate action isn’t taken to reverse it.
C. Have regular follow up care The administration of glucose and regular insulin, with
D. May engage in contact sports sodium bicarbonate if necessary, can temporarily prevent
cardiac arrest by moving potassium into the cells and
temporarily reducing serum potassium
levels. Hypernatremia, hypokalemia, and hypercalcemia
don’t usually occur with acute renal failure and aren’t calcium. A uric acid analysis of 3.5 mg/dl falls within the
treated with glucose, insulin, or sodium bicarbonate normal range of 2.7 to 7.7 mg/dl; PSP excretion of 75%
also falls with the normal range of 60% to 75%.
17.Ms. X has just been diagnosed with condylomata
acuminata (genital warts). What information is appropriate to 20. Katrina has an abnormal result on a Papanicolaou test.
tell this client? After admitting that she read her chart while the nurse was
A. This condition puts her at a higher risk for cervical out of the room, Katrina asks what dysplasia means. Which
cancer; therefore, she should have a Papanicolaou definition should the nurse provide?
(Pap) smear annually. A. Presence of completely undifferentiated tumor cells that
B. The most common treatment is metronidazole (Flagyl), don’t resemble cells of the tissues of their origin.
which should eradicate the problem within 7 to 10 days. B. Increase in the number of normal cells in a normal
C. The potential for transmission to her sexual partner will arrangement in a tissue or an organ.
be eliminated if condoms are used every time they have C. Replacement of one type of fully differentiated cell by
sexual intercourse. another in tissues where the second type normally isn’t
D. The human papillomavirus (HPV), which causes found.
condylomata acuminata, can’t be transmitted during D. Alteration in the size, shape, and organization of
oral sex. differentiated cells.

Answer: (A) This condition puts her at a higher risk for Answer: (D) Alteration in the size, shape, and organization
cervical cancer; therefore, she should have a Papanicolaou of differentiated cells. Dysplasia refers to an alteration in the
(Pap) smear annually. Women with condylomata acuminata size, shape, and organization of differentiated cells. The
are at risk for cancer of the cervix and vulva. Yearly Pap presence of completely undifferentiated tumor cells that
smears are very important for early detection. Because don’t resemble cells of the tissues of their origin is called
condylomata acuminata is a virus, there is no permanent anaplasia. An increase in the number of normal cells in
cure. Because condylomata acuminata can occur on the a normal arrangement in a tissue or an organ is called
vulva, a condom won’t protect sexual partners. HPV can be hyperplasia. Replacement of one type of fully differentiated
transmitted to other parts of the body, such as the mouth, cell by another in tissues where the second type normally
oropharynx, and larynx. isn’t found is called metaplasia.

18.Maritess was recently diagnosed with a genitourinary 21. During a routine checkup, Nurse Mariane assesses a male
problem and is being examined in the emergency client with acquired immunodeficiency syndrome (AIDS) for
department. When palpating the her kidneys, the nurse signs and symptoms of cancer. What is the most common
should keep which anatomical fact in mind? AIDS-related cancer?
A. The left kidney usually is slightly higher than the right A. Squamous cell carcinoma
one. B. Multiple myeloma
B. The kidneys are situated just above the adrenal glands. C. Leukemia
C. The average kidney is approximately 5 cm (2″) long and D. Kaposi’s sarcoma
2 to 3 cm (¾” to 1-1/8″) wide.
D. The kidneys lie between the 10th and 12th thoracic Answer: (D) Kaposi’s sarcoma. Kaposi’s sarcoma is the most
vertebrae. common cancer associated with AIDS. Squamous cell
carcinoma, multiple myeloma, and leukemia may occur in
Answer: (A) The left kidney usually is slightly higher than the anyone and aren’t associated specifically with AIDS.
right one. The left kidney usually is slightly higher than the
right one. An adrenal gland lies atop each kidney. The average 22.Ricardo is scheduled for a prostatectomy, and the
kidney measures approximately 11 cm (4-3/8″) long, 5 to 5.8 anesthesiologist plans to use a spinal (subarachnoid) block
cm (2″ to 2¼”) wide, and 2.5 cm (1″) thick. The kidneys are during surgery. In the operating room, the nurse positions the
located retroperitoneally, in the posterior aspect of the client according to the anesthesiologist’s instructions. Why
abdomen, on either side of the vertebral column. They does the client require special positioning for this type of
lie between the 12th thoracic and 3rd lumbar vertebrae. anesthesia?
A. To prevent confusion
19.Jestoni with chronic renal failure (CRF) is admitted to the B. To prevent seizures
urology unit. The nurse is aware that the diagnostic test are C. To prevent cerebrospinal fluid (CSF) leakage
consistent with CRF if the result is: D. To prevent cardiac arrhythmias
A. Increased pH with decreased hydrogen ions.
B. Increased serum levels of potassium, magnesium, and Answer: (C) To prevent cerebrospinal fluid (CSF) leakage. The
calcium. client receiving a subarachnoid block requires
C. Blood urea nitrogen (BUN) 100 mg/dl and serum special positioning to prevent CSF leakage and headache and
creatinine 6.5 mg/ dl. to ensure proper anesthetic distribution. Proper positioning
D. Uric acid analysis 3.5 mg/dl and phenolsulfonphthalein doesn’t help prevent confusion, seizures, or cardiac
(PSP) excretion 75%. arrhythmias.

Answer: (C) Blood urea nitrogen (BUN) 100 mg/dl and serum 23.A male client had a nephrectomy 2 days ago and is now
creatinine 6.5 mg/dl. The normal BUN level ranges 8 to 23 complaining of abdominal pressure and nausea. The first
mg/dl; the normal serum creatinine level ranges from 0.7 to nursing action should be to:
1.5 mg/dl. The test results in option C are abnormally A. Auscultate bowel sounds.
elevated, reflecting CRF and the kidneys’ decreased ability to B. Palpate the abdomen.
remove nonprotein nitrogen waste from the blood. CRF C. Change the client’s position.
causes decreased pH and increased hydrogen ions — not vice D. Insert a rectal tube.
versa. CRF also increases serum levels of potassium,
magnesium, and phosphorous, and decreases serum levels of
Answer: (A) Auscultate bowel sounds. If abdominal distention
is accompanied by nausea, the nurse must first auscultate Answer: (B) Urine output of 20 ml/hour. A urine output of less
bowel sounds. If bowel sounds are absent, the nurse should than 40 ml/hour in a client with burns indicates a fluid volume
suspect gastric or small intestine dilation and these findings deficit. This client’s PaO2 value falls within the normal range
must be reported to the physician. Palpation should be (80 to 100 mm Hg). White pulmonary secretions also
avoided postoperatively with abdominal distention. If are normal. The client’s rectal temperature isn’t significantly
peristalsis is absent, changing positions and inserting a rectal elevated and probably results from the fluid volume deficit.
tube won’t relieve the client’s discomfort.
28. Mr. Mendoza who has suffered a cerebrovascular accident
24.Wilfredo with a recent history of rectal bleeding is being (CVA) is too weak to move on his own. To help the client avoid
prepared for a colonoscopy. How should the nurse Patricia pressure ulcers, Nurse Celia should:
position the client for this test initially? A. Turn him frequently.
A. Lying on the right side with legs straight B. Perform passive range-of-motion (ROM) exercises.
B. Lying on the left side with knees bent C. Reduce the client’s fluid intake.
C. Prone with the torso elevated D. Encourage the client to use a footboard.
D. Bent over with hands touching the floor
Answer: (A) Turn him frequently. The most important
Answer: (B) Lying on the left side with knees bent. For a intervention to prevent pressure ulcers is frequent position
colonoscopy, the nurse initially should position the client on changes, which relieve pressure on the skin and underlying
the left side with knees bent. Placing the client on the right tissues. If pressure isn’t relieved, capillaries become
side with legs straight, prone with the torso elevated, or bent occluded, reducing circulation and oxygenation of the tissues
over with hands touching the floor wouldn’t allow proper and resulting in cell death and ulcer formation. During passive
visualization of the large intestine. ROM exercises, the nurse moves each joint through its range
of movement, which improves joint mobility and circulation
25.A male client with inflammatory bowel disease undergoes to the affected area but doesn’t prevent pressure ulcers.
an ileostomy. On the first day after surgery, Nurse Oliver notes Adequate hydration is necessary to maintain healthy skin
that the client’s stoma appears dusky. How should the nurse and ensure tissue repair. A footboard prevents plantar flexion
interpret this finding? and footdrop by maintaining the foot in a dorsiflexed position.
A. Blood supply to the stoma has been interrupted.
B. This is a normal finding 1 day after surgery. 29.Nurse Maria plans to administer dexamethasone cream to
C. The ostomy bag should be adjusted. a female client who has dermatitis over the anterior chest.
D. An intestinal obstruction has occurred. How should the nurse apply this topical agent?
A. With a circular motion, to enhance absorption.
Answer: (A) Blood supply to the stoma has been B. With an upward motion, to increase blood supply to the
interrupted. An ileostomy stoma forms as the ileum is affected area
brought through the abdominal wall to the surface skin, C. In long, even, outward, and downward strokes in the
creating an artificial opening for waste elimination. The stoma direction of hair growth
should appear cherry red, indicating adequate arterial D. In long, even, outward, and upward strokes in the
perfusion. A dusky stoma suggests decreased perfusion, direction opposite hair growth
which may result from interruption of the stoma’s blood
supply and may lead to tissue damage or necrosis. A dusky Answer: (C) In long, even, outward, and downward strokes in
stoma isn’t a normal finding. Adjusting the ostomy bag the direction of hair growth. When applying a topical agent,
wouldn’t affect stoma color, which depends on blood supply the nurse should begin at the midline and use long, even,
to the area. An intestinal obstruction also wouldn’t change outward, and downward strokes in the direction of hair
stoma color. growth. This application pattern reduces the risk of
follicle irritation and skin inflammation.
26.Anthony suffers burns on the legs, which nursing
intervention helps prevent contractures? 30.Nurse Kate is aware that one of the following classes of
A. Applying knee splints medication protect the ischemic myocardium by blocking
B. Elevating the foot of the bed catecholamines and sympathetic nerve stimulation is:
C. Hyperextending the client’s palms A. Beta -adrenergic blockers
D. Performing shoulder range-of-motion exercises B. Calcium channel blocker
C. Narcotics
Answer: (A) Applying knee splints. Applying knee splints D. Nitrates
prevents leg contractures by holding the joints in a position of
function. Elevating the foot of the bed can’t Answer: (A) Beta -adrenergic blockers. Beta-adrenergic
prevent contractures because this action doesn’t hold the blockers work by blocking beta receptors in the myocardium,
joints in a position of function. Hyperextending a body part reducing the response to catecholamines and sympathetic
for an extended time is inappropriate because it can cause nerve stimulation. They protect the myocardium, helping
contractures. Performing shoulder range-of-motion exercises to reduce the risk of another infraction by decreasing
can prevent contractures in the shoulders, but not in the legs. myocardial oxygen demand. Calcium channel blockers reduce
the workload of the heart by decreasing the heart rate.
27.Nurse Ron is assessing a client admitted with second- and Narcotics reduce myocardial oxygen demand, promote
third-degree burns on the face, arms, and chest. Which vasodilation, and decrease anxiety. Nitrates reduce
finding indicates a potential problem? myocardial oxygen consumption bt decreasing left ventricular
A. Partial pressure of arterial oxygen (PaO2) value of 80 end diastolic pressure (preload) and systemic vascular
mm Hg. resistance (afterload).
B. Urine output of 20 ml/hour.
C. White pulmonary secretions.
D. Rectal temperature of 100.6° F (38° C).
31.A male client has jugular distention. On what position condition. Because the client requested that the nurse
should the nurse place the head of the bed to obtain the most update his wife on his condition, doing so doesn’t breach
accurate reading of jugular vein distention? confidentiality
A. High Fowler’s
B. Raised 10 degrees 35. A male client arriving in the emergency department is
C. Raised 30 degrees receiving cardiopulmonary resuscitation from paramedics
D. Supine position who are giving ventilations through an endotracheal (ET) tube
that they placed in the client’s home. During a pause in
Answer: (C) Raised 30 degrees. Jugular venous pressure is compressions, the cardiac monitor shows narrow QRS
measured with a centimeter ruler to obtain the vertical complexes and a heart rate of beats/minute with a palpable
distance between the sternal angle and the point of highest pulse. Which of the following actions
pulsation with the head of the bed inclined between 15 to should the nurse take first?
30 degrees. Increased pressure can’t be seen when the client A. Start an L.V. line and administer amiodarone
is supine or when the head of the bed is raised 10 degrees (Cardarone), 300 mg L.V. over 10 minutes.
because the point that marks the pressure level is above the B. Check endotracheal tube placement.
jaw (therefore, not visible). In high Fowler’s position, the C. Obtain an arterial blood gas (ABG) sample.
veins would be barely discernible above the clavicle. D. Administer atropine, 1 mg L.V.

32.The nurse is aware that one of the following classes of Answer: (B) Check endotracheal tube placement. ET tube
medications maximizes cardiac performance in clients with placement should be confirmed as soon as the client arrives
heart failure by increasing ventricular contractility? in the emergency department. Once the airways is
A. Beta-adrenergic blockers secured, oxygenation and ventilation should be confirmed
B. Calcium channel blocker using an end-tidal carbon dioxide monitor and pulse oximetry.
C. Diuretics Next, the nurse should make sure L.V. access is established. If
D. Inotropic agents the client experiences symptomatic bradycardia, atropine is
administered as ordered 0.5 to 1 mg every 3 to 5 minutes to a
Answer: (D) Inotropic agents. Inotropic agents are total of 3 mg. Then the nurse should try to find the cause
administered to increase the force of the heart’s contractions, of the client’s arrest by obtaining an ABG sample. Amiodarone
thereby increasing ventricular contractility and ultimately is indicated for ventricular tachycardia, ventricular fibrillation
increasing cardiac output. Beta-adrenergic blockers and and atrial flutter – not symptomatic bradycardia.
calcium channel blockers decrease the heart rate and
ultimately decreased the workload of the heart. Diuretics are 36. After cardiac surgery, a client’s blood pressure measures
administered to decrease the overall vascular volume, also 126/80 mm Hg. Nurse Katrina determines that mean arterial
decreasing the workload of the heart. pressure (MAP) is which of the following?
A. 46 mm Hg
33.A male client has a reduced serum high-density lipoprotein B. 80 mm Hg
(HDL) level and an elevated low-density lipoprotein (LDL) C. 95 mm Hg
level. Which of the following dietary modifications is not D. 90 mm Hg
appropriate for this client?
A. Fiber intake of 25 to 30 g daily Answer: (C) 95 mm Hg. Use the following formula to calculate
B. Less than 30% of calories form fat MAP
C. Cholesterol intake of less than 300 mg daily  MAP = systolic + 2 (diastolic) /3
D. Less than 10% of calories from saturated fat  MAP=[126 mm Hg + 2 (80 mm Hg) ]/3
 MAP=286 mm HG/ 3
Answer: (B) Less than 30% of calories form fat. A client with  MAP=95 mm Hg
low serum HDL and high serum LDL levels should get less than
30% of daily calories from fat. The other modifications 37. A female client arrives at the emergency department with
are appropriate for this client. chest and stomach pain and a report of black tarry stool for
several months. Which of the following order should the
34. A 37-year-old male client was admitted to the coronary nurse Oliver anticipate?
care unit (CCU) 2 days ago with an acute myocardial A. Cardiac monitor, oxygen, creatine kinase and lactate
infarction. Which of the following actions would breach the dehydrogenase levels
client confidentiality? B. Prothrombin time, partial thromboplastin time,
A. The CCU nurse gives a verbal report to the nurse on the fibrinogen and fibrin split product values.
telemetry unit before transferring the client to that unit C. Electrocardiogram, complete blood count, testing for
B. The CCU nurse notifies the on-call physician about a occult blood, comprehensive serum metabolic panel.
change in the client’s condition D. Electroencephalogram, alkaline phosphatase and
C. The emergency department nurse calls up the latest aspartate aminotransferase levels, basic serum
electrocardiogram results to check the client’s metabolic panel
progress.
D. At the client’s request, the CCU nurse updates the Answer: (C) Electrocardiogram, complete blood count, testing
client’s wife on his condition for occult blood, comprehensive serum metabolic panel. An
electrocardiogram evaluates the complaints of chest
Answer: (C) The emergency department nurse calls up the pain, laboratory tests determines anemia, and the stool test
latest electrocardiogram results to check the client’s for occult blood determines blood in the stool. Cardiac
progress. The emergency department nurse is no longer monitoring, oxygen, and creatine kinase and lactate
directly involved with the client’s care and thus has no legal dehydrogenase levels are appropriate for a cardiac primary
right to information about his present condition. Anyone problem. A basic metabolic panel and alkaline phosphatase
directly involved in his care (such as the telemetry nurse and and aspartate aminotransferase levels assess liver function.
the on-call physician) has the right to information about his Prothrombin time, partial thromboplastin time, fibrinogen
and fibrin split products are measured to verify bleeding C. Essential thrombocytopenia
dyscrasias, An electroencephalogram evaluates brain D. Von Willebrand’s disease
electrical activity.
Answer: (C) Essential thrombocytopenia. Essential
38. Macario had coronary artery bypass graft (CABG) surgery thrombocytopenia is linked to immunologic disorders, such as
3 days ago. Which of the following conditions is suspected by SLE and human immunodeficiency vitus. The disorder known
the nurse when a decrease in platelet count from 230,000 ul as von Willebrand’s disease is a type of hemophilia and isn’t
to 5,000 ul is noted? linked to SLE. Moderate to severe anemia is associated with
A. Pancytopenia SLE, not polycythermia. Dressler’s syndrome is pericarditis
B. Idiopathic thrombocytopemic purpura (ITP) that occurs after a myocardial infarction and isn’t linked to
C. Disseminated intravascular coagulation (DIC) SLE.
D. Heparin-associated thrombosis and thrombocytopenia
(HATT) 43. The nurse is aware that the following symptoms is most
commonly an early indication of stage 1 Hodgkin’s disease?
Answer: (D) Heparin-associated thrombosis and A. Pericarditis
thrombocytopenia (HATT). HATT may occur after CABG B. Night sweat
surgery due to heparin use during surgery. Although DIC and C. Splenomegaly
ITP cause platelet aggregation and bleeding, neither is D. Persistent hypothermia
common in a client after revascularization
surgery. Pancytopenia is a reduction in all blood cells. Answer: (B) Night sweat. In stage 1, symptoms include a single
enlarged lymph node (usually), unexplained fever, night
39. Which of the following drugs would be ordered by the sweats, malaise, and generalized pruritis. Although
physician to improve the platelet count in a male client with splenomegaly may be present in some clients, night sweats
idiopathic thrombocytopenic purpura (ITP)? are generally more prevalent. Pericarditis isn’t associated
A. Acetylsalicylic acid (ASA) with Hodgkin’s disease, nor is hypothermia. Moreover,
B. Corticosteroids splenomegaly and pericarditis aren’t symptoms. Persistent
C. Methotrezate hypothermia is associated with Hodgkin’s but isn’t an early
D. Vitamin K sign of the disease.

Answer: (B) Corticosteroids. Corticosteroid therapy can 44. Francis with leukemia has neutropenia. Which of the
decrease antibody production and phagocytosis of the following functions must frequently assessed?
antibody-coated platelets, retaining more A. Blood pressure
functioning platelets. Methotrexate can cause B. Bowel sounds
thrombocytopenia. Vitamin K is used to treat an excessive C. Heart sounds
anticoagulate state from warfarin overload, and D. Breath sounds
ASA decreases platelet aggregation.
Answer: (D) Breath sounds. Pneumonia, both viral and
40. A female client is scheduled to receive a heart valve fungal, is a common cause of death in clients with
replacement with a porcine valve. Which of the following neutropenia, so frequent assessment of respiratory rate
types of transplant is this? and breath sounds is required. Although assessing blood
A. Allogeneic pressure, bowel sounds, and heart sounds is important, it
B. Autologous won’t help detect pneumonia
C. Syngeneic
D. Xenogeneic 45. The nurse knows that neurologic complications of
multiple myeloma (MM) usually involve which of the
Answer: (D) Xenogeneic. An xenogeneic transplant is between following body system?
is between human and another species. A syngeneic A. Brain
transplant is between identical twins, allogeneic transplant is B. Muscle spasm
between two humans, and autologous is a transplant from C. Renal dysfunction
the same individual. D. Myocardial irritability

41. Marco falls off his bicycle and injuries his ankle. Which of Answer: (B) Muscle spasm. Back pain or paresthesia in the
the following actions shows the initial response to the injury lower extremities may indicate impending spinal cord
in the extrinsic pathway? compression from a spinal tumor. This should be recognized
A. Release of Calcium and treated promptly as progression of the tumor may result
B. Release of tissue thromboplastin in paraplegia. The other options, which reflect parts of the
C. Conversion of factors XII to factor XIIa nervous system, aren’t usually affected by MM.
D. Conversion of factor VIII to factor VIIIa
46. Nurse Patricia is aware that the average length of time
Answer: (B). Tissue thromboplastin is released when from human immunodeficiency virus (HIV) infection to the
damaged tissue comes in contact with clotting factors. development of acquired immunodeficiency syndrome
Calcium is released to assist the conversion of factors X to (AIDS)?
Xa. Conversion of factors XII to XIIa and VIII to VIII a are part A. Less than 5 years
of the intrinsic pathway B. 5 to 7 years
C. 10 years
42. Instructions for a client with systemic lupus D. More than 10 years
erythematosus (SLE) would include information about which
of the following blood dyscrasias? Answer: (C)10 years. Epidermiologic studies show the
A. Dressler’s syndrome average time from initial contact with HIV to the
B. Polycythemia development of AIDS is 10 years
patient should stop the medication and notify the health
47. An 18-year-old male client admitted with heat stroke care provider. The other manifestations are expected side
begins to show signs of disseminated intravascular effects of chemotherapy
coagulation (DIC). Which of the following laboratory findings
is most consistent with DIC? 51. Stacy’s mother states to the nurse that it is hard to see
A. Low platelet count Stacy with no hair. The best response for the nurse is:
B. Elevated fibrinogen levels A. “Stacy looks very nice wearing a hat”.
C. Low levels of fibrin degradation products B. “You should not worry about her hair, just be glad that
D. Reduced prothrombin time she is alive”.
C. “Yes it is upsetting. But try to cover up your feelings
Answer: (A) Low platelet count. In DIC, platelets and clotting when you are with her or else she may be upset”.
factors are consumed, resulting in microthrombi and D. “This is only temporary; Stacy will re-grow new hair in
excessive bleeding. As clots form, fibrinogen levels decrease 3-6 months, but may be different in texture”.
and the prothrombin time increases. Fibrin
degeneration products increase as fibrinolysis takes places. Answer: (D) “This is only temporary; Stacy will re-grow new
hair in 3-6 months, but may be different in texture”. This is the
48. Mario comes to the clinic complaining of fever, drenching appropriate response. The nurse should help the mother how
night sweats, and unexplained weight loss over the past 3 to cope with her own feelings regarding the child’s disease
months. Physical examination reveals a single enlarged so as not to affect the child negatively. When the hair grows
supraclavicular lymph node. Which of the following is the back, it is still of the same color and texture.
most probable diagnosis?
A. Influenza 52. Stacy has beginning stomatitis. To promote oral hygiene
B. Sickle cell anemia and comfort, the nurse in-charge should:
C. Leukemia A. Provide frequent mouthwash with normal saline.
D. Hodgkin’s disease B. Apply viscous Lidocaine to oral ulcers as needed.
C. Use lemon glycerine swabs every 2 hours.
Answer: (D) Hodgkin’s disease. Hodgkin’s disease typically D. Rinse mouth with Hydrogen Peroxide.
causes fever night sweats, weight loss, and lymph mode
enlargement. Influenza doesn’t last for months. Clients with Answer: (B) Apply viscous Lidocaine to oral ulcers as
sickle cell anemia manifest signs and symptoms of needed. Stomatitis can cause pain and this can be relieved by
chronic anemia with pallor of the mucous membrane, applying topical anesthetics such as lidocaine before mouth
fatigue, and decreased tolerance for exercise; they don’t care. When the patient is already comfortable, the nurse can
show fever, night sweats, weight loss or lymph node proceed with providing the patient with oral rinses of saline
enlargement. Leukemia doesn’t cause lymph solution mixed with equal part of water or hydrogen peroxide
node enlargement mixed water in 1:3 concentrations to promote oral hygiene.
49. A male client with a gunshot wound requires an Every 2-4 hours.
emergency blood transfusion. His blood type is AB negative.
Which blood type would be the safest for him to receive?
A. AB Rh-Xpositive
B. A Rh-positive 53. During the administration of chemotherapy agents, Nurse
C. A Rh-negative Oliver observed that the IV site is red and swollen, when the
D. O Rh-positive IV is touched Stacy shouts in pain. The first nursing action to
take is:
Answer: (C) A Rh-negative. Human blood can sometimes A. Notify the physician
contain an inherited D antigen. Persons with the D antigen B. Flush the IV line with saline solution
have Rh-positive blood type; those lacking the antigen have C. Immediately discontinue the infusion
Rh-negative blood. It’s important that a person with D. Apply an ice pack to the site, followed by warm
Rhnegative blood receives Rh-negative blood. If Rh-positive compress.
blood is administered to an Rh-negative person, the
recipient develops anti-Rh agglutinins, and sub sequent Answer: (C) Immediately discontinue the infusion. Edema or
transfusions with Rh-positive blood may cause serious swelling at the IV site is a sign that the needle has been
reactions with clumping and hemolysis of red blood cells dislodged and the IV solution is leaking into the tissues
Situation: Stacy is diagnosed with acute lymphoid leukemia causing the edema. The patient feels pain as the nerves are
(ALL) and beginning chemotherapy. irritated by pressure and the IV solution. The first action of
the nurse would be to discontinue the infusion right away to
50. Stacy is discharged from the hospital following her prevent further edema and other complication
chemotherapy treatments. Which statement of Stacy’s
mother indicated that she understands when she will contact 54. The term “blue bloater” refers to a male client which of
the physician? the following conditions?
A. “I should contact the physician if Stacy has difficulty in A. Adult respiratory distress syndrome (ARDS)
sleeping”. B. Asthma
B. “I will call my doctor if Stacy has persistent vomiting C. Chronic obstructive bronchitis
and diarrhea”. D. Emphysema
C. “My physician should be called if Stacy is irritable and
unhappy”. Answer: (C) Chronic obstructive bronchitis. Clients with
D. “Should Stacy have continued hair loss, I need to call the chronic obstructive bronchitis appear bloated; they have large
doctor”. barrel chest and peripheral edema, cyanotic nail beds, and
at times, circumoral cyanosis. Clients with ARDS are acutely
Answer: (B) “I will call my doctor if Stacy has persistent short of breath and frequently need intubation for
vomiting and diarrhea”. Persistent (more than 24 hours) mechanical ventilation and large amount of oxygen. Clients
vomiting, anorexia, and diarrhea are signs of toxicity and the
with asthma don’t exhibit characteristics of chronic disease, B. Decreased serum acid phosphate level
and clients with emphysema appear pink and cachectic. C. Elevated white blood cell count
D. Elevated serum aminotransferase
55. The term “pink puffer” refers to the female client with
which of the following conditions? Answer: (D) Elevated serum aminotransferase. Hepatic cell
A. Adult respiratory distress syndrome (ARDS) death causes release of liver enzymes
B. Asthma alanine aminotransferase (ALT), aspartate aminotransferase
C. Chronic obstructive bronchitis (AST) and lactate dehydrogenase (LDH) into the circulation.
D. Emphysema Liver cirrhosis is a chronic and irreversible disease of the liver
characterized by generalized inflammation and fibrosis of the
Answer: (D) Emphysema. Because of the large amount of liver tissues.
energy it takes to breathe, clients with emphysema are
usually cachectic. They’re pink and usually breathe through 60.The biopsy of Mr. Gonzales confirms the diagnosis of
pursed lips, hence the term “puffer.” Clients with ARDS are cirrhosis. Mr. Gonzales is at increased risk for excessive
usually acutely short of breath. Clients with asthma don’t bleeding primarily because of:
have any particular characteristics, and clients with chronic A. Impaired clotting mechanism
obstructive bronchitis are bloated and cyanotic in B. Varix formation
appearance. C. Inadequate nutrition
D. Trauma of invasive procedure
56. Jose is in danger of respiratory arrest following the
administration of a narcotic analgesic. An arterial blood gas Answer: (A) Impaired clotting mechanism. Cirrhosis of the
value is obtained. Nurse Oliver would expect the paco2 to be liver results in decreased Vitamin K absorption and formation
which of the following values? of clotting factors resulting in impaired clotting mechanism.
A. 15 mm Hg
B. 30 mm Hg 61. Mr. Gonzales develops hepatic encephalopathy. Which
C. 40 mm Hg clinical manifestation is most common with this condition?
D. 80 mm Hg A. Increased urine output
B. Altered level of consciousness
Answer: D 80 mm Hg. A client about to go into respiratory C. Decreased tendon reflex
arrest will have inefficient ventilation and will be retaining D. Hypotension
carbon dioxide. The value expected would be around 80 mm
Hg. All other values are lower than expected. Answer: (B) Altered level of consciousness. Changes in
behavior and level of consciousness are the first sins of
57. Timothy’s arterial blood gas (ABG) results are as follows; hepatic encephalopathy. Hepatic encephalopathy is caused by
pH 7.16; Paco2 80 mm Hg; Pao2 46 mm Hg; HCO3- 24mEq/L; liver failure and develops when the liver is unable to convert
Sao2 81%. This ABG result represents which of the following protein metabolic product ammonia to urea. This results in
conditions? accumulation of ammonia and other toxic in the blood that
A. Metabolic acidosis damages the cells.
B. Metabolic alkalosis
C. Respiratory acidosis 62. When Mr. Gonzales regained consciousness, the physician
D. Respirator y alkalosis orders 50 ml of Lactose p.o. every 2 hours. Mr. Gozales
develops diarrhea. The nurse best action would be:
Answer: (C) Respiratory acidosis. Because Paco2 is high at 80 A. “I’ll see if your physician is in the hospital”.
mm Hg and the metabolic measure, HCO3- is normal, the B. “Maybe your reacting to the drug; I will withhold the
client has respiratory acidosis. The pH is less than 7.35, next dose”.
academic, which eliminates metabolic and C. “I’ll lower the dosage as ordered so the drug causes
respiratory alkalosis as possibilities. If the HCO3- was below only 2 to 4 stools a day”.
22 mEq/L the client would have metabolic acidosis. D. “Frequently, bowel movements are needed to reduce
sodium level”.
58. Norma has started a new drug for hypertension. Thirty
minutes after she takes the drug, she develops chest tightness Answer: (C) “I’ll lower the dosage as ordered so the drug
and becomes short of breath and tachypneic. She has a causes only 2 to 4 stools a day”. Lactulose is given to a
decreased level of consciousness. These signs indicate which patients with hepatic encephalopathy to reduce absorption of
of the following conditions? ammonia in the intestines by binding with ammonia and
A. Asthma attack promoting more frequent bowel movements. If the patient
B. Pulmonary embolism experience diarrhea, it indicates over dosage and the nurse
C. Respiratory failure must reduce the amount of medication given to the patient.
D. Rheumatoid arthritis The stool will be mashy or soft. Lactulose is also very sweet
and may cause cramping and bloating.
Answer: (C) Respiratory failure. The client was reacting to the
drug with respiratory signs of impending anaphylaxis, which 63. Which of the following groups of symptoms indicates a
could lead to eventually respiratory failure. Although the signs ruptured abdominal aortic aneurysm?
are also related to an asthma attack or a A. Lower back pain, increased blood pressure, decreased
pulmonary embolism, consider the new drug first. re blood cell (RBC) count, increased white blood (WBC)
Rheumatoid arthritis doesn’t manifest these signs. count.
B. Severe lower back pain, decreased blood pressure,
Situation: Mr. Gonzales was admitted to the hospital with decreased RBC count, increased WBC count.
ascites and jaundice. To rule out cirrhosis of the liver: C. Severe lower back pain, decreased blood pressure,
decreased RBC count, decreased RBC count, decreased
59. Which laboratory test indicates liver cirrhosis? WBC count.
A. Decreased red blood cell count
D. Intermitted lower back pain, decreased blood pressure,
decreased RBC count, increased WBC count. Answer: (C) Kidneys’ excretion of sodium and water. The
kidneys respond to rise in blood pressure by excreting sodium
Answer: (B) Severe lower back pain, decreased blood and excess water. This response ultimately affects sysmolic
pressure, decreased RBC count, increased WBC count.Severe blood pressure by regulating blood volume. Sodium or water
lower back pain indicates an aneurysm rupture, secondary to retention would only further increase blood pressure. Sodium
pressure being applied within the abdominal cavity. and water travel together across the membrane in the
When ruptured occurs, the pain is constant because it can’t kidneys; one can’t travel without the other.
be alleviated until the aneurysm is repaired. Blood pressure
decreases due to the loss of blood. After the aneurysm 68. Nurse Rose is aware that the statement that best explains
ruptures, the vasculature is interrupted and blood volume is why furosemide (Lasix) is administered to treat hypertension
lost, so blood pressure wouldn’t increase. For the is:
same reason, the RBC count is decreased – not increased. The A. It dilates peripheral blood vessels.
WBC count increases as cell migrate to the site of injury. B. It decreases sympathetic cardioacceleration.
C. It inhibits the angiotensin-coverting enzymes
64. After undergoing a cardiac catheterization, Tracy has a D. It inhibits reabsorption of sodium and water in the
large puddle of blood under his buttocks. Which of the loop of Henle
following steps should the nurse take first?
A. Call for help. Answer: (D) It inhibits reabsorption of sodium and water in
B. Obtain vital signs the loop of Henle. Furosemide is a loop diuretic that inhibits
C. Ask the client to “lift up” sodium and water reabsorption in the loop Henle, thereby
D. Apply gloves and assess the groin site causing a decrease in blood pressure. Vasodilators cause
dilation of peripheral blood vessels, directly relaxing vascular
Answer: (D) Apply gloves and assess the groin site. Observing smooth muscle and decreasing blood pressure. Adrenergic
standard precautions is the first priority when dealing with blockers decrease sympathetic cardioacceleration
any blood fluid. Assessment of the groin site is the and decrease blood pressure. Angiotensin-converting enzyme
second priority. This establishes where the blood is coming inhibitors decrease blood pressure due to their action on
from and determineshow much blood has been lost. The goal angiotensin.
in this situation is to stop the bleeding. The nurse would call .
for help if it were warranted after the assessment of the 69. Nurse Nikki knows that laboratory results supports the
situation. After determining the extent of the bleeding, vital diagnosis of systemic lupus erythematosus (SLE) is:
signs assessment is important. The nurse should never move A. Elavated serum complement level
the client, in case a clot has formed. Moving can disturb the B. Thrombocytosis, elevated sedimentation rate
clot and cause rebleeding. C. Pancytopenia, elevated antinuclear antibody (ANA)
titer
65. Which of the following treatment is a suitable surgical D. Leukocysis, elevated blood urea nitrogen (BUN) and
intervention for a client with unstable angina? creatinine levels
A. Cardiac catheterization Answer: (C) Pancytopenia, elevated antinuclear antibody
B. Echocardiogram (ANA) titer. Laboratory findings for clients with SLE usually
C. Nitroglycerin show pancytopenia, elevated ANA titer, and decreased
D. Percutaneous transluminal coronary angioplasty serum complement levels. Clients may have elevated BUN
(PTCA) and creatinine levels from nephritis, but the increase does
not indicate SLE
Answer: (D) Percutaneous transluminal coronary angioplasty
(PTCA). PTCA can alleviate the blockage and restore blood 70. Arnold, a 19-year-old client with a mild concussion is
flow and oxygenation. An echocardiogram is a noninvasive discharged from the emergency department. Before
diagnosis test. Nitroglycerin is an oral sublingual medication. discharge, he complains of a headache. When offered
Cardiac catheterization is a diagnostic tool – not a treatment acetaminophen, his mother tells the nurse the headache is
severe and she would like her son to have something stronger.
66. The nurse is aware that the following terms used to Which of the following responses by the nurse is appropriate?
describe reduced cardiac output and perfusion impairment A. “Your son had a mild concussion, acetaminophen is
due to ineffective pumping of the heart is: strong enough.”
A. Anaphylactic shock B. “Aspirin is avoided because of the danger of Reye’s
B. Cardiogenic shock syndrome in children or young adults.”
C. Distributive shock C. “Narcotics are avoided after a head injury because they
D. Myocardial infarction (MI) may hide a worsening condition.”
D. Stronger medications may lead to vomiting, which
Answer: (B) Cardiogenic shock. Cardiogenic shock is shock increases the intracarnial pressure (ICP).”
related to ineffective pumping of the heart. Anaphylactic
shock results from an allergic reaction. Distributive shock Answer: (C) Narcotics are avoided after a head injury because
results from changes in the intravascular volume distribution they may hide a worsening condition. Narcotics may mask
and is usually associated with increased cardiac output. MI changes in the level of consciousness that indicate increased
isn’t a shock state, though a severe MI can lead to shock. ICP and shouldn’t acetaminophen is strong enough ignores
the mother’s question and therefore isn’t appropriate. Aspirin
67. A client with hypertension ask the nurse which factors can is contraindicated in conditions that may have bleeding, such
cause blood pressure to drop to normal levels? as trauma, and for children or young adults with viral illnesses
A. Kidneys’ excretion to sodium only. due to the danger of Reye’s syndrome. Stronger medications
B. Kidneys’ retention of sodium and water may not necessarily lead to vomiting but will sedate the
C. Kidneys’ excretion of sodium and water client, thereby masking changes in his level of consciousness.
D. Kidneys’ retention of sodium and excretion of water
71. When evaluating an arterial blood gas from a male client C. Myxedema coma
with a subdural hematoma, the nurse notes the Paco2 is 30 D. Tibial myxedema
mm Hg. Which of the following responses best describes the
result? Answer: (C) Myxedema coma. Myxedema coma, severe
A. Appropriate; lowering carbon dioxide (CO2) reduces hypothyroidism, is a life-threatening condition that may
intracranial pressure (ICP) develop if thyroid replacement medication isn’t
B. Emergent; the client is poorly oxygenated taken. Exophthalmos, protrusion of the eyeballs, is seen with
C. Normal hyperthyroidism. Thyroid storm is life-threatening but is
D. Significant; the client has alveolar hypoventilation caused by severe hyperthyroidism. Tibial myxedema,
peripheral mucinous edema involving the lower leg,
Answer: (A) Appropriate; lowering carbon dioxide (CO2) is associated with hypothyroidism but isn’t life-threatening
reduces intracranial pressure (ICP). A normal Paco2 value is 35
to 45 mm Hg CO2 has vasodilating properties; therefore, 76. Nurse Sugar is assessing a client with Cushing’s syndrome.
lowering Paco2 through hyperventilation will lower ICP Which observation should the nurse report to the physician
caused by dilated cerebral vessels. Oxygenation is evaluated immediately?
through Pao2 and oxygen saturation. Alveolar A. Pitting edema of the legs
hypoventilation would be reflected in an increased Paco2. B. An irregular apical pulse
C. Dry mucous membranes
72. When prioritizing care, which of the following clients D. Frequent urination
should the nurse Olivia assess first?
A. A 17-year-old clients 24-hours postappendectomy Answer: (B) An irregular apical pulse. Because Cushing’s
B. A 33-year-old client with a recent diagnosis of Guillain- syndrome causes aldosterone overproduction, which
Barre syndrome increases urinary potassium loss, the disorder may lead to
C. A 50-year-old client 3 days postmyocardial infarction hypokalemia. Therefore, the nurse should immediately report
D. A 50-year-old client with diverticulitis signs and symptoms of hypokalemia, such as an irregular
apical pulse, to the physician. Edema is an expected finding
Answer: (B) A 33-year-old client with a recent diagnosis of because aldosterone overproduction causes sodium and fluid
Guillain-Barre syndrome . Guillain-Barre syndrome is retention. Dry mucous membranes and frequent urination
characterized by ascending paralysis and potential respiratory signal dehydration, which isn’t associated with Cushing’s
failure. The order of client assessment should follow client syndrome.
priorities, with disorder of airways, breathing, and
then circulation. There’s no information to suggest the 77. Cyrill with severe head trauma sustained in a car accident
postmyocardial infarction client has an arrhythmia or other is admitted to the intensive care unit. Thirty-six hours later,
complication. There’s no evidence to suggest hemorrhage or the client’s urine output suddenly rises above 200 ml/hour,
perforation for the remaining clients as a priority of care. leading the nurse to suspect diabetes insipidus. Which
laboratory findings support the nurse’s suspicion of diabetes
73. JP has been diagnosed with gout and wants to know why insipidus?
colchicine is used in the treatment of gout. Which of the A. Above-normal urine and serum osmolality levels
following actions of colchicines explains why it’s effective for B. Below-normal urine and serum osmolality levels
gout? C. Above-normal urine osmolality level, below-normal
A. Replaces estrogen serum osmolality level
B. Decreases infection D. Below-normal urine osmolality level, above-normal
C. Decreases inflammation serum osmolality level
D. Decreases bone demineralization
Answer: (D) Below-normal urine osmolality level, above-
Answer: (C) Decreases inflammation. Then action of normal serum osmolality level. In diabetes insipidus,
colchicines is to decrease inflammation by reducing the excessive polyuria causes dilute urine, resulting in a below-
migration of leukocytes to synovial fluid. Colchicine normal urine osmolality level. At the same time, polyuria
doesn’t replace estrogen, decrease infection, or decrease depletes the body of water, causing dehydration that leads to
bone demineralization. an above-normal serum osmolality level. For the same
reasons, diabetes insipidus doesn’t cause above-normal urine
74. Norma asks for information about osteoarthritis. Which of osmolality or below-normal serum osmolality levels.
the following statements about osteoarthritis is correct?
A. Osteoarthritis is rarely debilitating 78. Jomari is diagnosed with hyperosmolar hyperglycemic
B. Osteoarthritis is a rare form of arthritis nonketotic syndrome (HHNS) is stabilized and prepared for
C. Osteoarthritis is the most common form of arthritis discharge. When preparing the client for discharge and home
D. Osteoarthritis afflicts people over 60 management, which of the following statements indicates
that the client understands her condition and how to control
Answer: (C) Osteoarthritis is the most common form of it?
arthritis. Osteoarthritis is the most common form of arthritis A. “I can avoid getting sick by not becoming dehydrated
and can be extremely debilitating. It can afflict people of any and by paying attention to my need to urinate, drink,
age, although most are elderly or eat more than usual.”
B. “If I experience trembling, weakness, and headache, I
75. Ruby is receiving thyroid replacement therapy develops should drink a glass of soda that contains sugar.”
the flu and forgets to take her thyroid replacement medicine. C. “I will have to monitor my blood glucose level closely
The nurse understands that skipping this medication will put and notify the physician if it’s constantly elevated.”
the client at risk for developing which of the following D. “If I begin to feel especially hungry and thirsty, I’ll eat a
lifethreatening complications? snack high in carbohydrates.”
A. Exophthalmos
B. Thyroid storm
Answer: (A) “I can avoid getting sick by not becoming 82. A male client is scheduled for a transsphenoidal
dehydrated and by paying attention to my need to urinate, hypophysectomy to remove a pituitary tumor. Preoperatively,
drink, or eat more than usual.” Inadequate fluid intake during the nurse should assess for potential complications by doing
hyperglycemic episodes often leads to HHNS. By recognizing which of the following?
the signs of hyperglycemia (polyuria, polydipsia, and A. Testing for ketones in the urine
polyphagia) and increasing fluid intake, the client may prevent B. Testing urine specific gravity
HHNS. Drinking a glass of nondiet soda would be appropriate C. Checking temperature every 4 hours
for hypoglycemia. A client whose diabetes is controlled with D. Performing capillary glucose testing every 4 hours
oral antidiabetic agents usually doesn’t need to monitor
blood glucose levels. A highcarbohydrate diet would Answer: (D) Performing capillary glucose testing every 4
exacerbate the client’s condition, particularly if fluid intake is hours. The nurse should perform capillary glucose testing
low. every 4 hours because excess cortisol may cause insulin
resistance, placing the client at risk for hyperglycemia. Urine
79. A 66-year-old client has been complaining of sleeping ketone testing isn’t indicated because the client does secrete
more, increased urination, anorexia, weakness, irritability, insulin and, therefore, isn’t at risk for ketosis. Urine specific
depression, and bone pain that interferes with her going gravity isn’t indicated because although fluid balance can be
outdoors. Based on these assessment findings, the nurse compromised, it usually isn’t dangerously
would suspect which of the following disorders? imbalanced. Temperature regulation may be affected by
A. Diabetes mellitus excess cortisol and isn’t an accurate indicator of infection.
B. Diabetes insipidus
C. Hypoparathyroidism 83. Capillary glucose monitoring is being performed every 4
D. Hyperparathyroidism hours for a client diagnosed with diabetic ketoacidosis. Insulin
is administered using a scale of regular insulin according to
Answer: (D) Hyperparathyroidism. Hyperparathyroidism is glucose results. At 2 p.m., the client has a capillary glucose
most common in older women and is characterized by bone level of 250 mg/dl for which he receives 8 U of regular insulin.
pain and weakness from excess parathyroid hormone (PTH). Nurse Mariner should expect the dose’s:
Clients also exhibit hypercaliuria-causing polyuria. A. onset to be at 2 p.m. and its peak to be at 3 p.m.
While clients with diabetes mellitus and diabetes insipidus B. onset to be at 2:15 p.m. and its peak to be at 3 p.m.
also have polyuria, they don’t have bone pain and increased C. onset to be at 2:30 p.m. and its peak to be at 4 p.m.
sleeping. Hypoparathyroidism is characterized by urinary D. onset to be at 4 p.m. and its peak to be at 6 p.m.
frequency rather than polyuria.
Answer: (C) onset to be at 2:30 p.m. and its peak to be at 4
80. Nurse Lourdes is teaching a client recovering from p.m.. Regular insulin, which is a short-acting insulin, has an
addisonian crisis about the need to take fludrocortisone onset of 15 to 30 minutes and a peak of 2 to 4 hours. Because
acetate and hydrocortisone at home. Which statement by the the nurse gave the insulin at 2 p.m., the expected onset
client indicates an understanding of the instructions? would be from 2:15 p.m. to 2:30 p.m. and the peak from 4
A. “I’ll take my hydrocortisone in the late afternoon, before p.m. to 6 p.m.
dinner.”
B. “I’ll take all of my hydrocortisone in the morning, right 84. The physician orders laboratory tests to confirm
after I wake up.” hyperthyroidism in a female client with classic signs and
C. “I’ll take two-thirds of the dose when I wake up and symptoms of this disorder. Which test result would confirm
one-third in the late afternoon.” the diagnosis?
D. “I’ll take the entire dose at bedtime.” A. No increase in the thyroid-stimulating hormone (TSH)
level after 30 minutes during the TSH stimulation test
Answer: (C) “I’ll take two-thirds of the dose when I wake up B. A decreased TSH level
and one-third in the late afternoon.” Hydrocortisone, a C. An increase in the TSH level after 30 minutes during the
glucocorticoid, should be administered according to a TSH stimulation test
schedule that closely reflects the body’s own secretion of this D. Below-normal levels of serum triiodothyronine (T3) and
hormone; therefore, two-thirds of the dose of hydrocortisone serum thyroxine (T4) as detected by radioimmunoassay
should be taken in the morning and one-third in the late
afternoon. This dosage schedule reduces adverse effects. Answer: (A) No increase in the thyroid-stimulating hormone
(TSH) level after 30 minutes during the TSH stimulation
81. Which of the following laboratory test results would test. In the TSH test, failure of the TSH level to rise after
suggest to the nurse Len that a client has a corticotropin- 30 minutes confirms hyperthyroidism. A decreased TSH level
secreting pituitary adenoma? indicates a pituitary deficiency of this hormone. Below-
A. High corticotropin and low cortisol levels normal levels of T3 and T4, as detected by radioimmunoassay,
B. Low corticotropin and high cortisol levels signal hypothyroidism. A below-normal T4 level also occurs in
C. High corticotropin and high cortisol levels malnutrition and liver disease and may result
D. Low corticotropin and low cortisol levels from administration of phenytoin and certain other drugs.

Answer: (C) High corticotropin and high cortisol levels. A 85. Rico with diabetes mellitus must learn how to self-
corticotropin-secreting pituitary tumor would cause administer insulin. The physician has prescribed 10 U of U-100
high corticotropin and high cortisol levels. A high regular insulin and 35 U of U-100 isophane insulin suspension
corticotropin level with a low cortisol level and a low (NPH) to be taken before breakfast. When teaching the client
corticotropin level with a low cortisol level would how to select and rotate insulin injection sites, the nurse
be associated with hypocortisolism. Low corticotropin and should provide which instruction?
high cortisol levels would be seen if there was a primary A. “Inject insulin into healthy tissue with large blood
defect in the adrenal glands vessels and nerves.”
B. “Rotate injection sites within the same anatomic
region, not among different regions.”
C. “Administer insulin into areas of scar tissue or Colles’ fracture doesn’t refer to a fracture of the olecranon,
hypotrophy whenever possible.” humerus, or carpal scaphoid
D. “Administer insulin into sites above muscles that you
plan to exercise heavily later that day.” 89. Cleo is diagnosed with osteoporosis. Which electrolytes
are involved in the development of this disorder?
Answer: (B) “Rotate injection sites within the same anatomic A. Calcium and sodium
region, not among different regions.” The nurse should B. Calcium and phosphorous
instruct the client to rotate injection sites within the same C. Phosphorous and potassium
anatomic region. Rotating sites among different regions may D. Potassium and sodium
cause excessive day-to-day variations in the blood glucose
level; also, insulin absorption differs from one region to the Answer: (B) Calcium and phosphorous. In osteoporosis, bones
next. Insulin should be injected only into healthy tissue lose calcium and phosphate salts, becoming porous, brittle,
lacking large blood vessels, nerves, or scar tissue or other and abnormally vulnerable to fracture. Sodium and potassium
deviations. Injecting insulin into areas of hypertrophy may aren’t involved in the development of osteoporosis.
delay absorption. The client shouldn’t inject insulin into areas
of lipodystrophy (such as hypertrophy or atrophy); to prevent 90. Johnny a firefighter was involved in extinguishing a house
lipodystrophy, the client should rotate injection sites fire and is being treated to smoke inhalation. He develops
systematically. Exercise speeds drug absorption, so the client severe hypoxia 48 hours after the incident, requiring
shouldn’t inject insulin into sites above muscles that will be intubation and mechanical ventilation. He most likely has
exercised heavily. developed which of the following conditions?
A. Adult respiratory distress syndrome (ARDS)
86. Nurse Sarah expects to note an elevated serum glucose B. Atelectasis
level in a client with hyperosmolar hyperglycemic nonketotic C. Bronchitis
syndrome (HHNS). Which other laboratory finding should the D. Pneumonia
nurse anticipate?
A. Elevated serum acetone level Answer: (A) Adult respiratory distress syndrome
B. Serum ketone bodies (ARDS). Severe hypoxia after smoke inhalation is typically
C. Serum alkalosis related to ARDS. The other conditions listed aren’t typically
D. Below-normal serum potassium level associated with smoke inhalation and severe hypoxia.

Answer: (D) Below-normal serum potassium level. A client 91. A 67-year-old client develops acute shortness of breath
with HHNS has an overall body deficit of potassium resulting and progressive hypoxia requiring right femur. The hypoxia
from diuresis, which occurs secondary to the was probably caused by which of the following conditions?
hyperosmolar, hyperglycemic state caused by the relative A. Asthma attack
insulin deficiency. An elevated serum acetone level and serum B. Atelectasis
ketone bodies are characteristic of diabetic ketoacidosis. C. Bronchitis
Metabolic acidosis, not serum alkalosis, may occur in HHNS. D. Fat embolism

87. For a client with Graves’ disease, which nursing Answer: (D) Fat embolism. Long bone fractures are correlated
intervention promotes comfort? with fat emboli, whichcause shortness of breath and hypoxia.
A. Restricting intake of oral fluids It’s unlikely the client has developed asthma or bronchitis
B. Placing extra blankets on the client’s bed without a previous history. He could develop atelectasis but it
C. Limiting intake of high-carbohydrate foods typically doesn’t produce progressive hypoxia.
D. Maintaining room temperature in the low-normal range
92. A client with shortness of breath has decreased to absent
Answer: (D) Maintaining room temperature in the low-normal breath sounds on the right side, from the apex to the base.
range. Graves’ disease causes signs and symptoms Which of the following conditions would best explain this?
of hypermetabolism, such as heat intolerance, diaphoresis, A. Acute asthma
excessive thirst and appetite, and weight loss. To reduce heat B. Chronic bronchitis
intolerance and diaphoresis, the nurse should keep the C. Pneumonia
client’s room temperature in the low-normal range. To D. Spontaneous pneumothorax
replace fluids lost via diaphoresis, the nurse
should encourage, not restrict, intake of oral fluids. Placing Answer: (D) Spontaneous pneumothorax. A spontaneous
extra blankets on the bed of a client with heat intolerance pneumothorax occurs when the client’s lung collapses,
would cause discomfort. To provide needed energy and causing an acute decreased in the amount of functional
calories, the nurse should encourage the client to eat high- lung used in oxygenation. The sudden collapse was the cause
carbohydrate foods. of his chest pain and shortness of breath. An asthma attack
would show wheezing breath sounds, and bronchitis would
88. Patrick is treated in the emergency department for a have rhonchi. Pneumonia would have bronchial breath
Colles’ fracture sustained during a fall. What is a Colles’ sounds over the area of consolidation
fracture?
A. Fracture of the distal radius 93. A 62-year-old male client was in a motor vehicle accident
B. Fracture of the olecranon as an unrestrained driver. He’s now in the emergency
C. Fracture of the humerus department complaining of difficulty of breathing and chest
D. Fracture of the carpal scaphoid pain. On auscultation of his lung field, no breath sounds are
present in the upper lobe. This client may have which of the
Answer: (A) Fracture of the distal radius. Colles’ fracture is a following conditions?
fracture of the distal radius, such as from a fall on an A. Bronchitis
outstretched hand. It’s most common in women. B. Pneumonia
C. Pneumothorax
D. Tuberculosis (TB) lactate solution IV to run over 24 hours. The IV infusion set
has a drop factor of 10 drops per milliliter. The nurse should
Answer: (C) Pneumothorax. From the trauma the client regulate the client’s IV to deliver how many drops per
experienced, it’s unlikely he has bronchitis, pneumonia, or minute?
TB; rhonchi with bronchitis, bronchial breath sounds with TB A. 18
would be heard. B. 21
C. 35
94. If a client requires a pneumonectomy, what fills the area D. 40
of the thoracic cavity?
A. The space remains filled with air only Answer: (B) 21. 3000 x 10 divided by 24 x 60
B. The surgeon fills the space with a gel
C. Serous fluids fills the space and consolidates the region 99. Mickey, a 6-year-old child with a congenital heart disorder
D. The tissue from the other lung grows over to the other is admitted with congestive heart failure. Digoxin (lanoxin)
side 0.12 mg is ordered for the child. The bottle of Lanoxin
contains .05 mg of Lanoxin in 1 ml of solution. What amount
Answer: (C) Serous fluids fills the space and consolidates the should the nurse administer to the child?
region. Serous fluid fills the space and eventually A. 1.2 ml
consolidates, preventing extensive mediastinal shift of the B. 2.4 ml
heart and remaining lung. Air can’t be left in the space. C. 3.5 ml
There’s no gel that can be placed in the pleural space. The D. 4.2 ml
tissue from the other lung can’t cross the
mediastinum, although a temporary mediastinal shift exits Answer: (B) 2.4 ml. .05 mg/ 1 ml = .12mg/ x ml, .05x = .12, x =
until the space is filled. 2.4 ml.

95. Hemoptysis may be present in the client with a pulmonary 100. Nurse Alexandra teaches a client about elastic stockings.
embolism because of which of the following reasons? Which of the following statements, if made by the client,
A. Alveolar damage in the infracted area indicates to the nurse that the teaching was successful?
B. Involvement of major blood vessels in the occluded area A. “I will wear the stockings until the physician tells me to
C. Loss of lung parenchyma remove them.”
D. Loss of lung tissue B. “I should wear the stockings even when I am sleep.”
C. “Every four hours I should remove the stockings for a
Answer: (A) Alveolar damage in the infracted area. The half hour.”
infracted area produces alveolar damage that can lead to the D. “I should put on the stockings before getting out of bed
production of bloody sputum, sometimes in massive in the morning.”
amounts. Clot formation usually occurs in the legs. There’s a Answer: (D) “I should put on the stockings before getting out
loss of lung parenchyma and subsequent scar tissue of bed in the morning. Promote venous return by applying
formation external pressure on veins.
96. Aldo with a massive pulmonary embolism will have an
arterial blood gas analysis performed to determine the extent
of hypoxia. The acid-base disorder that may be present is? PNLE V for Care of Clients with
A. Metabolic acidosis
B. Metabolic alkalosis
Physiologic and Psychosocial
C. Respiratory acidosis Alterations (Part 3)
D. Respiratory alkalosis

Answer: (D) Respiratory alkalosis. A client with massive


1. Mr. Marquez reports of losing his job, not being able to
pulmonary embolism will have a large region and blow off
sleep at night, and feeling upset with his wife. Nurse John
large amount of carbon dioxide, which crosses the unaffected
responds to the client, “You may want to talk about your
alveolar-capillary membrane more readily than does
employment situation in group today.” The Nurse is using
oxygen and results in respiratory alkalosis.
which therapeutic technique?
97. After a motor vehicle accident, Armand an 22-year-old
client is admitted with a pneumothorax. The surgeon inserts a A. Observations
chest tube and attaches it to a chest drainage system. B. Restating
Bubbling soon appears in the water seal chamber. Which of C. Exploring
the following is the most likely cause of the bubbling? D. Focusing\
A. Air leak
B. Adequate suction Answer: (D) Focusing. The nurse is using focusing by
C. Inadequate suction suggesting that the client discuss a specific issue. The nurse
D. Kinked chest tube didn’t restate the question, make observation, or ask further
question (exploring).
Answer: (A) Air leak. Bubbling in the water seal chamber of a
chest drainage system stems from an air leak. In 2. Tony refuses his evening dose of Haloperidol (Haldol), then
pneumothorax an air leak can occur as air is pulled from the becomes extremely agitated in the dayroom while other
pleural space. Bubbling doesn’t normally occur with clients are watching television. He begins cursing and
either adequate or inadequate suction or any preexisting throwing furniture. Nurse Oliver first action is to:
bubbling in the water seal chamber.
A. Check the client’s medical record for an order for an as-
98. Nurse Michelle calculates the IV flow rate for a needed I.M. dose of medication for agitation.
postoperative client. The client receives 3,000 ml of Ringer’s B. Place the client in full leather restraints.
C. Call the attending physician and report the behavior. stealing, rationalizing instead that as long as no one was using
D. Remove all other clients from the dayroom. the items, it was all right to borrow them. It is important for
the nurse to understand the psychodynamically, this behavior
Answer: (D) Remove all other clients from the dayroom. The may be largely attributed to a developmental defect related
nurse’s first priority is to consider the safety of the clients in to the:
the therapeutic setting. The other actions are appropriate
responses after ensuring the safety of other clients. A. Id
B. Ego
3. Tina who is manic, but not yet on medication, comes to the C. Superego
drug treatment center. The nurse would not let this client join D. Oedipal complex
the group session because:
Answer: (C) Superego. This behavior shows a weak sense of
A. The client is disruptive. moral consciousness. According to Freudian theory,
B. The client is harmful to self. personality disorders stem from a weak superego.
C. The client is harmful to others.
D. The client needs to be on medication first. 8. In preparing a female client for electroconvulsive therapy
(ECT), Nurse Michelle knows that succinylcoline (Anectine)
Answer: (A) The client is disruptive. Group activity provides will be administered for which therapeutic effect?
too much stimulation, which the client will not be able to
handle (harmful to self) and as a result will be disruptive to A. Short-acting anesthesia
others. B. Decreased oral and respiratory secretions.
C. Skeletal muscle paralysis.
4. Dervid, an adolescent boy was admitted for substance D. Analgesia.
abuse and hallucinations. The client’s mother asks Nurse
Armando to talk with his husband when he arrives at the Answer: (C) Skeletal muscle paralysis. Anectine is a
hospital. The mother says that she is afraid of what the father depolarizing muscle relaxant causing paralysis. It is used to
might say to the boy. The most appropriate nursing reduce the intensity of muscle contractions during
intervention would be to: the convulsive stage, thereby reducing the risk of bone
fractures or dislocation
A. Inform the mother that she and the father can work
through this problem themselves. 9. Nurse Gina is aware that the dietary implications for a
B. Refer the mother to the hospital social worker. client in manic phase of bipolar disorder is:
C. Agree to talk with the mother and the father together.
D. Suggest that the father and son work things out. A. Serve the client a bowl of soup, buttered French bread,
and apple slices.
Answer: (C) Agree to talk with the mother and the father B. Increase calories, decrease fat, and decrease protein.
together. By agreeing to talk with both parents, the nurse can C. Give the client pieces of cut-up steak, carrots, and an
provide emotional support and further assess and validate apple.
the family’s needs. D. Increase calories, carbohydrates, and protein.

5. What is Nurse John likely to note in a male client being Answer: (D) Increase calories, carbohydrates, and protein.This
admitted for alcohol withdrawal? client increased protein for tissue building and
increased calories to replace what is burned up (usually via
A. Perceptual disorders. carbohydrates).
B. Impending coma.
C. Recent alcohol intake. 10.What parental behavior toward a child during an admission
D. Depression with mutism. procedure should cause Nurse Ron to suspect child abuse?

Answer: (A) Perceptual disorders. Frightening visual A. Flat affect


hallucinations are especially common in clients experiencing B. Expressing guilt
alcohol withdrawal. C. Acting overly solicitous toward the child.
D. Ignoring the child.
6. Aira has taken amitriptyline HCL (Elavil) for 3 days, but now
complains that it “doesn’t help” and refuses to take it. What Answer: (C) Acting overly solicitous toward the child. This
should the nurse say or do? behavior is an example of reaction formation, a
coping mechanism.
A. Withhold the drug.
B. Record the client’s response. 11.Nurse Lynnette notices that a female client with obsessive-
C. Encourage the client to tell the doctor. compulsive disorder washes her hands for long periods each
D. Suggest that it takes awhile before seeing the results. day. How should the nurse respond to this compulsive
behavior?
Answer: (D) Suggest that it takes awhile before seeing the
results. The client needs a specific response; that it takes 2 A. By designating times during which the client can focus
to 3 weeks (a delayed effect) until the therapeutic blood on the behavior.
level is reached B. By urging the client to reduce the frequency of the
behavior as rapidly as possible.
7. Dervid, an adolescent has a history of truancy from school, C. By calling attention to or attempting to prevent the
running away from home and “barrowing” other people’s behavior.
things without their permission. The adolescent denies D. By discouraging the client from verbalizing anxieties.
cause of her symptoms. After the psychological conflict is
Answer: (A) By designating times during which the client can resolved, her symptoms will disappear. Saying that it must be
focus on the behavior. The nurse should designate times awful not to be able to move her legs wouldn’t answer the
during which the client can focus on the compulsive behavior client’s question; knowing that the cause is psychological
or obsessive thoughts. The nurse should urge the client to wouldn’t necessarily make her feel better. Telling her that she
reduce the frequency of the compulsive behavior gradually, has developed paralysis to avoid leaving her parents or
not rapidly. She shouldn’t call attention to or try to prevent that her personality caused her disorder wouldn’t help her
the behavior. Trying to prevent the behavior may cause pain understand and resolve the underlying conflict.
and terror in the client. The nurse should encourage the client
to verbalize anxieties to help distract attention from the 14.Nurse Krina knows that the following drugs have been
compulsive behavior. known to be effective in treating obsessive-compulsive
disorder (OCD):
12.After seeking help at an outpatient mental health clinic,
Ruby who was raped while walking her dog is diagnosed with A. benztropine (Cogentin) and diphenhydramine
posttraumatic stress disorder (PTSD). Three months later, (Benadryl).
Ruby returns to the clinic, complaining of fear, loss of control, B. chlordiazepoxide (Librium) and diazepam (Valium)
and helpless feelings. Which nursing intervention is most C. fluvoxamine (Luvox) and clomipramine (Anafranil)
appropriate for Ruby? D. divalproex (Depakote) and lithium (Lithobid)

A. Recommending a high-protein, low-fat diet. Answer: (C) fluvoxamine (Luvox) and clomipramine
B. Giving sleep medication, as prescribed, to restore a (Anafranil). The antidepressants fluvoxamine and
normal sleepwake cycle. clomipramine have been effective in the treatment of OCD.
C. Allowing the client time to heal. Librium and Valium may be helpful in treating anxiety related
D. Exploring the meaning of the traumatic event with the to OCD but aren’t drugs of choice to treat the illness. The
client. other medications mentioned aren’t effective in the
treatment of OCD.
Answer: (D) Exploring the meaning of the traumatic event
with the client. The client with PTSD needs encouragement to 15.Alfred was newly diagnosed with anxiety disorder. The
examine and understand the meaning of the traumatic event physician prescribed buspirone (BuSpar). The nurse is aware
and consequent losses. Otherwise, symptoms may worsen that the teaching instructions for newly prescribed buspirone
and the client may become depressed or engage in self- should include which of the following?
destructive behavior such as substance abuse. The client must
explore the meaning of the event and won’t heal without A. A warning about the drugs delayed therapeutic effect,
this, no matter how much time passes. Behavioral techniques, which is from 14 to 30 days.
such as relaxation therapy, may help decrease the client’s B. A warning about the incidence of neuroleptic malignant
anxiety and induce sleep. The physician may prescribe syndrome (NMS).
antianxiety agents or antidepressants cautiously to avoid C. A reminder of the need to schedule blood work in 1
dependence; sleep medication is rarely appropriate. A special week to check blood levels of the drug.
diet isn’t indicated unless the client also has an eating D. A warning that immediate sedation can occur with a
disorder or a nutritional problem. resultant drop in pulse.

13.Meryl, age 19, is highly dependent on her parents and Answer: (A) A warning about the drugs delayed therapeutic
fears leaving home to go away to college. Shortly before the effect, which is from 14 to 30 days. The client should be
semester starts, she complains that her legs are paralyzed and informed that the drug’s therapeutic effect might not be
is rushed to the emergency department. When physical reached for 14 to 30 days. The client must be instructed
examination rules out a physical cause for her paralysis, the to continue taking the drug as directed. Blood level checks
physician admits her to the psychiatric unit where she is aren’t necessary. NMS hasn’t been reported with this drug,
diagnosed with conversion disorder. Meryl asks the nurse, but tachycardia is frequently reported
“Why has this happened to me?” What is the nurse’s best
response? 16.Richard with agoraphobia has been symptom-free for 4
months. Classic signs and symptoms of phobias include:
A. “You’ve developed this paralysis so you can stay with
your parents. You must deal with this conflict if you A. Insomnia and an inability to concentrate.
want to walk again.” B. Severe anxiety and fear.
B. “It must be awful not to be able to move your legs. You C. Depression and weight loss.
may feel better if you realize the problem is D. Withdrawal and failure to distinguish reality from
psychological, not physical.” fantasy.
C. “Your problem is real but there is no physical basis for
it. We’ll work on what is going on in your life to find Answer: (B) Severe anxiety and fear. Phobias cause severe
out why it’s happened.” anxiety (such as a panic attack) that is out of proportion to
D. “It isn’t uncommon for someone with your personality the threat of the feared object or situation. Physical signs and
to develop a conversion disorder during times of stress.” symptoms of phobias include profuse sweating, poor
motor control, tachycardia, and elevated blood pressure.
Answer: (C) “Your problem is real but there is no physical Insomnia, an inability to concentrate, and weight loss are
basis for it. We’ll work on what is going on in your life to find common in depression. Withdrawal and failure to distinguish
out why it’s happened.” The nurse must be honest with the reality from fantasy occur in schizophrenia.
client by telling her that the paralysis has no physiologic cause
while also conveying empathy and acknowledging that her 17.Which medications have been found to help reduce or
symptoms are real. The client will benefit from psychiatric eliminate panic attacks?
treatment, which will help her understand the underlying
A. Antidepressants Answer: (C) Emotional lability, euphoria, and impaired
B. Anticholinergics memory. Signs of antianxiety agent overdose include
C. Antipsychotics emotional lability, euphoria, and impaired memory.
D. Mood stabilizers Phencyclidine overdose can cause combativeness, sweating,
and confusion. Amphetamine overdose can result in agitation,
Answer: (A) Antidepressants. Tricyclic and monoamine hyperactivity, and grandiose ideation. Hallucinogen overdose
oxidase (MAO) inhibitor antidepressants have been found to can produce suspiciousness, dilated pupils, and increased
be effective in treating clients with panic attacks. Why these blood pressure.
drugs help control panic attacks isn’t clearly understood.
Anticholinergic agents, which are smooth-muscle 21.The nurse is caring for a client diagnosed with antisocial
relaxants, relieve physical symptoms of anxiety but don’t personality disorder. The client has a history of fighting,
relieve the anxiety itself. Antipsychotic drugs are cruelty to animals, and stealing. Which of the following traits
inappropriate because clients who experience panic attacks would the nurse be most likely to uncover during assessment?
aren’t psychotic. Mood stabilizers aren’t indicated
because panic attacks are rarely associated with mood A. History of gainful employment
changes. B. Frequent expression of guilt regarding antisocial
behavior
18.A client seeks care because she feels depressed and has C. Demonstrated ability to maintain close, stable
gained weight. To treat her atypical depression, the physician relationships
prescribes tranylcypromine sulfate (Parnate), 10 mg by mouth D. d. A low tolerance for frustration
twice per day. When this drug is used to treat atypical
depression, what is its onset of action? Answer: (D) A low tolerance for frustration. Clients with an
antisocial personality disorder exhibit a low tolerance for
A. 1 to 2 days frustration, emotional immaturity, and a lack of
B. 3 to 5 days impulse control. They commonly have a history of
C. 6 to 8 days unemployment, miss work repeatedly, and quit work without
D. 10 to 14 days other plans for employment. They don’t feel guilt about their
behavior and commonly perceive themselves as victims. They
Answer: (B) 3 to 5 days. Monoamine oxidase inhibitors, such also display a lack of responsibility for the outcome of
as tranylcypromine, have an onset of action of their actions. Because of a lack of trust in others, clients with
approximately 3 to 5 days. A full clinical response may be antisocial personality disorder commonly have difficulty
delayed for 3 to 4 weeks. The therapeutic effects may developing stable, close relationships.
continue for 1 to 2 weeks after discontinuation
22.Nurse Amy is providing care for a male client undergoing
19. A 65 years old client is in the first stage of Alzheimer’s opiate withdrawal. Opiate withdrawal causes severe physical
disease. Nurse Patricia should plan to focus this client’s care discomfort and can be life-threatening. To minimize these
on: effects, opiate users are commonly detoxified with:

A. Offering nourishing finger foods to help maintain the A. Barbiturates


client’s nutritional status. B. Amphetamines
B. Providing emotional support and individual counseling. C. Methadone
C. Monitoring the client to prevent minor illnesses from D. Benzodiazepines
turning into major problems.
D. Suggesting new activities for the client and family to do Answer: (C) Methadone. Methadone is used to detoxify
together. opiate users because it binds with opioid receptors at many
sites in the central nervous system but doesn’t have the same
Answer: (B) Providing emotional support and individual deterious effects as other opiates, such as cocaine, heroin,
counseling. Clients in the first stage of Alzheimer’s disease are and morphine. Barbiturates, amphetamines,
aware that something is happening to them and may become and benzodiazepines are highly addictive and would require
overwhelmed and frightened. Therefore, nursing care detoxification treatment.
typically focuses on providing emotional support and
individual counseling. The other options are appropriate 23.Nurse Cristina is caring for a client who experiences false
during the second stage of Alzheimer’s disease, when sensory perceptions with no basis in reality. These
the client needs continuous monitoring to prevent minor perceptions are known as:
illnesses from progressing into major problems and when
maintaining adequate nutrition may become a challenge. A. Delusions
During this stage, offering nourishing finger foods helps B. Hallucinations
clients to feed themselves and maintain adequate nutrition. C. Loose associations
D. Neologisms
20.The nurse is assessing a client who has just been admitted
to the emergency department. Which signs would suggest an Answer: (B) Hallucinations. Hallucinations are visual, auditory,
overdose of an antianxiety agent? gustatory, tactile, or olfactory perceptions that have no basis
in reality. Delusions are false beliefs, rather than perceptions,
A. Combativeness, sweating, and confusion that the client accepts as real. Loose associations are rapid
B. Agitation, hyperactivity, and grandiose ideation shifts among unrelated ideas. Neologisms are bizarre words
C. Emotional lability, euphoria, and impaired memory that have meaning only to the client.
D. Suspiciousness, dilated pupils, and increased blood
pressure 24. Nurse Marco is developing a plan of care for a client with
anorexia nervosa. Which action should the nurse include in
the plan?
A. Restricts visits with the family and friends until the response to stress, characterized by apathy. Logical thinking is
client begins to eat. the ability to think rationally and make responsible decisions,
B. Provide privacy during meals. which would lead the client admitting the problem and
C. Set up a strict eating plan for the client. seeking help. Repression is suppressing past events from the
D. Encourage the client to exercise, which will reduce her consciousness because of guilty association.
anxiety.
28.Richard is admitted with a diagnosis of schizotypal
Answer: (C) Set up a strict eating plan for the personality disorder. Which signs would this client exhibit
client. Establishing a consistent eating plan and monitoring during social situations?
the client’s weight are very important in this disorder. The
family and friends should be included in the client’s care. The A. Aggressive behavior
client should be monitored during meals-not given privacy. B. Paranoid thoughts
Exercise must be limited and supervised. C. Emotional affect
D. Independence needs
25.Tim is admitted with a diagnosis of delusions of grandeur.
The nurse is aware that this diagnosis reflects a belief that Answer: (B) Paranoid thoughts. Clients with schizotypal
one is: personality disorder experience excessive social anxiety that
can lead to paranoid thoughts. Aggressive behavior is
A. Highly important or famous. uncommon, although these clients may experience
B. Being persecuted agitation with anxiety. Their behavior is emotionally cold with
C. Connected to events unrelated to oneself a flattened affect, regardless of the situation. These clients
D. Responsible for the evil in the world. demonstrate a reduced capacity for close or dependent
relationships.
Answer: (A) Highly important or famous. A delusion of
grandeur is a false belief that one is highly important or 29. Nurse Mickey is caring for a client diagnosed with bulimia.
famous. A delusion of persecution is a false belief that one The most appropriate initial goal for a client diagnosed with
is being persecuted. A delusion of reference is a false belief bulimia is to:
that one is connected to events unrelated to oneself or a
belief that one is responsible for the evil in the world A. Avoid shopping for large amounts of food.
B. Control eating impulses.
26.Nurse Jen is caring for a male client with manic depression. C. Identify anxiety-causing situations
The plan of care for a client in a manic state would include: D. Eat only three meals per day.
Answer: (C) Identify anxiety-causing situations. Bulimic
A. Offering a high-calorie meals and strongly encouraging behavior is generally a maladaptive coping response to stress
the client to finish all food. and underlying issues. The client must identify anxiety-
B. Insisting that the client remain active through the day so causing situations that stimulate the bulimic behavior and
that he’ll sleep at night. then learn new ways of coping with the anxiety.
C. Allowing the client to exhibit hyperactive, demanding,
manipulative behavior without setting limits. 30.Rudolf is admitted for an overdose of amphetamines.
D. Listening attentively with a neutral attitude and When assessing the client, the nurse should expect to see:
avoiding power struggles.
A. Tension and irritability
Answer: (D) Listening attentively with a neutral attitude and B. Slow pulse
avoiding power struggles. The nurse should listen to the C. Hypotension
client’s requests, express willingness to seriously consider the D. Constipation
request, and respond later. The nurse should encourage the
client to take short daytime naps because he expends so Answer: (A) Tension and irritability. An amphetamine is a
much energy. The nurse shouldn’t try to restrain the nervous system stimulant that is subject to abuse because of
client when he feels the need to move around as long as his its ability to produce wakefulness and euphoria. An overdose
activity isn’t harmful. High calorie finger foods should be increases tension and irritability. Options B and C are
offered to supplement the client’s diet, if he can’t remain incorrect because amphetamines stimulate norepinephrine,
seated long enough to eat a complete meal. The nurse which increase the heart rate and blood flow. Diarrhea is a
shouldn’t be forced to stay seated at the table to finish common adverse effect so option D in is incorrect
a meal. The nurse should set limits in a calm, clear, and self-
confident tone of voice. 31.Nicolas is experiencing hallucinations tells the nurse, “The
voices are telling me I’m no good.” The client asks if the nurse
27.Ramon is admitted for detoxification after a cocaine hears the voices. The most appropriate response by the nurse
overdose. The client tells the nurse that he frequently uses would be:
cocaine but that he can control his use if he chooses. Which
coping mechanism is he using?
A. “It is the voice of your conscience, which only you can
control.”
A. Withdrawal B. “No, I do not hear your voices, but I believe you can
B. Logical thinking hear them”.
C. Repression C. “The voices are coming from within you and only you
D. Denial can hear them.”
D. “Oh, the voices are a symptom of your illness; don’t pay
Answer: (D) Denial. Denial is unconscious defense mechanism any attention to them.”
in which emotional conflict and anxiety is avoided by refusing
to acknowledge feelings, desires, impulses, or external facts
that are consciously intolerable. Withdrawal is a common
Answer: (B) “No, I do not hear your voices, but I believe you Answer: (D) Moderate-level anxiety. A moderately anxious
can hear them”. The nurse, demonstrating knowledge and person can ignore peripheral events and focuses on central
understanding, accepts the client’s perceptions even though concerns
they are hallucinatory
37.When assessing a premorbid personality characteristics of
32.The nurse is aware that the side effect of electroconvulsive a client with a major depression, it would be unusual for the
therapy that a client may experience: nurse to find that this client demonstrated:

A. Loss of appetite A. Rigidity


B. Postural hypotension B. Stubbornness
C. Confusion for a time after treatment C. Diverse interest
D. Complete loss of memory for a time D. Over meticulousness

Answer: (C) Confusion for a time after treatment. The Answer: (C) Diverse interest. Before onset of depression,
electrical energy passing through the cerebral cortex during these clients usually have very narrow, limited interest.
ECT results in a temporary state of confusion after treatment.
38.Nurse Krina recognizes that the suicidal risk for depressed
33.A dying male client gradually moves toward resolution of client is greatest:
feelings regarding impending death. Basing care on the theory
of Kubler-Ross, Nurse Trish plans to use nonverbal A. As their depression begins to improve
interventions when assessment reveals that the client is in B. When their depression is most severe
the: C. Before nay type of treatment is started
D. As they lose interest in the environment
A. Anger stage
B. Denial stage Answer: (A) As their depression begins to improve. At this
C. Bargaining stage point the client may have enough energy to plan
D. Acceptance stage and execute an attempt

Answer: (D) Acceptance stage. Communication and 39.Nurse Kate would expect that a client with vascular
intervention during this stage are mainly nonverbal, as when dementis would experience:
the client gestures to hold the nurse’s hand
34.The outcome that is unrelated to a crisis state is: A. Loss of remote memory related to anoxia
B. Loss of abstract thinking related to emotional state
A. Learning more constructive coping skills C. Inability to concentrate related to decreased stimuli
B. Decompensation to a lower level of functioning. D. Disturbance in recalling recent events related to
C. Adaptation and a return to a prior level of functioning. cerebral hypoxia.
D. A higher level of anxiety continuing for more than 3
months. Answer: (D) Disturbance in recalling recent events related to
cerebral hypoxia. Cell damage seems to interfere with
Answer: (D) A higher level of anxiety continuing for more than registering input stimuli, which affects the ability to register
3 months. This is not an expected outcome of a crisis because and recall recent events; vascular dementia is related to
by definition a crisis would be resolved in 6 weeks. multiple vascular lesions of the cerebral cortex
and subcortical structure.
35.Miranda a psychiatric client is to be discharged with orders
for haloperidol (haldol) therapy. When developing a teaching 40.Josefina is to be discharged on a regimen of lithium
plan for discharge, the nurse should include cautioning the carbonate. In the teaching plan for discharge the nurse should
client against: include:

A. Driving at night A. Advising the client to watch the diet carefully


B. Staying in the sun B. Suggesting that the client take the pills with milk
C. Ingesting wines and cheeses C. Reminding the client that a CBC must be done once a
D. Taking medications containing aspirin month.
D. Encouraging the client to have blood levels checked as
Answer: (B) Staying in the sun. Haldol causes ordered.
photosensitivity. Severe sunburn can occur on exposure to
the sun Answer: (D) Encouraging the client to have blood levels
checked as ordered. Blood levels must be checked monthly
36.Jen a nursing student is anxious about the upcoming board or bimonthly when the client is on maintenance therapy
examination but is able to study intently and does not because there is only a small range between therapeutic and
become distracted by a roommate’s talking and loud music. toxic levels.
The student’s ability to ignore distractions and to focus on
studying demonstrates: 41.The psychiatrist orders lithium carbonate 600 mg p.o t.i.d
for a female client. Nurse Katrina would be aware that the
A. Mild-level anxiety teaching about the side effects of this drug were understood
B. Panic-level anxiety when the client state, “I will call my doctor immediately if I
C. Severe-level anxiety notice any:
D. Moderate-level anxiety
A. Sensitivity to bright light or sun
B. Fine hand tremors or slurred speech
C. Sexual dysfunction or breast enlargement account for 90% of suicide attempts but males are three times
D. d. Inability to urinate or difficulty when urinating more successful because of methods used.

Answer: (B) Fine hand tremors or slurred speech. These are 47. Dervid with paranoid schizophrenia repeatedly uses
common side effects of lithium carbonate. profanity during an activity therapy session. Which response
by the nurse would be most appropriate?
42.Nurse Mylene recognizes that the most important factor
necessary for the establishment of trust in a critical care area A. “Your behavior won’t be tolerated. Go to your room
is: immediately.”
B. “You’re just doing this to get back at me for making you
A. Privacy come to therapy.”
B. Respect C. “Your cursing is interrupting the activity. Take time out
C. Empathy in your room for 10 minutes.”
D. Presence D. “I’m disappointed in you. You can’t control yourself even
for a few minutes.”
Answer: (D) Presence. The constant presence of a nurse
provides emotional support because the client knows that Answer: (C) “Your cursing is interrupting the activity. Take
someone is attentive and available in case of an emergency time out in your room for 10 minutes.” The nurse should set
limits on client behavior to ensure a comfortable
43.When establishing an initial nurse-client relationship, environment for all clients. The nurse should accept hostile
Nurse Hazel should explore with the client the: or quarrelsome client outbursts within limits without
becoming personally offended, as in option A. Option B is
A. Client’s perception of the presenting problem. incorrect because it implies that the client’s actions reflect
B. Occurrence of fantasies the client may experience. feelings toward the staff instead of the client’s own misery.
C. Details of any ritualistic acts carried out by the client Judgmental remarks, such as option D, may decrease the
D. Client’s feelings when external; controls are instituted. client’s self-esteem

Answer: (A) Client’s perception of the presenting 48.Nurse Maureen knows that the nonantipsychotic
problem. The nurse can be most therapeutic by starting medication used to treat some clients with schizoaffective
where the client is, because it is the client’s concept of the disorder is:
problem that serves as the starting point of the relationship.
A. phenelzine (Nardil)
44.Tranylcypromine sulfate (Parnate) is prescribed for a B. chlordiazepoxide (Librium)
depressed client who has not responded to the tricyclic C. lithium carbonate (Lithane)
antidepressants. After teaching the client about the D. imipramine (Tofranil)
medication, Nurse Marian evaluates that learning has
occurred when the client states, “I will avoid: Answer: (C) lithium carbonate (Lithane). Lithium carbonate,
an antimania drug, is used to treat clients with cyclical
A. Citrus fruit, tuna, and yellow vegetables.” schizoaffective disorder, a psychotic disorder once
B. Chocolate milk, aged cheese, and yogurt’” classified under schizophrenia that causes affective
C. Green leafy vegetables, chicken, and milk.” symptoms, including maniclike activity. Lithium helps control
D. Whole grains, red meats, and carbonated soda.” the affective component of this disorder. Phenelzine is a
monoamine oxidase inhibitor prescribed for clients
Answer: (B) Chocolate milk, aged cheese, and yogurt’. These who don’t respond to other antidepressant drugs such as
high-tyramine foods, when ingested in the presence of an imipramine. Chlordiazepoxide, an antianxiety agent,
MAO inhibitor, cause a severe hypertensive response. generally is contraindicated in psychotic clients. Imipramine,
primarily considered an antidepressant agent, is also used to
45.Nurse John is a aware that most crisis situations should treat clients with agoraphobia and that undergoing cocaine
resolve in about: detoxification

49.Which information is most important for the nurse Trinity


A. 1 to 2 weeks
to include in a teaching plan for a male schizophrenic client
B. 4 to 6 weeks
taking clozapine (Clozaril)?
C. 4 to 6 months
D. 6 to 12 months
A. Monthly blood tests will be necessary.
Answer: (B) 4 to 6 weeks. Crisis is self-limiting and lasts from 4 B. Report a sore throat or fever to the physician
to 6 weeks. immediately.
C. Blood pressure must be monitored for hypertension.
46. Nurse Judy knows that statistics show that in adolescent D. Stop the medication when symptoms subside.
suicide behavior:
Answer: (B) Report a sore throat or fever to the physician
immediately. A sore throat and fever are indications of an
A. Females use more dramatic methods than males
infection caused by agranulocytosis, a potentially life-
B. Males account for more attempts than do females
threatening complication of clozapine. Because of the risk of
C. Females talk more about suicide before attempting it
agranulocytosis, white blood cell (WBC) counts are necessary
D. Males are more likely to use lethal methods than are
weekly, not monthly. If the WBC count drops below
females
3,000/μl, the medication must be stopped. Hypotension may
occur in clients taking this medication. Warn the client to
Answer: (D) Males are more likely to use lethal methods than
stand up slowly to avoid dizziness from orthostatic
are females. This finding is supported by research; females
hypotension. The medication should be continued, even
when symptoms have been controlled. If the medication must manic signs and symptoms. Major depression is a recurring,
be stopped, it should be slowly tapered over 1 to 2 weeks and persistent sadness or loss of interest or pleasure in almost all
only under the supervision of a physician. activities, with signs and symptoms recurring for at least 2
weeks.
50.Ricky with chronic schizophrenia takes neuroleptic
medication is admitted to the psychiatric unit. Nursing 53. After taking an overdose of phenobarbital (Barbita), Mario
assessment reveals rigidity, fever, hypertension, and is admitted to the emergency department. Dr. Trinidad
diaphoresis. These findings suggest which prescribes activated charcoal (Charcocaps) to be administered
lifethreatening reaction: by mouth immediately. Before administering the dose, the
nurse verifies the dosage ordered. What is the usual
A. Tardive dyskinesia. minimum dose of activated charcoal?
B. Dystonia.
C. Neuroleptic malignant syndrome. A. 5 g mixed in 250 ml of water
D. Akathisia. B. 15 g mixed in 500 ml of water
C. 30 g mixed in 250 ml of water
Answer: (C) Neuroleptic malignant syndrome. The client’s D. 60 g mixed in 500 ml of water
signs and symptoms suggest neuroleptic malignant syndrome,
a life-threatening reaction to neuroleptic medication that Answer: (C) 30 g mixed in 250 ml of water. The usual adult
requires immediate treatment. Tardive dyskinesia causes dosage of activated charcoal is 5 to 10 times the estimated
involuntary movements of the tongue, mouth, facial muscles, weight of the drug or chemical ingested, or a minimum
and arm and leg muscles. Dystonia is characterized by cramps dose of 30 g, mixed in 250 ml of water. Doses less than this
and rigidity of the tongue, face, neck, and back muscles. will be ineffective; doses greater than this can increase the
Akathisia causes restlessness, anxiety, and jitteriness. risk of adverse reactions, although toxicity doesn’t occur with
activated charcoal, even at the maximum dose.
51.Which nursing intervention would be most appropriate if a
male client develop orthostatic hypotension while taking 54.What herbal medication for depression, widely used in
amitriptyline (Elavil)? Europe, is now being prescribed in the United States?

A. Consulting with the physician about substituting a A. Ginkgo biloba


different type of antidepressant. B. Echinacea
B. Advising the client to sit up for 1 minute before getting C. St. John’s wort
out of bed. D. Ephedra
C. Instructing the client to double the dosage until the
problem resolves. Answer: (C) St. John’s wort. St. John’s wort has been found to
D. Informing the client that this adverse reaction should have serotonin-elevating properties, similar to prescription
disappear within 1 week. antidepressants. Ginkgo biloba is prescribed to enhance
mental acuity. Echinacea has immune-stimulating properties.
Answer: (B) Advising the client to sit up for 1 minute before Ephedra is a naturally occurring stimulant that is similar
getting out of bed. To minimize the effects of amitriptyline- to ephedrine.
induced orthostatic hypotension, the nurse should advise the
client to sit up for 1 minute before getting out of bed. 55.Cely with manic episodes is taking lithium. Which
Orthostatic hypotension commonly occurs with tricyclic electrolyte level should the nurse check before administering
antidepressant therapy. In these cases, the dosage may this medication?
be reduced or the physician may prescribe nortriptyline,
another tricyclic antidepressant. Orthostatic hypotension A. Calcium
disappears only when the drug is discontinued. B. Sodium
C. Chloride
52.Mr. Cruz visits the physician’s office to seek treatment for D. Potassium
depression, feelings of hopelessness, poor appetite, insomnia,
fatigue, low selfesteem, poor concentration, and difficulty Answer: (B) Sodium. Lithium is chemically similar to sodium. If
making decisions. The client states that these symptoms sodium levels are reduced, such as from sweating or diuresis,
began at least 2 years ago. Based on this report, the nurse lithium will be reabsorbed by the kidneys, increasing the risk
Tyfany suspects: of toxicity. Clients taking lithium shouldn’t restrict their intake
of sodium and should drink adequate amounts of fluid each
A. Cyclothymic disorder. day. The other electrolytes are important for normal body
B. Atypical affective disorder. functions but sodium is most important to the absorption of
C. Major depression. lithium.
D. Dysthymic disorder.
56.Nurse Josefina is caring for a client who has been
Answer: (D) Dysthymic disorder. Dysthymic disorder is marked diagnosed with delirium. Which statement about delirium is
by feelings of depression lasting at least 2 years, accompanied true?
by at least two of the following symptoms: sleep disturbance,
appetite disturbance, low energy or fatigue, low A. It’s characterized by an acute onset and lasts about 1
selfesteem, poor concentration, difficulty making decisions, month.
and hopelessness. These symptoms may be relatively B. It’s characterized by a slowly evolving onset and lasts
continuous or separated by intervening periods of normal about 1 week.
mood that last a few days to a few weeks. Cyclothymic C. It’s characterized by a slowly evolving onset and lasts
disorder is a chronic mood disturbance of at least 2 years’ about 1 month.
duration marked by numerous periods of depression
and hypomania. Atypical affective disorder is characterized by
D. It’s characterized by an acute onset and lasts hours to a
number of days. Answer: (C) Monitor vital signs, serum electrolyte levels, and
acid-base balance. An anorexic client who requires
Answer: (D) It’s characterized by an acute onset and lasts hospitalization is in poor physical condition from starvation
hours to a number of days. Delirium has an acute onset and and may die as a result of arrhythmias, hypothermia,
typically can last from several hours to several days. malnutrition, infection, or cardiac abnormalities secondary
to electrolyte imbalances. Therefore, monitoring the client’s
57.Edward, a 66 year old client with slight memory vital signs, serum electrolyte level, and acid base balance is
impairment and poor concentration is diagnosed with crucial. Option A may worsen anxiety. Option B is incorrect
primary degenerative dementia of the Alzheimer’s type. Early because a weight obtained after breakfast is more accurate
signs of this dementia include subtle personality changes and than one obtained after the evening meal. Option D
withdrawal from social interactions. To assess for progression would reward the client with attention for not eating and
to the middle stage of Alzheimer’s disease, the nurse reinforce the control issues that are central to the underlying
should observe the client for: psychological problem; also, the client may record food and
fluid intake inaccurately.
A. Occasional irritable outbursts.
B. Impaired communication. 60.Celia with a history of polysubstance abuse is admitted to
C. Lack of spontaneity. the facility. She complains of nausea and vomiting 24 hours
D. Inability to perform self-care activities. after admission. The nurse assesses the client and notes
piloerection, pupillary dilation, and lacrimation. The nurse
Answer: (B) Impaired communication. Initially, memory suspects that the client is going through which of the
impairment may be the only cognitive deficit in a client with following withdrawals?
Alzheimer’s disease. During the early stage of this
disease, subtle personality changes may also be present. A. Alcohol withdrawal
However, other than occasional irritable outbursts and lack B. Cannibis withdrawal
of spontaneity, the client is usually cooperative and exhibits C. Cocaine withdrawal
socially appropriate behavior. Signs of advancement to the D. Opioid withdrawal
middle stage of Alzheimer’s disease include exacerbated
cognitive impairment with obvious personality changes Answer: (D) Opioid withdrawal. The symptoms listed are
and impaired communication, such as inappropriate specific to opioid withdrawal. Alcohol withdrawal would show
conversation, actions, and responses. During the late stage, elevated vital signs. There is no real withdrawal from cannibis.
the client can’t perform self-care activities and may become Symptoms of cocaine withdrawal include depression, anxiety,
mute and agitation.
58.Isabel with a diagnosis of depression is started on
imipramine (Tofranil), 75 mg by mouth at bedtime. The nurse 61.Mr. Garcia, an attorney who throws books and furniture
should tell the client that: around the office after losing a case is referred to the
psychiatric nurse in the law firm’s employee assistance
A. This medication may be habit forming and will be program. Nurse Beatriz knows that the client’s behavior most
discontinued as soon as the client feels better. likely represents the use of which defense mechanism?
B. This medication has no serious adverse effects.
C. The client should avoid eating such foods as aged A. Regression
cheeses, yogurt, and chicken livers while taking the B. Projection
medication. C. Reaction-formation
D. This medication may initially cause tiredness, which D. Intellectualization
should become less bothersome over time.
Answer: (A) Regression. An adult who throws temper
Answer: (D) This medication may initially cause tiredness, tantrums, such as this one, is displaying regressive behavior,
which should become less bothersome over time. Sedation is or behavior that is appropriate at a younger age. In
a common early adverse effect of imipramine, a tricyclic projection, the client blames someone or something
antidepressant, and usually decreases as tolerance other than the source. In reaction formation, the client acts in
develops. Antidepressants aren’t habit forming and don’t opposition to his feelings. In intellectualization, the client
cause physical or psychological dependence. However, after a overuses rational explanations orabstract thinking to decrease
long course of high-dose therapy, the dosage should be the significance of a feeling or event.
decreased gradually to avoid mild withdrawal symptoms.
Serious adverse effects, although rare, include myocardial 62.Nurse Anne is caring for a client who has been treated long
infarction, heart failure, and tachycardia. Dietary term with antipsychotic medication. During the assessment,
restrictions, such as avoiding aged cheeses, yogurt, and Nurse Anne checks the client for tardive dyskinesia. If tardive
chicken livers, are necessary for a client taking a monoamine dyskinesia is present, Nurse Anne would most likely observe:
oxidase inhibitor, not a tricyclic antidepressant.
A. Abnormal movements and involuntary movements of
59.Kathleen is admitted to the psychiatric clinic for treatment the mouth, tongue, and face.
of anorexia nervosa. To promote the client’s physical health, B. Abnormal breathing through the nostrils accompanied
the nurse should plan to: by a “thrill.”
C. Severe headache, flushing, tremors, and ataxia.
A. Severely restrict the client’s physical activities. D. Severe hypertension, migraine headache,
B. Weigh the client daily, after the evening meal.
C. Monitor vital signs, serum electrolyte levels, and acid- Answer: (A) Abnormal movements and involuntary
base balance. movements of the mouth, tongue, and face. Tardive
D. Instruct the client to keep an accurate record of food dyskinesia is a severe reaction associated with long term use
and fluid intake. of antipsychotic medication. The clinical manifestations
include abnormal movements (dyskinesia) and involuntary has an I.Q. of below 20; Mild mental retardation 50-70 and
movements of the mouth, tongue (fly catcher tongue), and Severe mental retardation has an I.Q. of 20-35.
face.
67.The therapeutic approach in the care of Armand an autistic
63.Dennis has a lithium level of 2.4 mEq/L. The nurse child include the following EXCEPT:
immediately would assess the client for which of the
following signs or symptoms? A. Engage in diversionary activities when acting -out
B. Provide an atmosphere of acceptance
A. Weakness C. Provide safety measures
B. Diarrhea D. Rearrange the environment to activate the child
C. Blurred vision
D. Fecal incontinence Answer: (D) Rearrange the environment to activate the
child. The child with autistic disorder does not want
Answer: (C) Blurred vision. At lithium levels of 2 to 2.5 mEq/L change. Maintaining a consistent environment is
the client will experienced blurred vision, muscle twitching, therapeutic. A. Angry outburst can be re-channeling through
severe hypotension, and persistent nausea and vomiting. safe activities. B. Acceptance enhances a trusting
With levels between 1.5 and 2 mEq/L the client experiencing relationship. C. Ensure safety from self-destructive behaviors
vomiting, diarrhea, muscle weakness, ataxia, like head banging and hair pulling
dizziness, slurred speech, and confusion. At lithium levels of
2.5 to 3 mEq/L or higher, urinary and fecal incontinence 68.Jeremy is brought to the emergency room by friends who
occurs, as well as seizures, cardiac dysrythmias, peripheral state that he took something an hour ago. He is actively
vascular collapse, and death. hallucinating, agitated, with irritated nasal septum.

64.Nurse Jannah is monitoring a male client who has been A. Heroin


placed inrestraints because of violent behavior. Nurse B. Cocaine
determines that it will be safe to remove the restraints when: C. LSD
D. Marijuana
A. The client verbalizes the reasons for the violent
behavior. Answer: (B) cocaine. The manifestations indicate intoxication
B. The client apologizes and tells the nurse that it will with cocaine, a CNS stimulant. A. Intoxication with heroine is
never happen again. manifested by euphoria then impairment in judgment,
C. No acts of aggression have been observed within 1 attention and the presence of papillary constriction. C.
hour after the release of two of the extremity Intoxication with hallucinogen like LSD is manifested
restraints. by grandiosity, hallucinations, synesthesia and increase in vital
D. The administered medication has taken effect. signs D. Intoxication with Marijuana, a cannabinoid is
manifested by sensation of slowed time, conjunctival redness,
Answer: (C) No acts of aggression have been observed social withdrawal, impaired judgment and hallucinations.
within 1 hour after the release of two of the extremity
restraints. The best indicator that the behavior is controlled, 69.Nurse Pauline is aware that Dementia unlike delirium is
if the client exhibits no signs of aggression after partial characterized by:
release of restraints. Options A, B, and D do not ensure that
the client has controlled the behavior. A. Slurred speech
B. Insidious onset
65.Nurse Irish is aware that Ritalin is the drug of choice for a C. Clouding of consciousness
child with ADHD. The side effects of the following may be D. Sensory perceptual change
noted by the nurse:
Answer: (B) insidious onset. Dementia has a gradual onset
A. Increased attention span and concentration and progressive deterioration. It causes pronounced memory
B. Increase in appetite and cognitive disturbances. A,C and D are all characteristics of
C. Sleepiness and lethargy delirium.
D. Bradycardia and diarrhea
70.A 35 year old female has intense fear of riding an elevator.
Answer: (A) increased attention span and concentration. The She claims “ As if I will die inside.” The client is suffering from:
medication has a paradoxic effect that decrease hyperactivity
and impulsivity among children with ADHD. B, C, D. A. Agoraphobia
Side effects of Ritalin include anorexia, insomnia, diarrhea B. Social phobia
and irritability. C. Claustrophobia
D. Xenophobia
66.Kitty, a 9 year old child has very limited vocabulary and
interaction skills. She has an I.Q. of 45. She is diagnosed to Answer: (C) Claustrophobia. Claustrophobia is fear of closed
have Mental retardation of this classification: space. A. Agoraphobia is fear of open space or being a
situation where escape is difficult. B. Social phobia is fear of
A. Profound performing in the presence of others in a way that will
B. Mild be humiliating or embarrassing. D. Xenophobia is fear of
C. Moderate strangers.
D. Severe
71.Nurse Myrna develops a counter-transference reaction.
Answer: (C) Moderate. The child with moderate mental This is evidenced by:
retardation has an I.Q. of 35- 50 Profound Mental retardation
A. Revealing personal information to the client someone in her past to the patient D. Resistance is the client’s
B. Focusing on the feelings of the client. refusal to submit himself to the care of the nurse
C. Confronting the client about discrepancies in verbal or
non-verbal behavior 75.Marielle, 17 years old was sexually attacked while on her
D. The client feels angry towards the nurse who resembles way home from school. She is brought to the hospital by her
his mother. mother. Rape is an example of which type of crisis:

Answer: (A) Revealing personal information to the A. Situational


client. Counter-transference is an emotional reaction of the B. Adventitious
nurse on the client based on her unconscious needs and C. Developmental
conflicts. B and C. These are therapeutic approaches. D. This is D. Internal
transference reaction where a client has an emotional
reaction towards the nurse based on her past. Answer: (B) Adventitious. Adventitious crisis is a crisis
involving a traumatic event. It is not part of everyday life. A.
72.Tristan is on Lithium has suffered from diarrhea and Situational crisis is from an external source that upset ones
vomiting. What should the nurse in-charge do first: psychological equilibrium C and D. Are the same. They are
transitional or developmental periods in life
A. Recognize this as a drug interaction
B. Give the client Cogentin 76. Nurse Greta is aware that the following is classified as an
C. Reassure the client that these are common side effects Axis I disorder by the Diagnosis and Statistical Manual of
of lithium therapy Mental Disorders, Text Revision (DSM-IV-TR) is:
D. Hold the next dose and obtain an order for a stat
serum lithium level A. Obesity
B. Borderline personality disorder
Answer: (D) Hold the next dose and obtain an order for a stat C. Major depression
serum lithium level. Diarrhea and vomiting are manifestations D. Hypertension
of Lithium toxicity. The next dose of lithium should be
withheld and test is done to validate the observation. A. The Answer: (C) Major depression. The DSM-IV-TR classifies major
manifestations are not due to drug interaction. B. Cogentin is depression as an Axis I disorder. Borderline personality
used to manage the extra pyramidal symptom side effects disorder as an Axis II; obesity and hypertension, Axis III.
of antipsychotics. C. The common side effects of Lithium are
fine hand tremors, nausea, polyuria and polydipsia. 77.Katrina, a newly admitted is extremely hostile toward a
staff member she has just met, without apparent reason.
73.Nurse Sarah ensures a therapeutic environment for all the According to Freudian theory, the nurse should suspect that
client. Which of the following best describes a therapeutic the client is experiencing which of the following phenomena?
milieu?
A. Intellectualization
A. A therapy that rewards adaptive behavior B. Transference
B. A cognitive approach to change behavior C. Triangulation
C. A living, learning or working environment. D. Splitting
D. A permissive and congenial environment
Answer: (B) Transference. Transference is the unconscious
Answer: (C) A living, learning or working environment. A assignment of negative or positive feelings evoked by a
therapeutic milieu refers to a broad conceptual approach significant person in the client’s past to another person.
in which all aspects of the environment are channeled to Intellectualization is a defense mechanism in which the client
provide a therapeutic environment for the client. The six avoids dealing with emotions by focusing on facts.
environmental elements include structure, safety, norms; Triangulation refers to conflicts involving three family
limit setting, balance and unit modification. A. Behavioral members. Splitting is a defense mechanism commonly seen in
approach in psychiatric care is based on the premise that clients with personality disorder in which the world is
behavior can be learned or unlearned through the use perceived as all good or all bad.
of reward and punishment. B. Cognitive approach to change
behavior is done by correcting distorted perceptions and 78.An 83year-old male client is in extended care facility is
irrational beliefs to correct maladaptive behaviors. D. This is anxious most of the time and frequently complains of a
not congruent with therapeutic milieu. number of vague symptoms that interfere with his ability to
eat. These symptoms indicate which of the following
74.Anthony is very hostile toward one of the staff for no disorders?
apparent reason. He is manifesting:
A. Conversion disorder
A. Splitting B. Hypochondriasis
B. Transference C. Severe anxiety
C. Countertransference D. Sublimation
D. Resistance
Answer: (B) Hypochondriasis. Complains of vague physical
Answer: (B) Transference. Transference is a positive or symptoms that have no apparent medical causes are
negative feeling associated with a significant person in the characteristic of clients with hypochondriasis. In many cases,
client’s past that are unconsciously assigned to another A. the GI system is affected. Conversion disorders are
Splitting is a defense mechanism commonly seen in a characterized by one or more neurologic symptoms. The
client with personality disorder in which the world is client’s symptoms don’t suggest severe anxiety. A client
perceived as all good or all bad C. Countert-transference is a
phenomenon where the nurse shifts feelings assigned to
experiencing sublimation channels maladaptive feelings or A. “I went to the mall with my friends last Saturday”
impulses into socially acceptable behavior B. “I’m hyperventilating only when I have a panic attack”
C. “Today I decided that I can stop taking my medication”
79. Charina, a college student who frequently visited the D. “Last night I decided to eat more than a bowl of cereal”
health center during the past year with multiple vague
complaints of GI symptoms before course examinations. Answer: (A) “I went to the mall with my friends last
Although physical causes have been eliminated, the Saturday”. Clients with panic disorder tent to be socially
student continues to express her belief that she has a serious withdrawn. Going to the mall is a sign of working on
illness. These symptoms are typically of which of the following avoidance behaviors. Hyperventilating is a key symptom of
disorders? panic disorder. Teaching breathing control is a
major intervention for clients with panic disorder. The client
A. Conversion disorder taking medications for panic disorder; such as tricylic
B. Depersonalization antidepressants and benzodiazepines, must be weaned off
C. Hypochondriasis these drugs. Most clients with panic disorder
D. Somatization disorder with agoraphobia don’t have nutritional problems.

Answer: (C) Hypochondriasis. Hypochodriasis in this case is 83. The effectiveness of monoamine oxidase (MAO) inhibitor
shown by the client’s belief that she has a serious illness, drug therapy in client with posttraumatic stress disorder can
although pathologic causes have been eliminated. The be demonstrated by which of the following client self –
disturbance usually lasts at lease 6 with identifiable reports?
life stressor such as, in this case, course examinations.
Conversion disorders are characterized by one or more A. “I’m sleeping better and don’t have nightmares”
neurologic symptoms. Depersonalization refers to persistent B. “I’m not losing my temper as much”
recurrent episodes of feeling detached from one’s self or C. “I’ve lost my craving for alcohol”
body. Somatoform disorders generally have a chronic course D. “I’ve lost my phobia for water”
with few remissions
Answer: (A) “I’m sleeping better and don’t have
80. Nurse Daisy is aware that the following pharmacologic nightmares” MAO inhibitors are used to treat sleep problems,
agents are sedative hypnotic medication is used to induce nightmares, and intrusive daytime thoughts in individual with
sleep for a client experiencing a sleep disorder is: posttraumatic stress disorder. MAO inhibitors aren’t used to
help control flashbacks or phobias or to decrease the craving
A. Triazolam (Halcion) for alcohol.
B. Paroxetine (Paxil)\
C. Fluoxetine (Prozac) 84. Mark, with a diagnosis of generalized anxiety disorder
D. Risperidone (Risperdal) wants to stop taking his lorazepam (Ativan). Which of the
following important facts should nurse Betty discuss with the
Answer: (A) Triazolam (Halcion). Triazolam is one of a group of client about discontinuing the medication?
sedative hypnotic medication that can be used for a limited
time because of the risk of dependence. Paroxetine is a A. Stopping the drug may cause depression
scrotonin-specific reutake inhibitor used for treatment B. Stopping the drug increases cognitive abilities
of depression panic disorder, and obsessive-compulsive C. Stopping the drug decreases sleeping difficulties
disorder. Fluoxetine is a scrotonin-specific reuptake inhibitor D. Stopping the drug can cause withdrawal symptoms
used for depressive disorders and obsessive-compulsive
disorders. Risperidome is indicated for psychotic disorders. Answer: (D) Stopping the drug can cause withdrawal
symptoms. Stopping antianxiety drugs such as
81. Aldo, with a somatoform pain disorder may obtain benzodiazepines can cause the client to have withdrawal
secondary gain. Which of the following statement refers to a symptoms. Stopping a benzodiazepine doesn’t tend to cause
secondary gain? depression, increase cognitive abilities, or decrease sleeping
difficulties.
A. It brings some stability to the family
B. It decreases the preoccupation with the physical illness 85. Jennifer, an adolescent who is depressed and reported by
C. It enables the client to avoid some unpleasant activity his parents as having difficulty in school is brought to the
D. It promotes emotional support or attention for the community mental health center to be evaluated. Which of
client the following other health problems would the
nurse suspect?
Answer: (D) It promotes emotional support or attention for
the client. Secondary gain refers to the benefits of the illness A. Anxiety disorder
that allow the client to receive emotional support or B. Behavioral difficulties
attention. Primary gain enables the client to avoid some C. Cognitive impairment
unpleasant activity. A dysfunctional family may disregard the D. Labile moods
real issue, although some conflict is relieved.
Somatoform pain disorder is a preoccupation with pain in the Answer: (B) Behavioral difficulties. Adolescents tend to
absence of physical disease. demonstrate severe irritability and behavioral problems
rather than simply a depressed mood. Anxiety disorder is
82. Dervid is diagnosed with panic disorder with agoraphobia more commonly associated with small children rather than
is talking with the nurse in-charge about the progress made in with adolescents. Cognitive impairment is typically
treatment. Which of the following statements indicates a associated with delirium or dementia. Labile mood is more
positive client response? characteristic of a client with cognitive impairment or
bipolar disorder
86. Ricardo, an outpatient in psychiatric facility is diagnosed difficulty in speech production. Flight of ideas is rapid shifting
with dysthymic disorder. Which of the following statement from one topic to another
about dysthymic disorder is true?
90. Which of the following descriptions of a client’s
A. It involves a mood range from moderate depression to experience and behavior can be assessed as an illusion?
hypomania
B. It involves a single manic depression A. The client tries to hit the nurse when vital signs must be
C. It’s a form of depression that occurs in the fall and taken
winter B. The client says, “I keep hearing a voice telling me to run
D. It’s a mood disorder similar to major depression but of away”
mild to moderate severity C. The client becomes anxious whenever the nurse leaves
the bedside
Answer: (D) It’s a mood disorder similar to major depression D. The client looks at the shadow on a wall and tells the
but of mild to moderate severity. Dysthymic disorder is a nurse she sees frightening faces on the wall.
mood disorder similar to major depression but it remains mild Answer: (D) The client looks at the shadow on a wall and tells
to moderate in severity. Cyclothymic disorder is a mood the nurse she sees frightening faces on the wall. Minor
disorder characterized by a mood range from memory problems are distinguished from dementia by their
moderate depression to hypomania. Bipolar I disorder is minor severity and their lack of significant interference with
characterized by a single manic episode with no past major the client’s social or occupational lifestyle. Other options
depressive episodes. Seasonalaffective disorder is a form of would be included in the history data but don’t directly
depression occurring in the fall and winter. correlate with the client’s lifestyle.

87. The nurse is aware that the following ways in vascular 91. During conversation of Nurse John with a client, he
dementia different from Alzheimer’s disease is: observes that the client shift from one topic to the next on a
regular basis. Which of the following terms describes this
A. Vascular dementia has more abrupt onset disorder?
B. The duration of vascular dementia is usually brief
C. Personality change is common in vascular dementia A. Flight of ideas
D. The inability to perform motor activities occurs in B. Concrete thinking
vascular dementia C. Ideas of reference
D. Loose association
Answer: (A) Vascular dementia has more abrupt
onset. Vascular dementia differs from Alzheimer’s disease in Answer: (D) Loose association. Loose associations are
that it has a more abrupt onset and runs a highly variable conversations that constantly shift in topic. Concrete
course. Personally change is common in Alzheimer’s disease. thinking implies highly definitive thought processes. Flight of
The duration of delirium is usually brief. The inability to carry ideas is characterized by conversation that’s disorganized
out motor activities is common in Alzheimer’s disease from the onset. Loose associations don’t necessarily start in
a cogently, then becomes loose
88. Loretta, a newly admitted client was diagnosed with
delirium and has history of hypertension and anxiety. She had 92. Francis tells the nurse that her coworkers are sabotaging
been taking digoxin, furosemide (Lasix), and diazepam the computer. When the nurse asks questions, the client
(Valium) for anxiety. This client’s impairment may be related becomes argumentative. This behavior shows personality
to which of the following conditions? traits associated with which of the following personality
disorder?
A. Infection
B. Metabolic acidosis A. Antisocial
C. Drug intoxication B. Histrionic
D. Hepatic encephalopathy C. Paranoid
D. Schizotypal
Answer: (C) Drug intoxication. This client was taking several
medications that have a propensity for producing delirium; Answer: (C) Paranoid. Because of their suspiciousness,
digoxin (a digitalis glycoxide), furosemide (a thiazide diuretic), paranoid personalities ascribe malevolent activities to others
and diazepam (a benzodiazepine). Sufficient supporting data and tent to be defensive, becoming quarrelsome and
don’t exist to suspect the other options as causes. argumentative. Clients with antisocial personality disorder
can also be antagonistic and argumentative but are
89. Nurse Ron enters a client’s room, the client says, “They’re less suspicious than paranoid personalities. Clients with
crawling on my sheets! Get them off my bed!” Which of the histrionic personality disorder are dramatic, not suspicious
following assessment is the most accurate? and argumentative. Clients with schizoid personality disorder
are usually detached from other and tend to have eccentric
A. The client is experiencing aphasia behavior.
B. The client is experiencing dysarthria
C. The client is experiencing a flight of ideas 93. Which of the following interventions is important for a
D. The client is experiencing visual hallucination Cely experiencing with paranoid personality disorder taking
olanzapine (Zyprexa)?
Answer: (D) The client is experiencing visual hallucination. The
presence of a sensory stimulus correlates with the definition A. Explain effects of serotonin syndrome
of a hallucination, which is a false sensory perception. B. Teach the client to watch for extrapyramidal adverse
Aphasia refers to a communication problem. Dysarthria is reaction
C. Explain that the drug is less affective if the client world. They need to have as in-depth assessment of physical
smokes complaints that may spill over into their delusional symptoms.
D. Discuss the need to report paradoxical effects such as Talking with the client won’t provide as assessment of his
euphoria itching, and itching isn’t as adverse reaction of antipsychotic
drugs, calling the physician to get the client’s
Answer: (C) Explain that the drug is less affective if the client medication increased doesn’t address his physical complaints.
smokes. Olanzapine (Zyprexa) is less effective for clients who
smoke cigarettes. Serotonin syndrome occurs with clients who 97. Ivy, who is on the psychiatric unit is copying and imitating
take a combination of antidepressant medications. the movements of her primary nurse. During recovery, she
Olanzapine doesn’t cause euphoria, and extrapyramidal says, “I thought the nurse was my mirror. I felt connected only
adverse reactions aren’t a problem. However, the client when I saw my nurse.” This behavior is known by which of the
should be aware of adverse effects such as tardive dyskinesia. following terms?

94. Nurse Alexandra notices other clients on the unit avoiding A. Modeling
a client diagnosed with antisocial personality disorder. When B. Echopraxia
discussing appropriate behavior in group therapy, which of C. Ego-syntonicity
the following comments is expected about this client by his D. Ritualism
peers?
Answer: (B) Echopraxia. Echopraxia is the copying of
A. Lack of honesty another’s behaviors and is the result of the loss of ego
B. Belief in superstition boundaries. Modeling is the conscious copying of someone’s
C. Show of temper tantrums behaviors. Ego-syntonicity refers to behaviors that
D. Constant need for attention correspond with the individual’s sense of self. Ritualism
behaviors are repetitive and compulsive
Answer: (A) Lack of honesty. Clients with antisocial
personality disorder tent to engage in acts of dishonesty, 98. Jun approaches the nurse and tells that he hears a voice
shown by lying. Clients with schizotypal personality disorder telling him that he’s evil and deserves to die. Which of the
tend to be superstitious. Clients with histrionic following terms describes the client’s perception?
personality disorders tend to overreact to frustrations and
disappointments, have temper tantrums, and seek attention. A. Delusion
B. Disorganized speech
95. Tommy, with dependent personality disorder is working to C. Hallucination
increase his selfesteem. Which of the following statements by D. Idea of reference
the Tommy shows teaching was successful?
Answer: (C) Hallucination. Hallucinations are sensory
A. “I’m not going to look just at the negative things about experiences that are misrepresentations of reality or have no
myself” basis in reality. Delusions are beliefs not based in reality.
B. “I’m most concerned about my level of competence and Disorganized speech is characterized by jumping from one
progress” topic to the next or using unrelated words. An idea
C. “I’m not as envious of the things other people have as I of reference is a belief that an unrelated situation holds
used to be” special meaning for the client.
D. “I find I can’t stop myself from taking over things other
should be doing” 99. Mike is admitted to a psychiatric unit with a diagnosis of
undifferentiated schizophrenia. Which of the following
Answer: (A) “I’m not going to look just at the negative things defense mechanisms is probably used by mike?
about myself”. As the clients makes progress on improving
self-esteem, selfblame and negative self evaluation will A. Projection
decrease. Clients with dependent personality disorder tend to B. Rationalization
feel fragile and inadequate and would be extremely unlikely C. Regression
to discuss their level of competence and progress. These D. Repression
clients focus on self and aren’t envious or jealous. Individuals
with dependent personality disorders don’t take over Answer: (C) Regression. Regression, a return to earlier
situations because they see themselves as inept and behavior to reduce anxiety, is the basic defense mechanism
inadequate. in schizophrenia. Projection is a defense mechanism in which
one blames others and attempts to justify actions; it’s used
96. Norma, a 42-year-old client with a diagnosis of chronic primarily by people with paranoid schizophrenia and
undifferentiated schizophrenia lives in a rooming house that delusional disorder. Rationalization is a defense mechanism
has a weekly nursing clinic. She scratches while she tells the used to justify one’s action. Repression is the basic defense
nurse she feels creatures eating away at her skin. Which of mechanism in the neuroses; it’s an involuntary exclusion of
the following interventions should be done first? painful thoughts, feelings, or experiences from awareness

A. Talk about his hallucinations and fears 100. Rocky has started taking haloperidol (Haldol). Which of
B. Refer him for anticholinergic adverse reactions the following instructions is most appropriate for Ricky before
C. Assess for possible physical problems such as rash taking haloperidol?
D. Call his physician to get his medication increased to
control his psychosis A. Should report feelings of restlessness or agitation at
once
Answer: (C) Assess for possible physical problems such as B. Use a sunscreen outdoors on a year-round basis
rash. Clients with schizophrenia generally have poor C. Be aware you’ll feel increased energy taking this drug
visceral recognition because they live so fully in their fantasy
D. This drug will indirectly control essential hypertension
Answer: C. RN floated from the obstetrics unit
Answer: (A) Should report feelings of restlessness or agitation should be able to care for a client with major
at once. Agitation and restlessness are adverse effect of abdominal surgery, because this nurse has
haloperidol and can be treated with antocholinergic drugs. experienced caring for clients with cesarean
Haloperidol isn’t likely to cause photosensitivity or control births.
essential hypertension. Although the client may experience
increased concentration and activity, these effects are due to 4. The registered nurse is planning to delegate
a decreased in symptoms, not the drug itself. task to a certified nursing assistant. Which of the
following clients should not be assigned to a
CAN?
SET 2

A. A client diagnosed with diabetes and


who has an infected toe
PNLE I Nursing Practice B. A client who had a CVA in the past two
months
Scope of this Nursing Test I is parallel to the NP1 C. A client with Chronic renal failure
NLE Coverage: D. A client with chronic venous insufficiency
 Foundation of Nursing
 Nursing Research Answer: A. The patient is experiencing a
 Professional Adjustment potentially serious complication related to
 Leadership and Management diabetes and needs ongoing assessment by an
1. The registered nurse is planning to delegate RN
tasks to unlicensed assistive personnel (UAP).
Which of the following task could the registered 5. The nurse in the medication unit passes the
nurse safely assigned to a UAP? medications for all the clients on the nursing unit.
The head nurse is making rounds with the
physician and coordinates clients’ activities with
A. Monitor the I&O of a comatose toddler client
other departments. The nurse assistant changes
with salicylate poisoning
the bed lines and answers call lights. A second
B. Perform a complete bed bath on a 2-year-old
nurse is assigned for changing wound dressings;
with multiple injuries from a serious fall
a licensed practitioner nurse takes vital signs and
C. Check the IV of a preschooler with Kawasaki
bathes theclients. This illustrates of what method
disease
of nursing care?
D. Give an outmeal bath to an infant with
eczema
A. Case management method
Answer: D. Bathing an infant with eczema can be B. Primary nursing method
safely delegated to an aide; this task is basic C. Team method
and can competently performed by an aid. D. Functional method

2. A nurse manager assigned a registered nurse Answer: D. It describes functional nursing. Staff
from telemetry unit to the pediatrics unit. There is assigned to specific task rather than specific
were three patients assigned to the RN. Which of clients.
the following patients should not be assigned to
the floated nurse? 6. A registered nurse has been assigned to six
clients on the 12-hour shift. The RN is responsible
for every aspect of care such as formulating the
A. A 9-year-old child diagnosed with rheumatic
care of plan, intervention and evaluating the care
fever
during her shift. At the end of her shift, the RN
B. A young infant after pyloromyotomy
will pass this same task to the next RN in charge.
C. A 4-year-old with VSD following cardiac
This nursing care illustrates of what kind of
catheterization
method?
D. A 5-month-old with Kawasaki disease

Answer: B. The RN floated from the telemetry A. primary nursing method


unit would be least prepared to care for a young B. case method
infant who has just had GI surgery and requires C. team method
a specific feeding regimen. D. functional method

3. A nurse in charge in the pediatric unit is Answer: B. Case management. The nurse
absent. The nurse manager decided to assign the assumes total responsibility for meeting the
nurse in the obstetrics unit to the pediatrics unit. needs of the client during her entire duty.
Which of the following patients could the nurse
manager safely assign to the float nurse? 7. A newly hired nurse on an adult medicine unit
with 3 months experience was asked to float to
pediatrics. The nurse hesitates to perform
A. A child who had multiple injuries from a
pediatric skills and receive an interesting
serious vehicle accident
assignment that feels overwhelming. The nurse
B. A child diagnosed with Kawasaki disease and
should:
with cardiac complications
C. A child who has had a nephrectomy for
Wilm’s tumor A. resign on the spot from the nursing position
D. A child receiving an IV chelating therapy for and apply for a position that does not require
lead poisoning floating
B. Inform the nursing supervisor and the prepared the consent form and it should be
charge nurse on the pediatric floor signed by:
about the nurse’s lack of skill and
feelings of hesitations and request A. The Physician
assistance B. The Registered Nurse caring for the client
C. Ask several other nurses how they feel about C. The 15-year-old mother of the baby boy
pediatrics and find someone else who is D. The mother of the girl
willing to accept the assignment
D. Refuse the assignment and leave the unit Answer: C. Even though the mother is a minor,
requesting a vacation a day she is legally able to sign consent for her own
child.
Answer: B. The nurse is ethically obligated to
inform the person responsible for the 12. A nurse caring to a client with Alzheimer’s
assignment and the person responsible for the disease overheard a family member say to the
unit about the nurse’s skill level. The nurse client, “if you pee one more time, I won’t give you
therefore avoids a situation of abandoningclients any more food and drinks”. What initial action is
and exposing them to greater risks best for the nurse to take?
8. An experienced nurse who voluntarily trained a
A. Take no action because it is the family
less experienced nurse with the intention of
enhancing the skills and knowledge and member saying that to the client
B. Talk to the family member and explain
promoting professional advancement to the nurse
is called a: that what she/he has said is not
appropriate for the client
C. Give the family member the number for an
A. mentor Elder Abuse Hot line
B. team leader D. Document what the family member has said
C. case manager
D. change agent Answer: B. This response is the most direct and
immediate. This is a case of potential need for
Answer: A. This describes a mentor advocacy and patient’s rights.
9. The pediatrics unit is understaffed and the 13. Which is true about informed consent?
nurse manager informs the nurses in the
obstetrics unit that she is going to assign one
A. A nurse may accept responsibility signing a
nurse to float in the pediatric units. Which
statement by the designated float nurse may put consent form if the client is unable
B. Obtaining consent is not the responsibility of
her job at risk?
the physician
C. A physician will not subject himself to liability
A. “I do not get along with one of the nurses on if he withholds any facts that are necessary
the pediatrics unit” to form the basis of an intelligent consent
B. “I have a vacation day coming and D. If the nurse witnesses a consent for
would like to take that now” surgery, the nurse is, in effect,
C. “I do not feel competent to go and work on indicating that the signature is that of
that area” the purported person and that the
D. “ I am afraid I will get the most serious person’s condition is as indicated at the
clients in the unit” time of signing

Answer:B. This action demonstrates a lack of Answer: D. The nurse who witness a consent for
responsibility and the nurse should attempt treatment or surgery is witnessing only that the
negotiation with the nurse manager client signed the form and that the client’s
condition is as indicated at the time of signing.
10. The newly hired staff nurse has been working The nurse is not witnessing that the client is
on a medical unit for 3 weeks. The nurse “informed”.
manager has posted the team leader
assignments for the following week. The new staff 14. A mother in labor told the nurse that she was
knows that a major responsibility of the team expecting that her baby has no chance to survive
leader is to: and expects that the baby will be born dead. The
mother accepts the fate of the baby and informs
A. Provide care to the most acutely ill client on the nurse that when the baby is born and requires
the team resuscitation, the mother refuses any treatment
B. Know the condition and needs of all the to her baby and expresses hostility toward the
patients on the team nurse while the pediatric team is taking care of
C. Document the assessments completed by the baby. The nurse is legally obligated to:
the team members
D. Supervise direct care by nursing assistants A. Notify the pediatric team that the mother has
refused resuscitation and any treatment for
Answer: B. The team leader is responsible for the the baby and take the baby to the mother
overall management of all clients and staff on B. Get a court order making the baby a ward of
the team, and this information is essential in the court
order to accomplish this C. Record the statement of the mother,
notify the pediatric team, and observe
11. A 15-year-old girl just gave birth to a baby carefully for signs of impaired bonding
boy who needs emergency surgery. The nurse
and neglect as a reasonable suspicion 18. A staff nurse has had a serious issue with her
of child abuse colleague. In this situation, it is best to:
D. Do nothing except record the mother’s
statement in the medical record A. Discuss this with the supervisor
B. Not discuss the issue with anyone. It will
Answer: C. Although the statements by the probably resolve itself
mother may not create a suspicion of neglect, C. Try to discuss with the colleague about
when they are coupled with observations about the issue and resolve it when both are
impaired bonding and maternal attachment, calmer
they may impose the obligation to report child D. Tell other members of the network what the
neglect. The nurse is further obligated to notify team member did
caregivers of refusal to consent to treatment
Answer: C. Waiting for emotions to dissipate and
15. The hospitalized client with a chronic cough is sitting down with the colleague is the first rule of
scheduled for bronchoscopy. The nurse is tasks to conflict resolution.
bring the informed consent document into the
client’s room for a signature. The client asks the 19. The nurse is caring to a client who just gave
nurse for details of the procedure and demands birth to a healthy baby boy. The nurse may not
an explanation why the process of informed disclose confidential information when:
consent is necessary. The nurse responds that
informed consent means: A. The nurse discusses the condition of the
client in a clinical conference with other
A. The patient releases the physician from all nurses
responsibility for the procedure. B. The client asks the nurse to discuss the her
B. The immediate family may make decision condition with the family
against the patient’s will. C. The father of a woman who just
C. The physician must give the client or delivered a baby is on the phone to find
surrogates enough information to make out the sex of the baby
health care judgments consistent with D. A researcher from an institutionally approved
their values and goals. research study reviews the medical record of
D. The patient agrees to a procedure ordered by a patient
the physician even if the client does not
understand what the outcome will be. Answer: C. The nurse has no idea who the
person is on the phone and therefore may not
1. Answer: C. It best explains what informed share the information even if the patient gives
consent is and provides for legal rights of the permission
patient
20. A 17-year-old married client is scheduled for
16. A hospitalized client with severe necrotizing surgery. The nurse taking care of the client
ulcer of the lower leg is schedule for an realizes that consent has not been signed after
amputation. The client tells the nurse that he will preoperative medications were given. What
not sign the consent form and he does not want should the nurse do?
any surgery or treatment because of religious
beliefs about reincarnation. What is the role of A. Call the surgeon
the RN? B. Ask the spouse to sign the consent
C. Obtain a consent from the client as soon as
A. call a family meeting possible
B. discuss the religious beliefs with the D. Get a verbal consent from the parents of the
physician client
C. encourage the client to have the surgery
D. inform the client of other options Answer: A. The priority is to let the surgeon
know, who in turn may ask the husband to sign
Answer: B. The physician may not be aware of the consent
the role that religious beliefs play in making a
decision about surgery. 21. A 12-year-old client is admitted to the
hospital. The physician ordered Dilantin to the
17. While in the hospital lobby, the RN overhears client. In administering IV phenytoin (Dilantin) to
the three staff discussing the health condition of a child, the nurse would be most correct in mixing
her client. What would be the appropriate nursing it with:
action for the RN to take?
A. Normal Saline
A. Tell them it is not appropriate to discuss B. Heparinized normal saline
the condition of the client C. 5% dextrose in water
B. Ignore them, because it is their right to D. Lactated Ringer’s solution
discuss anything they want to
C. Join in the conversation, giving them Answer: A. Phenytoin (Dilantin) can cause
supportive input about the case of the client venous irritation due to its alkalinity, therefore it
D. Report this incident to the nursing supervisor should be mixed with normal saline.

Answer: A. The behavior should be stopped. The 22. The nurse is caring to a client who is
first step is to remind the staff that hypotensive. Following a large hematemesis, how
confidentiality may be violated should the nurse position the client?
A. Feet and legs elevated 20 degrees, 27. A client admitted to the hospital and
trunk horizontal, head on small pillow diagnosed with Addison’s disease. What would be
B. Low Fowler’s with knees gatched at 30 the appropriate nursing action to the client?
degrees
C. Supine with the head turned to the left A. administering insulin-replacement therapy
D. Bed sloped at a 45 degree angle with the B. providing a low-sodium diet
head lowest and the legs highest C. restricting fluids to 1500 ml/day
D. reducing physical and emotional stress
Answer: A. This position increases venous return,
improves cardiac volume, and promotes Answer: D. Because the client’s ability to react
adequate ventilation and cerebral perfusion to stress is decreased, maintaining a quiet
environment becomes a nursing priority.
23. The client is brought to the emergency Dehydration is a common problem in Addison’s
department after a serious accident. What would disease, so close observation of the client’s
be the initial nursing action of the nurse to the hydration level is crucial.
client?
28. The nurse is to perform tracheal suctioning.
A. assess the level of consciousness and During tracheal suctioning, which nursing action
circulation is essential to prevent hypoxemia?
B. check respirations, circulation, neurological
response A. aucultating the lungs to determine the
C. align the spine, check pupils, check for baseline data to assess the effectiveness of
hemorrhage suctioning
D. check respiration, stabilize spine, check B. removing oral and nasal secretions
circulation C. encouraging the patient to deep breathe and
cough to facilitate removal of upper-airway
Answer: D. Checking the airway would be a secretions
priority, and a neck injury should be suspected D. administering 100% oxygen to reduce
the effects of airway obstruction during
24. A nurse is assigned to care to a client with suctioning.
Parkinson’s disease. What interventions are
important if the nurse wants to improve nutrition Answer: D. Presuctioning and postsuctioning
and promote effective swallowing of the client? ventilation with 100% oxygen is important in
reducing hypoxemia which occurs when the flow
A. Eat solid food of gases in the airway is obstructed by the
B. Give liquids with meals suctioning catheter.
C. Feed the client
D. Sit in an upright position to eat 29. An infant is admitted and diagnosed with
pneumonia and suspicious-looking red marks on
Answer: D. Client with Parkinson’s disease are at the swollen face resembling a handprint. The
a high risk for aspiration and undernutrition. nurse does further assessment to the client. How
Sitting upright promotes more effective would the nurse document the finding?
swallowing.
A. Facial edema with ecchymosis and handprint
25. During tracheal suctioning, the nurse should mark: crackles and wheezes
implement safety measures. Which of the B. Facial edema, with red marks; crackles
following should the nurse implements? in the lung
C. Facial edema with ecchymosis that looks like
A. limit suction pressure to 150-180 mmHg a handprint
B. suction for 15-20 seconds D. Red bruise mark and ecchymosis on face
C. wear eye goggles
D. remove the inner cannula Answer: B. This is an example of objective data
of both pulmonary status and direct observation
Answer: C. It is important to protect the RN’s on the skin by the nurse.
eyes from the possible contamination of
coughed-up secretions 30. On the evening shift, the triage nurse
evaluates several clients who were brought to the
26. The nurse is conducting a discharge emergency department. Which in the following
instructions to a client diagnosed with diabetes. clients should receive highest priority?
What sign of hypoglycemia should be taught to a
client? A. an elderly woman complaining of a loss of
appetite and fatigue for the past week
A. warm, flushed skin B. A football player limping and complaining of
B. hunger and thirst pain and swelling in the right ankle
C. increase urinary output C. A 50-year-old man, diaphoretic and
D. palpitation and weakness complaining of severe chest pain
radiating to his jaw
Answer: D. There has been too little food or too D. A mother with a 5-year-old boy who says her
much insulin. Glucose levels can be markedly son has been complaining of nausea and
decreased (less than 50 mg/dl). Severe vomited once since noon
hypoglycemia may be fatal if not detected
Answer: C. These are likely signs of an acute
myocardial infarction (MI). An acute MI is a
cardiovascular emergency requiring immediate
attention. Acute MI is potentially fatal if not 35. The physician instructed the nurse that
treated immediately. intravenous pyelogram will be done to the client.
The client asks the nurse what is the purpose of
31. A 80-year-old female client is brought to the the procedure. The appropriate nursing response
emergency department by her caregiver, on the is to:
nurse’s assessment; the following are the
manifestations of the client: anorexia, cachexia A. outline the kidney vasculature
and multiple bruises. What would be the best B. determine the size, shape, and placement of
nursing intervention? the kidneys
C. test renal tubular function and the
A. check the laboratory data for serum albumin, patency of the urinary tract
hematocrit, and hemoglobin D. measure renal blood flow
B. talk to the client about the caregiver and
support system Answer: C. Intravenous pyelogram tests both the
C. complete a police report on elder abuse function and patency of the kidneys. After the
D. complete a gastrointestinal and intravenous injection of a radiopaque contrast
neurological assessment medium, the size, location, and patency of the
kidneys can be observed by roentgenogram, as
Answer: D. Assessment and more data collection well as the patency of the urethra and bladder
are needed. The client may have gastrointestinal as the kidneys function to excrete the contrast
or neurological problems that account for the medium.
symptoms. The anorexia could result from
medications, poor dentition, or indigestion, and 36. A client visits the clinic for screening of
the bruises may be attributed to ataxia, frequent scoliosis. The nurse should ask the client to:
falls, vertigo or medication.
A. bend all the way over and touch the toes
32. The night shift nurse is making rounds. When B. stand up as straight and tall as possible
the nurse enters a client’s room, the client is on C. bend over at a 90-degree angle from
the floor next to the bed. What would be the the waist
initial action of the nurse? D. bend over at a 45-degree angle from the
waist
A. chart that the patient fell
B. call the physician Answer: C. This is the recommended position for
C. chart that the client was found on the floor screening for scoliosis. It allows the nurse to
next to the bed inspect the alignment of the spine, as well as to
D. fill out an incident report compare both shoulders and both hips.

Answer: B. This is closest to suggesting action- 37. A client with tuberculosis is admitted in the
assessment, rather than paperwork- and is hospital for 2 weeks. When a client’s family
therefore the best of the four. members come to visit, they would be adhering
to respiratory isolation precautions when they:

33. The nurse on the night shift is about to A. wash their hands when leaving
administer medication to a preschooler client and B. put on gowns, gloves and masks
notes that the child has no ID bracelet. The best C. avoid contact with the client’s roommate
way for the nurse to identify the client is to ask: D. keep the client’s room door open

A. The adult visiting, “The child’s name is Answer: A. Handwashing is the best method for
____________________?” reducing cross-contamination. Gowns and gloves
B. The child, “Is your name____________?” are not always required when entering a client’s
C. Another staff nurse to identify this child room.
D. The other children in the room what the
child’s name is 38. An infant is brought to the emergency
department and diagnosed with pyloric stenosis.
Answer: C. The only acceptable way to identify a The parents of the client ask the nurse, “Why
preschooler client is to have a parent or another does my baby continue to vomit?” Which of the
staff member identify the client. following would be the best nursing response of
the nurse?
34. The nurse caring to a client has completed
the assessment. Which of the following will be A. “Your baby eats too rapidly and overfills the
considered to be the most accurate charting of a stomach, which causes vomiting
lump felt in the right breast? B. “Your baby can’t empty the formula that
is in the stomach into the bowel”
A. “abnormally felt area in the right breast, C. “The vomiting is due to the nausea that
drainage noted” accompanies pyloric stenosis”
B. “hard nodular mass in right breast nipple” D. “Your baby needs to be burped more
C. “firm mass at five ‘ clock, outer thoroughly after feeding”
quadrant, 1cm from right nipple’
D. “mass in the right breast 4cmx1cm Answer: B. Pyloric stenosis is an anomaly of the
upper gastrointestinal tract. The condition
Answer: C. It describes the mass in the greatest involves a thickening, or hypertrophy, of the
detail. pyloric sphincter located at the distal end of the
stomach. This causes a mechanical intestinal
obstruction, which leads to vomiting after Answer:D. The first priority, beside maintaining a
feeding the infant. The vomiting associated with newborn’s patent airway, is body temperature.
pyloric stenosis is described as being projectile
in nature. This is due to the increasing amounts 43. A 2-year-old client is admitted to the hospital
of formula the infant begins to consume coupled with severe eczema lesions on the scalp, face,
with the increasing thickening of the pyloric neck and arms. The client is scratching the
sphincter. affected areas. What would be the best nursing
intervention to prevent the client from scratching
39. A 70-year-old client with suspected the affected areas?
tuberculosis is brought to the geriatric care
facilities. An intradermal tuberculosis test is A. elbow restraints to the arms
schedule to be done. The client asks the nurse B. Mittens to the hands
what is the purpose of the test. Which of the C. Clove-hitch restraints to the hands
following would be the best rationale for this? D. A posey jacket to the torso

A. reactivation of an old tuberculosis infection Answer: B. The purpose of restraints for this
B. increased incidence of new cases of child is to keep the child from scratching the
tuberculosis in persons over 65 years affected areas. Mittens restraint would prevent
old scratching, while allowing the most movement
C. greater exposure to diverse health care permissible.
workers
D. respiratory problems are characteristic in this 44. The parents of the hospitalized client ask the
population nurse how their baby might have gotten pyloric
stenosis. The appropriate nursing response would
Answer: B. Increased incidence of TB has been be:
seen in the general population with a high
incidence reported in hospitalized elderly clients. A. There is no way to determine this
Immunosuppression and lack of classic preoperatively
manifestations because of the aging process are B. Their baby was born with this condition
just two of the contributing factors of C. Their baby developed this condition
tuberculosis in the elderly. during the first few weeks of life
D. Their baby acquired it due to a formula
40. The nurse is making a health teaching to the allergy
parents of the client. In teaching parents how to
measure the area of induration in response to a Answer: C. Pyloric stenosis is not a congenital
PPD test, the nurse would be most accurate in anatomical defect, but the precise etiology is
advising the parents to measure: unknown. It develops during the first few weeks
of life.
A. both the areas that look red and feel raised
B. The entire area that feels itchy to the child 45. A male client comes to the clinic for check-up.
C. Only the area that looks reddened In doing a physical assessment, the nurse should
D. Only the area that feels raised report to the physician the most common
symptom of gonorrhea, which is:
Answer: D. Parents should be taught to feel the
area that is raised and measure only that A. pruritus
B. pus in the urine
41. A community health nurse is schedule to do C. WBC in the urine
home visit. She visits to an elderly person living D. Dysuria
alone. Which of the following observation would
be a concern? Answer: B. Pus is usually the first symptom,
because the bacteria reproduce in the bladder.
A. Picture windows
B. Unwashed dishes in the sink 46. Which of the following would be the most
C. Clear and shiny floors important goal in the nursing care of an infant
D. Brightly lit rooms client with eczema?

Answer: C. It is a safety hazard to have shiny A. preventing infection


floors because they can cause falls. B. maintaining the comfort level
C. providing for adequate nutrition
42. After a birth, the physician cut the cord of the D. decreasing the itching
baby, and before the baby is given to the mother,
what would be the initial nursing action of the Answer:A. Preventing infection in the infant with
nurse? eczema is the nurse’s most important goal. The
infant with eczema is at high risk for infection
A. examine the infant for any observable due to numerous breaks in the skin’s integrity.
abnormalities Intact skin is always the infant’s first line of
B. confirm identification of the infant and apply defense against infection.
bracelet to mother and infant
C. instill prophylactic medication in the infant’s 47. The nurse is making a discharge instruction to
eyes a client receiving chemotherapy. The client is at
D. wrap the infant in a prewarmed blanket risk for bone marrow depression. The nurse gives
and cover the head instructions to the client about how to prevent
infection at home. Which of the following health  Integrated Management of Childhood Illness
teaching would be included? 1. The student nurse is assigned to take the vital signs of the
clients in the pediatric ward. The student nurse reports to the
A. “Get a weekly WBC count” staff nurse that the parent of a toddler who is 2 days
B. “Do not share a bathroom with children or postoperative after a cleft palate repair has given the toddler
pregnant woman” a pacifier. What would be the best immediate action of the
C. “Avoid contact with others while receiving nurse?
chemotherapy”
D. “Do frequent hand washing and A. Notify the pediatrician of this finding
maintain good hygiene” B. Reassure the student that this is an acceptable action on
the parent’s part
Answer: D. Frequent hand washing and good
C. Discuss this action with the parents
hygiene are the best means of preventing
D. Ask the student nurse to remove the pacifier from the
infection.
toddler’s mouth
48. The nurse is assigned to care the client with
infectious disease. The best antimicrobial agent Answer: C. Nothing must be placed in the mouth of a toddler
for the nurse to use in handwashing is: who just undergone a cleft palate repair until the suture line
has completely healed. It is the nurse’s responsibility to
inform the parent of the client. Spoon, forks, straws, and
A. Isopropyl alcohol
B. Hexachlorophene (Phisohex) tongue blades are other unacceptable items to place in the
C. Soap and water mouth of a toddler who just undergone cleft palate repair.
D. Chlorhexidine gluconate (CHG) The general principle of care is that nothing should enter the
(Hibiclens) mouth until the suture line has completely healed.

Answer: D. CHG is a highly effective 2. The nurse is providing a health teaching to the mother of
antimicrobial ingredient, especially when it is an 8-year-old child with cystic fibrosis. Which of the following
used consistently over time. statement if made by the mother would indicate to the nurse
the need for further teaching about the medication regimen
49. The mother of the client tells the nurse, “ I’m of the child?
not going to have my baby get any
immunization”. What would be the best nursing A. “My child might need an extra capsule if the meal is
response to the mother? high in fat”
B. “I’ll give the enzyme capsule before every snack”
A. “You and I need to review your C. “I’ll give the enzyme capsule before every meal”
rationale for this decision” D. “My child hates to take pills, so I’ll mix the capsule into
B. “Your baby will not be able to attend day a cup of hot chocolate
care without immunizations”
C. “Your decision can be viewed as a form of Answer: D. The pancreatic capsules contain pancreatic
child abuse and neglect”
enzyme that should be administered in a cold, not a hot,
D. “You are needlessly placing other people at
medium (example: chilled applesauce versus hot chocolate)
risk for communicable diseases”
to maintain the medication’s integrity.
Answer: A. The mother may have many reasons
for such a decision. It is the nurse’s 3. The mother brought her child to the clinic for follow-up
responsibility to review this decision with the check up. The mother tells the nurse that 14 days after
mother and clarify any misconceptions regarding starting an oral iron supplement, her child’s stools are black.
immunizations that may exist. Which of the following is the best nursing response to the
mother?
50. The nurse is teaching the client about breast
self-examination. Which observation should the A. “I will notify the physician, who will probably decrease
client be taught to recognize when doing the the dosage slightly”
examination for detection of breast cancer? B. “This is a normal side effect and means the medication
is working”
A. tender, movable lump C. “You sound quite concerned. Would you like to talk
B. pain on breast self-examination about this further?”
C. round, well-defined lump D. “I will need a specimen to check the stool for possible
D. dimpling of the breast tissue bleeding”

Answer: D. The tumor infiltrates nearby tissue, it Answer: B. When oral iron preparations are given correctly,
can cause retraction of the overlying skin and the stools normally turn dark green or black. Parents of
create a dimpling appearance. children receiving this medication should be advised that
this side effect indicates the medication is being absorbed
PNLE II Nursing Practice and is working well.

4. An 8-year-old boy with asthma is brought to the clinic for


check up. The mother asks the nurse if the treatment given to
The scope of this Nursing Test II is parallel to the NP2 NLE
her son is effective. What would be the appropriate response
Coverage:
of the nurse?
 Maternal and Child Health
 Community Health Nursing
 Communicable Diseases
A. I will review first the child’s height on a growth chart to Answer: C. In selecting the correct needle to administer an
know if the treatment is working IM injection to a preschooler, the nurse should always look
B. I will review first the child’s weight on a growth chart to at the child and use judgment in evaluating muscle mass and
know if the treatment is working amount of subcutaneous fat. In this case, in the absence of
C. I will review first the number of prescriptions refills the further data, the nurse would be most correct in selecting a
child has required over the last 6 months to give you needle gauge and length appropriate for the “average’
an accurate answer preschool child. A medium-gauge needle (21G) that is 1 inch
D. I will review first the number of times the child has seen long would be most appropriate.
the pediatrician during the last 6 months to give you an
accurate answer 9. A 9-year-old boy is admitted to the hospital. The boy is
being treated with salicylates for the migratory polyarthritis
Answer: C. Reviewing the number of prescription refills the accompanying the diagnosis of rheumatic fever. Which of the
child has required over the last 6 months would be the best following activities performed by the child would give a best
indicator of how well controlled and thus how effective the sign that the medication is effective?
child’s asthma treatment is. Breakthrough wheezing,
shortness of breath, and upper respiratory infections would A. Listening to story of his mother
require that the child take additional medication. This would B. Listening to the music in the radio
be reflected in the number of prescription refills. C. Playing mini piano
D. Watching movie in the dvd mini player
5. The nurse is caring to a child client who is receiving
tetracycline. The nurse is aware that in taking this medication, Answer: C. The purpose of the salicylate therapy is to relieve
it is very important to: the pain associated with the migratory polyarthritis
accompanying the rheumatic fever. Playing mini piano would
A. Administer the drug between meals require movement of the child’s joints and would provide
B. Monitor the child’s hearing the nurse with a means of evaluating the child’s level of pain.
C. Give the drug through a straw
D. Keep the child out of the sunlight 10. The physician decided to schedule the 4-year-old client for
repair of left undescended testicle. The Injection of a
Answer: D. Tetracycline may cause a phototoxic reaction. hormone, HCG finds it less successful for treatment. To
administer a pentobarbital sodium (Nembutal) suppository
6. A 14 day-old infant with a cyanotic heart defects and mild preoperatively to this client, in which position should the
congestive heart failure is brought to the emergency nurse place him?
department. During assessment, the nurse checks the apical
pulse rate of the infant. The apical pulse rate is 130 beats per A. Supine with foot of bed elevated
minute. Which of the following is the appropriate nursing B. Prone with legs abducted
action? C. Sitting with foot of bed elevated
D. Side-lying with upper leg flexed
A. Retake the apical pulse in 15 minutes
B. Retake the apical pulse in 30 minutes Answer: D. The recommended position to administer rectal
C. Notify the pediatrician immediately medications to children is side-lying with the upper leg
D. Administer the medication as scheduled flexed. This position allows the nurse to safely and effectively
administer the medication while promoting comfort for the
Answer: D. The normal heart rate of an infant is 120-160 child.
beats per minute
11. The nurse is caring to a 24-month-old child diagnosed
7. The physician prescribed gentamicin (Garamycin) to a child with congenital heart defect. The physician prescribed digoxin
who is also receiving chemotherapy. Before administering the (Lanoxin) to the client. Before the administration of the drug,
drug, the nurse should check the results of the child’s: the nurse checks the apical pulse rate to be 110 beats per
minute and regular. What would be the next nursing action?
A. CBC and platelet count
B. Auditory tests A. Check the other vital signs and level of consciousness
C. Renal Function tests B. Withhold the digoxin and notify the physician
D. Abdominal and chest x-rays C. Give the digoxin as prescribed
D. Check the apical and radial simultaneously, and if they
Answer: C. Both gentamicin and chemotherapeutic agents are the same, give the digoxin.
can cause renal impairment and acute renal failure; thus
baseline renal function must be evaluated before initiating Answer:C. For a 12month-old child, 110 apical pulse rate is
either medication. normal and therefore it is safe to give the digoxin. A
toddler’s normal pulse rate is slightly lower than an infant’s
8. Which of the following is the suited size of the needle (120).
would the nurse select to administer the IM injection to a
preschool child? 12. An 8-year-old client with cystic fibrosis is admitted to the
hospital and will undergo a chest physiotherapy treatment.
A. 18 G, 1-1/2 inch The therapy should be properly coordinated by the nurse with
B. 25 G, 5/8 inch the respiratory therapy department so that treatments occur
C. 21 G, 1 inch during:
D. 18 G, 1inch
A. After meals
B. Between meals
C. After medication B. The older daughter not have interaction with the baby
D. Around the child’s play schedule at the hospital, because she may harm her new sibling
C. The older daughter stay with her grandmother for a few
Answer: B. Chest physiotherapy treatments are scheduled days until the parents and new baby are settled at home
between meals to prevent aspiration of stomach contents, D. The mother spend time alone with her older daughter
because the child is placed in a variety of positions during when the baby is sleeping
the treatment process.
Answer: D. The introduction of a baby into a family with one
13. The nurse is providing health teaching about the or more children challenges parent to promote acceptance
breastfeeding and family planning to the client who gave birth of the baby by siblings. The parent’s attitudes toward the
to a healthy baby girl. Which of the following statement arrival of the baby can set the stage for the other children’s
would alert the nurse that the client needs further teaching? reaction. Spending time with the older siblings alone will
also reassure them of their place in the family, even though
A. “I understand that the hormones for breastfeeding may the older children will have to eventually assume new
affect when my periods come” positions within the family hierarchy.
B. “Breastfeeding causes my womb to tighten and bleed
less after birth” 17. A 2-year-old client with cystic fibrosis is confined to bed
C. “I may not have periods while I am breastfeeding, so I and is not allowed to go to the playroom. Which of the
don’t need family planning” following is an appropriate toy would the nurse select for the
D. “I can get pregnant as early as one month after my baby child:
was born”
A. Puzzle
Answer: C. It is common misconception that breastfeeding B. Musical automobile
may prevent pregnancy. C. Arranging stickers in the album
D. Pounding board and hammer
14. A toddler is brought to the hospital because of severe
diarrhea and vomiting. The nurse assigned to the client enters Answer: D. The autonomous toddler would be frustrated by
the client’s room and finds out that the client is using a soiled being confined to be. The pounding board and hammer is
blanket brought in from home. The nurse attempts to remove developmentally appropriate and an excellent way for the
the blanket and replace it with a new and clean blanket. The toddler to release frustration.
toddler refuses to give the soiled blanket. The nurse realizes
that the best explanation for the toddler’s behavior is: 18. Which of the following clients is at high risk for
developmental problem?
A. The toddler did not bond well with the maternal figure
B. The blanket is an important transitional object A. A toddler with acute Glomerulonephritis on
C. The toddler is anxious about the hospital experience antihypertensive and antibiotics
D. The toddler is resistive to nursing interventions B. A 5-year-old with asthma on cromolyn sodium
C. A preschooler with tonsillitis
Answer: B. The “security blanket” is an important transitional D. A 2 1/2 –year old boy with cystic fibrosis
object for the toddler. It provides a feeling of comfort and
safety when the maternal figure is not present or when in a Answer: D. It is the developmental task of an 18-month-old
new situation for which the toddler was not prepared. toddler to explore and learn about the environment. The
Virtually any object (stuffed animal, doll, book etc) can respiratory complications associated with cystic fibrosis
become a security blanket for the toddler (which are present in almost all children with cystic fibrosis)
could prevent this development task from occurring.
15. The nurse has knowledge about the developmental task
of the child. In caring a 3-year-old-client, the nurse knows that 19. Which of the following would be the best divesionary
the suited developmental task of this child is to: activity for the nurse to select for a 2 weeks hospitalized 3-
year-old girl?
A. Learn to play with other children
B. Able to trust others A. Crayons and coloring books
C. Express all needs through speaking B. doll
D. Explore and manipulate the environment C. xylophone toy
D. Puzzles
Answer: D. Toddlers need to meet the developmental
milestone of autonomy versus shame and doubt. In order to Answer: C. The best diversion for a hospitalized child aged 2-
accomplish this, the toddler must be able to explore and 3 years old would be anything that makes noise or makes a
manipulate the environment. mess; xylophone which certainly makes noise or music
would be the best choice.
16. A mother who gave birth to her second daughter is so
concerned about her 2-year old daughter. She tells the nurse, 20. A nurse is providing safety instructions to the parents of
“I am afraid that my 2-year-old daughter may not accept her the 11-month-old child. Which of the following will the nurse
newly born sister”. It is appropriate to the nurse to response includes in the instructions?
that:
A. Plugging all electrical outlets in the house
A. The older daughter be given more responsibility and B. Installing a gate at the top and bottom of any stairs in
assure her “that she is a big girl now, and doesn’t need the home
Mommy as much” C. Purchasing an infant car seat as soon as possible
D. Begin to teach the child not to place small objects in the D. The adolescent has found a way to act out hostility
mouth toward the parent

Answer: B. An 11-month-old child stands alone and can walk Answer: B. Adolescents do feel indestructible, and this is
holding onto people or objects. Therefore the installation of a reflected in many risk-taking behaviors.
gate at the top and bottom of any stairs in the house is crucial
for the child’s safety 25. An 8-month-old infant is admitted to the hospital due to
diarrhea. The nurse caring for the client tells the mother to
21. An 8-year-old girl is in second grade and the parents stay beside the infant while making assessment. Which of the
decided to enroll her to a new school. While the child is following developmental milestones the infant has reached?
focusing on adjusting to new environment and peers, her
grades suffer. The child’s father severely punishes the child A. Has a three-word vocabulary
and forces her daughter to study after school. The father does B. Interacts with other infants
not allow also her daughter to play with other children. These C. Stands alone
data indicate to the nurse that this child is deprived of D. Recognizes but is fearful of strangers
forming which normal phase of development?
Answer: D. An 8-month-old infant both recognizes and is
A. Heterosexual relationships fearful of strangers. This developmental milestone is known
B. A love relationship with the father as “stranger anxiety”.
C. A dependency relationship with the father
D. Close relationship with peers 26. The community nurse is conducting a health teaching in
the group of married women. When teaching a woman about
Answer: D. In second grade a child needs to form a close fertility awareness, the nurse should emphasize that the basal
relationships with peers. body temperature:

22. A 5-year-old boy client is scheduled for hernia surgery. The A. Should be recorded each morning before any activity
nurse is preparing to do preoperative teaching with the child. B. Is the average temperature taken each morning
The nurse should knows that the 5-year-old would: C. Can be done with a mercury thermometer but not a
digital one
A. Expect a simple yet logical explanation regarding the D. Has a lower degree of accuracy in predicting ovulation
surgery than the cervical mucus test
B. Asks many questions regarding the condition and the
procedure Answer: A. The basal body temperature (BBT) is the lowest
C. Worry over the impending surgery body temperature of a healthy person that is taken
D. Be uninterested in the upcoming surgery immediately after waking and before getting out of bed. The
BBT usually varies from 36.2 – 36.3 degree Celsius during
Answer: B. A 5-year-old is highly concerned with body menses and for about 5-7 days afterward. About the time of
integrity. The preschool-age child normally asks many ovulation, a slight drop approximately 0.05 degree Celsius in
questions and in a situation such as this, could be expected to temperature may be seen; after ovulation, in concert with
ask even more the increasing progesterone levels of the early luteal phase,
the BBT rises 0.2-0.4 degree Celsius. This elevation remains
23. The nine-year-old client is admitted in the hospital for until 2-3 days before menstruation, or if pregnancy has
almost 1 week and is on bed rest. The child complains of occurred.
being bored and it seems tiresome to stay on bed and doing
nothing. What activity selected by the nurse would the child 27. The community nurse is providing an instruction to the
most likely find stimulating? clients in the health center about the use of diaphragm for
family planning. To evaluate the understanding of the woman,
A. Watching a video the nurse asks her to demonstrate the use of the diaphragm.
B. Putting together a puzzle Which of following statement indicates a need for further
C. Assembling handouts with the nurse for an upcoming health teaching?
staff development meeting
D. Listening to a compact disc A. “I should check the diaphragm carefully for holes every
time I use it.”
Answer: C. A 9-year-old enjoys working and feeling a sense B. “The diaphragm must be left in place for at least 6 hours
of accomplishment. The school-age child also enjoys after intercourse.”
“showing off,” and doing something with the nurse on the C. “I really need to use the diaphragm and jelly most
pediatric unit would allow this. This activity also provides the during the middle of my menstrual cycle
school-age child a needed opportunity to interact with D. “I may need a different size diaphragm if I gain or lose
others in the absence of school and personal friends. more than 20 pounds”

24. The parent of a 16-year-old boy tells the nurse that his son Answer: C. The woman must understand that, although the
is driving a motorbike very fast and with one hand. “It is “fertile” period is approximately midcycle, hormonal
making me crazy!” What would be the best explanation of the variations do occur and can result in early or late ovulations.
nurse to the behavior of the boy? To be effective, the diaphragm should be inserted before
every intercourse.
A. The adolescent might have an unconscious death wish
B. The adolescent feels indestructible 28. The client visits the clinic for prenatal check-up. While
C. The adolescent lacks life experience to realize how waiting for the physician, the nurse decided to conduct health
dangerous the behavior is teaching to the client. The nurse informed the client that
primigravida mother should go to the hospital when which 32. The nurse in the health center is providing immunization
patter is evident? to the children. The nurse is carefully assessing the condition
of the children before giving the vaccines. Which of the
A. Contractions are 2-3 minutes apart, lasting 90 seconds, following would the nurse note to withhold the infant’s
and membranes have ruptured scheduled immunizations?
B. Contractions are 5-10 minutes apart, lasting 30 seconds,
and are felt as strong menstrual cramps A. a dry cough
C. Contractions are 3-5 minutes apart, accompanied by B. a skin rash
rectal pressure and bloody show C. a low-grade fever
D. Contractions are 5 minutes apart, lasting 60 seconds, D. a runny nose
and increasing in intensity
Answer: B. A skin rash could indicate a concurrent infectious
Answer: D. Although instructions vary among birth centers, disease process in the infant. The scheduled immunizations
primigravidas should seek care when regular contractions should be withheld until the status of the infant’s health can
are felt about 5 minutes apart, becoming longer and be determined. Fevers above 38.5 degrees Celsius, alteration
stronger. in skin integrity, and infectious-appearing secretions are
indications to withhold immunizations.
29. A nurse is planning a home visit program to a new mother
who is 2 weeks postpartum and breastfeeding, the nurse 33. A mother brought her child in the health center for
includes in her health teaching about the resumption of hepatitis B vaccination in a series. The mother informs the
fertility, contraception and sexual activity. Which of the nurse that the child missed an appointment last month to
following statement indicates that the mother has understood have the third hepatitis B vaccination. Which of the following
the teaching? statements is the appropriate nursing response to the
mother?
A. “Because breastfeeding speeds the healing process after
birth, I can have sex right away and not worry about A. “I will examine the child for symptoms of hepatitis B”
infection” B. “Your child will start the series again”
B. “Because I am breastfeeding and my hormones are C. “Your child will get the next dose as soon as possible”
decreased, I may need to use a vaginal lubricant when I D. “Your child will have a hepatitis titer done to determine
have sex” if immunization has taken place.”
C. “After birth, you have to have a period before you can
get pregnant again’ Answer: C. Continuity is essential to promote active
D. “Breastfeeding protects me from pregnancy because it immunity and give hepatitis B lifelong prophylaxis. Optimally,
keeps my hormones down, so I don’t need any the third vaccination is given 6 months after the first.
contraception until I stop breastfeeding”
34. The community health nurse implemented a new program
Answer: B. Prolactin suppresses estrogen, which is needed about effective breast cancer screening technique for the
to stimulate vaginal lubrication during arousal. female personnel of the health department of Valenzuela.
Which of the following technique should the nurse consider
30. A community nurse enters the home of the client for to be of the lowest priority?
follow-up visit. Which of the following is the most appropriate
area to place the nursing bag of the nurse when conducting a A. Yearly breast exam by a trained professional
home visit? B. Detailed health history to identify women at risk
C. Screening mammogram every year for women over age
A. cushioned footstool 50
B. bedside wood table D. Screening mammogram every 1-2 years for women over
C. kitchen countertop age of 40.
D. living room sofa
Answer: B. Because of the high incidence of breast cancer, all
Answer: B. A wood surface provides the least chance for women are considered to be at risk regardless of health
organisms to be present. history.

31. The nurse in the health center is making an assessment to 35. Which of the following technique is considered an aseptic
the infant client. The nurse notes some rashes and small fluid- practice during the home visit of the community health
filled bumps in the skin. The nurse suspects that the infant nurse?
has eczema. Which of the following is the most important
nursing goal: A. Wrapping used dressing in a plastic bag before placing
them in the nursing bag
A. Preventing infection B. Washing hands before removing equipment from the
B. Providing for adequate nutrition nursing bag
C. Decreasing the itching C. Using the client’s soap and cloth towel for hand washing
D. Maintaining the comfort level D. Placing the contaminated needles and syringes in a
labeled container inside the nursing bag
Answer: A. Preventing infection in the infant with eczema is
the nurse’s most important goal. The infant with eczema is at Answer: B. Handwashing is the best way to prevent the
high risk for infection due to numerous breaks in the skin’s spread of infection.
integrity. Intact skin is always the infant’s first line of defense
against infection.
36. The nurse is planning to conduct a home visit in a small become pregnant. The nurse knows that further information
community. Which of the following is the most important is necessary when the woman states:
factor when planning the best time for a home care visit?
A. “Spontaneous abortion may occur in one out of five
A. Purpose of the home visit women who are infected”
B. Preference of the patient’s family B. “Pulmonary TB may jeopardize my pregnancy”
C. Location of the patient’s home C. “I know that I may not be able to have close contact
D. Length of time of the visit will take with my baby until contagious is no longer a problem
D. “I can get pregnant after I have been free of TB for 6
Answer: A. The purpose of the visit takes priority. months”

37. The nurse assigned in the health center is counseling a 30- Answer: D. Intervention is needed when the woman thinks
year-old client requesting oral contraceptives. The client tells that she needs to wait only 6 months after being free of TB
the nurse that she has an active yeast infection that has before she can get pregnant. She needs to wait 1.5-2years
recurred several times in the past year. Which statement by after she is declared to be free of TB before she should
the nurse is inaccurate concerning health promotion actions attempt pregnancy.
to prevent recurring yeast infection?
41. The Department of Health is alarmed that almost 33
A. “During treatment for yeast, avoid vaginal intercourse million people suffer from food poisoning every year.
for one week” Salmonella enteritis is responsible for almost 4 million cases
B. “Wear loose-fitting cotton underwear” of food poisoning. One of the major goals is to promote
C. “Avoid eating large amounts of sugar or sugar-bingeing” proper food preparation. The community health nurse is tasks
D. “Douche once a day with a mild vinegar and water to conduct health teaching about the prevention of food
solution” poisoning to a group of mother everyday. The nurse can help
identify signs and symptoms of specific organisms to help
Answer: D. Frequent douching interferes with the natural patients get appropriate treatment. Typical symptoms of
protective barriers in the vagina that resist yeast infection salmonella include:
and should be avoided.
A. Nausea, vomiting and paralysis
38. During immunization week in the health center, the B. Bloody diarrhea
parent of a 6-month-old infant asks the health nurse, “Why is C. Diarrhea and abdominal cramps
our baby going to receive so many immunizations over a long D. Nausea, vomiting and headache
time period?” The best nursing response would be:
Answer: C. Salmonella organisms cause lower GI symptoms
A. “The number of immunizations your baby will receive
shows how many pediatric communicable and 42. A community health nurse makes a home visit to an
infectious diseases can now be prevented.” elderly person living alone in a small house. Which of the
B. “You need to ask the physician” following observation would be a great concern?
C. “The number of immunizations your baby will receive is
determined by your baby’s health history and age” A. Big mirror in a wall
D. “It is easier on your baby to receive several B. Scattered and unwashed dishes in the sink
immunizations rather than one at a time” C. Shiny floors with scattered rugs
D. Brightly lit rooms
Answer: A. Completion for the recommended schedule of
infant immunizations does not require a large number of Answer: C. It is a safety hazard to have shiny floors and
immunizations, but it also provides protection against scattered rugs because they can cause falls and rugs should
multiple pediatric communicable and infectious diseases. be removed.

39. The community health nurse is conducting a health 43. The health nurse is conducting health teaching about
teaching about nutrition to a group of pregnant women who “safe” sex to a group of high school students. Which of the
are anemic and are lactose intolerant. Which of the following following statement about the use of condoms should the
foods should the nurse especially encourage during the third nurse avoid making?
trimester?
A. “Condoms should be used because they can prevent
A. Cheese, yogurt, and fish for protein and calcium needs infection and because they may prevent pregnancy”
plus prenatal vitamins and iron supplements B. “Condoms should be used even if you have recently
B. Prenatal iron and calcium supplements plus a regular tested negative for HIV”
adult diet C. “Condoms should be used every time you have sex
C. Red beans, green leafy vegetables, and fish for iron because condoms prevent all forms of sexually
and calcium needs plus prenatal vitamins and iron transmitted diseases”
supplements D. “Condoms should be used every time you have sex even
D. Red meat, milk and eggs for iron and calcium needs plus if you are taking the pill because condoms can prevent
prenatal vitamins and iron supplements the spread of HIV and gonorrhea”

Answer: C. This is appropriate foods that are high in iron and Answer: C. Condoms do not prevent ALL forms of sexually
calcium but would not affect lactose intolerance. transmitted diseases

40. A woman with active tuberculosis (TB) and has visited the 44. The department of health is promoting the breastfeeding
health center for regular therapy for five months wants to program to all newly mothers. The nurse is formulating a plan
of care to a woman who gave birth to a baby girl. The nursing 48. A community health nurse makes a home visit to a child
care plan for a breast-feeding mother takes into account that with an infectious and communicable disease. In planning
breast-feeding is contraindicated when the woman: care for the child, the nurse must determine that the primary
goal is that the:
A. Is pregnant
B. Has genital herpes infection A. Child will experience mild discomfort
C. Develops mastitis B. Child will experience only minor complications
D. Has inverted nipples C. Child will not spread the infection to others
D. Public health department will be notified
Answer: A. Pregnancy is one contraindication to breast-
feeding. Milk secretion is inhibited and the baby’s sucking Answer: C. The primary goal is to prevent the spread of the
may stimulate uterine contractions. disease to others. The child should experience no
complication. Although the health department may need to
45. The City health department conducted a medical mission be notified at some point, it is no the primary goal. It is also
in Barangay Marulas. Majority of the children in the Barangay important to prevent discomfort as much as possible
Marulas were diagnosed with pinworms. The community
health nurse should anticipate that the children’s chief 49. The mother brings her daughter to the health care clinic.
complaint would be: The child was diagnosed with conjunctivitis. The nurse
provides health teaching to the mother about the proper care
A. Lack of appetite of her daughter while at home. Which statement by the
B. Severe itching of the scalp mother indicates a need for additional information?
C. Perianal itching
D. Severe abdominal pain A. “I do not need to be concerned about the spreading of
this infection to others in my family”
Answer: C. Perianal itching is the child’s chief complaint B. “I should apply warm compresses before instilling
associated with the diagnosis of pinworms. The itching, in this antibiotic drops if purulent discharge is present in my
instance, is often described as being “intense” in nature. daughter’s eye”
Pinworms infestation usually occurs because the child is in the C. “I can use an ophthalmic analgesic ointment at
anus-to-mouth stage of development (child uses the toilet, nighttime if I have eye discomfort”
does not wash hands, places hands and pinworm eggs in D. “I should perform a saline eye irrigation before instilling,
mouth). Teaching the child hand washing before eating and the antibiotic drops into my daughter’s eye if purulent
after using the toilet can assist in breaking the cycle discharge is present”

46. The mother brought her daughter to the health center. Answer: A. Conjunctivitis is highly contagious. Antibiotic
The child has head lice. The nurse anticipates that the nursing drops are usually administered four times a day. When
diagnosis most closely correlated with this is: purulent discharge is present, saline eye irrigations or eye
applications of warm compresses may be necessary before
A. Fluid volume deficit related to vomiting instilling the medication. Ophthalmic analgesic ointment or
B. Altered body image related to alopecia drops may be instilled, especially at bedtime, because
C. Altered comfort related to itching discomfort becomes more noticeable when the eyelids are
D. Diversional activity deficit related to hospitalization closed.

Answer: C. Severe itching of the scalp is the classic sign and 50. A community health nurse is caring for a group of flood
symptom of head lice in a child. In turn, this would lead to victims in Marikina area. In planning for the potential needs
the nursing diagnosis of “altered comfort”. of this group, which is the most immediate concern?

47. The mother brings a child to the health care clinic because A. Finding affordable housing for the group
of severe headache and vomiting. During the assessment of B. Peer support through structured groups
the health care nurse, the temperature of the child is 40 C. Setting up a 24-hour crisis center and hotline
degree Celsius, and the nurse notes the presence of nuchal D. Meeting the basic needs to ensure that adequate food,
rigidity. The nurse is suspecting that the child might be shelter and clothing are available
suffering from bacterial meningitis. The nurse continues to
assess the child for the presence of Kernig’s sign. Which Answer: D. The question asks about the immediate concern.
finding would indicate the presence of this sign? The ABCs of community health care are always attending to
people’s basic needs of food, shelter, and clothing
A. Flexion of the hips when the neck is flexed from a lying
position
B. Calf pain when the foot is dorsiflexed
C. Inability of the child to extend the legs fully when lying PNLE III Nursing Practice
supine The scope of this Nursing Test III is parallel to the NP3 NLE
D. Pain when the chin is pulled down to the chest Coverage:
 Medical Surgical Nursing
Answer: C. Kernig’s sign is the inability of the child to extend 1. The nurse is going to replace the Pleur-O-Vac attached to
the legs fully when lying supine. This sign is frequently the client with a small, persistent left upper lobe
present in bacterial meningitis. Nuchal rigidity is also present pneumothorax with a Heimlich Flutter Valve. Which of the
in bacterial meningitis and occurs when pain prevents the following is the best rationale for this?
child from touching the chin to the chest.
A. Promote air and pleural drainage
B. Prevent kinking of the tube
C. Eliminate the need for a dressing and swing-through crutch gaits may also be used when only
D. Eliminate the need for a water-seal drainage one leg can be used for weight bearing

Answer: D. The Heimlich flutter valve has a one-way valve 6. The client is transferred to the nursing care unit from the
that allows air and fluid to drain. Underwater seal drainage is operating room after a transurethral resection of the prostate.
not necessary. This can be connected to a drainage bag for The client is complaining of pain in the abdomen area. The
the patient’s mobility. The absence of a long drainage tubing nurse suspects of bladder spasms, which of the following is
and the presence of a one-way valve promote effective the best nursing action to minimize the pain felt by the client?
therapy
A. Advising the client not to urinate around catheter
2. The client with acute pancreatitis and fluid volume deficit is B. Intermittent catheter irrigation with saline
transferred from the ward to the ICU. Which of the following C. Giving prescribed narcotics every 4 hour
will alert the nurse? D. Repositioning catheter to relieve pressure

A. Decreased pain in the fetal position Answer: A. The client needs to be told before surgery that
B. Urine output of 35mL/hr the catheter causes the urge to void. Attempts to void
C. CVP of 12 mmHg around the catheter cause the bladder muscles to contract
D. Cardiac output of 5L/min and result in painful spasms.

Answer: C. C = the normal CVP is 0-8 mmHg. This value 7. A client is diagnosed with peptic ulcer. The nurse caring for
reflects hypervolemia. The right ventricular function of this the client expects the physician to order which diet?
client reflects fluid volume overload, and the physician
should be notified. A. NPO
B. Small feedings of bland food
3. The nurse in the morning shift is making rounds in the C. A regular diet given frequently in small amounts
ward. The nurse enters the client’s room and found the client D. Frequent feedings of clear liquids
in discomfort condition. The client complains of stiffness in
the joints. To reduce the early morning stiffness of the joints Answer; B. Bland feedings should be given in small amounts
of the client,the nurse can encourage the client to: on a frequent basis to neutralize the hydrochloric acid and to
prevent overload
A. Sleep with a hot pad
B. Take to aspirins before arising, and wait 15 minutes 8. The nurse is going to insert a Miller-Abbott tube to the
before attempting locomotion client. Before insertion of the tube, the balloon is tested for
C. Take a hot tub bath or shower in the morning patency and capacity and then deflated. Which of the
D. Put joints through passive ROM before trying to move following nursing measure will ease the insertion to the tube?
them actively
A. Positioning the client in Semi-Fowler’s position
Answer: C. A hot tub bath or shower in the morning helps B. Administering a sedative to reduce anxiety
many patients limber up and reduces the symptoms of early C. Chilling the tube before insertion
morning stiffness. Cold and ice packs are used to a lesser D. Warming the tube before insertion
degree, though some clients state that cold decreases
localized pain, particularly during acute attacks. Answer: C. Chilling the tube before insertion assists in
relieving some of the nasal discomfort. Water-soluble
4. The nurse is planning of care to a client with peptic ulcer lubricants along with viscous lidocaine (Xylocaine) may also
disease. To avoid the worsening condition of the client, the be used. It is usually only lightly lubricated before insertion
nurse should carefully plan the diet of the client. Which of the
following will be included in the diet regime of the client? 9. The physician ordered a low-sodium diet to the client.
Which of the following food will the nurse avoid to give to the
A. Eating mainly bland food and milk or dairy products client?
B. Reducing intake of high-fiber foods
C. Eating small, frequent meals and a bedtime snack A. Orange juice.
D. Eliminating intake of alcohol and coffee B. Whole milk.
C. Ginger ale.
Answer: D. These substances stimulate the production of D. Black coffee.
hydrochloric acid, which is detrimental in peptic ulcer
disease. Answer: B. Whole milk should be avoided to include in the
client’s diet because it has 120 mg of sodium in 8 0z of milk
5. The physician has given instruction to the nurse that the
client can be ambulated on crutches, with no weight bearing 10. Mr. Bean, a 70-year-old client is admitted in the hospital
on the affected limb. The nurse is aware that the appropriate for almost one month. The nurse understands that prolonged
crutch gait for the nurse to teach the client would be: immobilization could lead to decubitus ulcers. Which of the
following would be the least appropriate nursing intervention
A. Tripod gait in the prevention of decubitus?
B. Two-point gait
C. Four-point gait A. Giving backrubs with alcohol
D. Three-point gait B. Use of a bed cradle
C. Frequent assessment of the skin
Answer: D. The three-point gait is appropriate when weight D. Encouraging a high-protein diet
bearing is not allowed on the affected limb. The swing-to
Answer: A. Alcohol is extremely drying and contributes to Answer: A. A positive nitrogen balance is important for
skin break down. An emollient lotion should be used. meeting metabolic needs, tissue repair, and resistance to
infection. Caloric goals may be as high as 5000 calories per
11. The physician prescribed digoxin 0.125 mg PO qd to a day.
client and instructed the nurse that the client is on high-
potassium diet. High potassium foods are recommended in 16. A client with multiple fractures of both lower extremities
the diet of a client taking digitalis preparations because a low is admitted for 3 days ago and is on skeletal traction. The
serum potassium has which of the following effects? client is complaining of having difficulty in bowel movement.
Which of the following would be the most appropriate
A. Potentiates the action of digoxin nursing intervention?
B. Promotes calcium retention
C. Promotes sodium excretion A. Administer an enema
D. Puts the client at risk for digitalis toxicity B. Perform range-of-motion exercise to all extremities
C. Ensure maximum fluid intake (3000ml/day)
Answer: D. Potassium influences the excitability of nerves D. Put the client on the bedpan every 2 hours
and muscles. When potassium is low and the client is on
digoxin, the risk of digoxin toxicity is increased. Answer: C. The best early intervention would be to increase
fluid intake, because constipation is common when activity is
12. The nurse is caring for a client who is transferred from the decreased or usual routines have been interrupted.
operating room for pneumonectomy. The nurse knows that
immediately following pneumonectomy; the client should be 17. John is diagnosed with Addison’s disease and admitted in
in what position? the hospital. What would be the appropriate nursing care for
John?
A. Supine on the unaffected side
B. Low-Fowler’s on the back A. Reducing physical and emotional stress
C. Semi-Fowler’s on the affected side B. Providing a low-sodium diet
D. Semi-Fowler’s on the unaffected side C. Restricting fluids to 1500ml/day
D. Administering insulin-replacement therapy
Answer: C. This position allows maximum expansion,
ventilation, and perfusion of the remaining lung. Answer: A. Because the client’s ability is to react to stress is
decreased, maintaining a quiet environment becomes A
13. A client is placed on digoxin, high potassium foods are nursing priority. Dehydration is a common problem in
recommended in the diet of the client. Which of the following Addison’s disease, so close observation of the client’s
foods willthe nurse give to the client? hydration level is crucial. To promote optimal hydration and
sodium intake, fluid intake is increased, particularly fluid
A. Whole grain cereal, orange juice, and apricots containing electrolytes, such as broths, carbonated
B. Turkey, green bean, and Italian bread beverages, and juices.
C. Cottage cheese, cooked broccoli, and roast beef
D. Fish, green beans and cherry pie 18. Mr. Smith is scheduled for an above-the-knee amputation.
After the surgery he was transferred to the nursing care unit.
Answer: C. This position allows maximum expansion, The nurse assigned to him knows that 72 hours after the
ventilation, and perfusion of the remaining lung. procedure the client should be positioned properly to prevent
contractures. Which of the following is the best position to
14. The nurse is assigned to care to a client who undergone the client?
thyroidectomy. What nursing intervention is important during
the immediate postoperative period following a A. Side-lying, alternating left and right sides
thyroidectomy? B. Sitting in a reclining chair twice a day
C. Lying on abdomen several times daily
A. Assess extremities for weakness and flaccidity D. Supine with stump elevated at least 30 degrees
B. Support the head and neck during position changes
C. Position the client in high Fowler’s Answer: C. At about 48-72 hours, the client must be turned
D. Medicate for restlessness and anxiety onto the abdomen to prevent flexion contractures.

Answer: B. Stress on the suture line should be avoided. 19. A client is scheduled to have an inguinal herniorraphy in
Prevent flexion or hyperextension of the neck, and provide a the outpatient surgical department. The nurse is providing
small pillow under thehead and neck. Neck muscles have health teaching about post surgical care to the client. Which
been affected during a thyroidectomy, support essential for of the following statement if made by the client would reflect
comfort and incisional support. the need for more teaching?

15. What would be the recommended diet the nurse will A. “I should call the physician if I have a cough or cold
implement to a client with burns of the head, face, neck and before surgery”
anterior chest? B. “I will be able to drive soon after surgery”
C. “I will not be able to do any heavy lifting for 3-6 weeks
A. Serve a high-protein, high-carbohydrate diet after surgery”
B. Encourage full liquid diet D. “I should support my incision if I have to cough or turn”
C. Serve a high-fat diet, high-fiber diet
D. Monitor intake to prevent weight gain Answer:B. The client should not drive for 2 weeks after
surgery to avoid stress on the incision. This reflects a need
for additional teaching.
regarding expectoration of thick sputum. Which nursing
20. Ms Jones is brought to the emergency room and is action is most effective?
complaining of muscle spasms, numbness, tremors and
weakness in the arms and legs. The client was diagnosed with A. Place the client in a lateral position every 2 hour
multiple sclerosis. The nurse assigned to Ms. Jones is aware B. Splint the patient’s chest with pillows when coughing
that she has to prevent fatigue to the client to alleviate the C. Use humified oxygen
discomfort. Which of the following teaching is necessary to D. Offer fluids at regular intervals
prevent fatigue?
Answer: D. Fluids liquefy secretions and therefore make it
A. Avoid extremes in temperature easier to expectorate
B. Install safety devices in the home
C. Attend support group meetings 25. The nurse is going to assess the bowel sound of the client.
D. Avoid physical exercise For accurate assessment of the bowel sound, the nurse
should listen for at least:
Answer: A. Extremes in heat and cold will exacerbate
symptoms. Heat delays transmission of impulses and A. 5 minutes
increases fatigue. B. 60 seconds
C. 30 seconds
D. 2 minutes
21. Mr. Stewart is in sickle cell crisis and complaining pain in
the joints and difficulty of breathing. On the assessment of Answer: D. Physical assessment guidelines recommend
the nurse, his temperature is 38.1 ºC. The physician ordered listening for atleast 2 minutes in each quadrant (and up to 5
Morphine sulfate via patient-controlled analgesia (PCA), and minutes, not at least 5 minutes).
oxygen at 4L/min. A priority nursing diagnosis to Mr. Stewart
is risk for infection. A nursing intervention to assist in 26. The nurse encourages the client to wear compression
preventing infection is: stockings. What is the rationale behind in using compression
stockings?
A. Using standard precautions and medical asepsis
B. Enforcing a “no visitors” rule A. Compression stockings promote venous return
C. Using moist heat on painful joints B. Compression stockings divert blood to major vessels
D. Monitoring a vital signs every 2 hour C. Compression stockings decreases workload on the heart
D. Compression stockings improve arterial circulation
Answer: A. Vigilant implementation of standard precautions
and medical asepsis is an effective means of preventing Answer: A. Compression stockings promote venous return
infection and prevent peripheral pooling.

22. Mrs. Maupin is a professor in a prestigious university for 27. Mr. Whitman is a stroke client and is having difficulty in
30 years. After lecture, she experience blurring of vision and swallowing. Which is the best nursing intervention is most
tiredness. Mrs. Maupin is brought to the emergency likely to assist the client?
department. On assessment, the nurse notes that the blood
pressure of the client is 139/90. Mrs. Maupin has been
A. Placing food in the unaffected side of the mouth
diagnosed with essential hypertension and placed on
B. Increasing fiber in the diet
medication to control her BP. Which potential nursing
C. Asking the patient to speak slowly
diagnosis will be a priority for discharge teaching?
D. Increasing fluid intake

A. Sleep Pattern disturbance Answer: A. Placing food in the unaffected side of the mouth
B. Impaired physical mobility assists in the swallowing process because the client has
C. Noncompliance sensation on that side and will have more control over the
D. Fluid volume excess swallowing process.

Answer: C. Noncompliance is a major problem in the 28. Following nephrectomy, the nurse closely monitors the
management of chronic disease. In hypertension, the client urinary output of the client. Which assessment finding is an
often does not feel ill and thus does not see a need to follow early indicator of fluid retention in the postoperative period?
a treatment regimen.
A. Periorbital edema
23. Following a needle biopsy of the kidney, which
B. Increased specific gravity of urine
assessment is an indication that the client is bleeding?
C. A urinary output of 50mL/hr
D. Daily weight gain of 2 lb or more
A. Slow, irregular pulse
B. Dull, abdominal discomfort Answer: D. Daily weights are taken following nephrectomy.
C. Urinary frequency Daily increases of 2 lb or more are indicative of fluid
D. Throbbing headache retention and should be reported to the physician. Intake
and output records may also reflect this imbalance.
Answer: B. An accumulation of blood from the kidney into
the abdomen would manifest itself with these symptoms 29. A nurse is completing an assessment to a client with
cirrhosis. Which of the following nursing assessment is
24. A client with acute bronchitis is admitted in the hospital. important to notify the physician?
The nurse assigned to the client is making a plan of care
A. Expanding ecchymosis
B. Ascites and serum albumin of 3.2 g/dl 34. A client with a diagnosis of gastric ulcer is complaining of
C. Slurred speech syncope and vertigo. What would be the initial nursing
D. Hematocrit of 37% and hemoglobin of 12g/dl intervention by the nurse?

Answer: A. Clients with cirrhosis have already coagulation A. Monitor the client’s vital signs
due to thrombocytopenia and vitamin K deficiency. This B. Keep the client on bed rest
could be a sign of bleeding C. Keep the patient on bed rest
D. Give a stat dose of Sucralfate (Carafate)
30. Mr. Park is 32-year-old, a badminton player and has a type
1 diabetes mellitus. After the game, the client complains of Answer: B. The priority is to maintain client’s safety. With
becoming diaphoretic and light-headedness. The client asks syncope and vertigo, the client is at high risk for falling.
the nurse how to avoid this reaction. The nurse will
recommend to: 35. After a right lower lobectomy on a 55-year-old client,
which action should the nurse initiate when the client is
A. Allow plenty of time after the insulin injection and transferred from the post anesthesia care unit?
before beginning the match
B. Eat a carbohydrate snack before and during the A. Notify the family to report the client’s condition
badminton match B. Immediately administer the narcotic as ordered
C. Drink plenty of fluids before, during, and after bed time C. Keep client on right side supported by pillows
D. Take insulin just before starting the badminton match D. Encourage coughing and deep breathing every 2 hours

Answer: B. Exercise enhances glucose uptake, and the client Answer: D. Coughing and deep breathing are essential for re-
is at risk for an insulin reaction. Snacks with carbohydrates expansion of the lung
will help.
36. The nurse is providing a discharge instruction about the
31. A client is rushed to the emergency room due to serious prevention of urinary stasis to a client with frequent bladder
vehicle accident. The nurse is suspecting of head injury. infection. Which of the following will the nurse include in the
Which of the following assessment findings would the nurse instruction?
report to the physician?
A. Drink 3-4 quarts of fluid every day
A. CVP of 5mmHa B. Empty the bladder every 2-4 hours while awake
B. Glasgow Coma Scale score of 13 C. Encourage the use of coffee, tea, and colas for their
C. Polyuria and dilute urinary output diuretic effect
D. Insomnia D. Teach Kegel exercises to control bladder flow

Answer: C. These are symptoms of diabetes insipidus. The Answer:B. Avoiding stasis of urine by emptying the bladder
patient can become hypovolemic and vasopressin may every 2-4 hours will prevent overdistention of the bladder
reverse the Polyuria. and future urinary tract infections.

32. Mrs. Moore, 62-year-old, with diabetes is in the 37. A male client visits the clinic for check-up. The client tells
emergency department. She stepped on a sharp sea shells the nurse that there is a yellow discharge from his penis. He
while walking barefoot along the beach. Mrs. Moore did not also experiences a burning sensation when urinating. The
notice that the object pierced the skin until later that evening. nurse is suspecting of gonorrhea. What teaching is necessary
What problem does the client most probably have? for this client?

A. Nephropathy A. Sex partner of 3 months ago must be treated


B. Macroangiopathy B. Women with gonorrhea are symptomatic
C. Carpal tunnel syndrome C. Use a condom for sexual activity
D. Peripheral neuropathy D. Sex partner needs to be evaluated

Answer: D. Peripheral neuropathy refers to nerve damage of Answer: D. If infected, the sex partner must be evaluated
the hands and feet. The client did not notice that the object and treated
pierced the skin.
38. A client with AIDS is admitted in the hospital. He is
33. A client with gangrenous foot has undergone a below- receiving intravenous therapy. While the nurse is assessing
knee amputation. The nurse in the nursing care unit knows the IV site, the client becomes confused and restless and the
that the priority nursing intervention in the immediate post intravenous catheter becomes disconnected and minimal
operative care of this client is: amount of the client’s blood spills onto the floor. Which
action will the nurse take to remove the blood spill?
A. Elevate the stump on a pillow for the first 24 hours
B. Encourage use of trapeze A. Promptly clean with a 1:10 solution of household
C. Position the client prone periodically bleach and water
D. Apply a cone-shaped dressing B. Promptly clean up the blood spill with full-strength
antimicrobial cleaning solution
Answer: A. The elevation of the stump on a pillow for the C. Immediately mop the floor with boiling water
first 24 hours decreases edema and increases venous return. D. Allow the blood to dry before cleaning to decrease the
possibility of cross-contamination
Answer: A. A 1:10 solution of household bleach and water is a day before discharge from the hospital. However, for the
recommended by the Centers for Disease Control and majority of clients, it takes 6-12 months before their surgically
Prevention to kill the human immunodeficiency virus (HIV). reduced stomach has stretched enough to accommodate a
larger meal
39. Before surgery, the physician ordered pentobarbital
sodium (Nembutal) for the client to sleep. The night before 43. A male client with cirrhosis is complaining of belly pain,
the scheduled surgery, the nurse gave the pre-medication. itchiness and his breasts are getting larger and also the
One hour later the client is still unable to sleep. The nurse abdomen. The client is so upset because of the discomfort
review the client’s chart and note the physician’s prescription and asks the nurse why his breast and abdomen are getting
with an order to repeat. What should the nurse do next? larger. Which of the following is the appropriate nursing
response?
A. Rub the client’s back until relaxed
B. Prepare a glass of warm milk A. “How much of a difference have you noticed”
C. Give the second dose of pentobarbital sodium B. “It’s part of the swelling your body is experiencing”
D. Explore the client’s feelings about surgery C. “It’s probably because you have been less physically
active”
Answer: D. Given the data, presurgical anxiety is suspected. D. “Your liver is not destroying estrogen hormones that all
The client needs an opportunity to talk about concerns men produce”
related to surgery before further actions (which may mask
the anxiety). Answer: A. This allows the client to elaborate his concern
and provides the nurse a baseline of assessment
40. The nurse on the night shift is making rounds in the
nursing care unit. The nurse is about to enter to the client’s 44. A client is diagnosed with detached retina and scheduled
room when a ventilator alarm sounds, what is the first action for surgery. Preoperative teaching of the nurse to the client
the nurse should do? includes:

A. Assess the lung sounds A. No eye pain is expected postoperatively


B. Suction the client right away B. Semi-fowler’s position will be used to reduce pressure in
C. Look at the client the eye.
D. Turn and position the client C. Eye patches may be used postoperatively
D. Return of normal vision is expected following surgery
Answer: C. A quick look at the client can help identify the
type and cause of the ventilator alarm. Disconnection of the Answer: C. Use of eye patches may be continued
tube from the ventilator, bronchospasm, and anxiety are postoperatively, depending on surgeon preference. This is
some of the obvious reasons that could trigger an alarm. done to achieve >90% success rate of the surgery.

41. What effective precautions should the nurse use to 45. A 70-year-old client is brought to the emergency
control the transmission of methicillin-resistant department with a caregiver. The client has manifestations of
Staphylococcus aureus (MRSA)? anorexia, wasting of muscles and multiple bruises. What
nursing interventions would the nurse implement?
A. Use gloves and handwashing before and after client
contact A. Talk to the client about the caregiver and support
B. Do nasal cultures on healthcare providers system
C. Place the client on total isolation B. Complete a gastrointestinal and neurological
D. Use mask and gown during care of the MRSA client assessment
C. Check the lab data for serum albumin, hematocrit and
Answer: A. Contact isolation has been advised by the hemoglobin
Centers for Disease Control and Prevention (CDC) to control D. Complete a police report on elder abuse
transmission of MRSA, which includes gloves and
handwashing. Answer: B. Assessment and more data collection are
needed. The client may have gastrointestinal or neurological
42. The postoperative gastrectomy client is scheduled for problems that account for the symptoms. The anorexia could
discharge. The client asks the nurse, “When I will be allowed result from medications, poor dentition, or indigestion, the
to eat three meals a day like the rest of my family?”. The bruises may be attributed to ataxia, frequent falls, vertigo, or
appropriate nursing response is: medication.

A. “You will probably have to eat six meals a day for the 46. A nurse is providing a discharge instruction to the client
rest of your life.” about the self-catheterization at home. Which of the
B. “Eating six meals a day can be a bother, can’t it?” following instructions would the nurse include?
C. “Some clients can tolerate three meals a day by the time
they leave the hospital. Maybe it will be a little longer A. Wash the catheter with soap and water after each use
for you.” B. Lubricate the catheter with Vaseline
D. “ It varies from client to client, but generally in 6-12 C. Perform the Valsalva maneuver to promote insertion
months most clients can return to their previous meal D. Replace the catheter with a new one every 24 hour
patterns”
Answer: A. The catheter should be washed with soap and
Answer:D. In response to the question of the client, the nurse water after withdrawal and placed in a clean container. It can
needs to provide brief, accurate information. Some clients be reused until it is too hard or too soft for insertion. Self-
who have had gastrectomies are able to tolerate three meals
care, prevention of complications, and cost-effectiveness are  Medical Surgical Nursing
important in home management. 1. Following spinal injury, the nurse should encourage the
client to drink fluids to avoid:
47. The nurse in the nursing care unit is assigned to care to a
client who is Immunocompromised. The client tells the nurse A. Urinary tract infection.
that his chest is painful and the blisters are itchy. What would B. Fluid and electrolyte imbalance.
be the nursing intervention to this client? C. Dehydration.
D. Skin breakdown.
A. Call the physician
B. Give a prn pain medication Answer: A. Clients in the early stage of spinal cord damage
C. Clarify if the client is on a new medication experience an atonic bladder, which is characterized by the
D. Use gown and gloves while assessing the lesions absence of muscle tone, an enlarged capacity, no feeling of
discomfort with distention, and overflow with a large
Answer: D. The client may have herpes zoster (shingles), a residual. This leads to urinary stasis and infection. High fluid
viral infection. The nurse should use standard precautions in intake limits urinary stasis and infection by diluting the urine
assessing the lesions. Immunocompromised clients are at and increasing urinary output.
risk for infection.
2. The client is transferred from the operating room to
48. A client is admitted and has been diagnosed with bacterial recovery room after an open-heart surgery. The nurse
(meningococcal) meningitis. The infection control registered assigned is taking the vital signs of the client. The nurse
nurse visits the staff nurse caring to the client. What notified the physician when the temperature of the client
statement made by the nurse reflects an understanding of the rises to 38.8 ºC or 102 ºF because elevated temperatures:
management of this client?
A. May be a forerunner of hemorrhage.
A. speech pattern may be altered B. Are related to diaphoresis and possible chilling.
B. Respiratory isolation is necessary for 24 hours after C. May indicate cerebral edema.
antibiotics are started D. Increase the cardiac output.
C. Perform skin culture on the macular popular rash
D. Expect abnormal general muscle contractions Answer: D. The temperature of 102 ºF (38.8ºC) or greater
lead to an increased metabolism and cardiac workload.
Answer: B. After a minimum of 24 hours of IV antibiotics, the
client is no longer considered communicable. Evaluation of 3. After radiation therapy for cancer of the prostate, the client
the nurse’s knowledge is needed for safe care and continuity experienced irritation in the bladder. Which of the following
of care sign of bladder irritability is correct?

49. A 18-year-old male client had sustained a head injury from A. Hematuria
a motorbike accident. It is uncertain whether the client may B. Dysuria
have minimal but permanent disability. The family is C. Polyuria
concerned regarding the client’s difficulty accepting the D. Dribbling
possibility of long term effects. Which nursing diagnosis is
best for this situation? ANSWER: B. Dysuria, nocturia, and urgency are all signs an
irritable bladder after radiation therapy.
A. Nutrition, less than body requirements
B. Injury, potential for sensory-perceptual alterations 4. A client is diagnosed with a brain tumor in the occipital
C. Impaired mobility, related to muscle weakness lobe. Which of the following will the client most likely
D. Anticipatory grieving, due to the loss of independence experience?

Answer: D. Stem of the question supports this choice by A. Visual hallucinations.


stating that the client has difficulty accepting the potential B. Receptive aphasia.
disability. C. Hemiparesis.
D. Personality changes.
50. A client with AIDS is scheduled for discharge. The client
tells the nurse that one of his hobbies at home is gardening. ANSWER: A. The occipital lobe is involve with visual
What will be the discharge instruction of the nurse to the interpretation.
client knowing that the client is prone to toxoplasmosis?
5. A client with Addison’s disease has a blood pressure of
A. Wash all vegetables before cooking 65/60. The nurse understands that decreased blood pressure
B. Wear gloves when gardening of the client with Addison’s disease involves a disturbance in
C. Wear a mask when travelling to foreign countries the production of:
D. Avoid contact with cats and birds
A. Androgens
Answer: B. Toxoplasmosis is an opportunistic infection and a B. Glucocorticoids
parasite of birds and mammals. The oocysts remain infectious C. Mineralocorticoids
in moist soil for about 1 year D. Estrogen

PNLE IV Nursing Practice ANSWER: C. Mineralocorticoids such as aldosterone cause


the kidneys to retain sodium ions. With sodium, water is also
The scope of this Nursing Test IV is parallel to the NP4 NLE retained, elevating blood pressure. Absence of this hormone
Coverage: thus causes hypotension.
ANSWER: C. Hemiparesis creates instability. Using a cane
6. The nurse is planning to teach the client about a provides a wider base of support and, therefore greater
spontaneous pneumothorax. The nurse would base the stability.
teaching on the understanding that:
11. The nurse is conducting a discharge teaching regarding
A. Inspired air will move from the lung into the pleural the prevention of further problems to a client who undergone
space. surgery for carpal tunnel syndrome of the right hand. Which
B. There is greater negative pressure within the chest of the following instruction will the nurse includes?
cavity.
C. The heart and great vessels shift to the affected side. A. Learn to type using your left hand only.
D. The other lung will collapse if not treated immediately. B. Avoid typing in a long period of time.
C. Avoid carrying heavy things using the right hand.
ANSWER: B. As a person with a tear in the lung inhales, air D. Do manual stretching exercise during breaks.
moves through that opening into the intrapleural and causes
partial or complete collapse of the lungs. ANSWER: D. Manual stretching exercises will assist in
keeping the muscles and tendons supple and pliable,
7. During an assessment, the nurse recognizes that the client reducing the traumatic consequences of repetitive activity.
has an increased risk for developing cancer of the tongue.
Which of the following health history will be a concern? 12. A female client is admitted because of recurrent urinary
tract infections. The client asks the nurse why she is prone to
A. Heavy consumption of alcohol. this disease. The nurse states that the client is most
B. Frequent gum chewing. susceptible because of:
C. Nail biting.
D. Poor dental habits. A. Continuity of the mucous membrane.
B. Inadequate fluid intake.
ANSWER: A. Heavy alcohol ingestion predisposes an C. The length of the urethra.
individual to the development of oral cancer. D. Poor hygienic practices.

8. The client in the orthopedic unit asks the nurse the reason ANSWER: C. The length of the urethra is shorter in females
behind why compact bone is stronger than cancellous bone. than in males; therefore microorganisms have a shorter
Which of the following is the correct response of the nurse? distance to travel to reach the bladder. The proximity of the
meatus to the anus in females also increases this incidence.
A. Compact bone is stronger than cancellous bone because
of its greater size. 13. A 55-year-old client is admitted with chest pain that
B. Compact bone is stronger than cancellous bone because radiates to the neck, jaw and shoulders that occurs at rest,
of its greater weight. with high body temperature, weak with generalized sweating
C. Compact bone is stronger than cancellous bone because and with decreased blood pressure. A myocardial infarction is
of its greater volume. diagnosed. The nurse knows that the most accurate
D. Compact bone is stronger than cancellous bone explanation for one of these presenting adaptations is:
because of its greater density.
A. Catecholamines released at the site of the infarction
ANSWER: D. The greater the density of compact bone makes causes intermittent localized pain.
it stronger than the cancellous bone. Compact bone forms B. Parasympathetic reflexes from the infarcted
from cancellous bone by the addition of concentric rings of myocardium causes diaphoresis.
bones substances to the marrow spaces of cancellous bone. C. Constriction of central and peripheral blood vessels
The large marrow spaces are reduced to haversian canals. causes a decrease in blood pressure.
D. Inflammation in the myocardium causes a rise in the
9. The nurse is reviewing the laboratory results of the client. systemic body temperature.
In reviewing the results of the RBC count, the nurse
understands that the higher the red blood cell count, the : ANSWER: D. Temperature may increase within the first 24
hours and persist as long as a week.
A. Greater the blood viscosity.
B. Higher the blood pH. 14. Following an amputation of a lower limb to a male client,
C. Less it contributes to immunity. the nurse provides an instruction on how to prevent a hip
D. Lower the hematocrit. flexion contracture. The nurse should instruct the client to:.

ANSWER: A. Viscosity, a measure of a fluid’s internal A. Perform quadriceps muscle setting exercises twice a day.
resistance to flow, is increased as the number of red cells B. Sit in a chair for 30 minutes three times a day.
suspended in plasma. C. Lie on the abdomen 30 minutes every four hours.
D. Turn from side to side every 2 hours.
10. The physician advised the client with Hemiparesis to use a
cane. The client asks the nurse why cane will be needed. The ANSWER: C. The hips are in extension when the client is
nurse explains to the client that cane is advised specifically to: prone; this keeps the hips from flexing.

A. Aid in controlling involuntary muscle movements. 15. The physician scheduled the client with rheumatoid
B. Relieve pressure on weight-bearing joints. arthritis for the injection of hydrocortisone into the knee
C. Maintain balance and improve stability. joint. The client asks the nurse why there is a need for this
D. Prevent further injury to weakened muscles. injection. The nurse explains that the most important reason
for doing this is to:
A. Lubricate the joint.
B. Prevent ankylosis of the joint. ANSWER: D. Clients adapting to illness frequently feel afraid
C. Reduce inflammation. and helpless and strike out at health team members as a
D. Provide physiotherapy. way of maintaining control or denying their fear.

ANSWER: C. Steroids have an anti-inflammatory effect that 20. Before discharge, the nurse scheduled the client who had
can reduce arthritic pannus formation. a colostomy for colorectal cancer for discharge instruction
about resuming activities. The nurse should plan to help the
16. The nurse is assigned to care for a 57-year-old female client understands that:
client who had a cataract surgery an hour ago. The nurse
should: A. After surgery, changes in activities must be made to
accommodate for the physiologic changes caused by the
A. Advise the client to refrain from vigorous brushing of operation.
teeth and hair. B. Most sports activities, except for swimming, can be
B. Instruct the client to avoid driving for 2 weeks. resumed based on the client’s overall physical condition.
C. Encourage eye exercises to strengthen the ocular C. With counseling and medical guidance, a near normal
musculature. lifestyle, including complete sexual function is possible.
D. Teach the client coughing and deep-breathing D. Activities of daily living should be resumed as quickly as
techniques. possible to avoid depression and further dependency.

ANSWER: A. Activities such as rigorous brushing of hair and ANSWER: C. There are few physical restraints on activity
teeth cause increased intraocular pressure and may lead to postoperatively, but the client may have emotional problems
hemorrhage in the anterior chamber. resulting from the body image changes.

17. A client with AIDS develops bacterial pneumonia is 21. A client is scheduled for bariatric surgery. Preoperative
admitted in the emergency department. The client’s arterial teaching is done. Which of the following statement would
blood gases is drawn and the result is PaO2 80mmHg. then alert the nurse that further teaching to the client is
arterial blood gases are drawn again and the level is reduced necessary?
from 80 mmHg to 65 mmHg. The nurse should;
A. “I will be limiting my intake to 600 to 800 calories a day
A. Have arterial blood gases performed again to check for once I start eating again.”
accuracy. B. “I’m going to have a figure like a model in about a
B. Increase the oxygen flow rate. year.”
C. Notify the physician. C. “I need to eat more high-protein foods.”
D. Decrease the tension of oxygen in the plasma. D. “I will be going to be out of bed and sitting in a chair the
first day after surgery.”.
ANSWER: C. This decrease in PaO2 indicates respiratory
failure; it warrants immediate medical evaluation. ANSWER: B. Clients need to be prepared emotionally for the
body image changes that occur after bariatric surgery.
18. An 18-year-old college student is brought to the Clients generally experience excessive abdominal skin folds
emergency department due to serious motor vehicle after weight stabilizes, which may require a panniculectomy.
accident. Right above-knee-amputation is done. Upon Body image disturbance often occurs in response to
awakening from surgery the client tells the nurse, “What incorrectly estimating one’s size; it is not uncommon for the
happened to me? I cannot remember anything?” Which of client to still feel fat no matter how much weight is lost.
the following would be the appropriate initial nursing
response? 22. The client who had transverse colostomy asks the nurse
about the possible effect of the surgery on future sexual
A. “You sound concerned; You’ll probably remember more relationship. What would be the best nursing response?
as you wake up.”
B. “Tell me what you think happened.” A. The surgery will temporarily decrease the client’s sexual
C. “You were in a car accident this morning.” impulses.
D. “An amputation of your right leg was necessary because B. Sexual relationships must be curtailed for several weeks.
of an accident.” C. The partner should be told about the surgery before any
sexual activity.
ANSWER: C. This is truthful and provides basic information D. The client will be able to resume normal sexual
that may prompt recollection of what happened; it is a relationships.
starting point.
ANSWER: D. Surgery on the bowel has no direct anatomic or
19. A 38-year-old client with severe hypertension is physiologic effect on sexual performance. However, the
hospitalized. The physician prescribed a Captopril (Capoten) nurse should encourage verbalization.
and Alprazolam (Xanax) for treatment. The client tells the
nurse that there is something wrong with the medication and 23. A 75-year-old male client tells the nurse that his wife has
nursing care. The nurse recognizes this behavior is probably a osteoporosis and asks what chances he had of getting also
manifestation of the client’s: osteoporosis like his wife. Which of the following is the
correct response of the nurse?
A. Reaction to hypertensive medications.
B. Denial of illness. A. “This is only a problem for women.”
C. Response to cerebral anoxia. B. “You are not at risk because of your small frame.”
D. Fear of the health problem. C. “You might think about having a bone density test,”
D. “Exercise is a good way to prevent this problem.” 28. A client is receiving diltiazem (Cardizem). What should the
nurse include in a teaching plan aimed at reducing the side
ANSWER: C. Osteoporosis is not restricted to women; it is a effects of this medication?
potential major health problem of all older adults; estimates
indicate that half of all women have at least one osteoporitic A. Take the drug with an antacid.
fracture and the risk in men is estimated between 13% and B. Lie down after meals.
25%; a bone mineral density measurement assesses the C. Avoid dairy products in diet.
mass of bone per unit volume or how tightly the bone is D. Change positions slowly.
packed.
ANSWER: D. Changing positions slowly will help prevent the
24. An older adult client with acute pain is admitted in the side effect of orthostatic hypotension.
hospital. The nurse understands that in managing acute pain
of the client during the first 24 hours, the nurse should ensure 29. A client is receiving simvastatin (Zocor). The nurse is
that: aware that this medication is effective when there is decrease
in:
A. Ordered PRN analgesics are administered on a
scheduled basis. A. The triglycerides
B. Patient controlled analgesia is avoided in this B. The INR
population. C. Chest pain
C. Pain medication is ordered via the intramuscular route. D. Blood pressure
D. An order for meperidine (Demerol) is secured for pain
relief. ANSWER: A. Therapeutic effects of simvastatin include
decreased serum triglyceries, LDL and cholesterol.
ANSWER: A. Around-the-clock administration of analgesics is
recommended for acute pain in the older adult population; 30. A client is taking nitroglycerine tablets, the nurse should
this help to maintain a therapeutic blood level of pain teach the client the importance of:
medication.
A. Increasing the number of tablets if dizziness or
25. A nurse is caring to an older adult with presbycusis. In
hypertension occurs.
formulating nursing care plan for this client, the nurse should
B. Limiting the number of tablets to 4 per day.
expect that hearing loss of the client that is caused by aging
C. Making certain the medication is stored in a dark
to have:
container.
D. Discontinuing the medication if a headache develops.
A. Overgrowth of the epithelial auditory lining.
B. Copious, moist cerumen. ANSWER: C. Nitroglycerine is sensitive to light and moisture
C. Difficulty hearing women’s voices. ad must be stored in a dark, airtight container.
D. Tears in the tympanic membrane.
31. The physician prescribes Ibuprofen (Motrin) and
ANSWER: C. Generally, female voices have a higher pitch hydroxychloroquine sulfate (Plaquenil) for a 58-year-old male
than male voices; older adults with presbycusis (hearing loss client with arthritis. The nurse provides information about
caused by the aging process) have more difficulty hearing toxicity of the hydroxychloroquine. The nurse can determine
higher-pitched sounds. if the information is clearly understood if the client states:

26. The nurse is reviewing the client’s chart about the ordered
A. “I will contact the physician immediately if I develop
medication. The nurse must observe for signs of hyperkalemia
blurred vision.”
when administering:
B. “I will contact the physician immediately if I develop
urinary retention.”
A. Furosemide (Lasix) C. “I will contact the physician immediately if I develop
B. Hydrochlorothiazide (HydroDIURIL) swallowing difficulty.”
C. Metolazone (Zaroxolyn) D. “I will contact the physician immediately if I develop
D. Spironolactone (Aldactone) feelings of irritability.”

ANSWER: D. Aldactone is a potassium-sparing diuretic; ANSWER: A. Visual disturbance are a sign of toxicity because
hyperkalemia is an adverse effect. retinopathy can occur with this drug.

27. The physician prescribed Albuterol (Proventil) to the client 32. The client with an acute myocardial infarction is
with severe asthma. After the administration of the hospitalized for almost one week. The client experiences
medication the nurse should monitor the client for: nausea and loss of appetite. The nurse caring for the client
recognizes that these symptoms may indicate the:
A. Palpitation
B. Visual disturbance A. Adverse effects of spironolactone (Aldactone)
C. Decreased pulse rate B. Adverse effects of digoxin (Lanoxin)
D. Lethargy C. Therapeutic effects of propranolol (Indiral)
D. Therapeutic effects of furosemide (Lasix)
ANSWER: A. Albuterol’s sympathomimetic effect causes
cardiac stimulation that may cause tachycardia and ANSWER: B. Toxic levels of Lanoxin stimulate the medullary
palpitation. chemoreceptor trigger zone, resulting in nausea and
subsequent anorexia.
33. A client with a partial occlusion of the left common Answer: C. Potassium iodide, which aids in decreasing the
carotid artery is scheduled for discharge. The client is still vascularity of the thyroid gland, decreases the risk for
receiving Coumadin. The nurse provided a discharge hemorrhage.
instruction to the client regarding adverse effects of
Coumadin. The nurse should tell the client to consult with the 38. A client with Addison’s disease is scheduled for discharge.
physician if: Before the discharge, the physician prescribes hydrocortisone
and fludrocortisone. The nurse expects the hydrocortisone to:
A. Swelling of the ankles increases.
B. Blood appears in the urine. A. Increase amounts of angiotensin II to raise the client’s
C. Increased transient Ischemic attacks occur. blood pressure.
D. The ability to concentrate diminishes. B. Control excessive loss of potassium salts.
C. Prevent hypoglycemia and permit the client to respond
Answer: B. Warfarin derivatives cause an increase in the to stress.
prothrombin time and INR, leading to an increased risk for D. Decrease cardiac dysrhythmias and dyspnea.
bleeding. Any abnormal or excessive bleeding must be
reported, because it may indicate toxic levels of the drug. Answer: C. Hydrocortisone is a glucocorticoid that has anti-
inflammatory action and aids in metabolism of
34. Levodopa is ordered for a client with Parkinson’s disease. carbohydrate, fat, and protein, causing elevation of blood
Before starting the medication, the nurse should know that: glucose. Thus it enables the body to adapt to stress.

A. Levodopa is inadequately absorbed if given with meals. 39. A client with diabetes insipidus is taking Desmopressin
B. Levodopa may cause the side effects of orthostatic acetate (DDAVP). To determine if the drug is effective, the
hypotension. nurse should monitor the client’s:
C. Levodopa must be monitored by weekly laboratory
tests. A. Arterial blood pH
D. Levodopa causes an initial euphoria followed by B. Pulse rate
depression. C. Serum glucose
D. Intake and output
Answer: B. Levodopa is the metabolic precursor of
dopamine. It reduces sympathetic outflow by limiting Answer: D. DDAVP replaces the ADH, facilitating
vasoconstriction, which may result in orthostatic reabsorption of water and consequent return of normal
hypotension. urine output and thirst.

35. In making a diagnosis of myasthenia gravis Edrophonium 40. A client with recurrent urinary tract infections is to be
HCI (Tensilon) is used. The nurse knows that this drug will discharged. The client will be taking nitrofurantoin (Macrobid)
cause a temporary increase in: 50 mg po every evening at home. The nurse provides
discharge instructions to the client. Which of the following
A. Muscle strength instructions will be correct?
B. Symptoms
C. Blood pressure A. Strain urine for crystals and stones
D. Consciousness B. Increase fluid intake.
C. Stop the drug if the urinary output increases
Answer: A. Tensilon, an anticholinesterase drug, causes D. Maintain the exact time schedule for drug taking.
temporary relief of symptoms of myasthenia gravis in client
who have the disease and is therefore an effective diagnostic Answer: B. To prevent crystal formation, the client should
aid. have sufficient intake to produce 1000 to 1500 mL of urine
daily while taking this drug.
36. The nurse can determine the effectiveness of
carbamazepine (Tegretol) in the management of trigeminal 41. A client with cancer of the lung is receiving chemotherapy.
neuralgia by monitoring the client’s: The physician orders antibiotic therapy for the client. The
nurse understands that chemotherapy destroys rapidly
A. Seizure activity growing leukocytes in the:
B. Liver function
C. Cardiac output A. Bone marrow
D. Pain relief B. Liver
C. Lymph nodes
Answer: D. Carbamazepine ( Tegretol) is administered to D. Blood
control pain by reducing the transmission of nerve impulses
in clients with trigeminal neuralgia. Answer: A. Prolonged chemotherapy may slow the
production of leukocytes in bone marrow, thus suppressing
37. Administration of potassium iodide solution is ordered to the activity of the immune system. Antibiotics may be
the client who will undergo a subtotal thyroidectomy. The required to help counter infections that the body can no
nurse understands that this medication is given to: longer handle easily.

A. Ablate the cells of the thyroid gland that produce T4. 42. The physician reduced the client’s Dexamethasone
B. Decrease the total basal metabolic rate. (Decadron) dosage gradually and to continue a lower
C. Decrease the size and vascularity of the thyroid. maintenance dosage. The client asks the nurse about the
D. Maintain function of the parathyroid gland. change of dosage. The nurse explains to the client that the
purpose of gradual dosage reduction is to allow:
A. Return of cortisone production by the adrenal glands. Answer: A. Once treatment with insulin for diabetic
B. Production of antibodies by the immune system ketoacidosis is begun, potassium ions reenter the cell,
C. Building of glycogen and protein stores in liver and causing hypokalemia; therefore potassium, along with the
muscle replacement fluid, is generally supplied.
D. Time to observe for return of increases intracranial
pressure 48. A female client is brought to the emergency unit. The
client is complaining of abdominal cramps. On assessment,
Answer: A. Any hormone normally produced by the body client is experiencing anorexia and weight is reduced. The
must be withdrawn slowly to allow the appropriate organ to physician’s diagnosis is colitis. Which of the following
adjust and resume production. symptoms of fluid and electrolyte imbalance should the nurse
report immediately?
43. The nurse is assigned to care for a client with diarrhea.
Excessive fluid loss is expected. The nurse is aware that fluid A. Skin rash, diarrhea, and diplopia
deficit can most accurately be assessed by: B. Development of tetaniy with muscles spasms
C. Extreme muscle weakness and tachycardia
A. The presence of dry skin D. Nausea, vomiting, and leg and stomach cramps.
B. A change in body weight
C. An altered general appearance Answer: C. Potassium, the major intracellular cation,
D. A decrease in blood pressure functions with sodium and calcium to regulate
neuromuscular activity and contraction of muscle fibers,
Answer: B. Dehydration is most readily and accurately particularly the heart muscle. In hypokalemia these
measured by serial assessment of body weight; 1 L of fluid symptoms develop.
weighs 2.2 pounds.
49. The client is to receive an IV piggyback medication. When
44. Which of the following is the most important electrolyte preparing the medication the nurse should be aware that it is
of intracellular fluid? very important to:

A. Potassium A. Use strict sterile technique


B. Sodium B. Use exactly 100mL of fluid to mix the medication
C. Chloride C. Change the needle just before adding the medication
D. Calcium D. Rotate the bag after adding the medication

Answer: A. The concentration of potassium is greater inside Answer: A. Because IV solutions enter the body’s internal
the cell and is important in establishing a membrane environment, all solutions and medications utilizing this
potential, a critical factor in the cell’s ability to function. route must be sterile to prevent the introduction of
microbes.
45. Which of the following client has a high risk for developing
hyperkalemia? 50. The nurse is reviewing the laboratory result of the client.
An arterial blood gas report indicates the client’s pH is 7.20,
A. Crohn’s disease PCO2 35 mmHg and HCO3 is 19 mEq/L. The results are
B. End-Stage renal disease consistent with:
C. Cushing’s syndrome
D. Chronic heart failure A. Metabolic acidosis
B. Metabolic alkalosis
Answer: B. The kidneys normally eliminate potassium from C. Respiratory acidosis
the body; hyperkalemia may necessitate dialysis. D. Respiratory alkalosis

46. The nurse is reviewing the laboratory result of the client. Answer: A. A low pH and bicarbonate level are consistent
The client’s serum potassium level is 5.8 mEq/L. Which of the with metabolic acidosis.
following is the initial nursing action?

A. Call the cardiac arrest team to alert them PNLE V Nursing Practice
B. Call the laboratory and repeat the test The scope of this Nursing Test V is parallel
C. Take the client’s vital signs and notify the physician to the NP5 NLE Coverage:
D. Obtain an ECG strip and have lidocaine available  Psychiatric Nursing
1. A 17-year-old client has a record of being
Answer: C. Vital signs monitor cardiorespiratory status; absent in the class without permission, and
hyperkalemia causes serious cardiac dysrhythmias. “borrowing” other people’s things without asking
permission. The client denies stealing;
47. Potassium chloride, 20 mEq, is ordered and to be added in rationalizing instead that as long as no one was
the IV solution of a client in a diabetic ketoacidosis. The using the items, there is no problem to use it by
primary reason for administering this drug is: other people. It is important for the nurse to
understand that psychodynamically, the behavior
of the client may be largely attributed to a
A. Replacement of excessive losses
development defect related to the:
B. Treatment of hyperpnea
C. Prevention of flaccid paralysis
D. Treatment of cardiac dysrhythmias A. Oedipal complex
B. Superego
C. Id
D. Ego
Answer: B. This shows a weak sense of moral 6. A 16-year-old girl was diagnosed with anorexia.
consciousness. According to Freudian theory, What would be the first assessment of the nurse?
personality disorders stem from a weak
superego. A. What food she likes.
B. Her desired weight.
2. A client tells the nurse, “Yesterday, I was C. Her body image.
planning to kill myself.” What is the best nursing D. What causes her behavior.
response to this cient?
Answer: A. Although all options may appear
A. “What are you going to do this time?” correct. A is the best because it focuses on a
B. Say nothing. Wait for the client’s next range of possible positive reinforcers, a basis for
comment an effective behavior modification program. It
C. “You seem upset. I am going to be here can lead to concrete, specific nursing
with you; perhaps you will want to talk interventions right away and provides a
about it” therapeutic use of “control” for the 16-year-old.
D. “Have you felt this way before?”
7. On an adolescent unit, a nurse caring to a
Answer: C. The client needs to have his or her client was informed that her client’s closest
feelings acknowledged, with encouragement to roommate dies at night. What would be the most
discuss feelings, and be reassured about the appropriate nursing action?
nurse’s presence.
A. Do not bring it up unless the client
3. In crisis intervention therapy, which of the asks.
following principle that the nurse will use to plan B. Tell the client that her roommate went home.
her/his goals? C. Tell the client, if asked, “You should ask the
doctor.”
A. Crises are related to deep, underlying D. Tell the client that her closest roommate
problems died.
B. Crises seldom occur in normal people’s lives
C. Crises may go on indefinitely. Answer: A. The nurse needs to wait and see: do
D. Crises usually resolved in 4-6 weeks. not “jump the gun”; do not assume that the
client wants to know now.
Answer: D. Part of the definition of a crisis is a
time span of 4-6 weeks. 8. A woman gave birth to an unhealthy infant,
and with some body defects. The nurse should
4. The nurse enters the room of the male client expect the woman’s initial reactions to include:
and found out that the client urinates on the floor.
The client hides when the nurse is about to talk to A. Depression
him. Which of the following is the best nursing B. Withdrawal
intervention? C. Apathy
D. Anger
A. Place restriction on the client’s activities
when his behavior occurs. Answer: D. The woman is experiencing an actual
B. Ask the client to clean the soiled floor. loss and will probably exhibit many of the same
C. Take the client to the bathroom at symptoms as a person who has lost someone to
regular intervals. death.
D. Limit fluid intake.
9. A client in the psychiatric unit is shouting out
Answer; C. The client is most likely confused, loud and tells the nurse, “Please, help me. They
rather than exhibiting acting-out, hostile are coming to get me.” What would be the
behavior. Frequent toileting will allow urination in appropriate nursing response?
an appropriate place.
A. “ I won’t let anyone get you.”
5. A young lady with a diagnosis of schizophrenic B. “Who are they?”
reaction is admitted to the psychiatric unit. In the C. “I don’t see anyone coming.”
past two months, the client has poor appetite, D. “You look frightened.”
experienced difficulty in sleeping, was mute for
long periods of time, just stayed in her room, Answer: C. This option is an example of pointing
grinning and pointing at things. What would be out reality- the nurse’s perception.
the initial nursing action on admitting the client
to the unit? 10. A client who is severely obese tells the nurse,
“My therapist told me that I eat a lot because I
A. Assure the client that “ You will be well cared didn’t get any attention and love from my
for.” mother. What does the therapist mean?” What is
B. Introduce the client to some of the other the best nursing response?
clients.
C. Ask “Do you know where you are?” A. “What do you think is the connection
D. Take the client to the assigned room. between your not getting enough love and
overeating?”
Answer: D. The client needs basic, simple B. “Tell me what you think the therapist
orientation that directly relates to the here-and- means.”
now, and does not require verbal interaction.
C. “You need to ask your therapist.” others in a socially comprehensible and
D. “ We are here to deal with your diet, not with acceptable way.
your psychological problems.”
15. The client is telling the nurse in the
Answer: B. This response asks information that psychiatric ward, “I hate them.” Which of the
the nurse can use. If the client understands the following is the most appropriate nursing
statement, the nurse can support the therapist response to the client?
when focusing on connection between food,
love, and mother. If the client does not A. “Tell me about your hate.”
understand thestatement, the nurse can help B. “I will stay with you as long as you feel this
get clarification from the therapist. way.”
C. “For whom do you have these feelings?”
11. After the discussion about the procedure the D. “I understand how you can feel this way.”
physician scheduled the client for mastectomy.
The client tells the nurse, “If my breasts will be Answer: A. The nurse is asking the client to
removed, I’m afraid my husband will not love me clarify and further discuss feelings.
anymore and maybe he will never touch me.”
What should the nurse’s response? 16. The mother visits her son with major
depression in the psychiatric unit. After the
A. “I doubt that he feels that way.” conversation of the client and the mother, the
B. “What makes you feel that way?” nurse asks the mother how it is talking to her son.
C. “Have you discussed your feelings with The mother tells the nurse that it was a stressful
your husband?” time. During an interview with the client, the
D. Ask the husband, in front of the wife, how he client says, “we had a marvelous visit.” Which of
feels about this. the following coping mechanism can be described
to thestatement of the client?
Answer: C. This option redirects the client to talk
to her husband. A. Identification.
B. Rationalization.
12. The child is brought to the hospital by the C. Denial.
parents. During assessment of the nurse, what D. Compensation.
parental behavior toward a child should alert the
nurse to suspect child abuse? Answer: C. Denial is the act of avoiding
disagreeable realities by ignoring them.
A. Ignoring the child.
B. Flat affect. 17. A male client is quiet when the physician told
C. Expressions of guilt. him that he has stage IV cancer and has 4
D. Acting overly solicitous toward the child months to live. The nurse determines that this
reaction may be an example of:
Answer: D. This is an example of reaction
formation, a coping mechanism. A. Indifference
B. Denial
13. A nurse is caring to a client with manic C. Resignation
disorder in the psychiatric ward. On the morning D. Anger
shift, the nurse is talking with the client who is
now exhibiting a manic episode with flight of Answer: B. Reactions when told of a life-
ideas. The nurse primarily needs to: threatening illness stem from Kübler-Ross’ ideas
on death and dying. Denial is a typical grief
A. Focus on the feelings conveyed rather response, and usually is a first reaction.
than the thoughts expressed.
B. Speak loudly and rapidly to keep the client’s 18. A nurse is caring to a female client with five
attention, because the client is easily young children. The family member told the client
distracted. that her ex-husband has died 2 days ago. The
C. Allow the client to talk freely. reaction of the client is stunned silence, followed
D. Encourage the client to complete one by anger that the ex-husband left no insurance
thought at a time. money for their young children. The nurse should
understand that:
Answer: A. Often the verbalized ideas are
jumbled, but the underlying feelings are A. The children and the injustice done to them
discernible and must be acknowledged. by their father’s death are the woman’s main
concern.
14. The nurse is caring to an autistic child. Which B. To explain the woman’s reaction, the nurse
of the following play behavior would the nurse needs more information about the
expect to see in a child? relationship and breakup.
C. The woman is not reacting normally to the
A. competitive play news.
B. nonverbal play D. The woman is experiencing a normal
C. cooperative play bereavement reaction.
D. solitary play
Answer: D. Shock and anger are commonly the
Answer: D. Autistic children do best with solitary primary initial reactions.
play because they typically do not interact with
19. A client who is manic comes to the outpatient A. Delusion.
department. The nurse is assigning an activity for B. Hallucination.
the client. What activity is best for the nurse to C. Negativism.
encourage for a client in a manic phase? D. Illusion.

A. Solitary activity, such as walking with Answer: A. This is a false belief developed in
the nurse, to decrease stimulation. response to an emotional need.
B. Competitive activity, such as bingo, to
increase the client’s self-esteem. 24. A client is admitted in the hospital. On
C. Group activity, such as basketball, to assessment, the nurse found out that the client
decrease isolation. had several suicidal attempts. Which of the
D. Intellectual activity, such as scrabble, to following is the most important nursing action?
increase concentration.
A. Ignore the client as long as he or she is
Answer: A. This option avoids external stimuli, talking about suicide, because suicide
yet channels the excess motor activity that is attempt is unlikely.
often part of the manic phase. B. Administer medication.
C. Relax vigilance when the client seems to be
20. The nurse is about to administer Imipramine recovering from depression.
HCI (Tofranil) to the client, the client says, “Why D. Maintain constant awareness of the
should I take this?” The doctor started me on this client’s whereabouts.
10days ago; it didn’t help me at all.” Which of the
following is the best nursing response: Answer:D. The client must be constantly
observed.
A. “What were you expecting to happen?”
B. “It usually takes 2-3 weeks to be 25. The nurse suspects that the client is suffering
effective.” from depression. During assessment, what are
C. “Do you want to refuse this medication? You the most characteristic signs and symptoms of
have the right.” depression the nurse would note?
D. “That’s a long time wait when you feel so
depressed.” A. Constipation, increased appetite.
B. Anorexia, insomnia.
Answer: B. The patient needs a brief, factual C. Diarrhea, anger.
answer. D. Verbosity, increased social interaction

21. Which of the following drugs the nurse should Answer: B. The appetite is diminished and
choose to administer to a client to prevent sleeping is affected to a client with depression.
pseudoparkinsonism? .
26. The client in the psychiatric unit states that,
A. Isocarboxazid (Marplan) “The goodas are coming! I must be ready.” In
B. Chlorpromazine HCI (Thorazine) response to this neologism, the nurse’s initial
C. Trihexyphenidyl HCI (Artane) response is to:
D. Trifluoperazine HCI (Stelazine)
A. Acknowledge that the word has some
Answer: C. Trihexyphenidyl HCI (Artane) is often special meaning for the client.
used to counteract side effect of B. Try to interpret what the client means.
pseudoparkinsonism, which often accompanies C. Divert the client’s attention to an aspect of
the use of phenothiazine, such as reality.
chlorpromazine HCI (Thorazine or Trifluoperazine D. State that what the client is saying has not
HCI (Stelazine). been understood and then divert attention to
something that is really bound.
22. The nurse is caring to an 80-year-old client
with dementia? What is the most important Answer: A. It is important to acknowledge a
psychosocial need for this client? statement, even if it is not understood.

A. Focus on the there-and-then rather the here- 27. A male client diagnosed with depression tells
and-now. the nurse, “I don’t want to look weak and I don’t
B. Limit in the number of visitors, to minimize even cry because my wife and my kids can’t bear
confusion. it.” The nurse understands that this is an example
C. Variety in their daily life, to decrease of:
depression.
D. A structured environment, to minimize A. Repression.
regressive behaviors. B. Suppression.
C. Undoing.
Answer: D. Persons with dementia needs D. Rationalization.
sameness, consistency, structure, routine, and
predictability. Answer: D. Rationalization is the process of
constructing plausible reasons for one’s
23. A client tells the nurse, “I don’t want to eat responses
any meals offered in this hospital because the
food is poisoned.” The nurse is aware that the 28. A female client tells the nurse that she is
client is expressing an example of: afraid to go out from her room because she
thinks that the other client might kill her. The 33. A nurse is going to give a rectal suppository
nurse is aware that this behavior is related to: as a preoperative medication to a 4-year-old boy.
The boy is very anxious and frightened. Which of
A. Hallucination. the following statement by the nurse would be
B. Ideas of reference. most appropriate to gain the child’s cooperation?
C. Delusion of persecution.
D. Illusion. A. “Be a big kid! Everyone’s waiting for you.”
B. “Lie still now and I’ll let you have one of your
Answer: C. The client has ideas that someone is presents before you even have your
out to kill her. operation.”
C. “Take a nice, big, deep breath and then
29. A female client is taking Imipramine HCI let me hear you count to five.”
(Tofranil) for almost 1 week and shows less D. “You look so scared. Want to know a secret?
awareness of the physical body. What problem This won’t hurt a bit!”
would the nurse be most concerned? Answer: C. Preschool children commonly
experience fears and fantasies regarding
A. Nausea. invasive procedures. The nurse should attempts
B. Gait disturbances. to momentarily distract the child with a simple
C. Bowel movements. task that can be easily accomplished while the
D. Voiding. child remains in the side-lying position. The
suppository can be slipped into place while the
Answer: D. A serious side effect of Imipramine child is counting, and then the nurse can praise
HCI (Tofranil) is urinary retention (voiding the child for cooperating, while holding the
problems) buttocks together to prevent expulsion of the
suppository.
30. A 6-year-old client dies in the nursing unit.
The parents want to see the child. What is the 34. A depressed client is on an MAO inhibitor?
most appropriate nursing action? What should the nurse watch out for?

A. Give the parents time alone with the A. Hypertensive crisis.


body. B. Diet restrictions.
B. Ask the physician for permission. C. Taking medication with meals.
C. Complete the postmortem care and quietly D. Exposure to sunlight.
accompany the family to the child’s room.
D. Suggest the parents to wait until the funeral Answer: A. This is the more inclusive answer,
service to say “good-bye.” although diet restrictions (answer1) are
important, their purpose is to prevent
Answer: A. This allows the parents/family to hypertensive crisis (answer 2).
grieve over the loss of the child, by going
through the steps of leave taking. 35. A 16-year-old girl is admitted for treatment of
a fracture. The client shares to the nurse caring
31. A 20-year-old female client is diagnosed with to her that her step-father has made sexual
anxiety disorder. The physician prescribed advances to her. She got the chance to tell it to
Flouxetine (Prozac). What is the most important her mother but refuses to believe. What is the
side effects should a nurse be concerned? most therapeutic action of the nurse would be:

A. Tremor, drowsiness. A. Tell the client to work it out with her father.
B. Seizures, suicidal tendencies. B. Tell the client to discuss it with her mother.
C. Visual disturbance, headache. C. Ask the father about it.
D. Excessive diaphoresis, diarrhea. D. Ask the mother what she thinks.

Answer: B. Assess for suicidal tendencies, Answer: D. This comes closest to beginning to
especially during early therapy. There is an focus on family-centered approach to intervene
increased risk of seizures in debilitated client in the “conspiracy of silence”. This is therefore
and those with a history of seizures. the best among the options.

32. A nurse is assigned to activate a client who is 36. A client with a diagnosis of paranoid disorder
withdrawn, hears voices and negativistic. What is admitted in the psychiatric hospital. The client
would be the best nursing approach? tells the nurse, “the FBI is following me. These
people are plotting against me.” With this
statement the nurse will need to:
A. Mention that the “voices” would want the
client to participate.
B. Demand that the client must join a group A. Acknowledge that this is the client’s
activity. belief but not the nurse’s belief.
C. Give the client a long explanation of the B. Ask how that makes the client feel.
benefits of activity. C. Show the client that no one is behind.
D. Tell the client that the nurse needs a D. Use logic to help the client doubt this belief.
partner for an activity.
Answer: A. The nurse should neither challenge
Answer: D. The nurse helps to activate by doing nor use logic to dispel an irrational belief.
something with the client.
37. A nurse is completing the routine physical
examination to a healthy 16-year-old male client.
The client shares to the nurse that he feels like Answer: D. This is the most neutral answer by
killing his girlfriend because he found out that her process of elimination.
girlfriend had another boyfriend. He then laughs,
and asks the nurse to keep this a secret just 41. A 3-year-old boy is brought to the emergency
between the two of them. The nurse reviews his department. After an hour, the boy dies of
chart and notes that there is no previously history respiratory failure. The mother of the boy
of violence or psychiatric illness. Which of the becomes upset, shouting and abusive, saying to
following would be the best action of the nurse to the nurse, “If it had been your son, they would
take at this time? have done more to save it. “What should the
nurse say or do?
A. Suggest the teen meet with a counselor to
discuss his feelings about his girlfriend. A. Touch her and tell her exactly what was done
B. Tell the teen that his feelings are normal, and for her baby.
recommend that he find another girlfriend to B. Allow the mother to continue her
take his mind off the problem. present behavior while sitting quietly
C. Recall the teenage boys often say things with her.
they really do not mean and ignore the C. “No, all clients are given the same good
comment. care.”
D. Regard the comment seriously and D. “Yes, you’re probably right. Your son did not
notify the teen’s primary health care get better care.”
provider and parents
Answer: B. This option allows a normal grief
Answer: D. Any threat to the safety of oneself or response (anger).
other should always be taken seriously and
never disregarded by the nurse. 42. The nurse is interacting to a client with an
antisocial personality disorder. What would be the
38. Which of the following person will be at most therapeutic approach of the nurse to an
highest risk for suicide? antisocial behavior?

A. A student at exam time A. Gratify the client’s inner needs.


B. A married woman, age 40, with 6 children. B. Give the client opportunities to test reality.
C. A person who is an alcoholic. C. Provide external controls.
D. A person who made a previous suicide D. Reinforce the client’s self-concept.
attempt.
Answer: C. Personality disorders stem from a
Answer: C. The likelihood of multiple contributing weak superego, implying a lack of adequate
factors may make this person at higher risk for controls.
suicide. Some factors that may exist are physical
illness related to alcoholism, emotional factors 43. A 55-year-old male client tells the nurse that
( anxiety, guilt, remorse), social isolation due to he needs his glasses and hearing aid with him in
impaired relationships and economic problems the recovery room after the surgery, or he will be
related to employment. upset for not granting his request. What is the
appropriate nursing response?
39. A male client is repetitively doing the
handwashing every time he touches things. It is A. “Do you get upset and confused often?”
important for a nurse to understand that the B. “You won’t need your glasses or hearing aid.
client’s behavior is probably an attempt to: The nurses will take care of you.”
C. “I understand. You will be able to
A. Seek attention from the staff. cooperate best if you know what is
B. Control unacceptable impulses or going on, so I will find out how I can
feelings. arrange to have your glasses and
C. Do what the voices the patient hears tell him hearing aid available to you in the
or her to do. recovery room.”
D. Punish himself or herself for guilt feeling. D. I understand you might be more cooperative
if you have your aid and glasses, but that is
Answer: B. A ritual, such as compulsive just not possible. Rules, you know.”
handwashing, is an attempt to allay anxiety
caused by unconscious impulses that are Answer: C. The client will be easier to care for if
frightening. he has his hearing aid and glasses.

40. In a mental health settings, the basic goal of 44. The male client had fight with his roommates
nursing is to: in the psychiatric unit. The client agitated client is
placed in isolation for seclusion. The nurse knows
A. Advance the science of psychiatry by it is essential that:
initiating research and gathering data for
current statistics on emotional illness. A. A staff member has frequent contacts
B. Plan activity programs for clients. with the client.
C. Understand various types of family therapy B. Restraints are applied.
and psychological tests and how to interpret C. The client is allowed to come out after 4
them. hours.
D. Maintain a therapeutic environment. D. All the furniture is removed form the isolation
room.
Answer: A. Frequent contacts at times of stress A. Use simple questions that call for a response.
are important, especially when a client is B. Encourage discussion of feelings.
isolated. C. Look through a photo album together.
D. Bring up neutral topics.
45. A medical representative comes to the
hospital unit for the promotion of a new product. Answer: D. Neutral, nonthreatening topics are
A female client, admitted for hysterical behavior, best in attempting to encourage a response.
is found embracing him. What should the nurse
say? 50. Which of the following nursing approach is
most important in a client with depression?
A. “Have you considered birth control?”
B. “This isn’t the purpose of either of you A. Deemphasizing preoccupation with
being here.” elimination, nourishment, and sleep.
C. “I see you’ve made a new friend.” B. Protecting against harm to others.
D. “Think about what you are doing.” C. Providing motor outlets for aggressive,
hostile feelings.
Answer: B. This response is aimed at redirecting D. Reducing interpersonal contacts.
the inappropriate behavior.
Answer: C. It is important to externalize the
46. A client with dementia is for discharge. The anger away from self.
nurse is providing a discharge instruction to the
family member regarding safety measures at
home. What suggestion can the nurse make to SET 3
the family members?

A. Avoid stairs without banisters.


B. Use restraints while the client is in bed to PNLE I for Foundation of
keep him or her from wandering off during
the night.
C. Use restraints while the client is sitting in a Nursing
chair to keep him or her from wandering off
1. Which element in the circular chain of infection
during the day.
can be eliminated by preserving skin integrity?
D. Provide a night-light and a big clock.
A. Host
Answer: D. This option is best to decrease B. Reservoir
confusion and disorientation to place and time C. Mode of transmission
D. Portal of entry
47. A 30-year-old married woman comes to the
hospital for treatment of fractures. The woman Answer: D. In the circular chain of infection,
tells the nurse that she was physically abused by pathogens must be able to leave their reservoir
her husband. The woman receives a call from her and be transmitted to a susceptible host through
husband telling her to get home and things will a portal of entry, such as broken skin.
be different. He felt sorry of what he did. What
can the nurse advise her? 2. Which of the following will probably result in a
break in sterile technique for respiratory
A. “Do you think so?” isolation?
B. “It’s not likely.”
C. “What will be different?” A. Opening the patient’s window to the outside
D. “I hope so, for your sake.” environment
B. Turning on the patient’s room ventilator
Answer: C. This option helps the woman to think C. Opening the door of the patient’s room
through and elaborate on her own thoughts and leading into the hospital corridor
prognosis. D. Failing to wear gloves when administering a
bed bath
48. A female client was diagnosed with breast
cancer. It is found to be stage IV, and a modified Answer: C. Respiratory isolation, like strict
mastectomy is performed. After the procedure, isolation, requires that the door to the door
what behaviors could the nurse expects the client patient’s room remain closed. However, the
to display? patient’s room should be well ventilated, so
opening the window or turning on the ventricular
A. Denial of the possibility of carcinoma. is desirable. The nurse does not need to wear
B. Signs of grief reaction. gloves for respiratory isolation, but good hand
C. Relief that the operation is over. washing is important for all types of isolation.
D. Signs of deep depression.
3. Which of the following patients is at greater
Answer:B. It is mostly likely that grief would be risk for contracting an infection?
expressed because of object loss.
A. A patient with leukopenia
B. A patient receiving broad-spectrum
49. A client is withdrawn and does not want to
antibiotics
interact to anybody even to the nurse. What is
C. A postoperative patient who has undergone
the best initial nursing approach to encourage
orthopedic surgery
communication with this client?
D. A newly diagnosed diabetic patient
equipment after a patient has been discharged
Answer: A. Leukopenia is a decreased number of to prepare them for reuse by another patient.
leukocytes (white blood cells), which are The purpose of protective (reverse) isolation is
important in resisting infection. None of the to prevent a person with seriously impaired
other situations would put the patient at risk for resistance from coming into contact who
contracting an infection; taking potentially pathogenic organisms.
broadspectrum antibiotics might actually reduce
the infection risk. 8. Which of the following constitutes a break in
sterile technique while preparing a sterile field for
4. Effective hand washing requires the use of: a dressing change?
A. Soap or detergent to promote A. Using sterile forceps, rather than sterile
emulsification gloves, to handle a sterile item
B. Hot water to destroy bacteria B. Touching the outside wrapper of sterilized
C. A disinfectant to increase surface tension material without sterile gloves
D. All of the above C. Placing a sterile object on the edge of
the sterile field
Answer: A. Soaps and detergents are used to D. Pouring out a small amount of solution (15 to
help remove bacteria because of their ability to 30 ml) before pouring the solution into a
lower the surface tension of water and act as sterile container
emulsifying agents. Hot water may lead to skin
irritation or burns. Answer: C. The edges of a sterile field are
considered contaminated. When sterile items are
5. After routine patient contact, hand washing allowed to come in contact with the edges of the
should last at least: field, the sterile items also become
contaminated.
A. 30 seconds
B. 1 minute 9. A natural body defense that plays an active
C. 2 minute role in preventing infection is:
D. 3 minutes
A. Yawning
Answer: A. Depending on the degree of B. Body hair
exposure to pathogens, hand washing may last C. Hiccupping
from 10 seconds to 4 minutes. After routine D. Rapid eye movements
patient contact, hand washing for 30 seconds
effectively minimizes the risk of Answer: B. Hair on or within body areas, such as
pathogen transmission. the nose, traps and holds particles that contain
microorganisms. Yawning and hiccupping do
6. Which of the following procedures always not prevent microorganisms from entering or
requires surgical asepsis? leaving the body. Rapid eye movement marks
the stage of sleep during which dreaming
A. Vaginal instillation of conjugated estrogen occurs.
B. Urinary catheterization
C. Nasogastric tube insertion 10. All of the following statement are true about
D. Colostomy irrigation donning sterile gloves except:
Answer: B. The urinary system is normally free A. The first glove should be picked up by
of microorganisms except at the urinary meatus. grasping the inside of the cuff.
Any procedure that involves entering this system B. The second glove should be picked up by
must use surgically aseptic measures to inserting the gloved fingers under the cuff
maintain a bacteria-free state. outside the glove.
C. The gloves should be adjusted by sliding the
7. Sterile technique is used whenever: gloved fingers under the sterile cuff and
pulling the glove over the wrist
A. Strict isolation is required D. The inside of the glove is considered
B. Terminal disinfection is performed sterile
C. Invasive procedures are performed
D. Protective isolation is necessary Answer: D. The inside of the glove is always
considered to be clean, but not sterile.
Answer: C. All invasive procedures, including
surgery, catheter insertion, and administration 11.When removing a contaminated gown, the
of parenteral therapy, require sterile technique
nurse should be careful that the first thing she
to maintain a sterile environment. All equipment
touches is the:
must be sterile, and the nurse and the physician
must wear sterile gloves and maintain surgical A. Waist tie and neck tie at the back of the
asepsis. In the operating room, the nurse and gown
physician are required to wear sterile gowns, B. Waist tie in front of the gown
gloves, masks, hair covers, and shoe covers for C. Cuffs of the gown
all invasive procedures. Strict isolation requires D. Inside of the gown
the use of clean gloves, masks, gowns and
equipment to prevent the transmission of highly Answer: A. The back of the gown is considered
communicable diseases by contact or by clean, the front is contaminated. So, after
airborne routes. Terminal disinfection is removing gloves and washing hands, the nurse
the disinfection of all contaminated supplies and should untie the back of the gown; slowly move
backward away from the gown, holding D. Presence of cardiac enzymes
the inside of the gown and keeping the edges off
the floor; turn and fold the gown inside out; Answer: A. Platelets are disk-shaped cells that
discard it in a contaminated linen container; are essential for blood coagulation. A platelet
then wash her hands again. count determines the number of thrombocytes
in blood available for promoting hemostasis and
12.Which of the following nursing interventions is assisting with blood coagulation after injury. It
considered the most effective form or universal also is used to evaluate the patient’s potential
precautions? for bleeding; however, this is not its primary
purpose. The normal count ranges from 150,000
A. Cap all used needles before removing them to 350,000/mm3. A count of 100,000/mm3 or
from their syringes less indicates a potential for bleeding; count of
B. Discard all used uncapped needles and less than 20,000/mm3 is associated with
syringes in an impenetrable protective spontaneous bleeding.
container
C. Wear gloves when administering IM 16.Which of the following white blood cell (WBC)
injections counts clearly indicates leukocytosis?
D. Follow enteric precautions
A. 4,500/mm³
Answer: B. According to the Centers for Disease B. 7,000/mm³
Control (CDC), blood-to-blood contact occurs C. 10,000/mm³
most commonly when a health care worker D. 25,000/mm³
attempts to cap a used needle. Therefore, used
needles should never be recapped; instead they Answer: D. Leukocytosis is any transient
should be inserted in a specially designed increase in the number of white blood cells
puncture resistant, labeled container. Wearing (leukocytes) in the blood. Normal WBC counts
gloves is not always necessary range from 5,000 to 100,000/mm3. Thus, a
when administering an I.M. injection. Enteric count of 25,000/mm3 indicates leukocytosis.
precautions prevent the transfer of pathogens
via feces. 17. After 5 days of diuretic therapy with 20mg of
furosemide (Lasix) daily, a patient begins to
13.All of the following measures are exhibit fatigue, muscle cramping and muscle
recommended to prevent pressure ulcers except: weakness. These symptoms probably indicate
that the patient is experiencing:
A. Massaging the reddened are with lotion
B. Using a water or air mattress A. Hypokalemia
C. Adhering to a schedule for positioning and B. Hyperkalemia
turning C. Anorexia
D. Providing meticulous skin care D. Dysphagia

Answer: A. Nurses and other health care Answer: A. Fatigue, muscle cramping, and
professionals previously believed that massaging muscle weaknesses are symptoms
a reddened area with lotion would promote of hypokalemia (an inadequate potassium level),
venous return and reduce edema to the area. which is a potential side effect of diuretic
However, research has shown that massage only therapy. The physician usually orders
increases the likelihood of cellular ischemia and supplemental potassium to prevent hypokalemia
necrosis to the area. in patients receiving diuretics. Anorexia is
another symptom of hypokalemia. Dysphagia
14.Which of the following blood tests should be means difficulty swallowing.
performed before a blood transfusion?
18.Which of the following statements about chest
A. Prothrombin and coagulation time X-ray is false?
B. Blood typing and cross-matching
C. Bleeding and clotting time A. No contradictions exist for this test
D. Complete blood count (CBC) and electrolyte B. Before the procedure, the patient should
levels. remove all jewelry, metallic objects, and
buttons above the waist
Answer: B. Before a blood transfusion is C. A signed consent is not required
performed, the blood of the donor and recipient D. Eating, drinking, and medications are
must be checked for compatibility. This is done allowed before this test
by blood typing (a test that determines a
person’s blood type) and cross-matching Answer: A. Pregnancy or suspected pregnancy is
(a procedure that determines the compatibility the only contraindication for a chest X-ray.
of the donor’s and recipient’s blood after the However, if a chest X-ray is necessary, the
blood types has been matched). If the blood patient can wear a lead apron to protect the
specimens are incompatible, hemolysis and pelvic region from radiation. Jewelry,
antigen-antibody reactions will occur. metallic objects, and buttons would interfere
with the X-ray and thus should not be worn
15.The primary purpose of a platelet count is to above the waist. A signed consent is not
evaluate the: required because a chest X-ray is not an invasive
examination. Eating, drinking and medications
A. Potential for clot formation are allowed because the X-ray is of the chest,
B. Potential for bleeding not the abdominal region.
C. Presence of an antigen-antibody response
19.The most appropriate time for the nurse to D. Divide the area between the greater
obtain a sputum specimen for culture is: femoral trochanter and the lateral
femoral condyle into thirds, and select
A. Early in the morning the middle third on the anterior of the
B. After the patient eats a light breakfast thigh
C. After aerosol therapy
D. After chest physiotherapy Answer: D. The vastus lateralis, a long, thick
muscle that extends the full length of the thigh,
Answer: A. Obtaining a sputum specimen early is viewed by many clinicians as the site of choice
in this morning ensures an adequate supply of for I.M. injections because it has relatively few
bacteria for culturing and decreases the risk major nerves and blood vessels. The middle
of contamination from food or medication. third of the muscle is recommended as the
injection site. The patient can be in a supine or
20.A patient with no known allergies is to receive sitting position for an injection into this site.
penicillin every 6 hours. When administering the
medication, the nurse observes a fine rash on the 23.The mid-deltoid injection site is seldom used
for I.M. injections because it:
patient’s skin. The most appropriate nursing
action would be to: A. Can accommodate only 1 ml or less of
medication
A. Withhold the moderation and notify the B. Bruises too easily
physician C. Can be used only when the patient is lying
B. Administer the medication and notify the down
physician D. Does not readily parenteral medication
C. Administer the medication with an
antihistamine Answer: A. The mid-deltoid injection site can
D. Apply corn starch soaks to the rash accommodate only 1 ml or less of medication
because of its size and location (on the deltoid
Answer: A. Initial sensitivity to penicillin is muscle of the arm, close to the brachial artery
commonly manifested by a skin rash, even in and radial nerve).
individuals who have not been allergic to it
previously. Because of the danger of 24.The appropriate needle size for insulin
anaphylactic shock, he nurse should withhold injection is:
the drug and notify the physician, who may
choose to substitute another drug. Administering A. 18G, 1 ½” long
an antihistamine is a dependent nursing B. 22G, 1” long
intervention that requires a written physician’s C. 22G, 1 ½” long
order. Although applying corn starch to the rash D. 25G, 5/8” long
may relieve discomfort, it is not the nurse’s top
priority in such a potentially life-threatening Answer: D. A 25G, 5/8” needle is the
situation. recommended size for insulin injection because
insulin is administered by the subcutaneous
21.All of the following nursing interventions are route. An 18G, 1 ½” needle is usually used for
correct when using the Ztrack method of drug I.M. injections in children, typically in the
injection except: vastus lateralis. A 22G, 1 ½” needle is usually
used for adult I.M. injections, which are typically
A. Prepare the injection site with alcohol administered in the vastus lateralis or
B. Use a needle that’s a least 1” long ventrogluteal site.
C. Aspirate for blood before injection
D. Rub the site vigorously after the 25.The appropriate needle gauge for intradermal
injection to promote absorption injection is:

Answer: D. The Z-track method is an I.M. A. 20G


injection technique in which the patient’s skin is B. 22G
pulled in such a way that the needle track is C. 25G
sealed off after the injection. This procedure D. 26G
seals medication deep into the muscle,
thereby minimizing skin staining and irritation. Answer:D. Because an intradermal injection
Rubbing the injection site is contraindicated does not penetrate deeply into the skin, a small-
because it may cause the medication to bore 25G needle is recommended. This type of
extravasate into the skin. injection is used primarily to administer antigens
to evaluate reactions for allergy or sensitivity
22.The correct method for determining the vastus studies. A 20G needle is usually used for I.M.
lateralis site for I.M. injection is to: injections of oilbased medications; a 22G needle
for I.M. injections; and a 25G needle, for I.M.
A. Locate the upper aspect of the upper outer injections; and a 25G needle, for subcutaneous
quadrant of the buttock about 5 to 8 cm insulin injections.
below the iliac crest
B. Palpate the lower edge of the acromion 26.Parenteral penicillin can be administered as
process and the midpoint lateral aspect of an:
the arm
C. Palpate a 1” circular area anterior to the A. IM injection or an IV solution
umbilicus B. IV or an intradermal injection
C. Intradermal or subcutaneous injection
D. IM or a subcutaneous injection
Answer: D. Phlebitis, the inflammation of a vein,
Answer: A. Parenteral penicillin can be can be caused by chemical irritants (I.V.
administered I.M. or added to a solution and solutions or medications), mechanical irritants
given I.V. It cannot be administered (the needle or catheter used during
subcutaneously or intradermally. venipuncture or cannulation), or a localized
allergic reaction to the needle or catheter. Signs
27.The physician orders gr 10 of aspirin for a and symptoms of phlebitis include pain or
patient. The equivalent dose in milligrams is: discomfort, edema and heat at the I.V. insertion
site, and a red streak going up the arm or leg
A. 0.6 mg from the I.V. insertion site.
B. 10 mg
C. 60 mg 32.The best way of determining whether a patient
D. 600 mg has learned to instill ear medication properly is
for the nurse to:
Answer:D. gr 10 x 60mg/gr 1 = 600 mg
A. Ask the patient if he/she has used ear drops
28.The physician orders an IV solution of dextrose before
5% in water at 100ml/hour. What would the flow B. Have the patient repeat the nurse’s
rate be if the drop factor is 15 gtt = 1 ml? instructions using her own words
C. Demonstrate the procedure to the patient
A. 5 gtt/minute and encourage to ask questions
B. 13 gtt/minute D. Ask the patient to demonstrate the
C. 25 gtt/minute procedure
D. 50 gtt/minute
Answer: D. Return demonstration provides the
Answer: C. 100ml/60 min X 15 gtt/ 1 ml = 25 most certain evidence for evaluating the
gtt/minute effectiveness of patient teaching.

29.Which of the following is a sign or symptom of 33.Which of the following types of medications
a hemolytic reaction to blood transfusion? can be administered via gastrostomy tube?
A. Hemoglobinuria A. Any oral medications
B. Chest pain B. Capsules whole contents are dissolve in
C. Urticaria water
D. Distended neck veins C. Enteric-coated tablets that are thoroughly
dissolved in water
Answer: A. Hemoglobinuria, the abnormal D. Most tablets designed for oral use,
presence of hemoglobin in the urine, indicates a except for extended-
hemolytic reaction (incompatibility of the duration compounds
donor’s and recipient’s blood). In this reaction,
antibodies in the recipient’s plasma combine Answer: D. Capsules, enteric-coated tablets, and
rapidly with donor RBC’s; the cells are most extended duration or sustained release
hemolyzed in either circulatory or products should not be dissolved for use in
reticuloendothelial system. Hemolysis occurs a gastrostomy tube. They are pharmaceutically
more rapidly in ABO incompatibilities than in Rh manufactured in these forms for valid reasons,
incompatibilities. Chest pain and urticaria may and altering them destroys their purpose. The
be symptoms of impending anaphylaxis. nurse should seek an alternate physician’s order
Distended neck veins are an indication of when an ordered medication is inappropriate for
hypervolemia. delivery by tube.

30.Which of the following conditions may require 34.A patient who develops hives after receiving
fluid restriction? an antibiotic is exhibiting drug:
A. Fever A. Tolerance
B. Chronic Obstructive Pulmonary Disease B. Idiosyncrasy
C. Renal Failure C. Synergism
D. Dehydration D. Allergy

Answer: C. In real failure, the kidney loses their Answer: D. A drug-allergy is an adverse reaction
ability to effectively eliminate wastes and fluids. resulting from an immunologic response
Because of this, limiting the patient’s intake of following a previous sensitizing exposure to the
oral and I.V. fluids may be necessary. Fever, drug. The reaction can range from a rash or
chronic obstructive pulmonary disease, and hives to anaphylactic shock. Tolerance to a drug
dehydration are conditions for which fluids means that the patient experiences a decreasing
should be encouraged. physiologic response to repeated administration
of the drug in the same dosage. Idiosyncrasy is
31.All of the following are common signs and an individual’s unique hypersensitivity to a drug,
symptoms of phlebitis except: food, or other substance; it appears to be
genetically determined. Synergism, is a drug
A. Pain or discomfort at the IV insertion site interaction in which the sum of the drug’s
B. Edema and warmth at the IV insertion site combined effects is greater than that of their
C. A red streak exiting the IV insertion site separate effects.
D. Frank bleeding at the insertion site
35.A patient has returned to his room after testing service to evaluate student nursing
femoral arteriography. All of the following are competence but it does not certify nurses. The
appropriate nursing interventions except: American Nurses Association identifies
requirements for certification and offers
A. Assess femoral, popliteal, and pedal pulses examinations for certification in many areas
every 15 minutes for 2 hours of nursing., such as medical surgical nursing.
B. Check the pressure dressing for sanguineous These certification (credentialing) demonstrates
drainage that the nurse has the knowledge and the ability
C. Assess a vital signs every 15 minutes for 2 to provide high quality nursing care in the area
hours of her certification. A graduate of an associate
D. Order a hemoglobin and hematocrit degree program is not a clinical nurse
count 1 hour after the arteriography specialist: however, she is prepared to provide
bed side nursing with a high degree of
Answer: D. A hemoglobin and hematocrit count knowledge and skill. She must successfully
would be ordered by the physician if bleeding complete the licensing examination to become a
were suspected. The other answers are registered professional nurse.
appropriate nursing interventions for a patient
who has undergone femoral arteriography. 39.The purpose of increasing urine acidity
through dietary means is to:
36.The nurse explains to a patient that a cough:
A. Decrease burning sensations
A. Is a protective response to clear the B. Change the urine’s color
respiratory tract of irritants C. Change the urine’s concentration
B. Is primarily a voluntary action D. Inhibit the growth of microorganisms
C. Is induced by the administration of an
antitussive drug Answer: D. Microorganisms usually do not grow
D. Can be inhibited by “splinting” the abdomen in an acidic environment.

Answer: A. Coughing, a protective response that 40.Clay colored stools indicate:


clears the respiratory tract of irritants, usually is
involuntary; however it can be voluntary, as A. Upper GI bleeding
when a patient is taught to perform coughing B. Impending constipation
exercises. An antitussive drug inhibits coughing. C. An effect of medication
Splinting the abdomen supports the abdominal D. Bile obstruction
muscles when a patient coughs.
Answer: D. Bile colors the stool brown. Any
37.An infected patient has chills and begins inflammation or obstruction that impairs bile
shivering. The best nursing intervention is to: flow will affect the stool pigment, yielding light,
clay-colored stool. Upper GI bleeding results in
A. Apply iced alcohol sponges black or tarry stool. Constipation is characterized
B. Provide increased cool liquids by small, hard masses. Many medications and
C. Provide additional bedclothes foods will discolor stool – for example, drugs
D. Provide increased ventilation containing iron turn stool black.; beets turn stool
red.
Answer: C. In an infected patient, shivering
results from the body’s attempt to increase heat 41.In which step of the nursing process would the
production and the production of neutrophils nurse ask a patient if the medication she
and phagocytotic action through increased administered relieved his pain?
skeletal muscle tension and contractions. Initial
vasoconstriction may cause skin to feel cold to A. Assessment
the touch. Applying additional bed clothes helps B. Analysis
to equalize the body temperature and stop the C. Planning
chills. Attempts to cool the body result in D. Evaluation
further shivering, increased metabloism, and
thus increased heat production. Answer: D. In the evaluation step of the nursing
process, the nurse must decide whether the
38.A clinical nurse specialist is a nurse who has: patient has achieved the expected outcome that
was identified in the planning phase.
A. Been certified by the National League for
Nursing 42.All of the following are good sources of vitamin
B. Received credentials from the Philippine A except:
Nurses’ Association
C. Graduated from an associate degree A. White potatoes
program and is a registered professional B. Carrots
nurse C. Apricots
D. Completed a master’s degree in the D. Egg yolks
prescribed clinical area and is a
registered professional nurse Answer: A. The main sources of vitamin A are
yellow and green vegetables (such as carrots,
Answer: .D. A clinical nurse specialist must have sweet potatoes, squash, spinach, collard greens,
completed a master’s degree in a clinical broccoli, and cabbage) and yellow fruits (such as
specialty and be a registered professional nurse. apricots, and cantaloupe). Animal sources
The National League of Nursing accredits include liver, kidneys, cream, butter, and egg
educational programs in nursing and provides a yolks.
immediately before surgery, not the day
43.Which of the following is a primary nursing before. A topical antiseptic would not remove
intervention necessary for all patients with a microorganisms and would be beneficial only
Foley Catheter in place? after proper cleaning and rinsing. Tub bathing
might transfer organisms to another body site
A. Maintain the drainage tubing and collection rather than rinse them away.
bag level with the patient’s bladder
B. Irrigate the patient with 1% Neosporin 47.When transferring a patient from a bed to a
solution three times a daily chair, the nurse should use which muscles to
C. Clamp the catheter for 1 hour every 4 hours avoid back injury?
to maintain the bladder’s elasticity
D. Maintain the drainage tubing and A. Abdominal muscles
collection bag below bladder level to B. Back muscles
facilitate drainage by gravity C. Leg muscles
D. Upper arm muscles
Answer: D. Maintaing the drainage tubing and
collection bag level with the patient’s bladder Answer: C. The leg muscles are the strongest
could result in reflux of urine into the kidney. muscles in the body and should bear the greatest
Irrigating the bladder with Neosporin and stress when lifting. Muscles of the abdomen,
clamping the catheter for 1 hour every 4 hours back, and upper arms may be easily injured
must be prescribed by a physician.
48.Thrombophlebitis typically develops in
44.The ELISA test is used to: patients with which of the following conditions?
A. Screen blood donors for antibodies to human A. Increases partial thromboplastin time
immunodeficiency virus (HIV) B. Acute pulsus paradoxus
B. Test blood to be used for transfusion for HIV C. An impaired or traumatized blood vessel wall
antibodies D. Chronic Obstructive Pulmonary Disease
C. Aid in diagnosing a patient with AIDS (COPD)
D. All of the above
Answer: C. The factors, known as Virchow’s
Answer: D. The ELISA test of venous blood is triad, collectively predispose a patient to
used to assess blood and potential blood donors thromboplebitis; impaired venous return to the
to human immunodeficiency virus (HIV). A heart, blood hypercoagulability, and injury to a
positive ELISA test combined with various signs blood vessel wall. Increased
and symptoms helps to diagnose acquired partial thromboplastin time indicates a
immunodeficiency syndrome (AIDS) prolonged bleeding time during fibrin
clot formation, commonly the result of
45.The two blood vessels most commonly used anticoagulant (heparin) therapy. Arterial blood
for TPN infusion are the: disorders (such as pulsus paradoxus) and lung
diseases (such as COPD) do not necessarily
A. Subclavian and jugular veins impede venous return of injure vessel walls.
B. Brachial and subclavian veins
C. Femoral and subclavian veins 49.In a recumbent, immobilized patient, lung
D. Brachial and femoral veins ventilation can become altered, leading to such
respiratory complications as:
: D. Tachypnea (an abnormally rapid rate of
breathing) would indicate that the patient was A. Respiratory acidosis, ateclectasis, and
still hypoxic (deficient in oxygen).The partial hypostatic pneumonia
pressures of arterial oxygen and carbon dioxide B. Appneustic breathing, atypical pneumonia
listed are within the normal range. Eupnea refers and respiratory alkalosis
to normal respiration. C. Cheyne-Strokes respirations and
spontaneous pneumothorax
46.Effective skin disinfection before a surgical D. Kussmail’s respirations and hypoventilation
procedure includes which of the following
methods? Answer: A. Because of restricted respiratory
movement, a recumbent, immobilize patient is
A. Shaving the site on the day before surgery at particular risk for respiratory acidosis from
B. Applying a topical antiseptic to the skin on poor gas exchange; atelectasis from reduced
the evening before surgery surfactant and accumulated mucus in the
C. Having the patient take a tub bath on the bronchioles, and hypostatic pneumonia from
morning of surgery bacterial growth caused by stasis of mucus
D. Having the patient shower with an secretions.
antiseptic soap on the
evening v=before and the morning of 50.Immobility impairs bladder elimination,
surgery resulting in such disorders as
Answer: D. Studies have shown that showering A. Increased urine acidity and relaxation of the
with an antiseptic soap before surgery is the perineal muscles, causing incontinence
most effective method of removing B. Urine retention, bladder distention, and
microorganisms from the skin. Shaving the site infection
of the intended surgery might cause breaks in C. Diuresis, natriuresis, and decreased urine
the skin, thereby increasing the risk of infection; specific gravity
however, if indicated, shaving, should be done
D. Decreased calcium and phosphate levels in A. Spermicides
the urine
B. Diaphragm
Answer: B. The immobilized patient commonly C. Condoms
suffers from urine retention caused by decreased D. Vasectomy
muscle tone in the perineum. This leads to
bladder distention and urine stagnation, which
provide an excellent medium for bacterial growth 1. Answer:C . Condoms, when used
leading to infection. Immobility also results in correctly and consistently, are
more alkaline urine with excessive amounts of the most effective contraceptive
calcium, sodium and phosphate, a gradual
decrease in urine production, and an increased
method or barrier against
specific gravity. bacterial and viral
sexually transmitted infections.
Although spermicides kill sperm,
PNLE II for they do not provide reliable
protection against the spread of
Maternal and sexually transmitted infections,
especially intracellular organisms
such as HIV. Insertion
Child Health and removal of the diaphragm
along with the use of the
1. For the client who is using oral
spermicides may cause vaginal
contraceptives, the nurse informs irritations, which could place the
the client about the need to take the client at risk for
pill at the same time each day to infection transmission. Male
accomplish which of the following? sterilization eliminates
spermatozoa from the
A. Decrease the incidence of
ejaculate, but it does not
nausea
eliminate bacterial and/or viral
B. Maintain hormonal levels
microorganisms that can cause
C. Reduce side effects
sexually transmitted infections.
D. Prevent drug interactions
3. When preparing a woman who is 2
Answer: B . Regular timely ingestion
of oral contraceptives is necessary days postpartum for
to maintain hormonal levels of the discharge, recommendations for
drugs to suppress the action of the which of the following contraceptive
hypothalamus and anterior pituitary methods would be avoided?
leading to inappropriate secretion of
A. Diaphragm
FSH and LH. Therefore, follicles do
B. Female condom
not mature, ovulation is inhibited,
C. Oral contraceptives
and pregnancy is prevented. The
D. Rhythm method
estrogen content of the oral site
contraceptive may cause the
Answer: A . The diaphragm must be
nausea, regardless of when the pill
fitted individually to ensure
is taken. Side effects and
effectiveness. Because of the
drug interactions may occur with
changes to the reproductive
oral contraceptives regardless of
structures during pregnancy and
the time the pill is taken.
following delivery, the diaphragm
must be refitted, usually at the
2. When teaching a client about
6 weeks’ examination following
contraception. Which of the childbirth or after a weight loss of 15
following would the nurse include as lbs or more. In addition, for
the most effective method for maximum effectiveness, spermicidal
preventing sexually transmitted jelly should be placed in the dome
infections? and around the rim. However,
spermicidal jelly should not be
inserted into the vagina until time!” Which of the following should
involution is completed the nurse recommend?
at approximately 6 weeks. Use of a
female condom protects the A. Daily enemas
reproductive system from the B. Laxatives
introduction of semen or C. Increased fiber intake
spermicides into the vagina and may D. Decreased fluid intake
be used after childbirth. Oral
contraceptives may be started Answer: C . During the third
within the first postpartum week to trimester, the enlarging uterus
ensure suppression of ovulation. For places pressure on the intestines.
the couple who has determined the This coupled with the effect of
female’s fertile period, using the hormones on smooth
rhythm method, avoidance of muscle relaxation causes decreased
intercourse during this period, is intestinal motility (peristalsis).
safe and effective Increasing fiber in the diet will help
fecal matter pass more quickly
4. For which of the following clients through the intestinal tract, thus
would the nurse expect that decreasing the amount of water
that is absorbed. As a result, stool is
an intrauterine device would not be
softer and easier to pass. Enemas
recommended?
could precipitate preterm labor
A. Woman over age 35 and/or electrolyte loss and should
B. Nulliparous woman be avoided. Laxatives may
C. Promiscuous young adult cause preterm labor by stimulating
D. Postpartum client peristalsis and may interfere with
the absorption of nutrients. Use for
Answer: C . An IUD may increase more than 1 week can also lead to
the risk of pelvic inflammatory laxative dependency. Liquid in the
disease, especially in women with diet helps provide a semisolid, soft
more than one sexual partner, consistency to the stool. Eight to
because of the increased risk of ten glasses of fluid per day are
sexually transmitted infections. An essential to maintain hydration and
UID should not be used if promote stool evacuation.
the woman has an active or chronic
pelvic infection, postpartum 6. Which of the following would the
infection, endometrial hyperplasia nurse use as the basis for the
or carcinoma, or uterine teaching plan when caring for a
abnormalities. Age is not a factor in pregnant teenager concerned about
determining the risks associated gaining too much weight during
with IUD use. Most IUD users are pregnancy?
over the age of 30. Although there
is a slightly higher risk for A. 10 pounds per trimester
infertility in women who have never B. 1 pound per week for 40 weeks
been pregnant, the IUD is an C. ½ pound per week for 40 weeks
acceptable option as long as the D. A total gain of 25 to 30
risk-benefit ratio is discussed. IUDs pounds
may be inserted immediately after
delivery, but this is not Answer: D . To ensure adequate
recommended because of fetal growth and development
the increased risk and rate of during the 40 weeks of a
expulsion at this time. pregnancy, a total weight gain 25 to
30 pounds is recommended:
5. A client in her third trimester tells 1.5 pounds in the first 10 weeks; 9
the nurse, “I’m constipated all the pounds by 30 weeks; and 27.5
pounds by 40 weeks. The pregnant record her obstetrical history as
woman should gain less weight in which of the following?
the first and second trimester than
in the third. During the first A. G2 T2 P0 A0 L2
trimester, the client should only B. G3 T1 P1 A0 L2
gain 1.5 pounds in the first 10 C. G3 T2 P0 A0 L2
weeks, not 1 pound per week. D. G4 T1 P1 A1 L2
A weight gain of ½ pound per week
would be 20 pounds for the Answer: D. The client has been
total pregnancy, less than the pregnant four times, including
recommended amount. current pregnancy (G). Birth at 38
weeks’ gestation is considered full
7. The client tells the nurse that her term (T), while birth form 20 weeks
last menstrual period started on to 38 weeks is considered preterm
(P). A spontaneous abortion
January 14 and ended on January 20.
occurred at 8 weeks (A). She has
Using Nagele’s rule, the nurse
two living children (L).
determines her EDD to be which of
the following? 9. When preparing to listen to the
A. September 27 fetal heart rate at 12 weeks’
B. October 21 gestation, the nurse would use
C. November 7 which of the following?
D. December 27
A. Stethoscope placed midline at
the umbilicus
Answer: B . To calculate the EDD by
B. Doppler placed midline at the
Nagele’s rule, add 7 days to the first
suprapubic region
day of the last menstrual period and
C. Fetoscope placed midway
count back 3 months, changing the
between the umbilicus and the
year appropriately. To obtain a date
xiphoid process
of September 27, 7 days have been
D. External electronic fetal monitor
added to the last day of the LMP
placed at the umbilicus
(rather than the first day of the
LMP), plus 4 months (instead of 3
Answer: B. At 12 weeks gestation,
months) were counted back. To
the uterus rises out of the pelvis
obtain the date of November 7, 7
and is palpable above the
days have been subtracted (instead
symphysis pubis. The Doppler
of added) from the first day of LMP
intensifies the sound of the
plus November indicates counting
fetal pulse rate so it is audible. The
back 2 months (instead of 3
uterus has merely risen out of the
months) from January. To obtain the
pelvis into the abdominal cavity and
date of December 27, 7 days were
is not at the level of the umbilicus.
added to the last day of the LMP
The fetal heart rate at this age is
(rather than the first day of
not audible with a stethoscope. The
the LMP) and December indicates
uterus at 12 weeks is just above the
counting back only 1 month
symphysis pubis in the abdominal
(instead of 3 months) from January.
cavity, not midway between the
umbilicus and the xiphoid process.
8. When taking an obstetrical history
At 12 weeks the FHR would be
on a pregnant client who states, “I
difficult to auscultate with a
had a son born at 38 weeks fetoscope. Although the external
gestation, a daughter born at 30 electronic fetal monitor would
weeks gestation and I lost a baby at project the FHR, the uterus has
about 8 weeks,” the nurse should not risen to the umbilicus at 12
weeks.
edema, and proteinuria. Although
10.When developing a plan of care urine is checked for glucose at each
for a client newly diagnosed clinic visit, this is not the priority.
with gestational diabetes, which of Depression may cause either
the following instructions would be anorexia or excessive food intake,
the priority? leading to excessive weight gain or
loss. This is not, however, the
A. Dietary intake priority consideration at this time.
B. Medication Weight gain thought to be caused
C. Exercise by excessive food intake would
D. Glucose monitoring require a 24-hour diet recall.
However, excessive intake would
Answer: A . Although all of the not be the primary consideration for
choices are important in the this client at this time.
management of diabetes, diet
therapy is the mainstay of the 12. A client 12 weeks’ pregnant
treatment plan and should always come to the emergency department
be the priority. Women diagnosed with abdominal cramping and
with gestational diabetes generally moderate vaginal bleeding.
need only diet therapy without Speculum examination reveals 2 to 3
medication to control their
cms cervical dilation. The nurse
blood sugar levels. Exercise, is
important for all pregnant women would document these findings as
and especially for diabetic women, which of the following?
because it burns up glucose, thus A. Threatened abortion
decreasing blood sugar. However, B. Imminent abortion
dietary intake, not exercise, is the C. Complete abortion
priority. All pregnant women with D. Missed abortion
diabetes should have periodic
monitoring of serum Answer:B. Cramping and vaginal
glucose. However, those with bleeding coupled with cervical
gestational diabetes generally do dilation signifies that termination of
not need daily glucose monitoring. the pregnancy is inevitable and
The standard of care recommends a cannot be prevented. Thus, the
fasting and 2- hour postprandial nurse would document an imminent
blood sugar level every 2 weeks. abortion. In a threatened abortion,
cramping and vaginal bleeding are
11.A client at 24 weeks gestation present, but there is no cervical
has gained 6 pounds in 4 weeks. dilation. The symptoms may subside
Which of the following would be the or progress to abortion. In
priority when assessing the client? a complete abortion all the products
of conception are expelled. A
A. Glucosuria missed abortion is early fetal
B. Depression intrauterine death without expulsion
C. Hand/face edema of the products of conception.
D. Dietary intake
13.Which of the following would be
Answer: C. After 20 weeks’ the priority nursing diagnosis for a
gestation, when there is a rapid client with an ectopic pregnancy?
weight gain, preeclampsia should
be suspected, which may be caused A. Risk for infection
by fluid retention manifested by B. Pain
edema, especially of the hands and C. Knowledge Deficit
face. The three classic signs of D. Anticipatory Grieving
preeclampsia are hypertension,
Answer:B . For the client with an C. Encourage her to wear a nursing
ectopic pregnancy, lower abdominal brassiere
pain, usually unilateral, is the D. Use soap and water to clean the
primary symptom. Thus, pain is the nipples
priority. Although the potential for
infection is always present, the risk Answer:A. Feeding more frequently,
is low in ectopic pregnancy because about every 2 hours, will decrease
pathogenic microorganisms have the infant’s frantic, vigorous sucking
not been introduced from external from hunger and will decrease
sources. The client may have a breast engorgement, soften the
limited knowledge of the pathology breast, and promote ease of correct
and treatment of the condition and latching-on for feeding. Narcotics
will most likely experience grieving, administered prior to breast feeding
but this is not the priority at this are passed through the breast milk
time. to the infant, causing excessive
sleepiness. Nipple soreness is not
14.Before assessing the postpartum severe enough to warrant narcotic
client’s uterus for firmness and analgesia. All postpartum clients,
position in relation to the umbilicus especially lactating mothers, should
and midline, which of the following wear a supportive brassiere with
should the nurse do first? wide cotton straps. This does not,
however, prevent or reduce nipple
A. Assess the vital signs soreness. Soaps are drying to the
B. Administer analgesia skin of the nipples and should not
C. Ambulate her in the hall be used on the breasts of lactating
D. Assist her to urinate mothers. Dry nipple
skin predisposes to cracks and
Answer: D. Before uterine fissures, which can become sore
assessment is performed, it is and painful.
essential that the woman empty her
bladder. A full bladder will interfere 16.The nurse assesses the vital
with the accuracy of signs of a client, 4 hours’
the assessment by elevating the postpartum that are as follows: BP
uterus and displacing to the side of 90/60; temperature 100.4ºF; pulse
the midline. Vital sign assessment is 100 weak, thready; R 20 per minute.
not necessary unless an
Which of the following should the
abnormality in uterine assessment
is identified. Uterine assessment nurse do first?
should not cause acute pain that A. Report the temperature to the
requires administration of physician
analgesia. Ambulating the client is B. Recheck the blood pressure with
an essential component of another cuff
postpartum care, but is not C. Assess the uterus for firmness
necessary prior to assessment of and position
the uterus. D. Determine the amount of
lochia
15.Which of the following should the
nurse do when a primipara who Answer: D. A weak, thready pulse
is lactating tells the nurse that she elevated to 100 BPM may indicate
has sore nipples? impending hemorrhagic shock. An
increased pulse is a compensatory
A. Tell her to breast feed more mechanism of the body in response
frequently to decreased fluid volume. Thus,
B. Administer a narcotic before the nurse should check the amount
breast feeding
of lochia present. Temperatures up serosa is a pink to
to 100.48F in the first 24 hours after brownish serosanguineous
birth are related to the dehydrating discharge occurring from 3 to 10
effects of labor and are considered days after delivery that contains
normal. Although rechecking the decidua, erythrocytes, leukocytes,
blood pressure may be a correct cervical mucus,
choice of action, it is not the first and microorganisms. Lochia alba is
action that should be implemented an almost colorless to yellowish
in light of the other data. The data discharge occurring from 10 days to
indicate a potential impending 3 weeks after delivery and
hemorrhage. Assessing the uterus containing leukocytes, decidua,
for firmness and position in relation epithelial cells, fat, cervical mucus,
to the umbilicus and midline is cholesterol crystals, and bacteria.
important, but the nurse
should check the extent of vaginal 18.A postpartum client has a
bleeding first. Then it would be temperature of 101.4ºF, with a
appropriate to check the uterus, uterus that is tender when palpated,
which may be a possible cause of remains unusually large, and not
the hemorrhage. descending as normally expected.
Which of the following should the
17.The nurse assesses the
nurse assess next?
postpartum vaginal discharge
(lochia) on four clients. Which of the A. Lochia
following assessments would B. Breasts
warrant notification of the physician? C. Incision
D. Urine
A. A dark red discharge on a 2-day
postpartum client Answer: A. The data suggests an
B. A pink to brownish discharge on a infection of the endometrial lining of
client who is 5 days postpartum the uterus. The lochia may be
C. Almost colorless to creamy decreased or copious, dark brown in
discharge on a client 2 weeks appearance, and foul smelling,
after delivery providing further evidence of a
D. A bright red discharge 5 days possible infection. All the client’s
after delivery data indicate a uterine problem, not
a breast problem.
Answer: D. Any bright red vaginal Typically, transient fever, usually
discharge would be considered 101ºF, may be present with breast
abnormal, but especially 5 days engorgement. Symptoms of mastitis
after delivery, when the lochia is include influenza-like
typically pink to brownish. Lochia manifestations. Localized infection
rubra, a dark red discharge, is of an episiotomy or C-section
present for 2 to 3 days after incision rarely causes
delivery. Bright red vaginal bleeding systemic symptoms, and uterine
at this time suggests involution would not be affected.
late postpartum hemorrhage, which The client data do not include
occurs after the first 24 hours dysuria, frequency, or urgency,
following delivery and is generally symptoms of urinary
caused by retained placental tract infections, which would
fragments or bleeding disorders. necessitate assessing the client’s
Lochia rubra is the normal dark red urine.
discharge occurring in the first 2 to
3 days after delivery, containing 19.Which of the following is the
epithelial cells, erythrocyes, priority focus of nursing practice
leukocytes and decidua. Lochia
with the current early postpartum prevents heat loss from the head a
discharge? large head, a large body surface
area of the newborn’s body.
A. Promoting comfort and
restoration of health 21.A newborn who has an
B. Exploring the emotional status of asymmetrical Moro reflex response
the family
should be further assessed for which
C. Facilitating safe and effective
of the following?
self-and newborn care
D. Teaching about the importance of A. Talipes equinovarus
family planning B. Fractured clavicle
C. Congenital hypothyroidism
Answer: C. Because of early D. Increased intracranial pressure
postpartum discharge and limited
time for teaching, the nurse’s Answer: B. A fractured clavicle
priority is to facilitate the safe and would prevent the normal Moro
effective care of the client and response of symmetrical sequential
newborn. Although promoting extension and abduction of the
comfort and restoration of arms followed by flexion and
health, exploring the family’s adduction. In talipes equinovarus
emotional status, and teaching (clubfoot) the foot is
about family planning are important turned medially, and in plantar
in postpartum/newborn nursing flexion, with the heel elevated. The
care, they are not the priority focus feet are not involved with the Moro
in the limited time presented by reflex. Hypothyroiddism has no
early post-partum discharge. effect on the primitive reflexes.
Absence of the Moror reflex is the
20. Which of the following actions most significant single indicator of
would be least effective in central nervous system status, but
maintaining a neutral thermal it is not a sign of increased
environment for the newborn? intracranial pressure.
A. Placing infant under radiant 22.During the first 4 hours after a
warmer after bathing male circumcision, assessing for
B. Covering the scale with a
which of the following is the priority?
warmed blanket prior to weighing
C. Placing crib close to nursery A. Infection
window for family viewing B. Hemorrhage
D. Covering the infant’s head with a C. Discomfort
knit stockinette D. Dehydration

Answer: C. Heat loss by radiation Answer:B. Hemorrhage is a


occurs when the infant’s crib is potential risk following any surgical
placed too near cold walls or procedure. Although the infant has
windows. Thus placing the been given vitamin K to facilitate
newborn’s crib close to the viewing clotting, the prophylactic dose is
window would be least effective. often not sufficient to prevent
Body heat is lost bleeding. Although infection is a
through evaporation during bathing. possibility, signs will not appear
Placing the infant under the radiant within 4 hours after the surgical
warmer after bathing will assist the procedure. The primary discomfort
infant to be rewarmed. Covering the of circumcision occurs during the
scale with a warmed blanket prior surgical procedure, not afterward.
to weighing prevents heat loss Although feedings are withheld
through conduction. A knit cap prior to the circumcision, the
chances of dehydration are D. Recognize this as normal first
minimal. period of reactivity

23.The mother asks the nurse. Answer: D . The first 15 minutes to


“What’s wrong with my son’s 1 hour after birth is the first period
breasts? Why are they so enlarged?” of reactivity involving respiratory
Whish of the following would be the and circulatory adaptation to
best response by the nurse? extrauterine life. The data given
reflect the normal changes during
A. “The breast tissue is inflamed this time period. The
from the trauma experienced infant’s assessment data reflect
with birth” normal adaptation. Thus, the
B. “A decrease in material physician does not need to be
hormones present before notified and oxygen is not needed.
birth causes enlargement,” The data do not indicate
C. “You should discuss this with the presence of choking, gagging or
your doctor. It could be a coughing, which are signs of
malignancy” excessive secretions. Suctioning is
D. “The tissue has hypertrophied not necessary.
while the baby was in the uterus”
25.The nurse hears a mother telling
Answer: B . The presence of a friend on the telephone about
excessive estrogen and umbilical cord care. Which of the
progesterone in the maternal following statements by the mother
fetal blood followed by prompt indicates effective teaching?
withdrawal at birth precipitates
breast engorgement, which will A. “Daily soap and water cleansing
spontaneously resolve in 4 to 5 is best”
days after birth. The trauma of the B. ‘Alcohol helps it dry and kills
birth process does not cause germs”
inflammation of the newborn’s C. “An antibiotic ointment applied
breast tissue. Newborns do not daily prevents infection”
have breast malignancy. This reply D. “He can have a tub bath each
by the nurse would cause the day”
mother to have undue anxiety.
Breast tissue does not hypertrophy Answer: B. Application of 70%
in the fetus or newborns. isopropyl alcohol to the cord
minimizes microorganisms
24. Immediately after birth the nurse (germicidal) and promotes drying.
notes the following on a male The cord should be kept dry until it
newborn: respirations 78; apical falls off and the stump has healed.
hearth rate 160 BPM, nostril flaring; Antibiotic ointment should only be
mild intercostal used to treat an infection, not as a
prophylaxis. Infants should not be
retractions; and grunting at the end submerged in a tub of water until
of expiration. Which of the the cord falls off and the stump has
following should the nurse do? completely healed.
A. Call the assessment data to the 26.A newborn weighing 3000 grams
physician’s attention and feeding every 4 hours needs
B. Start oxygen per nasal cannula at
120 calories/kg of body weight every
2 L/min.
C. Suction the infant’s mouth and 24 hours for proper growth
nares and development. How many ounces
of 20 cal/oz formula should this
newborn receive at each feeding to B. From the symphysis pubis to the
meet nutritional needs? xiphoid process
C. From the symphysis pubis to
A. 2 ounces the fundus
B. 3 ounces D. From the fundus to the umbilicus
C. 4 ounces
D. 6 ounces Answer: C . The nurse should use a
nonelastic, flexible, paper
Answer:B. To determine the amount measuring tape, placing the zero
of formula needed, do the point on the superior border of the
following mathematical calculation. symphysis pubis and stretching the
3 kg x 120 cal/kg per day = 360 tape across the abdomen at the
calories/day feeding q 4 hours = 6 midline to the top of the fundus.
feedings per day = 60 calories per The xiphoid and umbilicus are not
feeding: 60 calories per feeding; 60 appropriate landmarks to use when
calories per feeding with formula 20 measuring the height of the fundus
cal/oz = 3 ounces per feeding. (McDonald’s measurement).
Based on the calculation. 2, 4 or 6
ounces are incorrect. 29.A client with severe preeclampsia
is admitted with of BP
27.The postterm neonate with
160/110, proteinuria, and severe
meconium-stained amniotic fluid
pitting edema. Which of the
needs care designed to especially
following would be most important
monitor for which of the following?
to include in the client’s plan of
A. Respiratory problems care?
B. Gastrointestinal problems
A. Daily weights
C. Integumentary problems
B. Seizure precautions
D. Elimination problems
C. Right lateral positioning
D. Stress reduction
Answer: A. Intrauterine anoxia may
cause relaxation of the anal
Answer: B . Women hospitalized
sphincter and emptying of
with severe preeclampsia need
meconium into the amniotic fluid. At
decreased CNS stimulation to
birth some of the meconium fluid
prevent a seizure. Seizure
may be aspirated, causing
precautions provide environmental
mechanical obstruction or chemical
safety should a seizure occur.
pneumonitis. The infant is not at
Because of edema, daily weight is
increased risk for gastrointestinal
important but not the priority.
problems. Even though the skin is
Preclampsia causes vasospasm and
stained with meconium, it is
therefore can reduce utero-
noninfectious (sterile) and
placental perfusion. The client
nonirritating. The postterm
should be placed on her left side to
meconiumstained infant is not at
maximize blood flow, reduce blood
additional risk for bowel or urinary
pressure, and promote diuresis.
problems.
Interventions to reduce stress and
anxiety are very important to
28.When measuring a client’s fundal
facilitate coping and a sense of
height, which of the following
control, but seizure precautions are
techniques denotes the correct the priority.
method of measurement used by the
nurse? 30. A postpartum primipara asks the
A. From the xiphoid process to the nurse, “When can we have
umbilicus sexual intercourse again?” Which of
the following would be the nurse’s newborn but is not the safest.
best response? Because of the proximity of the
sciatic nerve, the gluteus maximus
A. “Anytime you both want to.” muscle should not be until the child
B. “As soon as choose a has been walking 2 years.
contraceptive method.”
C. “When the discharge has 32.When performing a pelvic
stopped and the incision is examination, the nurse observes a
healed.”
red swollen area on the right side of
D. “After your 6 weeks
the vaginal orifice. The nurse would
examination.”
document this as enlargement of
Answer: C. Cessation of the lochial which of the following?
discharge signifies healing of the A. Clitoris
endometrium. Risk of hemorrhage B. Parotid gland
and infection are minimal 3 weeks C. Skene’s gland
after a normal vaginal delivery. D. Bartholin’s gland
Telling the client anytime is
inappropriate because this response Answer: D . Bartholin’s glands are
does not provide the client with the the glands on either side of the
specific information she vaginal orifice. The clitoris is female
is requesting. Choice of a erectile tissue found in the perineal
contraceptive method is important, area above the urethra. The parotid
but not the specific criteria for safe glands are open into the mouth.
resumption of sexual activity. Skene’s glands open into the
Culturally, the 6- weeks’ posterior wall of the female urinary
examination has been used as the meatus.
time frame for resuming
sexual activity, but it may be 33.To differentiate as a female, the
resumed earlier. hormonal stimulation of the embryo
that must occur involves which of
31.When preparing to administer the
the following?
vitamin K injection to a neonate,
the nurse would select which of the A. Increase in maternal estrogen
following sites as appropriate for secretion
the injection? B. Decrease in maternal androgen
secretion
A. Deltoid muscle C. Secretion of androgen by the
B. Anterior femoris muscle fetal gonad
C. Vastus lateralis muscle D. Secretion of estrogen by the
D. Gluteus maximus muscle fetal gonad

Answer: C . The middle third of the Answer: D . The fetal gonad must
vastus lateralis is the preferred secrete estrogen for the embryo to
injection site for vitamin K differentiate as a female. An
administration because it is free of increase in maternal estrogen
blood vessels and nerves and is secretion does not
large enough to absorb the effect differentiation of the embryo,
medication. The deltoid muscle of and maternal estrogen secretion
a newborn is not large enough for a occurs in every pregnancy. Maternal
newborn IM injection. Injections into androgen secretion remains the
this muscle in a small child might same as before pregnancy and does
cause damage to the radial nerve. not effect differentiation. Secretion
The anterior femoris muscle is the of androgen by the fetal gonad
next safest muscle to use in a would produce a male fetus.
A. Braxton-Hicks sign
34.A client at 8 weeks’ gestation B. Chadwick’s sign
calls complaining of slight nausea in C. Goodell’s sign
the morning hours. Which of the D. McDonald’s sign
following client interventions should
the nurse question? Answer: B . Chadwick’s sign refers
to the purple-blue tinge of the
A. Taking 1 teaspoon of cervix. Braxton Hicks contractions
bicarbonate of soda in an 8- are painless contractions beginning
ounce glass of water around the 4th month. Goodell’s
B. Eating a few low-sodium crackers sign indicates softening of the
before getting out of bed cervix. Flexibility of the uterus
C. Avoiding the intake of liquids in against the cervix is known as
the morning hours McDonald’s sign.
D. Eating six small meals a day
instead of thee large meals 37.During a prenatal class, the nurse
explains the rationale for
Answer: A . Using bicarbonate breathing techniques during
would increase the amount of preparation for labor based on the
sodium ingested, which can cause understanding that breathing
complications. Eating low-sodium
techniques are most important in
crackers would be appropriate.
Since liquids can increase nausea achieving which of the following?
avoiding them in the morning hours A. Eliminate pain and give the
when nausea is usually the expectant parents something to
strongest is appropriate. Eating six do
small meals a day would keep the B. Reduce the risk of fetal distress
stomach full, which often by increasing
decrease nausea. uteroplacental perfusion
C. Facilitate relaxation, possibly
35.The nurse documents positive reducing the perception of
ballottement in the client’s prenatal pain
record. The nurse understands that D. Eliminate pain so that less
this indicates which of the following? analgesia and anesthesia are
needed
A. Palpable contractions on the
abdomen Answer: C . Breathing techniques
B. Passive movement of the can raise the pain threshold and
unengaged fetus reduce the perception of pain. They
C. Fetal kicking felt by the client also promote relaxation. Breathing
D. Enlargement and softening of the techniques do not eliminate pain,
uterus but they can reduce it. Positioning,
not breathing, increases
Answer: B . Ballottement indicates uteroplacental perfusion.
passive movement of the
unengaged fetus. Ballottement is 38.After 4 hours of active labor, the
not a contraction. Fetal kicking felt nurse notes that the contractions of
by the client represents quickening.
a primigravida client are not strong
Enlargement and softening of the
uterus is known as Piskacek’s sign. enough to dilate the cervix. Which of
the
36.During a pelvic exam the nurse following would the nurse anticipate
notes a purple-blue tinge of the doing?
cervix. The nurse documents this as
which of the following?
A. Obtaining an order to begin she must have a cesarean delivery if
IV oxytocin infusion she has a complete placenta previa?
B. Administering a light sedative to
allow the patient to rest for A. “You will have to ask your
several hour physician when he returns.”
C. Preparing for a cesarean section B. “You need a cesarean to prevent
for failure to progress hemorrhage.”
D. Increasing the encouragement to C. “The placenta is covering most of
the patient when pushing begins your cervix.”
D. “The placenta is covering the
Answer:A . The client’s labor is opening of the uterus and
hypotonic. The nurse should call the blocking your baby.”
physical and obtain an order for an
infusion of oxytocin, which will Answer: D . A complete placenta
assist the uterus to contact more previa occurs when the placenta
forcefully in an attempt to dilate the covers the opening of the uterus,
cervix. Administering light sedative thus blocking the passageway for
would be done for hypertonic the baby. This response explains
uterine contractions. Preparing for what a complete previa is and the
cesarean section is unnecessary at reason the baby cannot come out
this time. Oxytocin would except by cesarean delivery. Telling
increase the uterine contractions the client to ask the physician is a
and hopefully progress labor before poor response and would increase
a cesarean would be necessary. It is the patient’s anxiety. Although a
too early to anticipate client cesarean would help to prevent
pushing with contractions. hemorrhage, the statement does
not explain why the hemorrhage
39.A multigravida at 38 weeks’ could occur. With a complete previa,
gestation is admitted with painless, the placenta is covering all the
cervix, not just most of it.
bright red bleeding and mild
contractions every 7 to 10 minutes.
41.The nurse understands that the
Which of the following assessments fetal head is in which of the
should be avoided?
following positions with a face
A. Maternal vital sign presentation?
B. Fetal heart rate
A. Completely flexed
C. Contraction monitoring
B. Completely extended
D. Cervical dilation
C. Partially extended
D. Partially flexed
Answer: D . The signs indicate
placenta previa and vaginal exam
Answer: B . With a face
to determine cervical dilation would
presentation, the head is
not be done because it could cause
completely extended. With a vertex
hemorrhage. Assessing maternal
presentation, the head is
vital signs can help determine
completely or partially flexed. With
maternal physiologic status. Fetal
a brow (forehead) presentation, the
heart rate is important to assess
head would be partially extended.
fetal well-being and should be done.
Monitoring the contractions will help
42.With a fetus in the left-anterior
evaluate the progress of labor.
breech presentation, the nurse
40.Which of the following would be would expect the fetal heart rate
the nurse’s most appropriate would be most audible in which of
response to a client who asks why the following areas?
A. Above the maternal umbilicus neonatorum usually results from
and to the right of midline maternal gonorrhea and is
B. In the lower-left maternal conjunctivitis. Pica refers to the oral
abdominal quadrant intake of nonfood substances.
C. In the lower-right maternal
abdominal quadrant 45.When describing dizygotic twins
D. Above the maternal umbilicus to a couple, on which of the
and to the left of midline following would the nurse base the
explanation?
Answer: D . With this presentation,
the fetal upper torso and back face A. Two ova fertilized by
the left upper maternal abdominal separate sperm
wall. The fetal heart rate would be B. Sharing of a common placenta
most audible above the maternal C. Each ova with the same
umbilicus and to the left of the genotype
middle.A The other positions would D. Sharing of a common chorion
be incorrect.
Answer: A . Dizygotic (fraternal)
43.The amniotic fluid of a client has twins involve two ova fertilized by
a greenish tint. The nurse interprets separate sperm. Monozygotic
this to be the result of which of the (identical) twins involve a common
following? placenta, same genotype, and
common chorion.
A. Lanugo
B. Hydramnio 46.Which of the following refers to
C. Meconium the single cell that reproduces itself
D. Vernix after conception?

Answer: C. The greenish tint is due A. Chromosome


to the presence of meconium. B. Blastocyst
Lanugo is the soft, downy hair on C. Zygote
the shoulders and back of the fetus. D. Trophoblast
Hydramnios represents excessive
amniotic fluid. Vernix is the white, Answer: C . The zygote is the single
cheesy substance covering the cell that reproduces itself after
fetus. conception. The chromosome is the
material that makes up the cell and
44.A patient is in labor and has just is gained from each parent.
been told she has a breech Blastocyst and trophoblast are later
presentation. The nurse should be terms for the embryo after zygote.
particularly alert for which of the
47.In the late 1950s, consumers and
following?
health care professionals
A. Quickening began challenging the routine use of
B. Ophthalmia neonatorum analgesics and anesthetics during
C. Pica childbirth. Which of the following
D. Prolapsed umbilical cord was an outgrowth of this concept?

Answer: D . In a breech position, A. Labor, delivery, recovery,


because of the space between the postpartum (LDRP)
presenting part and the cervix, B. Nurse-midwifery
prolapse of the umbilical cord is C. Clinical nurse specialist
common. Quickening is the D. Prepared childbirth
woman’s first perception of fetal
movement. Ophthalmia
Answer: D . Prepared childbirth was ischemic phases do not contribute
the direct result of the 1950’s to this variation.
challenging of the routine use of
analgesic and anesthetics during 50.When teaching a group of
childbirth. The LDRP was a much adolescents about male hormone
later concept and was not a direct production, which of the following
result of the challenging of routine would the nurse include as being
use of analgesics and anesthetics produced by the Leydig cells?
during childbirth. Roles for nurse
midwives and clinical nurse A. Follicle-stimulating hormone
specialists did not develop from B. Testosterone
this challenge. C. Leuteinizing hormone
D. Gonadotropin releasing hormone
48.A client has a midpelvic
contracture from a previous pelvic Answer: B . Testosterone is
injury due to a motor vehicle produced by the Leyding cells in the
accident as a teenager. The nurse is seminiferous tubules. Follicle-
aware that this could stimulating hormone and
leuteinzing hormone are released
prevent a fetus from passing by the anterior pituitary gland. The
through or around which structure hypothalamus is responsible for
during childbirth? releasing gonadotropin-releasing
hormone.
A. Symphysis pubis
B. Sacral promontory
C. Ischial spines
D. Pubic arch
PNLE III for
Answer: C . The ischial spines are
Medical Surgical
located in the mid-pelvic region and
could be narrowed due to the
Nursing
previous pelvic injury. The
symphysis pubis, 1. Marco who was diagnosed
sacral promontory, and pubic arch with brain tumor was scheduled for
are not part of the mid-pelvis. craniotomy. In preventing the
development of cerebral edema
49.When teaching a group of after surgery, the nurse should
adolescents about variations in the expect the use of:
length of the menstrual cycle, the
nurse understands that the A. Diuretics
underlying mechanism is B. Antihypertensive
C. Steroids
due to variations in which of the D. Anticonvulsants
following phases?
Answer: C . Glucocorticoids
A. Menstrual phase (steroids) are used for their anti-
B. Proliferative phase inflammatory action,
C. Secretory phase which decreases the development
D. Ischemic phase of edema.
Answer: B . Variations in the length 2. Halfway through the
of the menstrual cycle are due to administration of blood, the female
variations in the proliferative phase. client complains of lumbar pain.
The menstrual, secretory and After stopping the infusion Nurse
Hazel should:
A. Increase the flow of normal to develop complications. Which
saline among the following complications
B. Assess the pain further should the nurse anticipates:
C. Notify the blood bank
D. Obtain vital signs. A. Flapping hand tremors
B. An elevated hematocrit level
Answer: A . The blood must be C. Hypotension
stopped at once, and then normal D. Hypokalemia
saline should be infused to keep the
line patent and maintain blood Answer: A . Elevation of uremic
volume. waste products causes irritation of
the nerves, resulting in flapping
3. Nurse Maureen knows that the hand tremors.
positive diagnosis for HIV infection is
made based on which of the 6. A client is admitted to the hospital
following: with benign prostatic hyperplasia,
the nurse most relevant assessment
A. A history of high risk sexual would be:
behaviors.
B. Positive ELISA and western A. Flank pain radiating in the groin
blot tests B. Distention of the lower
C. Identification of an associated abdomen
opportunistic infection C. Perineal edema
D. Evidence of extreme weight loss D. Urethral discharge
and high fever
Answer: B . This indicates that the
Answer:B . These tests confirm the bladder is distended with urine,
presence of HIV antibodies that therefore palpable.
occur in response to the presence of
the human immunodeficiency virus 7. A client has undergone with penile
(HIV). implant. After 24 hrs of surgery, the
client’s scrotum was edematous and
4. Nurse Maureen is aware that a painful. The nurse should:
client who has been diagnosed with
chronic renal failure recognizes an A. Assist the client with sitz bath
adequate amount of high-biologic- B. Apply war soaks in the scrotum
value protein when the food the C. Elevate the scrotum using a
client selected from the menu was: soft support
D. Prepare for a possible incision
A. Raw carrots and drainage.
B. Apple juice
C. Whole wheat bread Answer: C . Elevation increases
D. Cottage cheese lymphatic drainage, reducing
edema and pain.
Answer: D . One cup of cottage
cheese contains approximately 225 8. Nurse hazel receives emergency
calories, 27 g of protein, 9 g of fat, laboratory results for a client with
30 mg cholesterol, and 6 g of chest pain and immediately informs
carbohydrate. Proteins of high the physician. An increased
biologic value (HBV) contain optimal myoglobin level suggests which of
levels of amino acids essential the following?
for life.
A. Liver disease
5. Kenneth who has diagnosed with B. Myocardial damage
uremic syndrome has the potential C. Hypertension
D. Cancer
Answer: C . Because of its
Answer: B . Detection of myoglobin widespread vasodilating effects,
is a diagnostic tool to determine nitroglycerin often produces side
whether myocardial damage has effects such as headache,
occurred. hypotension and dizziness.

9. Nurse Maureen would expect the 12. The following are lipid
a client with mitral stenosis would abnormalities. Which of the following
demonstrate symptoms associated is a risk factor for the development
with congestion in the: of atherosclerosis and PVD?

A. Right atrium A. High levels of low density


B. Superior vena cava lipid (LDL) cholesterol
C. Aorta B. High levels of high density lipid
D. Pulmonary (HDL) cholesterol
C. Low concentration triglycerides
Answer: D . When mitral stenosis is D. Low levels of LDL cholesterol.
present, the left atrium has
difficulty emptying its contents into Answer: A. An increased in LDL
the left ventricle because there is cholesterol concentration has been
no valve to prevent back ward flow documented at risk factor for the
into the pulmonary vein, the development of atherosclerosis. LDL
pulmonary circulation is under cholesterol is not broken down into
pressure. the liver but is deposited into the
wall of the blood vessels.
10. A client has been diagnosed with
hypertension. The nurse priority 13. Which of the following
nursing diagnosis would be: represents a significant risk
immediately after surgery for repair
A. Ineffective health of aortic aneurysm?
maintenance
B. Impaired skin integrity A. Potential wound infection
C. Deficient fluid volume B. Potential ineffective coping
D. Pain C. Potential electrolyte balance
D. Potential alteration in renal
Answer: A . Managing hypertension perfusion
is the priority for the client with
hypertension. Clients with Answer: D . There is a potential
hypertension frequently do not alteration in renal perfusion
experience pain, deficient manifested by decreased urine
volume, or impaired skin integrity. It output. The altered renal perfusion
is the asymptomatic nature of may be related to renal
hypertension that makes it so artery embolism, prolonged
difficult to treat. hypotension, or prolonged aortic
cross-clamping during the surgery.
11. Nurse Hazel teaches the client
with angina about common 14. Nurse Josie should instruct the
expected side effects of nitroglycerin client to eat which of the following
including: foods to obtain the best supply of
Vitamin B12?
A. high blood pressure
B. stomach cramps A. dairy products
C. headache B. vegetables
D. shortness of breath C. Grains
D. Broccoli
18. Marie with acute lymphocytic
Answer: A . Good source of vitamin leukemia suffers from nausea and
B12 are dairy products and meats. headache. These clinical
manifestations may indicate all of
15. Karen has been diagnosed with the following except
aplastic anemia. The nurse monitors
for changes in which of the following A. effects of radiation
physiologic functions? B. chemotherapy side effects
C. meningeal irritation
A. Bowel function D. gastric distension
B. Peripheral sensation
C. Bleeding tendencies Answer: D . Acute Lymphocytic
D. Intake and out put Leukemia (ALL) does not cause
gastric distention. It does invade
Answer: C . Aplastic anemia the central nervous system, and
decreases the bone marrow clients experience headaches and
production of RBC’s, white blood vomiting from meningeal irritation.
cells, and platelets. The client is at
risk for bruising and 19. A client has been diagnosed with
bleeding tendencies. Disseminated Intravascular
Coagulation (DIC). Which of the
16. Lydia is scheduled for elective following is contraindicated with the
splenectomy. Before the clients goes client?
to surgery, the nurse in charge final
assessment would be: A. Administering Heparin
B. Administering Coumadin
A. signed consent C. Treating the underlying cause
B. vital signs D. Replacing depleted blood
C. name band products
D. empty bladder
Answer: B . Disseminated
Answer: B . An elective procedure is Intravascular Coagulation (DIC) has
scheduled in advance so that all not been found to respond to oral
preparations can be completed anticoagulants such as Coumadin.
ahead of time. The vital signs are
the final check that must 20. Which of the following findings is
be completed before the client the best indication that fluid
leaves the room so that continuity replacement for the client with
of care and assessment is provided hypovolemic shock is adequate?
for.
A. Urine output greater than
17. What is the peak age range in 30ml/hr
acquiring acute lymphocytic B. Respiratory rate of 21
leukemia (ALL)? breaths/minute
C. Diastolic blood pressure greater
A. 4 to 12 years. than 90 mmhg
B. 20 to 30 years D. Systolic blood pressure greater
C. 40 to 50 years than 110 mmhg
D. 60 60 70 years
Answer: A . Urine output provides
Answer: A . The peak incidence of the most sensitive indication of the
Acute Lymphocytic Leukemia (ALL) client’s response to therapy for
is 4 years of age. It is uncommon hypovolemic shock. Urine output
after 15 years of age.
should be consistently greater than
30 to 35 mL/hr. Answer: C . The osmotic diuretic
mannitol is contraindicated in the
21. Which of the following signs and presence of inadequate renal
symptoms would Nurse Maureen function or heart failure because it
include in teaching plan as an early increases the intravascular volume
manifestation of laryngeal cancer? that must be filtered and excreted
by the kidney.
A. Stomatitis
B. Airway obstruction 24. Patricia a 20 year old college
C. Hoarseness student with diabetes mellitus
D. Dysphagia requests additional information
about the advantages of using a pen
Answer: C . Early warning signs of like insulin
laryngeal cancer can vary delivery devices. The nurse explains
depending on tumor location. that the advantages of these
Hoarseness lasting 2 weeks should devices over syringes includes:
be evaluated because it is one of
the most common warning signs. A. Accurate dose delivery
B. Shorter injection time
22. Karina a client with myasthenia C. Lower cost with reusable insulin
gravis is to receive cartridges
immunosuppressive therapy. The D. Use of smaller gauge needle.
nurse understands that this therapy
is effective because it: Answer: A . These devices are more
accurate because they are easily to
A. Promotes the removal of used and have improved adherence
antibodies that impair the in insulin regimens by young people
transmission of impulses because the medication can be
B. Stimulates the production of administered discreetly.
acetylcholine at the
neuromuscular junction. 25. A male client’s left tibia is
C. Decreases the production of fractures in an automobile accident,
autoantibodies that attack and a cast is applied. To assess for
the acetylcholine receptors. damage to major blood vessels from
D. Inhibits the breakdown of the fracture tibia, the nurse in
acetylcholine at the charge should monitor the client for:
neuromuscular junction.
A. Swelling of the left thigh
Answer: C . Steroids decrease the B. Increased skin temperature of
body’s immune response thus the foot
decreasing the production of C. Prolonged reperfusion of the
antibodies that attack the toes after blanching
acetylcholine receptors at D. Increased blood pressure
the neuromuscular junction
Answer: C . Damage to blood
23. A female client is receiving IV vessels may decrease the
Mannitol. An assessment specific to circulatory perfusion of the toes,
safe administration of the said drug this would indicate the lack of blood
is: supply to the extremity.

A. Vital signs q4h 26. After a long leg cast is removed,


B. Weighing daily the male client should:
C. Urine output hourly
D. Level of consciousness q4h
A. Cleanse the leg by scrubbing A. Active joint flexion and
with a brisk motion extension
B. Put leg through full range of B. Continued immobility until pain
motion twice daily subsides
C. Report any discomfort or stiffness C. Range of motion exercises twice
to the physician daily
D. Elevate the leg when sitting D. Flexion exercises three times
for long periods of time. daily

Answer: D . Elevation will help Answer: A . Active exercises,


control the edema that usually alternating extension, flexion,
occurs. abduction, and adduction, mobilize
exudates in the joints relieves
27. While performing a physical stiffness and pain.
assessment of a male client with
gout of the great toe, NurseVivian 30. A male client has undergone
should assess for additional tophi spinal surgery, the nurse should:
(urate deposits) on the:
A. Observe the client’s bowel
A. Buttocks movement and voiding patterns
B. Ears B. Log-roll the client to prone
C. Face position
D. Abdomen C. Assess the client’s feet for
sensation and circulation
Answer: B . Uric acid has a low D. Encourage client to drink plenty
solubility, it tends to precipitate and of fluids
form deposits at various sites where
blood flow is least active, including Answer: C . Alteration in sensation
cartilaginous tissue such as the and circulation indicates damage to
ears. the spinal cord, if these occurs
notify physician immediately.
28. Nurse Katrina would recognize
that the demonstration of crutch 31. Marina with acute renal failure
walking with tripod gait was moves into the diuretic phase after
understood when the client places one week of therapy. During this
weight on the: phase the client must be assessed
for signs of developing:
A. Palms of the hands and axillary
regions A. Hypovolemia
B. Palms of the hand B. renal failure
C. Axillary regions C. metabolic acidosis
D. Feet, which are set apart D. Hyperkalemia

Answer: B . The palms should bear Answer: A . In the diuretic phase


the client’s weight to avoid damage fluid retained during the oliguric
to the nerves in the axilla. phase is excreted and may reach 3
to 5 liters daily, hypovolemia may
29. Mang Jose with rheumatoid occur and fluids should be replaced.
arthritis states, “the only time I am
without pain is when I lie in bed 32. Nurse Judith obtains a specimen
perfectly still”. During the of clear nasal drainage from a client
convalescent stage, the nurse in with a head injury. Which of the
charge with Mang Jose should following tests differentiates mucus
encourage: from cerebrospinal fluid (CSF)?
A. Protein the following instruction is most
B. Specific gravity appropriate?
C. Glucose
D. Microorganism A. “Practice using the mechanical
aids that you will need when
Answer: C . The constituents of CSF future disabilities arise”.
are similar to those of blood B. “Follow good health habits to
plasma. An examination for glucose change the course of the
content is done to determine disease”.
whether a body fluid is a mucus or a C. “Keep active, use stress
CSF. A CSF normally contains reduction strategies, and
glucose. avoid fatigue.
D. “You will need to accept the
33. A 22 year old client suffered necessity for a quiet and inactive
from his first tonic-clonic seizure. lifestyle”.
Upon awakening the client asks the
nurse, “What caused me to have a Answer: C . The nurse most positive
seizure? Which of the following approach is to encourage the client
would the nurse include in the with multiple sclerosis to stay
primary cause of tonic clonic active, use stress reduction
seizures in adults more the 20 techniques and avoid
years? fatigue because it is important to
support the immune system while
A. Electrolyte imbalance remaining active.
B. Head trauma
C. Epilepsy 36. The nurse is aware the early
D. Congenital defect indicator of hypoxia in the
unconscious client is:
Answer: B . Trauma is one of the
primary cause of brain damage and A. Cyanosis
seizure activity in adults. Other B. Increased respirations
common causes of seizure activity C. Hypertension
in adults include D. Restlessness
neoplasms, withdrawal from drugs
and alcohol, and vascular disease. Answer: D . Restlessness is an early
indicator of hypoxia. The nurse
34. What is the priority nursing should suspect hypoxia in
assessment in the first 24 hours unconscious client who suddenly
after admission of the client with becomes restless.
thrombotic CVA?
37. A client is experiencing spinal
A. Pupil size and papillary shock. Nurse Myrna should expect
response the function of the bladder to be
B. cholesterol level which of the following?
C. Echocardiogram
D. Bowel sounds A. Normal
B. Atonic
Answer: A . It is crucial to monitor C. Spastic
the pupil size and papillary D. Uncontrolled
response to indicate changes
around the cranial nerves. Answer: B . In spinal shock, the
bladder becomes completely atonic
35. Nurse Linda is preparing a client and will continue to fill unless the
with multiple sclerosis for discharge client is catheterized.
from the hospital to home. Which of
38. Which of the following stage the Answer: C . Atropine sulfate is
carcinogen is irreversible? contraindicated with glaucoma
patients because it increases
A. Progression stage intraocular pressure.
B. Initiation stage
C. Regression stage 42. Among the following clients,
D. Promotion stage which among them is high risk for
potential hazards from the surgical
Answer: A . Progression stage is the experience?
change of tumor from the
preneoplastic state or low degree of A. 67-year-old client
malignancy to a fast growing tumor B. 49-year-old client
that cannot be reversed. C. 33-year-old client
D. 15-year-old client
39. Among the following
components thorough pain Answer: A . A 67 year old client is
assessment, which is the most greater risk because the older adult
significant? client is more likely to have a less-
effective immune system.
A. Effect
B. Cause 43. Nurse Jon assesses vital signs on
C. Causing factors a client undergone epidural
D. Intensity anesthesia.
Which of the following would the
Answer: D . Intensity is the major nurse assess next?
indicative of severity of pain and it
is important for the evaluation of A. Headache
the treatment. B. Bladder distension
C. Dizziness
40. A 65 year old female is D. Ability to move legs
experiencing flare up of pruritus.
Which of the client’s action could Answer: B . The last area to return
aggravate the cause of flare ups? sensation is in the perineal area,
and the nurse in charge should
A. Sleeping in cool and humidified monitor the client for distended
environment bladder.
B. Daily baths with fragrant
soap 44. Nurse Katrina should anticipate
C. Using clothes made from 100% that all of the following drugs may
cotton be used in the attempt to control the
D. Increasing fluid intake symptoms of Meniere’s disease
except:
Answer: B . The use of fragrant
soap is very drying to skin hence A. Antiemetics
causing the pruritus. B. Diuretics
C. Antihistamines
41. Atropine sulfate (Atropine) is D. Glucocorticoids
contraindicated in all but one of the
following client? Answer: D . Glucocorticoids play no
significant role in disease
A. A client with high blood treatment.
B. A client with bowel obstruction
C. A client with glaucoma 45. Which of the following
D. A client with U.T.I complications associated with
tracheostomy tube?
A. Increased cardiac output 48. Nurse Anna is aware that early
B. Acute respiratory distress adaptation of client with renal
syndrome (ARDS) carcinoma is:
C. Increased blood pressure
D. Damage to laryngeal nerves A. Nausea and vomiting
B. flank pain
Answer: D . Tracheostomy tube has C. weight gain
several potential complications D. intermittent hematuria
including bleeding, infection and
laryngeal nerve damage. Answer: D . Intermittent pain is the
classic sign of renal carcinoma. It is
46. Nurse Faith should recognize primarily due to capillary erosion by
that fluid shift in an client with burn the cancerous growth.
injury results from increase in the:
49. A male client with tuberculosis
A. Total volume of circulating whole asks Nurse Brian how long the
blood chemotherapy must be continued.
B. Total volume of intravascular Nurse Brian’s accurate reply would
plasma be:
C. Permeability of capillary
walls A. 1 to 3 weeks
D. Permeability of kidney tubules B. 6 to 12 months
C. 3 to 5 months
Answer: C . In burn, the capillaries D. 3 years and more
and small vessels dilate, and cell
damage cause the release of a Answer: B . Tubercle bacillus is a
histamine-like substance. The drug resistant organism and takes a
substance causes the capillary walls long time to be eradicated. Usually
to become more permeable and a combination of three drugs is
significant quantities of fluid are used for minimum of 6 months and
lost. at least six months beyond culture
conversion.
47. An 83-year-old woman has
several ecchymotic areas on her 50. A client has undergone
right arm. The bruises are probably laryngectomy. The immediate
caused by: nursing priority would be:

A. increased capillary fragility A. Keep trachea free of


and permeability secretions
B. increased blood supply to the B. Monitor for signs of infection
skin C. Provide emotional support
C. self inflicted injury D. Promote means of
D. elder abuse communication

Answer: A . Aging process involves Answer: A . Patent airway is the


increased capillary fragility and most priority; therefore removal of
permeability. Older adults have a secretions is necessary.
decreased amount of subcutaneous
fat and cause an
increased incidence of bruise like
PNLE IV for
lesions caused by collection of
extravascular blood inloosely Psychiatric
structured dermis.
Nursing
Answer: B . Establishing a consistent
eating plan and monitoring client’s
1. Marco approached Nurse Trish asking
weight are important to this disorder.
for advice on how to deal with his alcohol
addiction. Nurse Trish should tell the
5. A client is experiencing anxiety attack.
client that the only effective treatment
The most appropriate
for alcoholism is:
nursing intervention should include?
A. Psychotherapy
A. Turning on the television
B. Alcoholics anonymous (A.A.)
B. Leaving the client alone
C. Total abstinence
C. Staying with the client and
D. Aversion Therapy
speaking in short sentences
D. Ask the client to play with other
Answer: C . Total abstinence is the only
clients
effective treatment for alcoholism
Answer: C . Appropriate nursing
2. Nurse Hazel is caring for a male client
interventions for an anxiety attack
who experience false
include using short sentences, staying
sensory perceptions with no basis in
with the client, decreasing stimuli,
reality. This perception is known as:
remaining calm and medicating as
needed.
A. Hallucinations
B. Delusions 6. A female client is admitted with a
C. Loose associations diagnosis of delusions of GRANDEUR. This
D. Neologisms diagnosis reflects a belief that one is:
Answer: A . Hallucinations are visual,
A. Being Killed
auditory, gustatory, tactile or
B. Highly famous and important
olfactory perceptions that have no basis
C. Responsible for evil world
in reality.
D. Connected to client unrelated to
oneself
3. Nurse Monet is caring for a female
client who has suicidal tendency. When
Answer: B . Delusion of grandeur is a
accompanying the client to the restroom,
false belief that one is highly famous
Nurse Monet should…
and important.
A. Give her privacy 7. A 20 year old client was diagnosed
B. Allow her to urinate with dependent personality
C. Open the window and allow her to disorder. Which behavior is not likely to
get some fresh air be evidence of ineffective individual
D. Observe her coping?
Answer: D . The Nurse has a
A. Recurrent self-destructive behavior
responsibility to observe continuously
B. Avoiding relationship
the acutely suicidal client. The Nurse
C. Showing interest in solitary activities
should watch for clues, such as
D. Inability to make choices and
communicating suicidal thoughts, and
decision without advise
messages; hoarding medications and
talking about death.
Answer: D . Individual with dependent
personality disorder typically
4. Nurse Maureen is developing a plan of
shows indecisiveness submissiveness
care for a female client with anorexia
and clinging behavior so that others
nervosa. Which action should the nurse
will make decisions with them.
include in the plan?
8. A male client is diagnosed with
A. Provide privacy during meals schizotypal personality disorder.
B. Set-up a strict eating plan for the Which signs would this client exhibit
client during social situation?
C. Encourage client to exercise to
reduce anxiety
A. Paranoid thoughts
D. Restrict visits with the family
B. Emotional affect
C. Independence need The symptom that is unrelated to
D. Aggressive behavior depression would be?

Answer: A . Clients with schizotypal A. Apathetic response to the


personality disorder experience environment
excessive social anxiety that can lead to B. “I don’t know” answer to questions
paranoid thoughts C. Shallow of labile effect
D. Neglect of personal hygiene
9. Nurse Claire is caring for a client
diagnosed with bulimia. The Answer: C . With depression, there is
most appropriate initial goal for a client little or no emotional involvement
diagnosed with bulimia is? therefore little alteration in affect.

A. Encourage to avoid foods 13.Nurse Trish is working in a mental


B. Identify anxiety causing health facility; the nurse priority
situations nursing intervention for a newly admitted
C. Eat only three meals a day client with bulimia nervosa would be to?
D. Avoid shopping plenty of groceries
A. Teach client to measure I & O
Answer: B . Bulimia disorder generally is B. Involve client in planning daily meal
a maladaptive coping response to stress C. Observe client during meals
and underlying issues. The client should D. Monitor client continuously
identify anxiety causing situation
that stimulate the bulimic behavior and Answer: D . These clients often hide food
then learn new ways of coping with or force vomiting; therefore they must
the anxiety. be carefully monitored.

10. Nurse Tony was caring for a 41 year 14.Nurse Patricia is aware that the major
old female client. Which behavior by the health complication associated
client indicates adult cognitive with intractable anorexia nervosa would
development? be?

A. Generates new levels of A. Cardiac dysrhythmias resulting


awareness to cardiac arrest
B. Assumes responsibility for her actions B. Glucose intolerance resulting in
C. Has maximum ability to solve protracted hypoglycemia
problems and learn new skills C. Endocrine imbalance causing cold
D. Her perception are based on reality amenorrhea
D. Decreased metabolism causing cold
Answer: A . An adult age 31 to 45 intolerance
generates new level of awareness.
Answer: A . These clients have severely
11.A neuromuscular blocking agent is depleted levels of sodium and
administered to a client before potassium because of their starvation
ECT therapy. The Nurse should carefully diet and energy expenditure, these
observe the client for? electrolytes are necessary for cardiac
functioning.
A. Respiratory difficulties
B. Nausea and vomiting 15.Nurse Anna can minimize agitation in
C. Dizziness a disturbed client by?
D. Seizures
A. Increasing stimulation
Answer: A . Neuromuscular Blocker, B. limiting unnecessary interaction
such as SUCCINYLCHOLINE C. increasing appropriate sensory
(Anectine) produces respiratory perception
depression because it inhibits D. ensuring constant client and staff
contractions of respiratory muscles. contact

12.A 75 year old client is admitted to the Anaswer: B . Limiting unnecessary


hospital with the diagnosis of dementia interaction will decrease stimulation and
of the Alzheimer’s type and depression. agitation.
19.Joey a client with antisocial
16.A 39 year old mother with obsessive- personality disorder belches loudly. A
compulsive disorder has staff member asks Joey, “Do you know
become immobilized by her elaborate why people find you repulsive?”
hand washing and walking rituals. this statement most likely would elicit
Nurse Trish recognizes that the basis of which of the following client reaction?
O.C. disorder is often:
A. Depensiveness
A. Problems with being too B. Embarrassment
conscientious C. Shame
B. Problems with anger and remorse D. Remorsefulness
C. Feelings of guilt and inadequacy
D. Feeling of unworthiness and A . When the staff member ask the
hopelessness client if he wonders why others find
him repulsive, the client is likely to feel
Answer: C . Ritualistic behavior seen in defensive because the question
this disorder is aimed at controlling guilt is belittling. The natural tendency is to
and inadequacy by maintaining an counterattack the threat to self image.
absolute set pattern of behavior.
20.Which of the following approaches
17.Mario is complaining to other clients would be most appropriate to use with a
about not being allowed by staff to keep client suffering from narcissistic
food in his room. Which of the following personality disorder when
interventions would be most appropriate? discrepancies exist between what the
client states and what actually exist?
A. Allowing a snack to be kept in his
room A. Rationalization
B. Reprimanding the client B. Supportive confrontation
C. Ignoring the clients behavior C. Limit setting
D. Setting limits on the behavior D. Consistency

Answer: D . The nurse needs to set Answer: B . The nurse would specifically
limits in the client’s manipulative use supportive confrontation with the
behavior to help the client control client to point out discrepancies
dysfunctional behavior. A consistent between what the client states and what
approach by the staff is necessary to actually exists to increase responsibility
decrease manipulation. for self.

18.Conney with borderline personality 21.Cely is experiencing alcohol


disorder who is to be discharge withdrawal exhibits tremors, diaphoresis
soon threatens to “do something” to and hyperactivity. Blood pressure is
herself if discharged. Which of the 190/87 mmhg and pulse is 92 bpm.
following actions by the nurse would be Which of the medications would the
most important? nurse expect to administer?

A. Ask a family member to stay with the A. Naloxone (Narcan)


client at home temporarily B. Benzlropine (Cogentin)
B. Discuss the meaning of the C. Lorazepam (Ativan)
client’s statement with her D. Haloperidol (Haldol)
C. Request an immediate extension for
the client Answer: C . The nurse would most likely
D. Ignore the clients statement because administer benzodiazepine, such as
it’s a sign of manipulation lorazepan (ativan) to the client who is
experiencing symptom: The client’s
Answer: B . Any suicidal statement must experiences symptoms of withdrawal
be assessed by the nurse. The nurse because of the rebound phenomenon
should discuss the client’s statement when the sedation of the CNS from
with her to determine its meaning in alcohol begins to decrease.
terms of suicide
22.Which of the following foods would D. Use natural remedies rather than
the nurse Trish eliminate from the diet of drugs to control behavior
a client in alcohol withdrawal?
Answer: A . Environmental (MILIEU)
A. Milk therapy aims at having everything in the
B. Orange Juice client’s surrounding area toward helping
C. Soda the client.
D. Regular Coffee
26.Nurse Trish would expect a child with
Answer: D . Regular coffee contains a diagnosis of reactive
caffeine which acts as psychomotor attachment disorder to:
stimulants and leads to feelings of
anxiety and agitation. Serving coffee top A. Have more positive relation with the
the client may add to tremors or father than the mother
wakefulness. B. Cling to mother & cry on separation
C. Be able to develop only
23.Which of the following would Nurse superficial relation with the
Hazel expect to assess for a client who is others
exhibiting late signs of heroin D. Have been physically abuse
withdrawal?
Answer: C . Children who have
A. Yawning & diaphoresis experienced attachment difficulties with
B. Restlessness & Irritability primary caregiver are not able to trust
C. Constipation & steatorrhea others and therefore relate superficially
D. Vomiting and Diarrhea
27.When teaching parents about
Answer: D . Vomiting and diarrhea are childhood depression Nurse Trina
usually the late signs of heroin should say?
withdrawal, along with muscle spasm,
fever, nausea, repetitive, abdominal A. It may appear acting out
cramps and backache. behavior
B. Does not respond to conventional
24.To establish open and trusting treatment
relationship with a female client who C. Is short in duration & resolves easily
has been hospitalized with severe D. Looks almost identical to adult
anxiety, the nurse in charge should? depression

A. Encourage the staff to have frequent Answer: A . Children have difficulty


interaction with the client verbally expressing their feelings, acting
B. Share an activity with the client out behavior, such as temper tantrums,
C. Give client feedback about behavior may indicate underlying depression.
D. Respect client’s need for
personal space 28.Nurse Perry is aware that language
development in autistic child resembles:
Answer: D . Moving to a client’s personal
space increases the feeling of threat, A. Scanning speech
which increases anxiety. B. Speech lag
C. Shuttering
25. Nurse Monette recognizes that the D. Echolalia
focus of environmental (MILIEU) therapy Answer: D . The autistic child repeat
is to: sounds or words spoken by others.

A. Manipulate the environment to 29.A 60 year old female client who lives
bring about positive changes alone tells the nurse at the community
in behavior health center “I really don’t need anyone
B. Allow the client’s freedom to to talk to”. The TV is my best friend. The
determine whether or not they will nurse recognizes that the client is using
be involved in activities the defense mechanism known as?
C. Role play life events to meet
individual needs A. Displacement
B. Projection
C. Sublimation induced persisting dementia; the client
D. Denial cannot remember facts and fills in the
gaps with imaginary information. Nurse
Answer: D . The client statement is an Benjie is aware that this is typical of?
example of the use of denial, a defense
that blocks problem by unconscious A. Flight of ideas
refusing to admit they exist B. Associative looseness
C. Confabulation
30.When working with a male client D. Concretism
suffering phobia about black cats,
Nurse Trish should anticipate that a Answer: C . Confabulation or the filling in
problem for this client would be? of memory gaps with imaginary facts is
a defense mechanism used by people
A. Anxiety when discussing phobia experiencing memory deficits.
B. Anger toward the feared object
C. Denying that the phobia exist 34.Nurse Joey is aware that the signs &
D. Distortion of reality when completing symptoms that would be most specific
daily routines for diagnosis anorexia are?

Answer: A . Discussion of the feared A. Excessive weight loss,


object triggers an emotional response to amenorrhea & abdominal
the object. distension
B. Slow pulse, 10% weight loss &
31.Linda is pacing the floor and appears alopecia
extremely anxious. The duty C. Compulsive behavior, excessive fears
nurse approaches in an attempt to & nausea
alleviate Linda’s anxiety. The D. Excessive activity, memory lapses &
most therapeutic question by the nurse an increased pulse
would be?
Answer: A . These are the major signs of
A. Would you like to watch TV? anorexia nervosa. Weight loss is
B. Would you like me to talk with excessive (15% of expected weight)
you?
C. Are you feeling upset now? 35.A characteristic that would suggest to
D. Ignore the client Nurse Anne that an adolescent may have
bulimia would be:
Answer: B . The nurse presence may
provide the client with support & feeling A. Frequent regurgitation & re-
of control. swallowing of food
B. Previous history of gastritis
32.Nurse Penny is aware that the C. Badly stained teeth
symptoms that distinguish post D. Positive body image
traumatic stress disorder from other
anxiety disorder would be: Answer: C . Dental enamel erosion
occurs from repeated self-induced
A. Avoidance of situation & certain vomiting.
activities that resemble the stress
B. Depression and a blunted affect when 36.Nurse Monette is aware that
discussing the traumatic situation extremely depressed clients seem to do
C. Lack of interest in family & others best in settings where they have:
D. Re-experiencing the trauma in
dreams or flashback A. Multiple stimuli
B. Routine Activities
Answer: D . Experiencing the actual C. Minimal decision making
trauma in dreams or flashback is the D. Varied Activities
major symptom that distinguishes post
traumatic stress disorder from other Answer: B . Depression usually is both
anxiety disorder. emotional & physical. A simple daily
routine is the best, least stressful and
33.Nurse Benjie is communicating with a least anxiety producing.
male client with substance-
37.To further assess a client’s suicidal
potential. Nurse Katrina should Answer:C . A person with this disorder
be especially alert to the client would not have adequate self-
expression of: boundaries

A. Frustration & fear of death 41.A 23 year old client has been
B. Anger & resentment admitted with a diagnosis of
C. Anxiety & loneliness schizophrenia says to the nurse “Yes, its
D. Helplessness & hopelessness march, March is little woman”. That’s
literal you know”. These statement
Answer: D . The expression of these illustrate:
feeling may indicate that this client is
unable to continue the struggle of life. A. Neologisms
B. Echolalia
38.A nursing care plan for a male client C. Flight of ideas
with bipolar I disorder should include: D. Loosening of association

A. Providing a structured Answer: D . Loose associations are


environment thoughts that are presented without the
B. Designing activities that will require logical connections usually necessary for
the client to maintain contact with the listening to interpret the message.
reality
C. Engaging the client in conversing 42.A long term goal for a paranoid male
about current affairs client who has unjustifiably accused his
D. Touching the client provide assurance wife of having many extramarital affairs
would be to help the client develop:
Answer: A . Structure tends to decrease
agitation and anxiety and to increase A. Insight into his behavior
the client’s feeling of security. B. Better self control
C. Feeling of self worth
39.When planning care for a female D. Faith in his wife
client using ritualistic behavior,
Nurse Gina must recognize that the Answer: C . Helping the client to develop
ritual: feeling of self worth would reduce the
client’s need to use pathologic defenses.
A. Helps the client focus on the inability
to deal with reality 43.A male client who is experiencing
B. Helps the client control the disordered thinking about food
anxiety being poisoned is admitted to the mental
C. Is under the client’s conscious control health unit. The nurse uses
D. Is used by the client primarily for which communication technique to
secondary gains encourage the client to eat dinner?

Answer: B . The rituals used by a client A. Focusing on self-disclosure of own


with obsessive compulsive disorder food preference
help control the anxiety level by B. Using open ended question and
maintaining a set pattern of action. silence
C. Offering opinion about the need to
40.A 32 year old male graduate student, eat
who has become increasingly withdrawn D. Verbalizing reasons that the client
and neglectful of his work and personal may not choose to eat
hygiene, is brought to the psychiatric
hospital by his parents. After detailed Answer: B . Open ended questions and
assessment, a diagnosis of schizophrenia silence are strategies used to
is made. It is unlikely that the client encourage clients to discuss their
will demonstrate: problem in descriptive manner.

A. Low self esteem 44.Nurse Nina is assigned to care for a


B. Concrete thinking client diagnosed with Catatonic Stupor.
C. Effective self boundaries When Nurse Nina enters the client’s
D. Weak ego room, the client is found lying on the bed
with a body pulled into a fetal position.
Nurse Nina should? 47.During electroconvulsive therapy
(ECT) the client receives oxygen by mask
A. Ask the client direct questions to via positive pressure ventilation. The
encourage talking nurse assisting with this procedure knows
B. Rake the client into the dayroom to that positive pressure ventilation is
be with other clients necessary because?
C. Sit beside the client in silence
and occasionally ask open- A. Anesthesia is administered during the
ended question procedure
D. Leave the client alone and continue B. Decrease oxygen to the brain
with providing care to the increases confusion
other clients and disorientation
C. Grand mal seizure activity depresses
Answer: C . Clients who are withdrawn respirations
may be immobile and mute, and D. Muscle relaxations given to
require consistent, repeated prevent injury during seizure
interventions. Communication with activity depress respirations.
withdrawn clients requires much
patience from the nurse. The nurse Answer: D . A short acting skeletal
facilitates communication with the client muscle relaxant such as succinylcholine
by sitting in silence, asking open-ended (Anectine) is administered during this
question and pausing to provide procedure to prevent injuries during
opportunities for the client to respond. seizure.

45.Nurse Tina is caring for a client with 48.When planning the discharge of a
delirium and states that “look at client with chronic anxiety, Nurse
the spiders on the wall”. What should the Chris evaluates achievement of the
nurse respond to the client? discharge maintenance goals. Which
goal would be most appropriately having
A. “You’re having hallucination, there been included in the plan of
are no spiders in this room at all” care requiring evaluation?
B. “I can see the spiders on the wall, but
they are not going to hurt you” A. The client eliminates all anxiety from
C. “Would you like me to kill the daily situations
spiders” B. The client ignores feelings of anxiety
D. “I know you are frightened, but I C. The client identifies anxiety
do not see spiders on the wall” producing situations
D. The client maintains contact with a
Answer: D . When hallucination is crisis counselor
present, the nurse should reinforce
reality with the client. Answer: C . Recognizing situations that
produce anxiety allows the client to
46.Nurse Jonel is providing information to prepare to cope with anxiety or avoid
a community group about violence in the specific stimulus.
family. Which statement by a group
member would indicate a need to provide 49.Nurse Tina is caring for a client with
additional information? depression who has not responded
to antidepressant medication. The nurse
A. “Abuse occurs more in low- anticipates that what
income families” treatment procedure may be prescribed.
B. “Abuser Are often jealous or self-
centered” A. Neuroleptic medication
C. “Abuser use fear and intimidation” B. Short term seclusion
D. “Abuser usually have poor self- C. Psychosurgery
esteem” D. Electroconvulsive therapy

Answer: A . Personal characteristics of Answer: D . Electroconvulsive therapy is


abuser include low self-esteem, an effective treatment for depression
immaturity, dependence, insecurity and that has not responded to medication
jealousy
50.Mario is admitted to the emergency CONCEPT. The most popular theory
room with drug-included anxiety related was perhaps Nightingale’s. She
to over ingestion of prescribed defined nursing as the utilization of
antipsychotic medication. The
the persons environment to assist
most important piece of information the
nurse in charge should obtain initially him towards recovery. She defined
is the: the person as somebody who has a
reparative capabilities mediated
A. Length of time on the med. and enhanced by factors in his
B. Name of the ingested medication environment. She describes the
& the amount ingested environment as something that
C. Reason for the suicide attempt would facilitate the person’s
D. Name of the nearest relative & their reparative process and identified
phone number
different factors like sanitation,
Answer: B . In an emergency, lives noise, etc. that affects a person’s
saving facts are obtained first. The reparative state.
name and the amount of medication
ingested are of outmost important in 2. The act of utilizing the
treating this potentially life threatening environment of the patient to assist
situation. him in his recovery is theorized by

A. Nightingale
B. Benner
FOUNDATION OF NURSING C. Swanson
D. King
PNLE: FON Answer: A. Nightingale. Florence

Practice Exam for nightingale do not believe in the


germ theory, and perhaps this was
her biggest mistake. Yet, her theory
History , was the first in nursing. She
believed that manipulation of

Concepts and environment that includes


appropriate noise, nutrition,
hygiene, light, comfort, sanitation
Theories etc. could provide the client’s body
the nurturance it needs for repair
1. The four major concepts in and recovery.
nursing theory are the
3. For her, Nursing is a theoretical
A. Person, Environment, Nurse, system of knowledge that prescribes
Health a process of analysis and action
B. Nurse, Person, Environment, Cure related to care of the ill person
C. Promotive, Preventive, Curative,
Rehabilitative A. King
D. Person, Environment, B. Henderson
Nursing, Health C. Roy
D. Leininger
Answer: D. Person, Environment,
Nursing, Health. This is an actual Answer: C. Roy. Remember the
board exam question and is a word “ THEOROYTICAL “ For Callista
common board question. Theorist Roy, Nursing is a theoretical body of
always describes The nursing knowledge that prescribes analysis
profession by first defining what is and action to care for an ill person.
NURSING, followed by the PERSON, She introduced the ADAPTATION
ENVIRONMENT and HEALTH MODEL and viewed person as a
BIOSPSYCHOSOCIAL BEING. She the necessary strength, will and
believed that by adaptation, Man knowledge, and do this in such a
can maintain homeostasis. way as to help him gain
independence as rapidly as possible.
4. According to her, Nursing is a
helping or assistive profession to A. Henderson
persons who are wholly or partly B. Abdellah
dependent or when those who are C. Levin
supposedly caring for them are no D. Peplau
longer able to give care.
Answer: A. Henderson. This was an
A. Henderson actual board question. Remember
B. Orem this definition and associate it with
C. Swanson Virginia Henderson. Henderson also
D. Neuman describes the NATURE OF NURSING
theory. She identified 14 basic
Answer: B. Orem. In self care deficit needs of the client. She describes
theory, Nursing is defined as A nursing roles as SUBSTITUTIVE :
helping or assistive profession to Doing everything for the client,
person who are wholly or partly SUPPLEMENTARY : Helping the client
dependent or when people who are and COMPLEMENTARY : Working
to give care to them are no longer with the client. Breathing normally,
available. Self care, are the Eliminating waste, Eating and
activities that a person do for drinking adquately, Worship and
himself to maintain health, life and Play are some of the basic needs
well being. according to her.

5. Nursing is a unique profession, 7. Caring is the essence and central


Concerned with all the variables unifying, a dominant domain that
affecting an individual’s response to distinguishes nursing from other
stressors, which are intra, inter and health disciplines. Care is an
extra personal in nature. essential human need.

A. Neuman A. Benner
B. Johnson B. Watson
C. Watson C. Leininger
D. Parse D. Swanson

Answer: A. Neuman. Neuman Answer:C. Leininger. There are


divided stressors as either intra, many theorist that describes
inter and extra personal in nature. nursing as CARE. The most popular
She said that NURSING is concerned was JEAN WATSON’S Human Caring
with eliminating these stressors to Model. But this question pertains to
obtain a maximum level of Leininger’s definition of caring. CUD
wellness. The nurse helps the client I LIE IN GER? [ Could I Lie In There ]
through PRIMARY, SECONDARY AND Is the Mnemonics I am using not to
TERTIARY prevention modes. Please get confused. C stands for CENTRAL
do not confuse this with LEAVELL , U stands for UNIFYING, D stands
and CLARK’S level of prevention. for DOMINANT DOMAIN. I
emphasize on this matter due to
6. The unique function of the nurse feedback on the last June 2006
is to assist the individual, sick or batch about a question about
well, in the performance of those CARING.
activities contributing to health that
he would perform unaided if he has
8. Caring involves 5 processes,
KNOWING, BEING WITH, DOING FOR, Answer: A. Benner. I think of CARE
ENABLING and MAINTAINING BELIEF. BEAR to facilitate retainment of
BENNER. As in, Care Benner. For
A. Benner her, Caring means being
B. Watson CONNECTED or making things
C. Leininger matter to people. Caring according
D. Swanson to Benner give meaning to illness
and re establish connection.
Answer:D. Swanson . Caring
according to Swanson involves 5 11. Which of the following is NOT
processes. Knowing means TRUE about profession according to
understanding the client. Being with Marie Jahoda?
emphasizes the Physical presence
of the nurse for the patient. Doing A. A profession is an organization of
for means doing things for the an occupational group based on
patient when he is incapable of the application of special
doing it for himself. Enabling means knowledge
helping client transcend B. It serves specific interest of a
maturational and developmental group
stressors in life while Maintaining C. It is altruistic
belief is the ability of the Nurse to D. Quality of work is of greater
inculcate meaning to these events. importance than the rewards

9. Caring is healing, it is Answer: B. It serves specific interest


communicated through the of a group.Believe it or not, you
consciousness of the nurse to the should know the definition of
individual being cared for. It allows profession according to Jahoda
access to higher human spirit. because it is asked in the Local
boards. A profession should serve
A. Benner the WHOLE COMMUNITY and not
B. Watson just a specific intrest of a group.
C. Leininger Everything else, are correct.
D. Swanson
12. Which of the following is NOT an
Answer: B. Watson. The deepest attribute of a professional?
and spiritual definition of Caring
came from Jean watson. For her, A. Concerned with quantity
Caring expands the limits of B. Self directed
openess and allows access to C. Committed to spirit of inquiry
higher human spirit. D. Independent

10. Caring means that person, Answer: A. Concerned with


events, projects and things matter to quantity. A professional is
people. It reveals stress and coping concerned with QUALITY and not
options. Caring creates QUANTITY. In nursing, We have
responsibility. It is an inherent methods of quality assurance and
feature of nursing practice. It helps control to evaluate the
the nurse assist clients to recover in effectiveness of nursing care.
the face of the illness. Nurses, are never concerned with
QUANTITY of care provided.
A. Benner
B. Watson 13. The most unique characteristic
C. Leininger of nursing as a profession is
D. Swanson
A. Education A. Initiate modification on client’s
B. Theory lifestyle
C. Caring B. Protect client’s right
D. Autonomy C. Coordinates the activities of
other members of the health
Answer: C. Caring. Caring and team in managing patient care
caring alone, is the most unique D. Provide in service education
quality of the Nursing Profession. It programs, Use accurate
is the one the delineate Nursing nursing audit, formulate
from other professions. philosophy and vision of the
institution
14. This is the distinctive individual
qualities that differentiate a person Answer:Provide in service education
to another programs, Use accurate nursing
audit, formulate philosophy and
A. Philosophy vision of the institution
B. Personality
C. Charm 17. What best describes nurses as a
D. Character care provider?

Answer: B. Personality. Personality A. Determine client’s need


are qualities that make us different B. Provide direct nursing care
from each other. These are C. Help client recognize and cope
impressions that we made, or the with stressful psychological
footprints that we leave behind. situation
This is the result of the integration D. Works in combined effort with all
of one’s talents, behavior, those involved in patient’s care
appearance, mood, character,
morals and impulses into one Answer: A. Determine client’s
harmonious whole. Philosophy is the need.You can never provide nursing
basic truth that fuel our soul and care if you don’t know what are the
give our life a purpose, it shapes needs of the client. How can you
the facets of a person’s character. provide an effective postural
Charm is to attract other people to drainage if you do not know where
be a change agent. Character is our is the bulk of the client’s secretion.
moral values and belief that guides Therefore, the best description of a
our actions in life. care provider is the accurate and
prompt determination of the client’s
15. Refers to the moral values and need to be able to render an
beliefs that are used as guides to appropriate nursing care.
personal behavior and actions
18. The nurse questions a doctors
A. Philosophy order of Morphine sulfate 50 mg, IM
B. Personality for a client with pancreatitis. Which
C. Charm role best fit that statement?
D. Character
A. Change agent
Answer:D. Character.Rationale: Refe B. Client advocate
r to number 14 C. Case manager
D. Collaborator
16. As a nurse manager, which of
the following best describes this Answer: B. Client advocate. As a
function? client’s advocate, Nurses are to
protect the client’s right and
promotes what is best for the client.
Knowing that Morphine causes are the method use by Nursing
spasm of the sphincter of Oddi and students, Private duty nurses and
will lead to further increase in the those in critical or isolation units.
client’s pain, The nurse knew that
the best treatment option for the 21. This is the best patient care
client was not provided and model when there are many nurses
intervene to provide the best but few patients.
possible care.
A. Functional nursing
19. These are nursing intervention B. Team nursing
that requires knowledge, skills and C. Primary nursing
expertise of multiple health D. Total patient care
professionals.
Answer: D. Total patient care .Total
A. Dependent patient care works best if there are
B. Independent many nurses but few patients.
C. Interdependent
D. Intradependent 22. This patient care model works
best when there are plenty of
Answer: patient but few nurses
C. Interdependent. Interdependent
functions are those that needs A. Functional nursing
expertise and skills of multiple B. Team nursing
health professionals. Example is C. Primary nursing
when A child was diagnosed with D. Total patient care
nephrotic syndrome and the doctor
ordered a high protein diet, Budek ANSWER:A. Functional
then work together with the nursing. Functional nursing is task
dietician about the age appropriate oriented, One nurse is assigned on
high protein foods that can be given a particular task leading to task
to the child, Including the expertise and efficiency. The nurse
preparation to entice the child into will work fast because the
eating the food. NOTE : It is still procedures are repetitive leading to
debated if the diet in NS is low, task mastery. This care is not
moderate or high protein, In the recommended as this leads
U.S, Protein is never restricted and fragmented nursing care.
can be taken in moderate amount.
As far as the local examination is 23. RN assumes 24 hour
concerned, answer LOW PROTEIN responsibility for the client to
HIGH CALORIC DIET. maintain continuity of care across
shifts, days or visits.
20. What type of patient care model
is the most common for student A. Functional nursing
nurses and private duty nurses? B. Team nursing
C. Primary nursing
A. Total patient care D. Total patient care
B. Team nursing
C. Primary Nursing Answer: C. Primary nursing. Your
D. Case management keyword in Primary nursing is the
24 hours. This does not necessarily
Answer: A. Total patient care. This is means the nurse is awake for 24
also known as case nursing. It is a hours, She can have a SECONDARY
method of nursing care wherein, NURSES that will take care of the
one nurse is assigned to one patient patient in shifts where she is not
for the delivery of total care. These arround.
mus maintain the integrity of our
organs, tissues and systems to be
24. Who developed the first theory able to function and prevent harmful
of nursing? agents entering our body. PERSONAL
INTEGRITY : These refers to our self
A. Hammurabi esteem, self worth, self concept,
B. Alexander identify and personality. SOCIAL
C. Fabiola INTEGRITY : Reflects our societal
D. Nightingale roles to our society, community,
family, friends and fellow individuals
Answer: D. Nightingale . Refer to
question # 2. Hammurabi is the 27. Proposed the HEALTH CARE
king of babylon that introduces the SYSTEM MODEL.
LEX TALIONES law, If you kill me,
you should be killed… If you rob A. Henderson
me, You should be robbed, An eye B. Orem
for an eye and a tooth for a tooth. C. Parse
Alexander the great was the son of D. Neuman
King Philip II and is from macedonia
but he ruled Greece including Persia Answer: D. Neuman . Betty Neuman
and Egypt. He is known to use a asserted that nursing is a unique
hammer to pierce a dying soldier’s profession and is concerned with all
medulla towards speedy death the variables affecting the
when he thinks that the soldier will individual’s response to stressors.
die anyway, just to relieve their These are INTRA or within
suffering. Fabiola was a beautiful ourselves, EXTRA or outside the
roman matron who converted her individual, INTER means between
house into a hospital. two or more people. She proposed
the HEALTH CARE SYSTEM MODEL
25. She introduces the NATURE OF which states that by PRIMARY,
NURSING MODEL. SECONDARY and TERTIARY
prevention, The nurse can help the
A. Henderson client maintain stability against
B. Nightingale these stressors.
C. Parse
D. Orlando 28. Conceptualized the BEHAVIORAL
SYSTEM MODEL
Answer: A. Henderson. Refer to
question # 6. A. Orem
B. Johnson
26. She described the four C. Henderson
conservation principle. D. Parse

A. Levin Answer: B. Johnson. According to


B. Leininger Dorothy Johnson, Each person is a
C. Orlando behavioral system that is composed
D. Parse of 7 subsystems. Man adjust or
adapt to stressors by a using a
Answer: A. Levin. Myra Levin LEARNED PATTERN OF RESPONSE.
described the 4 Conservation Man uses his behavior to meet the
principles which are concerned with demands of the environment, and is
the Unity and Integrity of an able to modified his behavior to
individual. These are ENERGY : Our support these demands
output to facilitate meeting of our
needs. STRUCTURAL INTEGRITY : We
29. Developed the CLINICAL that affects this individual to cope
NURSING – A HELPING ART MODEL with illness, recover and FIND
MEANING to this experience. For
A. Swanson her, Nursing is a HUMAN TO HUMAN
B. Hall relationship that is formed during
C. Weidenbach illness. To her, an individual is a
D. Zderad UNIQUE and irreplaceable being in
continuous process of becoming,
Asnwer: C. Weidenbach.Just evolving and changing. PLEASE do
remember ERNESTINE remember, that it is PARSE who
WEIDENBACHLINICAL. postulated the theory of HUMAN
BECOMING and not TRAVELBEE, for I
30. Developed the ROLE MODELING read books that say it was
and MODELING theory TRAVELBEE and not PARSE.

A. Erickson,Tomlin,Swain 33. He proposed the theory of


B. Neuman morality that is based on MUTUAL
C. Newman TRUST
D. Benner and Wrubel
A. Freud
Answer: A. Erickson,Tomlin,Swain B. Erikson
C. Kohlberg
31. Proposed the GRAND THEORY OF D. Peters
NURSING AS CARING
Answer: C. Kohlberg. Kohlber states
A. Erickson, Tomlin, Swain that relationships are based on
B. Peterson,Zderad mutual trust. He postulated the
C. Bnner,Wrubel levels of morality development. At
D. Boykin,Schoenhofer the first stage called the PREMORAL
or preconventional, A child do things
Answer: and label them as BAD or GOOD
D. Boykin,Schoenhofer . This theory depending on the PUNISHMENT or
was called GRAND THEORY because REWARD they get. They have no
boykin and schoenofer thinks that concept of justice, fairness and
ALL MAN ARE CARING, And that equity, for them, If I punch this kid
nursing is a response to this unique and mom gets mad, thats WRONG.
call. According to them, CARING IS But if I dance and sing, mama smiles
A MORAL IMPERATIVE, meaning, ALL and give me a new toy, then I am
PEOPLE will tend to help a man who doing something good. In the
fell down the stairs even if he is not Conventional level, The individual
trained to do so. actuates his act based on the
response of the people around him.
32. Postulated the INTERPERSONAL He will follow the rules, regulations,
ASPECT OF NURSING laws and morality the society
upholds. If the law states that I
A. Travelbee
should not resuscitate this man with
B. Swanson
a DNR order, then I would not.
C. Zderad
However, in the Post conventional
D. Peplau
level or the AUTONOMOUS level, the
individual still follows the rules but
Answer: A. Travelbee. Travelbee’s
can make a rule or bend part of
theory was referred to as
these rules according to his own
INTERPERSONAL theory because
MORALITY. He can change the rules if
she postulated that NURSING is to
he thinks that it is needed to be
assist the individual and all people
changed. Example is that, A nurse
still continue resuscitating the client A. Zderad and Peterson
even if the client has a DNR order B. Benner and Wrubel
because he believes that the client C. Fowler and Westerhoff
can still recover and his mission is to D. Schulman and Mekler
save lives, not watch patients die
34. He proposed the theory of Answer: D. Schulman and
morality based on PRINCIPLES Mekler . According to Schulman and
Mekler, there are 2 components
A. Freud that makes an action MORAL : The
B. Erikson intention should be good and the
C. Kohlberg Act must be just. A good example is
D. Peters ROBIN HOOD, His intention is GOOD
but the act is UNJUST, which makes
Answer: D. Peters . Remember his action IMMORAL.
PETERS for PRINCIPLES. P is to P. He
believes that morality has 3 37. Postulated that FAITH is the way
components : EMOTION or how one of behaving. He developed four
feels, JUDGEMENT or how one
theories of faith and development
reason and BEHAVIOR or how one
based on his experience.
actuates his EMOTION and
JUDGEMENT. He believes that A. Giligan
MORALITY evolves with the B. Westerhoff
development of PRINCPLES or the C. Fowler
person’s vitrue and traits. He also D. Freud
believes in AUTOMATICITY of virtues
or he calls HABIT, like kindness, Answer: B. Westerhoff. There are
charity, honesty, sincerity and thirft only 2 theorist of FAITH that might
which are innate to a person and be asked in the board examinations.
therfore, will be performed Fowler and Westerhoff. What differs
automatically. them is that, FAITH of fowler is
defined abstractly, Fowler defines
35. Freud postulated that child faith as a FORCE that gives a
adopts parental standards and traits meaning to a person’s life while
through Westerhoff defines faith as a
behavior that continuously develops
A. Imitation through time.
B. Introjection
C. Identification 38. He described the development of
D. Regression
faith. He suggested that faith is a
Answer:C. Identification. A child, spiritual dimension that gives
according to Freud adopts parental meaning to a persons life. Faith
standards, traits, habits and norms according to him, is a relational
through identication. A good phenomenon.
example is the corned beef A. Giligan
commercial ” WALK LIKE A MAN, B. Westerhoff
TALK LIKE A MAN ” Where the child C. Fowler
identifies with his father by wearing D. Freud
the same clothes and doing the
same thing. Answer:C. Fowler. Rationale: Refer
to # 37
36. According to them, Morality is
measured of how people treat 39. Established in 1906 by the
human being and that a moral child Baptist foreign mission society of
strives to be kind and just
America. Miss rose nicolet, was it’s C. Isabel Hampton Robb
first superintendent. D. Caroline Hampton Robb
A. St. Paul Hospital School of Answer:A. Clara louise Maas. Clara
nursing Louise Maas sacrificed her life in
B. Iloilo Mission Hospital School research of YELLOW FEVER. People
of nursing during her time do not believe that
C. Philippine General Hospital yellow fever was brought by
School of nursing mosquitoes. To prove that they are
D. St. Luke’s Hospital School of wrong, She allowed herself to be
nursing bitten by the vector and after days,
She died.
Answer:B. Iloilo Mission Hospital
School of nursing 43. He was called the father of
sanitation.
40. Anastacia Giron-Tupas was the A. Abraham
first Filipino nurse to occupy the B. Hippocrates
position of chief nurse in this C. Moses
hospital. D. Willam Halstead

A. St. Paul Hospital Answer: C. Moses


B. Iloilo Mission Hospital
C. Philippine General Hospital 44. The country where SHUSHURUTU
D. St. Luke’s Hospital originated

Answer:C. Philippine General A. China


Hospital B. Egypt
C. India
41. She was the daughter of D. Babylonia
Hungarian kings, who feed 300-900
people everyday in their gate, builds Answer: india
hospitals, and care of the poor and
45. They put girls clothes on male
sick herself.
infants to drive evil forces away
A. Elizabeth
B. Catherine A. Chinese
C. Nightingale B. Egyptian
D. Sairey Gamp C. Indian
D. Babylonian
Answer:A. Elizabeth.Saint Elizabeth
of Hungary was a daughter of a Answer: A. Chinese. Chinese
King and is the patron saint of believes that male newborns are
nurses. She build hospitals and feed demon magnets. To fool those
hungry people everyday using the demons, they put female clothes to
kingdom’s money. She is a princess, their male newborn.
but devoted her life in feeding the
hungry and serving the sick. 46. In what period of nursing does
people believe in TREPHINING to
42. She dies of yellow fever in her drive evil forces away?
search for truth to prove that yellow A. Dark period
fever is carried by a mosquitoes. B. Intuitive period
A. Clara louise Maas C. Contemporary period
B. Pearl Tucker D. Educative period
Answer: B. Intuitive C. St. Clare
period.Egyptians believe that a sick D. St. Elizabeth
person is someone with an evil
force or demon that is inside their Answer:C. St. Clare. The poor clares,
heads. To release these evil spirits, is the second order of St. Francis of
They would tend to drill holes on assisi. The first order was founded
the patient’s skull and it is called by St. Francis himself. St. Catherine
TREPHINING. of Siena was the first lady with the
lamp. St. Anne is the mother of
47. This period ended when Pastor mama mary. St. Elizabeth is the
Fliedner, build Kaiserwerth institute patron saint of Nursing.
for the training of Deaconesses
50. This period marked the religious
A. Apprentice period upheaval of Luther, Who questions
B. Dark period the Christian faith.
C. Contemporary period
D. Educative period A. Apprentice period
B. Dark period
Answer: A. Apprentice period.What C. Contemporary period
dilineates apprentice period among D. Educative period
others is that, it ENDED when
formal schools were established. Answer:B. Dark
During the apprentice period, There period. Protestantism emerged with
is no formal educational institution Martin Luther questions the Pope
for nurses. Most of them receive and Christianity. This started the
training inside the convent or Dark period of nursing when the
church. Some of them are trained christian faith was smeared by
just for the purpose of nursing the controversies. These leads to
wounded soldiers. But almost all of closure of some hospital and
them are influenced by the christian schools run by the church. Nursing
faith to serve and nurse the sick. became the work of prostitutes,
When Fliedner build the first formal slaves, mother and least desirable
school for nurses, It marked the end of women.
of the APPRENTICESHIP period.
51. According to the Biopsychosocial
48. Period of nursing where religious and spiritual theory of Sister Callista
Christian orders emerged to take Roy, Man, As a SOCIAL being is
care of the sick
A. Like all other men
A. Apprentice period B. Like some other men
B. Dark period C. Like no other men
C. Contemporary period D. Like men
D. Educative period
Answer:B. Like some other
Answer: A. Apprentice men.According to ROY, Man as a
period. Apprentice period is marked social being is like some other man.
by the emergence of religious As a spiritual being and Biologic
orders the are devoted to religious being, Man are all alike. As a
life and the practice of nursing. psychologic being, No man thinks
alike. This basically summarized her
49. Founded the second order of St. BIOPSYHOSOCIAL theory which is
Francis of Assisi included in our licensure exam
coverage.
A. St. Catherine
B. St. Anne
52. She conceptualized that man, as mother of Queen Elizabeth and the
an Open system is in constant wife of King Henry VIII. She was
interaction and transaction with a beheaded because Henry wants to
changing environment. mary another wife and that his
divorce was not approved by the
A. Roy pope. Outraged, He insisted on the
B. Levin separation of the Church and State
C. Neuman and divorce Anne himself by making
D. Newman everyone believe that Anne is
having an affair to another man.
Answer:A. Roy. OPEN system theory Anne was beheaded while her lips is
is ROY. As an open system, man still saying a prayer. Even without
continuously allows input from the her head, People still gave respect
environment. Example is when you to her diseased body and a
tell me Im good looking, I will be separate head. She was still
happy the entire day, Because I am remembered as Anne boleyn,
an open system and continuously Mother of Elizabeth who lead
interact and transact with my england to their GOLDEN AGE.
environment. A close system is best
exemplified by a CANDLE. When 55. She theorized that man is
you cover the candle with a glass, it composed of sub and supra systems.
will die because it will eventually Subsystems are cells, tissues,
use all the oxygen it needs inside organs and systems while the
the glass for combustion. A closed
suprasystems are family, society
system do not allow inputs and
and community.
output in its environment.
A. Roy
53. In a CLOSED system, which of B. Rogers
the following is true? C. Henderson
D. Johnson
A. Affected by matter
B. A sole island in vast ocean
Answer: B. Rogers. According to
C. Allows input
Martha Rogers, Man is composed of
D. Constantly affected by matter,
2 systems : SUB which includes
energy, information
cells, tissues, organs and system
and SUPRA which includes our
Answer: B. A sole island in vast
famly, community and society. She
ocean
stated that when any of these
systems are affected, it will affect
54. Who postulated the WHOLISTIC
the entire individual.
concept that the totality is greater
than sum of its parts? 56. Which of the following is not true
A. Roy about the human needs?
B. Rogers A. Certain needs are common to all
C. Henderson people
D. Johnson B. Needs should be followed
exactly in accordance with
Answer: B. Rogers. The wholistic their hierarchy
theory by Martha Rogers states that C. Needs are stimulated by internal
MAN is greater than the sum of all factors
its parts and that his dignity and D. Needs are stimulated by external
worth will not be lessen even if one factors
of this part is missing. A good
example is ANNE BOLEYN, The
Answer: B. Needs should be Answer: A. Makes decision contrary
followed exactly in accordance with to public opinion. Refer to question
their hierarchy.Needs can be # 58.
deferred. I can urinate later as not
to miss the part of the movie’s 60. This is the essence of mental
climax. I can save my money that health
are supposedly for my lunch to
watch my idols in concert. The A. Self awareness
physiologic needs can be meet later B. Self actualization
for some other needs and need not C. Self esteem
be strictly followed according to D. Self worth
their hierarchy.
Answer: B. Self actualization. The
57. Which of the following is TRUE peak of maslow’s hierarchy is the
about the human needs? essence of mental health

A. May not be deferred 61. Florence nightingale is born in


B. Are not interrelated
C. Met in exact and rigid way A. Germany
D. Priorities are alterable B. Britain
C. France
Answer: D. Priorities are D. Italy
alterable. Refer to question # 56.
Answer: D. Italy. Florence
58. According to Maslow, which of Nightingale was born in Florence,
Italy, May 12, 1820. Studied in
the following is NOT TRUE about a
Germany and Practiced in England.
self actualized person?
A. Understands poetry, music, 62. Which is unlikely of Florence
philosophy, science etc. Nightingale?
B. Desires privacy, autonomous
A. Born May 12, 1840
C. Follows the decision of the
B. Built St. Thomas school of
majority, uphold justice and
nursing when she was 40 years
truth
old
D. Problem centered
C. Notes in nursing
D. Notes in hospital
Answer: C. Follows the decision of
the majority, uphold justice and
1. Answer: A. Born May 12, 1840
truth. A,B and D are all qualities of a
self actualized person. A self
63. What country did Florence
actualized person do not follow the
decision of majority but is self Nightingale train in nursing?
directed and can make decisions A. Belgium
contrary to a popular opinion. B. US
C. Germany
59. According to Maslow, which of D. England
the following is TRUE about a self
actualized person? Answer: germany
A. Makes decision contrary to 64. Which of the following is
public opinion
recognized for developing the
B. Do not predict events
C. Self centered concept of HIGH LEVEL WELLNESS?
D. Maximum degree of self conflict A. Erikson
B. Madaw
C. Peplau 2004 also known as the Nursing
D. Dunn Code Of ethics which states that
[ SECTION 17, A ] A nurse should be
Answer:D. Dunn. According to Dunn, a member of an accredited
High level wellness is the ability of professional organization which is
an individual to maximize his full the PNA.
potential with the limitations
imposed by his environment. 68. Which of the following best
According to him, An individual can describes the action of a nurse who
be healthy or ill in both favorable documents her nursing diagnosis?
and unfavorable environment.
A. She documents it and charts it
65. One of the expectations is for whenever necessary
nurses to join professional B. She can be accused of
association primarily because of malpractice
C. She does it regularly as an
A. Promotes advancement and important responsibility
professional growth among D. She charts it only when the
its members patient is acutely ill
B. Works for raising funds for
nurse’s benefit Answer: C. She does it regularly as
C. Facilitate and establishes an important responsibility
acquaintances
D. Assist them and securing jobs 69. Which of the following does not
abroad govern nursing practice?

Answer: A. Promotes advancement A. RA 7164


and professional growth among its B. RA 9173
members C. BON Res. Code Of Ethics
D. BON Res. Scope of Nursing
66. Founder of the PNA Practice

A. Julita Sotejo Answer: A. RA 7164. 7164 is an old


B. Anastacia Giron Tupas law. This is the 1991 Nursing Law
C. Eufemia Octaviano which was repealed by the newer
D. Anesia Dionisio 9173.

Answer: Anastacia Giron Tupas 70. A nurse who is maintaining a


private clinic in the community
67. Which of the following provides renders service on maternal and
that nurses must be a member of a child health among the
national nurse organization? neighborhood for a fee is:
A. R.A 877 A. Primary care nurse
B. 1981 Code of ethics approved by B. Independent nurse
the house of delegates and the practitioner
PNA C. Nurse-Midwife
C. Board resolution No. 1955 D. Nurse specialist
Promulgated by the BON
D. RA 7164 Answer: B. Independent nurse
practitioner
Answer: C. Board resolution No.
1955 Promulgated by the BON. This 71. When was the PNA founded?
is an old board resolution. The new
Board resolution is No. 220 series of A. September 22, 1922
B. September 02, 1920
C. October 21, 1922 75. What regulates HOMEOSTASIS
D. September 02, 1922 according to the theory of Walter
Cannon?
Answer: D. September 02,
1922. According to the official PNA A. Positive feedback
website, they are founded B. Negative feedback
September 02, 1922. C. Buffer system
D. Various mechanisms
72. Who was the first president of
the PNA ? Answer: B. Negative feedback. The
theory of Health as the ability to
A. Anastacia Giron-Tupas maintain homeostasis was
B. Loreto Tupas postulated by Walter Cannon.
C. Rosario Montenegro According to him, There are certain
D. Ricarda Mendoza FEEDBACK Mechanism that
regulates our Homeostasis. A good
Answer: C. Rosario example is that when we overuse
Montenegro. Anastacia Giron Tupas our arm, it will produce pain. PAIN is
founded the FNA, the former name a negative feedback that signals us
of the PNA but the first President that our arm needs a rest.
was Rosario Montenegro.
76. Stated that health is WELLNESS.
73. Defines health as the ability to A termed define by the culture or an
maintain internal milieu. Illness individual.
according to him/her/them is the
failure to maintain internal A. Roy
B. Henderson
environment.
C. Rogers
A. Cannon D. King
B. Bernard
C. Leddy and Pepper Answer: C. Rogers. Martha Rogers
D. Roy states that HEALTH is synonymous
with WELLNESS and that HEALTH
Answer: B. Bernard. According to and WELLNESS is subjective
Bernard, Health is the ability to depending on the definition of one’s
maintain and Internal Milieu and culture.
Illness is the failure to maintain the
internal environment. 77. Defined health as a dynamic
state in the life cycle, and Illness as
74. Postulated that health is a state interference in the life cycle.
and process of being and becoming
A. Roy
an integrated and whole person.
B. Henderson
A. Cannon C. Rogers
B. Bernard D. King
C. Dunn
D. Roy Answer: D. King .Emogene King
states that health is a state in the
Answer: D. Roy. According to ROY, life cycle and Illness is any
Health is a state and process of interference on this cycle. I enjoyed
becoming a WHOLE AND the Movie LION KING and like what
INTEGRATED Person. Mufasa said that they are all part of
the CIRCLE OF LIFE, or the Life
cycle.
78. She defined health as the C. Benner and Wruber
soundness and wholness of D. Leddy and Pepper
developed human structure and
bodily mental functioning. Answer: D. Leddy and
Pepper .According to Leddy and
A. Orem Pepper, Wellness is subjective and
B. Henderson depends on an individuals
C. Neuman perception of balance, harmony and
D. Clark vitality. Leavell and Clark postulared
the ecologic model of health and
Answer: A. Orem. Orem defined illness or the AGENT-HOST-
health as the SOUNDNESS and ENVIRONMENT model. Peterson and
WHOLENESS of developed human Zderad developed the HUMANISTIC
structure and of bodily and mental NURSING PRACTICE theory while
functioning. Benner and Wruber postulate the
PRIMACY OF CARING MODEL.
79. According to her, Wellness is a
condition in which all parts and 82. He describes the WELLNESS-
subparts of an individual are in ILLNESS Continuum as interaction of
harmony with the whole system. the environment with well being and
A. Orem illness.
B. Henderson A. Cannon
C. Neuman B. Bernard
D. Johnson C. Dunn
D. Clark
Answer: C. Neuman. Neuman
believe that man is composed of Answer: Dunn
subparts and when this subparts
are in harmony with the whole 83. An integrated method of
system, Wellness results. Please do functioning that is oriented towards
not confuse this with the SUB and maximizing one’s potential within
SUPRA systems of martha rogers. the limitation of the environment.
80. Postulated that health is A. Well being
reflected by the organization, B. Health
interaction, interdependence and C. Low level Wellness
integration of the subsystem of the D. High level Wellness
behavioral system.
Answer: High level Wellness
A. Orem
B. Henderson 84. What kind of illness precursor,
C. Neuman according to DUNN is cigarette
D. Johnson smoking?

Answer: D. Johnson . Once you see A. Heredity


the phrase BEHAVIORAL SYSTEM, B. Social
answer Dorothy Johnson. C. Behavioral
D. Environmental
81. According to them, Well being is
a subjective perception of BALANCE, Answer: C. Behavioral. Behavioral
HARMONY and VITALITY precursors includes smoking,
alcoholism, high fat intake and
A. Leavell and Clark other lifestyle choices.
B. Peterson and Zderad Environmental factors involved poor
sanitation and over crowding. and it is a threat to his life and
Heridity includes congenital and functions, he will use preventive
diseases acquired through the behaviors to avoid the occurence of
genes. There are no social this threat.
precursors according to DUNN.
88. Which of the following is not a
85. According to DUNN, PERCEIVED BARRIER in preventive
Overcrowding is what type of illness action?
precursor?
A. Difficulty adhering to the
A. Heredity lifestyle
B. Social B. Economic factors
C. Behavioral C. Accessibility of health care
D. Environmental facilities
D. Increase adherence to medical
Answer: Environmental therapies

86. Health belief model was Answer:A. Difficulty adhering to the


formulated in 1975 by who? lifestyle and B. Economic
factors. Perceived barriers are those
A. Becker factors that affects the individual’s
B. Smith health preventive actions. Both A
C. Dunn and B can affect the individual’s
D. Leavell and Clark ability to prevent the occurence of
diseases. C and D are called
Answer:A. Becker. According to Preventive Health Behaviors which
Becker, The belief of an individual enhances the individual’s
greatly affects his behavior. If a preventive capabilities.
man believes that he is susceptible
to an illness, He will alter his 89. Conceptualizes that health is a
behavior in order to prevent its condition of actualization or
occurence. For example, If a man
realization of person’s potential.
thinks that diabetes is acquired
through high intake of sugar and Avers that the highest aspiration of
simple carbohydrates, then he will people is fulfillment and complete
limit the intake of foods rich in development actualization.
these components. A. Clinical Model
B. Role performance Model
87. In health belief model, Individual C. Adaptive Model
perception matters. Which of the D. Eudaemonistic Model
following is highly UNLIKELY to
influence preventive behavior? Answer: D. Eudaemonistic
Model . Smith formulated 5 models
A. Perceived susceptibility to an
of health. Clinical model simply
illness
states that when people experience
B. Perceived seriousness of an
sign and symptoms, they would
illness
think that they are unhealthy
C. Perceived threat of an illness
therefore, Health is the absence of
D. Perceived curability of an
clinical sign and symptoms of a
illness
disease. Role performance model
states that when a person does his
Answer: D. Perceived curability of an
role and activities without deficits,
illness . If a man think he is
he is healthy and the inability to
susceptibe to a certain disease,
perform usual roles means that the
thinks that the disease is serious
person is ill. Adaptive Model states
that if a person adapts well with his
environment, he is healthy and Answer: D. Cues to action . Refer to
maladaptation equates illness. question # 91.
Eudaemonistic Model of health
according to smith is the 93. Influence from peers and social
actualization of a person’s fullest pressure is included in what variable
potential. If a person functions of HBM?
optimally and develop self
actualization, then, no doubt that A. Demographic
person is healthy. B. Sociopsychologic
C. Structural
90. Views people as physiologic D. Cues to action
system and Absence of sign and
symptoms equates health. Answer: B. Sociopsychologic. Refer
to question # 91.
A. Clinical Model
B. Role performance Model 94. Age, Sex, Race etc. is included in
C. Adaptive Model what variable of HBM?
D. Eudaemonistic Model
A. Demographic
Answer:A. Clinical B. Sociopsychologic
Model. Rationale: Refer to question C. Structural
# 89. D. Cues to action

91. Knowledge about the disease Answer: A. Demographic. Refer to


and prior contact with it is what type question # 91.
of VARIABLE according to the health
95. According to Leavell and Clark’s
belief model?
ecologic model, All of this are factors
A. Demographic that affects health and illness except
B. Sociopsychologic
C. Structural A. Reservoir
D. Cues to action B. Agent
C. Environment
Answer: C. Structural. Modifying D. Host
variables in Becker’s health belief
model includes DEMOGRAPHIC : Age, Answer: A. Reservoir. According to
sex, race etc. SOCIOPSYCHOLOGIC : L&C’s Ecologic model, there are 3
Social and Peer influence. factors that affect health and
STRUCTURAL : Knowledge about the illness. These are the AGENT or the
disease and prior contact with it and factor the leads to illness, either a
CUES TO ACTION : Which are the bacteria or an event in life. HOST
sign and symptoms of the disease or are persons that may or may not be
advice from friends, mass media and affected by these agents.
others that forces or makes the ENVIRONMENT are factors external
individual seek help to the host that may or may not
predispose him to the AGENT.
92. It includes internal and external
96. Is a multi dimensional model
factors that leads the individual to
developed by PENDER that describes
seek help
the nature of persons as they
A. Demographic interact within the environment to
B. Sociopsychologic pursue health
C. Structural
D. Cues to action A. Ecologic Model
B. Health Belief Model Answer:A. Illness. Illness is
C. Health Promotion Model something PERSONAL. Unlike
D. Health Prevention Model disease, Illness are personal state in
which person feels unhealthy. An
Answer: C. Health Promotion old person might think he is ILL but
Model. Pender developed the in fact, he is not due, to diminishing
concept of HEALTH PROMOTION functions and capabilities, people
MODEL which postulated that an might think they are ILL. Disease
individual engages in health however, is something with tangible
promotion activities to increase well basis like lab results, X ray films or
being and attain self actualization. clinical sign and symptoms.
These includes exercise,
immunization, healthy lifestyle, 100. According to her, Caring is
good food, self responsibility and all defined as a nurturant way of
other factors that minimize if not responding to a valued client
totally eradicate risks and threats of towards whom the nurse feels a
health. sense of commitment and
responsibility.
97. Defined by Pender as all
activities directed toward increasing A. Benner
the level of well being and self B. Watson
actualization. C. Leininger
D. Swanson
A. Health prevention
B. Health promotion
C. Health teaching
D. Self actualization Answer: B. Watson. This is Jean
Watson’s definition of Nursing as
Answer:B. Health promotion. Refer caring. This was asked word per
to question # 96. word last June 06′ NLE. Benner
defines caring as something that
98. Defined as an alteration in matters to people. She postulated
normal function resulting in the responsibility created by Caring
reduction of capacities and in nursing. She was also responsible
shortening of life span. for the PRIMACY OF CARING MODEL.
Leininger defind the 4 conservation
A. Illness principle while Swanson introduced
B. Disease the 5 processes of caring.
C. Health
D. Wellness

Answer: B. Disease. Disease are PNLE: FON


alteration in body functions
resulting in reduction of capabilities
or shortening of life span.
Practice Exam for
99. Personal state in which a person Infection,
feels unhealthy
A. Illness Asepsis, Basic
B. Disease
C. Health
D. Wellness
concept of Stress
and Illness
1. When the General A. The client will not urinate due to
adaptation syndrome is activated, relaxation of the detrusor muscle
FLIGHT OR FIGHT response sets in. B. The client will be restless and
Sympathetic nervous system alert
releases norepinephrine while the C. Clients BP will increase, there
adrenal medulla secretes will be vasodilation
epinephrine. Which of the following D. There will be increase
is true with regards to that glycogenolysis, Pancrease will
statement? decrease insulin secretion

A. Pupils will constrict Answer: C. Clients BP will increase,


B. Client will be lethargic there will be vasodilation. If
C. Lungs will bronchodilate vasodilation will occur, The BP will
D. Gastric motility will increase not increase but decrease. It is true
that Blood pressure increases
Answer: C. Lungs will during SNS Stimulation due to the
bronchodilate. To better understand fact that we need more BLOOD to
the concept : The autonomic circulate during the FIGHT or
nervous system is composed of FLIGHT Response because the
SYMPATHETIC and oxygen demand has increased, but
PARASYMPATHETIC Nervous system. this is facilitated by
It is called AUTONOMIC Because it is vasoconstriction and not
Involuntary and stimuli based. You vasodilation. A,B and D are all
cannot tell your heart to kindly beat correct. The liver will increase
for 60 per minute, Nor, Tell your glycogenolysis or glycogen store
blood vessels, Please constrict, utilization due to a heightened
because you need to wear skirt demand for energy. Pancrease will
today and your varicosities are decrease insulin secretion because
bulging. Sympathetic Nervous almost every aspect of digestion
system is the FIGHT or FLIGHT that is controlled by
mechanism. When people FIGHT or Parasympathetic nervous system is
RUN, we tend to stimulate the ANS inhibited when the SNS dominates.
and dominate over SNS. Just
Imagine a person FIGHTING and 3. State in which a person’s physical,
RUNNING to get the idea on the emotional, intellectual and social
signs of SNS Domination. Imagine a development or spiritual functioning
resting and digesting person to get is diminished or impaired compared
a picture of PNS Domination. A with a previous experience.
person RUNNING or FIGHTING
Needs to bronchodilate, because A. Illness
the oxygen need is increased due to B. Disease
higher demand of the body. Pupils C. Health
will DILATE to be able to see the D. Wellness
enemy clearly. Client will be fully
alert to dodge attacks and leap Answer: A. Illness. Disease is a
through obstacles during running. PROVEN FACT based on a medical
The client’s gastric motility will theory, standards, diagnosis and
DECREASE Because you cannot clinical feature while ILLNESS Is a
afford to urinate or defecate during subjective state of not feeling well
fighting nor running. based on subjective appraisal,
previous experience, peer advice etc
2. Which of the following response is
not expected to a person whose GAS 4. This is the first stage of illness
is activated and the FIGHT OR wherein, the person starts to believe
FLIGHT response sets in? that something is wrong. Also known
as the transition phase from 6. In this stage of illness, The person
wellness to illness. learns to accept the illness.

A. Symptom Experience A. Symptom Experience


B. Assumption of sick role B. Assumption of sick role
C. Medical care contact C. Medical care contact
D. Dependent patient role D. Dependent patient role

Answer: A. Symptom Experience. A Answer:B. Assumption of sick


favorite board question are Stages role. Acceptance of illness occurs in
of Illness. When a person starts to the Assumption of sick role phase of
believe something is wrong, that illness.
person is experiencing signs and
symptoms of an illness. The patient 7. In this stage, the person tries to
will then ASSUME that he is sick. find answers for his illness. He wants
This is called assumption of the sick his illness to be validated, his
role where the patient accepts he is symptoms explained and the
Ill and try to give up some activities. outcome reassured or predicted
Since the client only ASSUMES his
illness, he will try to ask someone to A. Symptom Experience
validate if what he is experiencing B. Assumption of sick role
is a disease, This is now called as C. Medical care contact
MEDICAL CARE CONTACT. The client D. Dependent patient role
seeks professional advice for
validation, reassurance, clarification Answer: C. Medical care contact. At
and explanation of the symptoms this stage, The patient seeks for
he is experiencing. client will then validation of his symptom
start his dependent patient role of experience. He wants to find out if
receiving care from the health care what he feels are normal or not
providers. The last stage of Illness is normal. He wants someone to
the RECOVERY stage where the explain why is he feeling these
patient gives up the sick role and signs and symptoms and wants to
assumes the previous normal know the probable outcome of this
gunctions. experience.

5. In this stage of illness, the person 8. The following are true with
accepts or rejects a professionals regards to aspect of the sick role
suggestion. The person also except
becomes passive and may regress to
an earlier stage. A. One should be held
responsible for his condition
A. Symptom Experience B. One is excused from his societal
B. Assumption of sick role role
C. Medical care contact C. One is obliged to get well as soon
D. Dependent patient role as possible
D. One is obliged to seek competent
Answer: D. Dependent patient help
role. In the dependent patient role
stage, Client needs professionals for Answer: A. One should be held
help. They have a choice either to responsible for his condition. The
accept or reject the professional’s nurse should not judge the patient
decisions but patients are usually and not view the patient as the
passive and accepting. Regression cause or someone responsible for
tends to occur more in this period. his illness. A sick client is excused
from his societal roles, Oblige to get
well as soon as possible and Obliged manifest TICS, but this alone is not
to seek competent help. enough to diagnose the patient as
other diseases has the same tic
9. Refers to conditions that increases manifestation. Syndrome means
vulnerability of individual or group to COLLECTION of these symptoms
illness or accident that occurs together to characterize
a certain disease. Tics with
A. Predisposing factor coprolalia, echolalia, palilalia,
B. Etiology choreas or other movement
C. Risk factor disorders are characteristics of
D. Modifiable Risks TOURETTE SYNDROME.

Answer: C. Risk factor 12. A woman undergoing radiation


therapy developed redness and
10. Refers to the degree of burning of the skin around the best.
resistance the potential host has This is best classified as what type of
against a certain pathogen disease?

A. Susceptibility A. Neoplastic
B. Immunity B. Traumatic
C. Virulence C. Nosocomial
D. Etiology D. Iatrogenic

Answer: A. Susceptibility. Immunity Answer: D. Iatrogenic. Iatrogenic


is the ABSOLUTE Resistance to a diseases refers to those that
pathogen considering that person resulted from treatment of a certain
has an INTACT IMMUNITY while disease. For example, A child
susceptibility is the DEGREE of frequently exposed to the X-RAY
resistance. Degree of resistance Machine develops redness and
means how well would the partial thickness burns over the
individual combat the pathogens chest area. Neoplastic are
and repel infection or invasion of malignant diseases cause by
these disease causing organisms. A proliferation of abnormally growing
susceptible person is someone who cells. Traumatic are brought about
has a very low degree of resistance by injuries like Motor vehicular
to combat pathogens. An Immune accidents. Nosocomial are
person is someone that can easily infections that acquired INSIDE the
repel specific pathogens. However, hospital. Example is UTI Because of
Remember that even if a person is catheterization, This is commonly
IMMUNE [ Vaccination ] Immunity caused by E.Coli.
can always be impaired in cases of
chemotherapy, HIV, Burns, etc. 13. The classification of CANCER
according to its etiology Is best
11. A group of symptoms that sums described as:
up or constitute a disease
1. Nosocomial
A. Syndrome 2. Idiopathic
B. Symptoms 3. Neoplastic
C. Signs 4. Traumatic
D. Etiology 5. Congenital
6. Degenrative
Answer: A. Syndrome. Symptoms A. 5 and 2
are individual manifestation of a B. 2 and 3
certain disease. For example, In C. 3 and 4
Tourette syndrome, patient will D. 3 and 5
evident structural, anatomical or
Answer: B. 2 and 3. Aside from physical change in the structure of
being NEOPLASTIC, Cancer is the organ or system but function is
considered as IDIOPATHIC because altered due to other causes, which
the cause is UNKNOWN. is usually due to abnormal response
of the organ to stressors. Therefore,
14. Term to describe the ORGANIC BRAIN SYNDROME are
reactiviation and recurrence of anatomic and physiologic change in
pronounced symptoms of a disease the BRAIN that is NON
PROGRESSIVE BUT IRREVERSIBLE
A. Remission caused by alteration in structure of
B. Emission the brain and it’s supporting
C. Exacerbation structure which manifests different
D. Sub acute sign and symptoms of neurological,
physiologic and psychologic
Answer:C. Exacerbation alterations. Mental disorders
manifesting symptoms of psychoses
15. A type of illness characterized by without any evident organic or
periods of remission and structural damage are termed as
exacerbation INORGANIC PSYCHOSES while
alteration in the organ structures
A. Chronic that causes symptoms of bizaare
B. Acute pyschotic behavior is termed as
C. Sub acute ORGANIC PSYCHOSES.
D. Sub chronic
17. It is the science of organism as
Answer: A. Chronic. A good example affected by factors in their
is Multiple sclerosis that environment. It deals with the
characterized by periods of relationship between disease and
remissions and exacerbation and it geographical environment.
is a CHRONIC Disease. An acute and
sub acute diseases occurs too short A. Epidemiology
to manifest remissions. Chronic B. Ecology
diseases persists longer than 6 C. Statistics
months that is why remissions and D. Geography
exacerbation are observable.
Answer: B. Ecology. Ecology is the
16. Diseases that results from science that deals with the
changes in the normal structure, ECOSYSTEM and its effects on living
from recognizable anatomical things in the biosphere. It deals with
changes in an organ or body tissue diseases in relationship with the
is termed as environment. Epidimiology is simply
the Study of diseases and its
A. Functional occurence and distribution in man
B. Occupational for the purpose of controlling and
C. Inorganic preventing diseases. This was asked
D. Organic during the previous boards.

Answer:D. Organic. As the word 18. This is the study of the patterns
implies, ORGANIC Diseases are of health and disease. Its occurrence
those that causes a CHANGE in the and distribution in man, for the
structure of the organs and purpose of control and prevention of
systems. Inorganic diseases is disease.
synonymous with FUNCTIONAL
diseases wherein, There is no A. Epidemiology
B. Ecology treatment in tertiary. To best
C. Statistics differentiate the two, A client with
D. Geography ANEMIA that is being treated with
ferrous sulfate is considered being
Answer: A. Epidemiology. Refer to in the SECONDARY PREVENTION
number 17. because ANEMIA once treated, will
move the client on PRE ILLNESS
19. Refers to diseases that produced STATE again. However, In cases of
no anatomic changes but as a result ASPIRING Therapy in cases of
from abnormal response to a stimuli. stroke, ASPIRING no longer cure the
patient or PUT HIM IN THE PRE
A. Functional ILLNESS STATE. ASA therapy is done
B. Occupational in order to prevent coagulation of
C. Inorganic the blood that can lead to thrombus
D. Organic formation and a another possible
stroke. You might wonder why I
Answer: C. Inorganic. Refer to spelled ASPIRIN as ASPIRING, Its
number 16. side effect is OTOTOXICITY [ CN
VIII ] that leads to TINNITUS or
20. In what level of prevention ringing of the ears.
according to Leavell and Clark does
the nurse support the client in 21. In what level of prevention does
obtaining OPTIMAL HEALTH STATUS the nurse encourage optimal health
after a disease or injury? and increases person’s susceptibility
to illness?
A. Primary
B. Secondary A. Primary
C. Tertiary B. Secondary
D. None of the above C. Tertiary
D. None of the above
Answer: C. Tertiary. Perhaps one of
the easiest concept but asked Answer: D. None of the above. The
frequently in the NLE. Primary refers nurse never increases the person’s
to preventions that aims in susceptibility to illness but rather,
preventing the disease. Examples LESSEN the person’s susceptibility
are healthy lifestyle, good nutrition, to illness.
knowledge seeking behaviors etc.
Secondary prevention are those 22. Also known as HEALTH
that deals with early diagnostics, MAINTENANCE prevention.
case finding and treatments.
Examples are monthly breast self A. Primary
exam, Chest X-RAY, Antibiotic B. Secondary
treatment to cure infection, Iron C. Tertiary
therapy to treat anemia etc. Tertiary D. None of the above
prevention aims on maintaining
optimum level of functioning during Answer: B. Secondary. Secondary
or after the impact of a disease that prevention is also known as HEALTH
threatens to alter the normal body MAINTENANCE Prevention. Here,
functioning. Examples are The person feels signs and
prosthetis fitting for an amputated symptoms and seeks Diagnosis and
leg after an accident, Self treatment in order to prevent
monitoring of glucose among deblitating complications. Even if
diabetics, TPA Therapy after stroke the person feels healthy, We are
etc. The confusing part is between required to MAINTAIN our health by
the treatment in secondary and
monthly check ups, Physical 27. Which is the best way to
examinations, Diagnostics etc. disseminate information to the
public?
23. PPD In occupational health
nursing is what type of prevention? A. Newspaper
B. School bulletins
A. Primary C. Community bill boards
B. Secondary D. Radio and Television
C. Tertiary
D. None of the above Answer: D. Radio and Television. An
actual board question, The best way
Answer: A. Primary. PPD or to disseminate information to the
PERSONAL PROTECTIVE DEVICES public is by TELEVISION followed by
are worn by the workes in a RADIO. This is how the DOH
hazardous environment to protect establish its IEC Programs other
them from injuries and hazards. than publising posters, leaflets and
This is considered as a PRIMARY brochures. An emerging new way to
prevention because the nurse disseminate is through the internet.
prevents occurence of diseases and
injuries. 28. Who conceptualized health as
integration of parts and subparts of
24. BCG in community health an individual?
nursing is what type of prevention?
A. Newman
A. Primary B. Neuman
B. Secondary C. Watson
C. Tertiary D. Rogers
D. None of the above
Answer: B. Neuman. The supra and
Answer: primary subsystems are theories of Martha
Rogers but the parts and subparts
25. A regular pap smear for woman are Betty Neuman’s. She stated
every 3 years after establishing that HEALTH is a state where in all
normal pap smear for 3 consecutive parts and subparts of an individual
years Is advocated. What level of are in harmony with the whole
prevention does this belongs? system. Margarex Newman defined
health as an EXPANDING
A. Primary CONSCIOUSNESS. Her name is
B. Secondary Margaret not Margarex, I just used
C. Tertiary that to help you remember her
D. None of the above theory of health.

Answer: secondary 29. The following are concept of


health:
26. Self monitoring of blood glucose
for diabetic clients is on what level 1. Health is a state of complete
of prevention? physical, mental and social
wellbeing and not merely an
A. Primary absence of disease or infirmity.
B. Secondary 2. Health is the ability to maintain
C. Tertiary balance
D. None of the above 3. Health is the ability to maintain
internal milieu
Answer: tertiary 4. Health is integration of all parts
and subparts of an individual
A. 1,2,3
B. 1,3,4 33. Also known as STERILE
C. 2,3,4 TECHNIQUE
D. 1,2,3,4
A. Surgical Asepsis
Answer: D. 1,2,3,4. All of the B. Medical Asepsis
following are correct statement C. Sepsis
about health. The first one is the D. Asepsis
definition by WHO, The second one
is from Walter Cannon’s Answer: A. Surgical
homeostasis theory. Third one is Asepsis. Surgical Asepsis is also
from Claude Bernard’s concept of known as STERILE TECHNIQUE while
Health as Internal Milieu and the Medical Asepsis is synonymous with
last one is Neuman’s Theory. CLEAN TECHNIQUE.

30. The theorist the advocated that 34. This is a person or animal, who is
health is the ability to maintain without signs of illness but harbors
dynamic equilibrium is pathogen within his body and can be
transferred to another
A. Bernard
B. Selye A. Host
C. Cannon B. Agent
D. Rogers C. Environment
D. Carrier
Answer: C. Cannon. Walter Cannon
advocated health as HOMEOSTASIS Answer: carrier
or the ability to maintain dynamic
equilibrium. Hans Selye postulated 35. Refers to a person or animal,
Concepts about Stress and known or believed to have been
Adaptation. Bernard defined health exposed to a disease.
as the ability to maintain internal
milieu and Rogers defined Health as A. Carrier
Wellness that is influenced by B. Contact
individual’s culture. C. Agent
D. Host
31. Excessive alcohol intake is what
type of risk factor? Answer: contact

A. Genetics 36. A substance usually intended for


B. Age use on inanimate objects, that
C. Environment destroys pathogens but not the
D. Lifestyle spores.

Answer: lifestyle A. Sterilization


B. Disinfectant
32. Osteoporosis and degenerative C. Antiseptic
diseases like Osteoarthritis belongs D. Autoclave
to what type of risk factor?
Answer: B.
A. Genetics Disinfectant. Disinfectants are used
B. Age on inanimate objects while
C. Environment Antiseptics are intended for use on
D. Lifestyle persons and other living things.
Both can kill and inhibit growth of
Answer: age microorganism but cannot kill their
spores. That is when autoclaving or the disease process or it’s gradual
steam under pressure gets in, disappearance.
Autoclaving can kill almost ALL type
of microoganism including their 39. A child with measles developed
spores. fever and general weakness after
being exposed to another child with
37. This is a process of removing rubella. In what stage of infectious
pathogens but not their spores process does this child belongs?

A. Sterilization A. Incubation period


B. Auto claving B. Prodromal period
C. Disinfection C. Illness period
D. Medical asepsis D. Convalescent period

Answer: C. Disinfection. Both A and Answer: B. Prodromal period. To be


B are capable on killing spores. able to categorize MEASLES in the
Autoclaving is a form of Illness period, the specific signs of
Sterilization. Medical Asepsis is a Fever, Koplik’s Spot and Rashes
PRACTICE designed to minimize or must appear. In the situation above,
reduce the transfer of pathogens, Only general signs and symptoms
also known as your CLEAN appeared and the Specific signs and
TECHNIQUE. Disinfection is the symptoms is yet to appear,
PROCESS of removing pathogens therefore, the illness is still in the
but not their spores. Prodromal period. Signs and
symptoms of measles during the
38. The third period of infectious prodromal phase are Fever, fatigue,
processes characterized by runny nose, cough and
development of specific signs and conjunctivitis. Koplik’s spot heralds
symptoms the Illness period and cough is the
last symptom to disappear. All of
A. Incubation period this processes take place in 10 days
B. Prodromal period that is why, Measles is also known
C. Illness period as 10 day measles.
D. Convalescent period
40. A 50 year old mailman carried a
Answer: C. Illness period. In mail with anthrax powder in it. A
incubation period, The disease has minute after exposure, he still hasn’t
been introduced to the body but no developed any signs and symptoms
sign and symptom appear because of anthrax. In what stage of
the pathogen is not yet strong infectious process does this man
enough to cause it and may still belongs?
need to multiply. The second period
is called prodromal period. This is A. Incubation period
when the appearance of non B. Prodromal period
specific signs and symptoms sets C. Illness period
in, This is when the sign and D. Convalescent period
symptoms starts to appear. Illness
period is characterized by the Answer: A. Incubation
appearance of specific signs and period. Anthrax can have an
symptoms or refer tp as time with incubation period of hours to 7 days
the greatest symptom experience. with an average of 48 hours. Since
Acme is the PEAK of illness intensity the question stated exposure, we
while the convalescent period is can now assume that the mailman
characterized by the abatement of is in the incubation period.
41. Considered as the WEAKEST of foods by ENTERING / INVADING
LINK in the chain of infection that [PORTAL OF ENTRY] our HOUSE
nurses can manipulate to prevent [SUSCEPTIBLE HOST]. By imagining
spread of infection and diseases the Ant’s life cycle, we can easily
arrange the chain of infection.
A. Etiologic/Infectious agent
B. Portal of Entry 43. Markee, A 15 year old high
C. Susceptible host school student asked you. What is
D. Mode of transmission the mode of transmission of Lyme
disease. You correctly answered him
Answer: D. Mode of that Lyme disease is transmitted via
transmission. Mode of transmission
is the weakest link in the chain of A. Direct contact transmission
infection. It is easily manipulated by B. Vehicle borne transmission
the Nurses using the tiers of C. Air borne transmission
prevention, either by instituting D. Vector borne transmission
transmission based precautions,
Universal precaution or Isolation Answer: D. Vector borne
techniques. transmission. Lyme disease is
caused by Borrelia Burdorferi and is
42. Which of the following is the transmitted by a TICK BITE.
exact order of the infection chain?
44. The ability of the infectious
1. Susceptible host agent to cause a disease primarily
2. Portal of entry depends on all of the following
3. Portal of exit except
4. Etiologic agent
5. Reservoir A. Pathogenicity
6. Mode of transmission B. Virulence
A. 1,2,3,4,5,6 C. Invasiveness
B. 5,4,2,3,6,1 D. Non Specificity
C. 4,5,3,6,2,1
D. 6,5,4,3,2,1 Answer: D. Non Specificity. To be
able to cause a disease, A pathogen
Answer: C. 4,5,3,6,2,1. Chain of should have a TARGET ORGAN/S.
infection starts with the SOURCE : The pathogen should be specific to
The etiologic agent itself. It will first these organs to cause an infection.
proliferate on a RESERVOIR and will Mycobacterium Avium is NON
need a PORTAL OF EXIT to be able SPECIFIC to human organs and
to TRANSMIT irslef using a PORTAL therefore, not infective to humans
OF ENTRY to a SUSCEPTIBLE HOST. but deadly to birds. An
A simple way to understand the immunocompromised individual,
process is by looking at the lives of specially AIDS Patient, could be
a young queen ant that is starting infected with these NON SPECIFIC
to build her colony. Imagine the diseases due to impaired immune
QUEEN ANT as a SOURCE or the system.
ETIOLOGIC AGENT. She first need to
build a COLONY, OR the RESERVOIR 45. Contact transmission of
where she will start to lay the first infectious organism in the hospital is
eggs to be able to produce her usually cause by
worker ants and soldier ants to be
able to defend and sustain the new A. Urinary catheterization
colony. They need to EXIT [PORTAL B. Spread from patient to patient
OF EXIT] their colony and crawl
[MODE OF TRANSMISSION] in search
C. Spread by cross A. Creed
contamination via hands of B. Immunization
caregiver C. Current medication being taken
D. Cause by unclean instruments D. Color of the skin
used by doctors and nurses
Answer: A. Creed. Creed, Faith or
Answer: C. Spread by cross religious belief do not affect
contamination via hands of person’s susceptibility to illness.
caregiver. The hands of the Medication like corticosteroids could
caregiver like nurses, is the main supress a person’s immune system
cause of cross contamination in that will lead to increase
hospital setting. That is why susceptibility. Color of the skin could
HANDWASHING is the single most affect person’s susceptibility to
important procedure to prevent the certain skin diseases. A dark
occurence of cross contamination skinned person has lower risk of
and nosocomial infection. D refers skin cancer than a fair skinned
to Nosocomial infection and UTI is person. Fair skinned person also has
the most common noscomial a higher risk for cholecystitis and
infection in the hospital caused by cholelithiasis.
urinary catheterization. E.Coli
seems to be the major cause of this 49. Graciel has been injected TT5,
incident. B best fits Cross her last dosed for tetanus toxoid
Contamination, It is the spread of immunization. Graciel asked you,
microogranisms from patient o what type of immunity is TT
patient. Injections? You correctly answer her
by saying Tetanus toxoid
46. Transmission occurs when an immunization is a/an
infected person sneezes, coughs or
laugh that is usually projected at a A. Natural active immunity
distance of 3 feet. B. Natural passive immunity
C. Artificial active immunity
A. Droplet transmission D. Artificial passive immunity
B. Airborne transmission
C. Vehicle transmission Answer: C. Artificial active
D. Vector borne transmission immunity. TT1 ti TT2 are considered
the primary dose, while TT3 to TT5
Answer: Droplet transmission are the booster dose. A woman with
completed immunization of DPT
47. Considered as the first line of need not receive TT1 and TT2.
defense of the body against Tetanus toxoid is the actual toxin
infection produce by clostridium tetani but on
its WEAK and INACTIVATED form. It
A. Skin is Artificial because it did not occur
B. WBC in the course of actual illness or
C. Leukocytes infection, it is Active because what
D. Immunization has been passed is an actual toxin
and not a ready made
Answer: A. Skin. Remember that immunoglobulin.
intact skin and mucus membrane is
our first line of defense against 50. Agatha, was hacked and slashed
infection. by a psychotic man while she was
crossing the railway. She suffered
48. All of the following contributes to multiple injuries and was injected
host susceptibility except Tetanus toxoid Immunoglobulin.
Agatha asked you, What immunity
does TTIg provides? You best FRICTION. The rest, will just enhance
answered her by saying TTIg friction. The use of soap lowers the
provides surface tension thereby increasing
the effectiveness of friction. Water
A. Natural active immunity helps remove transient bacteria by
B. Natural passive immunity working with soap to create the
C. Artificial active immunity lather that reduces surface tension.
D. Artificial passive immunity Time is of essence but friction is the
most essential aspect of
Answer: D. Artificial passive handwashing
immunity. In this scenario, Agatha
was already wounded and has 53. In handwashing by medical
injuries. Giving the toxin [TT asepsis, Hands are held ….
Vaccine] itself would not help
Agatha because it will take time A. Above the elbow, The hands
before the immune system produce must always be above the waist
antitoxin. What agatha needs now is B. Above the elbow, The hands are
a ready made anti toxin in the form cleaner than the elbow
of ATS or TTIg. This is artificial, C. Below the elbow, Medical asepsis
because the body of agatha did not do not require hands to be above
produce it. It is passive because her the waist
immune system is not stimulated D. Below the elbow, Hands are
but rather, a ready made Immune dirtier than the lower arms
globulin is given to immediately
supress the invasion. Answer: D. Below the elbow, Hands
are dirtier than the lower
51. This is the single most important arms. Hands are held BELOW the
procedure that prevents cross elbow in medical asepsis in contrast
contamination and infection with surgical asepsis, wherein,
nurses are required to keep the
A. Cleaning hands above the waist. The
B. Disinfecting rationale is because in medical
C. Sterilizing asepsis, Hands are considered
D. Handwashing dirtier than the elbow and therefore,
to limit contamination of the lower
Answer: D. Handwashing. When you arm, The hands should always be
see the word HANDWASHING as one below the elbow.
of the options, 90% Chance it is the
correct answer in the local board. Or 54. The suggested time per hand on
should I say, 100% because I have handwashing using the time method
yet to see question from 1988 to is
2005 board questions that has
option HANDWASHING on it but is A. 5 to 10 seconds each hand
not the correct answer. B. 10 to 15 seconds each hand
C. 15 to 30 seconds each hand
52. This is considered as the most D. 30 to 60 seconds each hand
important aspect of handwashing
Answer: C. 15 to 30 seconds each
A. Time hand. Each hands requires atleast
B. Friction 15 to 30 seconds of handwashing to
C. Water effectively remove transient
D. Soap microorganisms.

Answer: B. Friction. The most 55. The minimum time in washing


important aspect of handwashing is each hand should never be below
A. 5 seconds A. The minimum time for boiling
B. 10 seconds articles is 5 minutes
C. 15 seconds B. Boil the glass baby bottler and
D. 30 seconds other articles for atleast 10
minutes
Answer:B. 10 seconds. According to C. For boiling to be effective, a
Kozier, The minimum time required minimum of 15 minutes is
for watching each hands is 10 required
seconds and should not be lower D. It doesn’t matter how long you
than that. The recommended time, boil the articles, as long as the
again, is 15 to 30 seconds. water reached 100 degree
Celsius
56. How many ml of liquid soap is
recommended for handwashing Answer: C. For boiling to be
procedure? effective, a minimum of 15 minutes
is required. Boiling is the most
A. 1-2 ml common and least expensive
B. 2-3 ml method of sterilization used in
C. 2-4 ml home. For it to be effective, you
D. 5-10 ml should boil articles for atleast 15
minutes.
Answer: C. 2-4 ml. If a liquid soap is
to be used, 1 tsp [ 5ml ] of liquid 59. This type of disinfection is best
soap is recommended for done in sterilizing drugs, foods and
handwashing procedure. other things that are required to be
sterilized before taken in by the
57. Which of the following is not true human body
about sterilization, cleaning and
disinfection? A. Boiling Water
B. Gas sterilization
A. Equipment with small lumen C. Steam under pressure
are easier to clean D. Radiation
B. Sterilization is the complete
destruction of all viable Answer: D. Radiation. Imagine foods
microorganism including spores and drugs that are being sterilized
C. Some organism are easily by a boiling water, ethylene oxide
destroyed, while other, with gas and autoclave or steam under
coagulated protein requires pressure, They will be inactivated
longer time by these methods. Ethylene oxide
D. The number of organism is gas used in gas sterlization is TOXIC
directly proportional to the length to humans. Boiling the food will
of time required for sterilization alter its consistency and nutrients.
Autoclaving the food is never
Answer: A. Equipment with small performed. Radiation using
lumen are easier to microwave oven or Ionizing
clean. Equipments with LARGE radiation penetrates to foods and
LUMEN are easier to clean than drugs thus, sterilizing them.
those with small lumen. B C and D
are all correct. 60. A TB patient was discharged in
the hospital. A UV Lamp was placed
58. Karlita asked you, How long in the room where he stayed for a
should she boil her glass baby bottle week. What type of disinfection is
in water? You correctly answered her this?
by saying
A. Concurrent disinfection
B. Terminal disinfection C. The instruments are put into
C. Regular disinfection unlocked position, on their
D. Routine disinfection hinge, during the autoclave
D. Autoclaving different kinds of
Answer: B. Terminal metals at one time is advisable
disinfection. Terminal disinfection
refers to practices to remove Answer: C. The instruments are put
pathogens that stayed in the into unlocked position, on their
belongings or immediate hinge, during the autoclave. Only C
environemnt of an infected client is correct. Metals with locks, like
who has been discharged. An clamps and scissors should be
example would be Killing airborne UNLOCKED in order to minimize
TB Bacilli using UV Light. stiffening caused by autoclave to
Concurrent disinfection refers to the hinges of these metals. NOT ALL
ongoing efforts implented during microorganism are destroyed by
the client’s stay to remove or limit autoclaving. There are recently
pathogens in his supplies, discovered microorganism that is
belongings, immediate environment invulnarable to extreme heat.
in order to control the spread of the Autoclaved instruments are to be
disease. An example is cleaning the used within 2 weeks. Only the same
bedside commode of a client with type of metals should be autoclaved
radium implant on her cervix with a as this will alteration in plating of
bleach disinfectant after each these metals.
voiding.
63. Which of the following is true
61. Which of the following is not true about masks?
in implementing medical asepsis
A. Mask should only cover the nose
A. Wash hand before and after B. Mask functions better if they are
patient contact wet with alcohol
B. Keep soiled linens from touching C. Masks can provide durable
the clothings protection even when worn for a
C. Shake the linens to remove long time and after each and
dust every patient care
D. Practice good hygiene D. N95 Mask or particulate
masks can filter organism as
Answer: C. Shake the linens to mall as 1 micromillimeter
remove dust. NEVER shake the
linens. Once soiled, fold it inwards Answer: D. N95 Mask or particulate
clean surface out. Shaking the linen masks can filter organism as mall as
will further spread pathogens that 1 micromillimeter. Mask should
has been harbored by the fabric. cover both nose and mouth. Masks
will not function optimally when
62. Which of the following is true wet. Masks should be worn not
about autoclaving or steam under greater than 4 hours, as it will lose
pressure? effectiveness after 4 hours. N95
mask or particulate mask can filter
A. All kinds of microorganism and organism as small as 1
their spores are destroyed by micromillimeter.
autoclave machine
B. The autoclaved instruments can 64. Where should you put a wet
be used for 1 month considering adult diaper?
the bags are still intact
A. Green trashcan
B. Black trashcan
C. Orange trashcan physician is the second most
D. Yellow trashcan appropriate action among the
choices. A nurse should never
Answer: D. Yellow attempt to put it back nor, touch it
trashcan. Infectious waste like blood with her bare hands.
and blood products, wet diapers
and dressings are thrown in yellow 67. After leech therapy, Where
trashcans. should you put the leeches?

65. Needles, scalpels, broken glass A. In specially marked BIO


and lancets are considered as HAZARD Containers
injurious wastes. As a nurse, it is B. Yellow trashcan
correct to put them at disposal via C. Black trashcan
a/an D. Leeches are brought back to the
culture room, they are not thrown
A. Puncture proof container away for they are reusable
B. Reused PET Bottles
C. Black trashcan Answer: A. In specially marked BIO
D. Yellow trashcan with a tag HAZARD Containers. Leeches, in
“INJURIOUS WASTES” leech therapy or LEECH
PHLEBOTOMY are to be disposed on
Answer: A. Puncture proof a BIO HAZARD container. They are
container. Needles, scalpels and never re used as this could cause
other sharps are to be disposed in a transfer of infection. These leeches
puncture proof container. are hospital grown and not the
usual leeches found in swamps.
66. Miranda Priestly, An executive of
RAMP magazine, was diagnosed with 68. Which of the following should the
cancer of the cervix. You noticed nurse AVOID doing in preventing
that the radioactive internal implant spread of infection?
protrudes to her vagina where
supposedly, it should be in her A. Recapping the needle before
cervix. What should be your initial disposal to prevent injuries
action? B. Never pointing a needle towards
a body part
A. Using a long forceps, Push it back C. Using only Standard precaution
towards the cervix then call the to AIDS Patients
physician D. Do not give fresh and uncooked
B. Wear gloves, remove it gently fruits and vegetables to Mr.
and place it on a lead container Gatchie, with Neutropenia
C. Using a long forceps, Remove
it and place it on a lead Answer: A. Recapping the needle
container before disposal to prevent
D. Call the physician, You are not injuries. Never recap needles. They
allowed to touch, re insert or are directly disposed in a puncture
remove it proof container after used.
Recapping the needles could cause
Answer: C. Using a long forceps, injury to the nurse and spread of
Remove it and place it on a lead infection. B C and D are all
container. A dislodged radioactive appropriate. Standard precaution is
cervical implant in brachytherapy sufficient for an HIV patient. A client
are to be picked by a LONG FORCEP with neutropenia are not given fresh
and stored in a LEAD CONTAINER in and uncooked fruits and vegetables
order to prevent damage on the for even the non infective
client’s normal tissue. Calling the organisms found in these foods
could cause severe infection on an Answer: B. Airborne
immunocompromised patients. precaution. Measles is highly
communicable and more contagious
69. Where should you put Mr. Alejar, than Rubella, It requires airborne
with Category II TB? precaution as it is spread by small
particle droplets that remains
A. In a room with positive air suspended in air and disperesed by
pressure and atleast 3 air air movements.
exchanges an hour
B. In a room with positive air 72. A client has been diagnosed with
pressure and atleast 6 air IMPETIGO. What precaution is used
exchanges an hour for this patient?
C. In a room with negative air
pressure and atleast 3 air A. Standard precaution
exchanges an hour B. Airborne precaution
D. In a room with negative air C. Droplet precaution
pressure and atleast 6 air D. Contact precaution
exchanges an hour
Answer:D. Contact
Answer: D. In a room with negative precaution. Impetigo causes blisters
air pressure and atleast 6 air or sores in the skin. It is generally
exchanges an hour. TB patients caused by GABS or Staph Aureaus.
should have a private room with It is spread by skin to skin contact
negative air pressure and atleast 6 or by scratching the lesions and
to 12 air exhanges per hour. touching another person’s skin.
Negative pressure room will prevent
air inside the room from escaping. 73. The nurse is to insert an NG Tube
Air exchanges are necessary since when suddenly, she accidentally dip
the client’s room do not allow air to the end of the tube in the client’s
get out of the room. glass containing distilled drinking
water which is definitely not sterile.
70. A client has been diagnosed with As a nurse, what should you do?
RUBELLA. What precaution is used
for this patient? A. Don’t mind the incident,
continue to insert the NG
A. Standard precaution Tube
B. Airborne precaution B. Obtain a new NG Tube for the
C. Droplet precaution client
D. Contact precaution C. Disinfect the NG Tube before
reinserting it again
Answer: C. Droplet D. Ask your senior nurse what to do
precaution. Droplet precaution is
sufficient on client’s with RUBELLA Answer: A. Don’t mind the incident,
or german measles. continue to insert the NG Tube. The
digestive tract is not sterile, and
71. A client has been diagnosed with therefore, simple errors like this
MEASLES. What precaution is used would not cause harm to the
for this patient? patient. NGT tube need not be
sterile, and so is colostomy and
A. Standard precaution rectal tubes. Clean technique is
B. Airborne precaution sufficient during NGT and colostomy
C. Droplet precaution care.
D. Contact precaution
74. All of the following are principle
of SURGICAL ASEPSIS except
A. Microorganism travels to moist A. The dominant hand
surfaces faster than with dry B. The non dominant hand
surfaces C. The left hand
B. When in doubt about the sterility D. No specific order, Its up to the
of an object, consider it not nurse for her own convenience
sterile
C. Once the skin has been Answer: B. The non dominant
sterilized, considered it hand. Gloves are put on the non
sterile dominant hands first and then, the
D. If you can reach the object by dominant hand. The rationale is
overreaching, just move around simply because humans tend to use
the sterile field to pick it rather the dominant hand first before the
than reaching for it non dominant hand. Out of 10
humans that will put on their sterile
Answer: C. Once the skin has been gloves, 8 of them will put the gloves
sterilized, considered it on their non dominant hands first.
sterile. Human skin is impossible to
be sterilized. It contains normal 77. As the scrubbed nurse, when
flora of microorganism. A B and D should you apply the goggles, shoe
are all correct. cap and mask prior to the operation?

75. Which of the following is true in A. Immediately after entering the


SURGICAL ASEPSIS? sterile field
B. After surgical hand scrub
A. Autoclaved linens and gowns are C. Before surgical hand scrub
considered sterile for about 4 D. Before entering the sterile field
months as long as the bagging is
intact Answer: C. Before surgical hand
B. Surgical technique is a sole effort scrub. The nurse should put his
of each nurse goggles, cap and mask prior to
C. Sterile conscience, is the best washing the hands. If he wash his
method to enhance sterile hands prior to putting all these
technique equipments, he must wash his
D. If a scrubbed person leaves the hands again as these equipments
area of the sterile field, He/she are said to be UNSTERILE.
must do handwashing and
gloving again, but the gown need 78. Which of the following should the
not be changed nurse do when applying gloves prior
to a surgical procedure?
Answer:C. Sterile conscience, is the
best method to enhance sterile A. Slipping gloved hand with all
technique. Sterile conscience, or the fingers when picking up the
moral imperative of a nurse to be second glove
honest in practicing sterile B. Grasping the first glove by
technique, is the best method to inserting four fingers, with
enhance sterile technique. thumbs up underneath the cuff
Autoclaved linens are considered C. Putting the gloves into the
sterile only within 2 weeks even if dominant hand first
the bagging is intact. Surgical D. Adjust only the fitting of the
technique is a team effort of each gloves after both gloves are
nurse. If a scrubbed person leave on
the sterile field and area, he must do
the process all over again. Answer: D. Adjust only the fitting of
76. In putting sterile gloves, Which the gloves after both gloves are
should be gloved first? on. The nurse should only adjust
fitting of the gloves when they are for eye goggles. The nurse will do
both on the hands. Not doing so will handwashing and then [HAND], Don
break the sterile technique. Only 4 the gloves first and wear the Gown
gingers are slipped when picking up [BODY]. I created this mnemonic
the second gloves. You cannot slip and I advise you use it because you
all of your fingers as the cuff is can never forget Camey hand and
limited and the thumb would not be body lotion. [ Yes, I know it is
able to enter the cuff. The first spelled as CAMAY ]]
glove is grasp by simply picking it
up with the first 2 fingers and a 81. In removing protective devices,
thumb in a pinching motion. Gloves which should be the exact
are put on the non dominant hands sequence?
first.
1. Eye wear or goggles
79. Which gloves should you remove 2. Cap
first? 3. Mask
4. Gloves
A. The glove of the non 5. Gown
dominant hand A. 4,3,5,1,2
B. The glove of the dominant hand B. 2,3,1,5,4
C. The glove of the left hand C. 5,4,3,2,1
D. Order in removing the gloves Is D. 1,2,3,4,5’
unnecessary
Answer: A. 4,3,5,1,2. When the
Answer: A. The glove of the non nurse is about to remove his
dominant hand. Gloves are worn in protective devices, The nurse will
the non dominant hand first, and is remove the GLOVES first followed
removed also from the non by the MASK and GOWN then, other
dominant hand first. Rationale is devices like cap, shoe cover, etc.
simply because in 10 people This is to prevent contamination of
removing gloves, 8 of them will use hair, neck and face area.
the dominant hand first and remove
the gloves of the non dominant 82. In pouring a plain NSS into a
hand. receptacle located in a sterile field,
how high should the nurse hold the
80. Before a surgical procedure, Give bottle above the receptacle?
the sequence on applying the
protective items listed below A. 1 inch
B. 3 inches
1. Eye wear or goggles C. 6 inches
2. Cap D. 10 inches
3. Mask
4. Gloves Answer: C. 6 inches. Even if you do
5. Gown not know the answer to this
A. 3,2,1,5,4 question, you can answer it
B. 3,2,1,4,5 correctly by imagining. If you pour
C. 2,3,1,5,4 the NSS into a receptacle 1 to 3
D. 2,3,1,4,5 inch above it, Chances are, The
mouth of the NSS bottle would dip
Answer: D. 2,3,1,4,5. The nurse into the receptacle as you fill it,
should use CaMEy Hand and Body making it contaminated. If you pour
Lotion in moisturizing his hand the NSS bottle into a receptacle 10
before surgical procedure and after inches above it, that is too high,
handwashing. Ca stands for chances are, as you pour the NSS,
CAP, Mstands for MASK, Ey stands most will spill out because the force
will be too much for the buoyant 3. She removes gloves and hands
force to handle. It will also be before leaving the client’s room
difficult to pour something precisely 4. She discards contaminated
into a receptacle as the height suction catheter tip in trashcan
increases between the receptacle found in the clients room
and the bottle. 6 inches is the A. 1,2
correct answer. It is not to low nor B. 1,2,3
too high. C. 1,2,3,4
D. 1,3
83. The tip of the sterile forceps is
considered sterile. It is used to Answer: C. 1,2,3,4. All soiled
manipulate the objects in the sterile equipments use in an infectious
field using the non sterile hands. client are disposed INSIDE the
How should the nurse hold a sterile client’s room to prevent
forceps? contamination outside the client’s
room. The nurse is correct in using
A. The tip should always be Mask the covers both nose and
lower than the handle mouth. Hands are washed before
B. The tip should always be above and after removing the gloves and
the handle before and after you enter the
C. The handle and the tip should be client’s room. Gloves and
at the same level contaminated suction tip are thrown
D. The handle should point in trashcan found in the clients
downward and the tip, always room.
upward
85. When performing surgical hand
Answer: A. The tip should always be scrub, which of the following nursing
lower than the handle. A sterile action is required to prevent
forcep is usually dipped into a contamination?
disinfectant or germicidal solution.
Imagine, if the tip is HIGHER than 1. Keep fingernail short, clean and
the handle, the solution will go into with nail polish
the handle and into your hands and 2. Open faucet with knee or foot
as you use the forcep, you will control
eventually lower its tip making the 3. Keep hands above the elbow
solution in your hand go BACK into when washing and rinsing
the tip thus contaminating the 4. Wear cap, mask, shoe cover after
sterile area of the forcep. To prevent you scrubbed
this, the tip should always be lower A. 1,2
than the handle. In situation B. 2,3
questions like this, IMAGINATION is C. 1,2,3
very important. D. 2,3,4

84. The nurse enters the room of the Answer: C. 1,2,3. Cap, mask and
client on airborne precaution due to shoe cover are worn BEFORE
tuberculosis. Which of the following scrubbing.
are appropriate actions by the
nurse? 86. When removing gloves, which of
the following is an inappropriate
1. She wears mask, covering the nursing action?
nose and mouth
2. She washes her hands before A. Wash gloved hand first
and after removing gloves, after B. Peel off gloves inside out
suctioning the client’s secretion C. Use glove to glove skin to skin
technique
D. Remove mask and gown has been widely used and accepted
before removing gloves by professionals today. He
conceptualized two types of human
Answer: D. Remove mask and gown response to stress, The GAS or
before removing gloves. Gloves are general adaptation syndrome which
the dirtiest protective item nurses is characterized by stages of
are wearing and therefore, the first ALARM, RESISTANCE and
to be removed to prevent spread of EXHAUSTION. The Local adaptation
microorganism as you remove the syndrome controls stress through a
mask and gown. particular body part. Example is
when you have been wounded in
87. Which of the following is TRUE in your finger, it will produce PAIN to
the concept of stress? let you know that you should
protect that particular damaged
A. Stress is not always present in area, it will also produce
diseases and illnesses inflammation to limit and control
B. Stress are only psychological and the spread of injury and facilitate
manifests psychological healing process. Another example is
symptoms when you are frequently lifting
C. All stressors evoke common heavy objects, eventually, you arm,
adaptive response back and leg muscles hypertorphies
D. Hemostasis refers to the dynamic to adapt to the stress of heavy
state of equilibrium lifting.

Answer: C. All stressors evoke 89. Which of the following is NOT


common adaptive response. All TRUE with regards to the concept of
stressors evoke common adaptive Modern Stress Theory?
response. A psychologic fear like
nightmare and a real fear or real A. Stress is not a nervous energy
perceive threat evokes common B. Man, whenever he
manifestation like tachycardia, encounters stresses, always
tachypnea, sweating, increase adapts to it
muscle tension etc. ALL diseases C. Stress is not always something to
and illness causes stress. Stress can be avoided
be both REAL or IMAGINARY. D. Stress does not always lead to
Hemostasis refers to the ARREST of distress
blood flowing abnormally through a
damage vessel. Homeostasis is the Answer: B. Man, whenever he
one that refers to dynamic state of encounters stresses, always adapts
equilibrium according to Walter to it. Man, do not always adapt to
Cannon. stress. Sometimes, stress can lead
to exhaustion and eventually,
88. According to this theorist, in his death. A,C and D are all correct.
modern stress theory, Stress is the
non specific response of the body to 90. Which of the following is TRUE
any demand made upon it. with regards to the concept of
Modern Stress Theory?
A. Hans Selye
B. Walter Cannon A. Stress is essential
C. Claude Bernard B. Man does not encounter stress if
D. Martha Rogers he is asleep
C. A single stress can cause a
answer: A. Hans Selye. Hans Selye disease
is the only theorist who proposed an D. Stress always leads to distress
intriguing theory about stress that
Answer:A. Stress is essential. Stress Answer: A. Stage of
is ESSENTIAL. No man can live Alarm. Adaptation mechanisms
normally without stress. It is begin in the stage of alarm. This is
essential because it is evoked by when the adaptive mechanism are
the body’s normal pattern of mobilized. When someone shouts
response and leads to a favorable SUNOG!!! your heart will begin to
adaptive mechanism that are beat faster, you vessels constricted
utilized in the future when more and bp increased.
stressors are encountered by the
body. Man can encounter stress 93. Stage of GAS Characterized by
even while asleep, example is adaptation
nightmare. Disease are
multifactorial, No diseases are A. Stage of Alarm
caused by a single stressors. Stress B. Stage of Resistance
are sometimes favorable and are C. Stage of Homeostasis
not always a cause for distress. An D. Stage of Exhaustion
example of favorable stress is when
a carpenter meets the demand and Answer: Stage of Resistance
stress of everyday work. He then
develops calluses on the hand to 94. Stage of GAS wherein, the Level
lessen the pressure of the hammer of resistance are decreased
against the tissues of his hand. He
also develop larger muscle and A. Stage of Alarm
more dense bones in the arm, thus, B. Stage of Resistance
a stress will lead to adaptations to C. Stage of Homeostasis
decrease that particular stress. D. Stage of Exhaustion

91. Which of the following is TRUE in Answer: A. Stage of


the stage of alarm of general Alarm. Resistance are decreased in
adaptation syndrome? the stage of alarm. Resistance is
absent in the stage of exhaustion.
A. Results from the prolonged Resistance is increased in the stage
exposure to stress of resistance.
B. Levels or resistance is increased
C. Characterized by adaptation 95. Where in stages of GAS does a
D. Death can ensue person moves back into
HOMEOSTASIS?
Answer: D. Death can ensue. Death
can ensue as early as the stage of A. Stage of Alarm
alarm. Exhaustion results to a B. Stage of Resistance
prolonged exposure to stress. C. Stage of Homeostasis
Resistance is when the levels of D. Stage of Exhaustion
resistance increases and
characterized by being able to Answer: Stage of Resistance
adapt.
96. Stage of GAS that results from
92. The stage of GAS where the prolonged exposure to stress. Here,
adaptation mechanism begins death will ensue unless extra
adaptive mechanisms are utilized
A. Stage of Alarm
B. Stage of Resistance A. Stage of Alarm
C. Stage of Homeostasis B. Stage of Resistance
D. Stage of Exhaustion C. Stage of Homeostasis
D. Stage of Exhaustion
Answer:Stage of Exhaustion communication, dressing, acting
and socializing in line with the
97. All but one is a characteristic of social and cultural standard of the
adaptive response people around the adapting
individual.
A. This is an attempt to maintain
homeostasis 100. Andy made an error and his
B. There is a totality of response senior nurse issued a written
C. Adaptive response is warning. Andy arrived in his house
immediately mobilized, mad and kicked the door hard to
doesn’t require time shut it off. What adaptation mode is
D. Response varies from person to this?
person
A. Biologic/Physiologic adaptive
Answer: C. Adaptive response is mode
immediately mobilized, doesn’t B. Psychologic adaptive mode
require time. Aside from having C. Sociocultural adaptive mode
limits that leads to exhaustion. D. Technological adaptive mode
Adaptive response requires time for
it to act. It requires energy, physical Answer: Psychologic adaptive mode
and psychological taxes that needs
time for our body to mobilize and PNLE: FON Practice Exam for
utilize. Stress, Crisis, Crisis
98. Andy, a newly hired nurse, starts
Intervention, Communication,
to learn the new technology and Recording, Learning and
electronic devices at the hospital. Documentation
Which of the following mode of 1. The coronary vessels, unlike any
adaptation is Andy experiencing? other blood vessels in the body,
respond to sympathetic stimulation
A. Biologic/Physiologic adaptive by
mode
B. Psychologic adaptive mode A. Vasoconstriction
C. Sociocultural adaptive mode B. Vasodilatation
D. Technological adaptive mode C. Decreases force of contractility
D. Decreases cardiac output
Answer: Technological adaptive
mode Answer: Vasodilatation

99. Andy is not yet fluent in French, 2. What stress response can you
but he works in Quebec where expect from a patient with blood
majority speaks French. He is sugar of 50 mg / dl?
starting to learn the language of the
people. What type of adaptation is A. Body will try to decrease the
Andy experiencing? glucose level
B. There will be a halt in release of
A. Biologic/Physiologic adaptive sex hormones
mode C. Client will appear restless
B. Psychologic adaptive mode D. Blood pressure will increase
C. Sociocultural adaptive mode
D. Technological adaptive mode Answer: Blood pressure will increase

Answer: C. Sociocultural adaptive 3. All of the following are purpose of


mode. Sociocultural adaptive inflammation except
modes include language,
A. Increase heat, thereby B. Swelling of the affected area
produce abatement of C. Pain, which causes guarding of
phagocytosis the area
B. Localized tissue injury by D. Increase heat due to transient
increasing capillary permeability vasodilation
C. Protect the issue from injury by
producing pain Answer: Redness on the affected
D. Prepare for tissue repair area

Answer: A. Increase heat, thereby


produce abatement of phagocytosis 8. The client has a chronic tissue
injury. Upon examining the client’s
4. The initial response of tissue after antibody for a particular cellular
injury is response, Which of the following
WBC component is responsible for
A. Immediate Vasodilation phagocytosis in chronic tissue
B. Transient Vasoconstriction injury?
C. Immediate Vasoconstriction
D. Transient Vasodilation A. Neutrophils
B. Basophils
Answer: C. Immediate C. Eosinophils
Vasoconstriction D. Monocytes

5. The last expected process in the Answer: Monocytes


stages of inflammation is
characterized by 9. Which of the following WBC
component proliferates in cases of
A. There will be sudden redness of Anaphylaxis?
the affected part
B. Heat will increase on the affected A. Neutrophils
part B. Basophils
C. The affected part will loss its C. Eosinophil
normal function D. Monocytes
D. Exudates will flow from the
injured site Answer: Eosinophil

Answer: . The affected part will loss 10. Icheanne, ask you, her Nurse,
its normal function about WBC Components. She got an
injury yesterday after she twisted
6. What kind of exudates is expected her ankle accidentally at her
when there is an antibody-antigen gymnastic class. She asked you,
reaction as a result of which WBC Component is
microorganism infection? responsible for proliferation at the
injured site immediately following an
A. Serous injury. You answer:
B. Serosanguinous
C. Purulent A. Neutrophils
D. Sanguinous B. Basophils
C. Eosinophils
Answer: Purulent D. Monocytes

7. The first manifestation of Answer: Neutrophils


inflammation is
11. Icheanne then asked you, what
A. Redness on the affected area is the first process that occurs in the
inflammatory response after injury, a nurse, you know that the type of
You tell her: healing that will most likely occur to
Miss Imelda is
A. Phagocytosis
B. Emigration A. First intention
C. Pavementation B. Second intention
D. Chemotaxis C. Third intention
D. Fourth intention
Answer: Pavementation
Answer: Second intention
12. Icheanne asked you again, What
is that term that describes the 16. Imelda is in the recovery stage
magnetic attraction of injured tissue after the incident. As a nurse, you
to bring phagocytes to the site of know that the diet that will be
injury? prescribed to Miss Imelda is

A. Icheanne, you better sleep now, A. Low calorie, High protein with
you asked a lot of questions Vitamin A and C rich foods
B. It is Diapedesis B. High protein, High calorie
C. We call that Emigration with Vitamin A and C rich
D. I don’t know the answer, foods
perhaps I can tell you after I C. High calorie, Low protein with
find it out later Vitamin A and C rich foods
D. Low calorie, Low protein with
Answer: D. I don’t know the answer, Vitamin A and C rich foods
perhaps I can tell you after I find it
out later B. High protein, High calorie with
Vitamin A and C rich foods
13. This type of healing occurs when
there is a delayed surgical closure of 17. Miss Imelda asked you, What is
infected wound WET TO DRY Dressing method? Your
best response is
A. First intention
B. Second intention A. It is a type of mechanical
C. Third intention debridement using Wet
D. Fourth intention dressing that is applied and
left to dry to remove dead
Answer: Third intention tissues
B. It is a type of surgical
14. Type of healing when scars are debridement with the use of Wet
minimal due to careful surgical dressing to remove the necrotic
incision and good healing tissues
C. It is a type of dressing where in,
A. First intention The wound is covered with Wet
B. Second intention or Dry dressing to prevent
C. Third intention contamination
D. Fourth intention D. It is a type of dressing where in,
A cellophane or plastic is placed
Answer: First intention on the wound over a wet
dressing to stimulate healing of
15. Imelda, was slashed and hacked the wound in a wet medium
by an unknown suspects. She
suffered massive tissue loss and Answer: A. It is a type of mechanical
laceration on her arms and elbow in debridement using Wet dressing
an attempt to evade the criminal. As
that is applied and left to dry to and which of the following below is
remove dead tissues an expected response?

18. The primary cause of pain in A. Low BP


inflammation is B. Decrease Urine output
C. Warm, flushed, dry skin
A. Release of pain mediators D. Low serum sodium levels
B. Injury to the nerve endings
C. Compression of the local Answer: Decrease Urine output
nerve endings by the edema
fluids 22. Which of the following is true
D. Circulation is lessen, Supply of about therapeutic relationship?
oxygen is insufficient
A. Directed towards helping an
Answer: C. Compression of the local individual both physically and
nerve endings by the edema fluids emotionally
B. Bases on friendship and mutual
19. The client is in stress because he trust
was told by the physician he needs C. Goals are set by the solely nurse
to undergo surgery for removal of D. Maintained even after the client
tumor in his bladder. Which of the doesn’t need anymore of the
following are effects of sympatho- Nurse’s help
adreno-medullary response by the
client? Answer:A. Directed towards helping
an individual both physically and
1. Constipation emotionally
2. Urinary frequency
3. Hyperglycemia 23. According to her, A nurse patient
4. Increased blood pressure relationship is composed of 4
A. 3,4 stages : Orientation, Identification,
B. 1,3,4 Exploitation and Resolution
C. 1,2,4
D. 1,4 A. Roy
B. Peplau
Answer: B. 1,3,4 C. Rogers
D. Travelbee
20. The client is on NPO post
midnight. Which of the following, if Answer: Peplau
done by the client, is sufficient to
cancel the operation in the morning? 24. In what phase of Nurse patient
relationship does a nurse review the
A. Eat a full meal at 10:00 P.M client’s medical records thereby
B. Drink fluids at 11:50 P.M learning as much as possible about
C. Brush his teeth the morning the client?
before operation
D. Smoke cigarette around 3:00 A. Pre Orientation
A.M B. Orientation
C. Working
Answer: D. Smoke cigarette around D. Termination
3:00 A.M
Answer: Pre Orientation
21. The client place on NPO for
preparation of the blood test. 25. Nurse Aida has seen her patient,
Adreno-cortical response is activated Roger for the first time. She
establish a contract about the
frequency of meeting and introduce D. Self awareness
to Roger the expected termination.
She started taking baseline Answer: Positive regard
assessment and set interventions
and outcomes. On what phase of 29. Which of the following statement
NPR Does Nurse Aida and Roger is not true about stress?
belong?
A. It is a nervous energy
A. Pre Orientation B. It is an essential aspect of
B. Orientation existence
C. Working C. It has been always a part of
D. Termination human experience
D. It is something each person has
Answer: orientation to cope

26. Roger has been seen agitated, Answer: A. It is a nervous energy


shouting and running. As Nurse Aida
approaches, he shouts and swear, 30. Martina, a Tennis champ was
calling Aida names. Nurse Aida told devastated after many new
Roger “That is an unacceptable competitors outpaced her in the
behavior Roger, Stop and go to your Wimbledon event. She became
room now.” The situation is most depressed and always seen crying.
likely in what phase of NPR? Martina is clearly on what kind of
situation?
A. Pre Orientation
B. Orientation A. Martina is just stressed out
C. Working B. Martina is Anxious
D. Termination C. Martina is in the exhaustion
stage of GAS
Answer: working D. Martina is in Crisis

27. Nurse Aida, in spite of the Answer: D. Martina is in Crisis


incident, still consider Roger as
worthwhile simply because he is a 31. Which of the following statement
human being. What major ingredient is not true with regards to anxiety?
of a therapeutic communication is
Nurse Aida using? A. It has physiologic component
B. It has psychologic component
A. Empathy C. The source of dread or
B. Positive regard uneasiness is from an
C. Comfortable sense of self unrecognized entity
D. Self awareness D. The source of dread or
uneasiness is from a
Answer: Positive regard recognized entity

28. Nurse Irma saw Roger and told Answer: D. The source of dread or
Nurse Aida “ Oh look at that uneasiness is from a recognized
psychotic patient “ Nurse Aida entity
should intervene and correct Nurse
Irma because her statement shows 32. Lorraine, a 27 year old executive
that she is lacking? was brought to the ER for an
unknown reason. She is starting to
A. Empathy speak but her speech is disorganized
B. Positive regard and cannot be understood. On what
C. Comfortable sense of self
level of anxiety does this features Answer: B. When the client starts to
belongs? have a narrow perceptual field and
selective inattentiveness
A. Mild
B. Moderate 36. Which of the following behavior
C. Severe is not a sign or a symptom of
D. Panic Anxiety?

Answer: Panic A. Frequent hand movement


B. Somatization
33. Elton, 21 year old nursing C. The client asks a question
student is taking the board D. The client is acting out
examination. She is sweating
profusely, has decreased awareness Answer: C. The client asks a
of his environment and is purely question
focused on the exam questions
characterized by his selective 37. Which of the following
attentiveness. What anxiety level is intervention is inappropriate for
Elton exemplifying? client’s with anxiety?

A. Mild A. Offer choices


B. Moderate B. Provide a quiet and calm
C. Severe environment
D. Panic C. Provide detailed explanation on
each and every procedures and
Answer: Moderate equipments
D. Bring anxiety down to a
34. You noticed the patient chart : controllable level
ANXIETY +3 What will you expect to
see in this client? Answer:Offer choices

A. An optimal time for learning, 38. Which of the following


Hearing and perception is greatly statement, if made by the nurse, is
increased considered not therapeutic?
B. Dilated pupils
C. Unable to communicate A. “How did you deal with your
D. Palliative Coping Mechanism anxiety before?”
B. “It must be awful to feel
Answer: B. Dilated pupils anxious.”
C. “How does it feel to be anxious?”
35. When should the nurse starts D. “What makes you feel
giving XANAX? anxious?”

A. When anxiety is +1 Answer: D. “What makes you feel


B. When the client starts to anxious?”
have a narrow perceptual
field and selective 39. Marissa Salva, Uses Benson’s
inattentiveness relaxation. How is it done?
C. When problem solving is not
possible A. Systematically tensing muscle
D. When the client is immobile and groups from top to bottom for 5
disorganized seconds, and then releasing
them
B. Concentrating on breathing
without tensing the muscle,
Letting go and repeating a 1. Spiritual Pain
word or sound after each 2. Spiritual Anxiety
exhalation 3. Spiritual Guilt
C. Using a strong positive, feeling- 4. Spiritual Despair
rich statement about a desired A. 1,2
change B. 2,3
D. Exercise combined with C. 3,4
meditation to foster relaxation D. 1,4
and mental alacrity
Answer:
Answer: B. Concentrating on 44. Grace, believes that her
breathing without tensing the relationship with God is broken. She
muscle, Letting go and repeating a tried to go to church to ask
word or sound after each exhalation forgiveness everyday to remedy her
feelings. What kind of spiritual
40. What type of relaxation distress is Grace experiencing?
technique does Lyza uses if a
machine is showing her pulse rate, A. Spiritual Pan
temperature and muscle tension B. Spiritual Alienation
which she can visualize and assess? C. Spiritual Guilt
D. Spiritual Despair
A. Biofeedback 45. Remedios felt “EMPTY” She felt
B. Massage that she has already lost God’s favor
C. Autogenic training and love because of her sins. This is
D. Visualization and Imagery a type of what spiritual crisis?

Answer: biofeedback A. Spiritual Anger


B. Spiritual Loss
41. This is also known as Self- C. Spiritual Despair
suggestion or Self-hypnosis D. Spiritual Anxiety
46. Budek is working with a
A. Biofeedback schizophrenic patient. He noticed
B. Meditation that the client is agitated, pacing
C. Autogenic training back and forth, restless and
D. Visualization and Imagery experiencing Anxiety +3. Budek said
“You appear restless” What
Answer:Autogenic training therapeutic technique did Budek
used?
42. Which among these drugs is NOT
an anxiolytic? A. Offering general leads
B. Seeking clarification
A. Valium C. Making observation
B. Ativan D. Encouraging description of
C. Milltown perception
D. Luvox 47. Rommel told Budek “ I SEE DEAD
PEOPLE “ Budek responded “You see
Answer: Luvox dead people?” This Is an example of
therapeutic communication
43. Kenneth, 25 year old diagnosed technique?
with HIV felt that he had not lived up
with God’s expectation. He fears A. Reflecting
that in the course of his illness, God B. Restating
will be punitive and not be C. Exploring
supportive. What kind of spiritual D. Seeking clarification
crisis is Kenneth experiencing?
48. Rommel told Budek, “Do you B. “Oh hail GOD Tadle, everyone
think Im crazy?” Budek responded, bow or face his wrath!”
“Do you think your crazy?” Budek C. “Hello Mr. Tadle, You are here
uses what example of therapeutic in the hospital, I am your
communication? nurse and you are a patient
here”
A. Reflecting D. “How can you be a GOD Mr.
B. Restating Tadle? Can you tell me more
C. Exploring about it?”
D. Seeking clarification 53. Erik John Senna, Told Nurse
49. Myra, 21 year old nursing Budek “ I don’t want to that, I don’t
student has difficulty sleeping. She want that thing.. that’s too painful!”
told Nurse Budek “I really think a lot Which of the following response is
about my x boyfriend recently” NON THERAPEUTIC
Budek told Myra “And that causes
you difficulty sleeping?” Which A. “ This must be difficult for you,
therapeutic technique is used in this But I need to inject you this for
situation? your own good”
B. “ You sound afraid”
A. Reflecting C. “Are you telling me you don’t
B. Restating want this injection?”
C. Exploring D. “Why are you so anxious?
D. Seeking clarification Please tell me more about
50. Myra told Budek “I cannot sleep, your feelings Erik”
I stay away all night” Budek told her 54. Legrande De Salvaje Y Cobrador
“You have difficulty sleeping” This is La Jueteng, was caught by the
what type of therapeutic bacolod police because of his illegal
communication technique? activities. When he got home after
paying for the bail, He shouted at his
A. Reflecting son. What defense mechanism did
B. Restating Mr. La Jueteng used?
C. Exploring
D. Seeking clarification A. Restitution
51. Myra said “I saw my dead B. Projection
grandmother here at my bedside a C. Displacement
while ago” Budek responded D. Undoing
“Really? That is hard to believe, How 55. Later that day, he bought his son
do you feel about it?” What ice cream and food. What defense
technique did Budek used? mechanism is Legrande
unconsciously doing?
A. Disproving
B. Disagreeing A. Restitution
C. Voicing Doubt B. Conversion
D. Presenting Reality C. Redoing
52. Which of the following is a D. Reaction formation
therapeutic communication in 56. Crisis is a sudden event in ones
response to “I am a GOD, bow life that disturbs a person’s
before me Or ill summon the homeostasis. Which of the following
dreaded thunder to burn you and is NOT TRUE in crisis?
purge you to pieces!”
A. The person experiences
A. “You are not a GOD, you are heightened feeling of stress
Professor Tadle and you are a PE B. Inability to function in the usual
Teacher, not a Nurse. I am Glen, organized manner
Your nurse.” C. Lasts for 4 months
D. Indicates unpleasant emotional 62. What is the best intervention
feelings when the client has just experienced
57. Which of the following is a the crisis and still at the first phase
characteristic of crisis? of the crisis?

A. Lasts for an unlimited period of A. Behavior therapy


time B. Gestalt therapy
B. There is a triggering event C. Cognitive therapy
C. Situation is not dangerous to the D. Milieu Therapy
person 63. Therapeutic nurse client
D. Person totality is not involved relationship is describes as follows
58. Levito Devin, The Italian prime
minister, is due to retire next week. 1. Based on friendship and mutual
He feels depressed due to the interest
enormous loss of influence, power, 2. It is a professional relationship
fame and fortune. What type of 3. It is focused on helping the
crisis is Devin experiencing? patient solve problems and
achieve health-related goals
A. Situational 4. Maintained only as long as the
B. Maturational patient requires professional
C. Social helpA. 1,2,3
D. Phenomenal B. 1,2,4
59. Estrada, The Philippine C. 2,3,4
president, has been unexpectedly D. 1,3,4
impeached and was out of office 64. The client is scheduled to have
before the end of his term. He is in surgical removal of the tumor on her
what type of crisis? left breast. Which of the following
manifestation indicates that she is
A. Situational experiencing Mild Anxiety?
B. Maturational
C. Social A. She has increased awareness
D. Phenomenal of her environmental details
60. The tsunami in Thailand and B. She focused on selected aspect
Indonesia took thousands of people of her illness
and change million lives. The people C. She experiences incongruence of
affected by the Tsunami are action, thoughts and feelings
saddened and do not know how to D. She experiences random motor
start all over again. What type of activities
crisis is this? 65. Which of the following nursing
intervention would least likely be
A. Situational effective when dealing with a client
B. Maturational with aggressive behavior?
C. Social
D. Phenomenal A. Approach him in a calm manner
61. Which of the following is the B. Provide opportunities to
BEST goal for crisis intervention? express feelings
C. Maintain eye contact with the
A. Bring back the client in the pre client
crisis state D. Isolate the client from others
B. Make sure that the client 66. Whitney, a patient of nurse
becomes better Budek, verbalizes… “I have nothing,
C. Achieve independence nothing… nothing! Don’t make me
D. Provide alternate coping close one more door, I don’t wanna
mechanism hurt anymore!” Which of the
following is the most appropriate C. Traditional
response by Budek? D. Resource oriented
72. Type of recording that integrates
A. Why are you singing? all data about the problem, gathered
B. What makes you say that? by members of the health team.
C. Ofcourse you are everything!
D. What is that you said? A. POMR
67. Whitney verbalizes that she is B. Traditional
anxious that the diagnostic test C. Resource oriented
might reveal laryngeal cancer. Which D. Source oriented
of the following is the most 73. These are data that are
appropriate nursing intervention? monitored by using graphic charts or
graphs that indicated the
A. Tell the client not to worry until progression or fluctuation of client’s
the results are in Temperature and Blood pressure.
B. Ask the client to express
feelings and concern A. Progress notes
C. Reassure the client everything B. Kardex
will be alright C. Flow chart
D. Advice the client to divert his D. Flow sheet
attention by watching television 74. Provides a concise method of
and reading newspapers organizing and recording data about
68. Considered as the most accurate the client. It is a series of flip cards
expression of person’s thought and kept in portable file used in change
feelings of shift reports.

A. Verbal communication A. Kardex


B. Non verbal communication B. Progress Notes
C. Written communication C. SOAPIE
D. Oral communication D. Change of shift report
69. Represents inner feeling that a 75. You are about to write an
person do not like talking about. information on the Kardex. There are
4 available writing instruments to
A. Overt communication use. Which of the following should
B. Covert communication you use?
C. Verbal communication
D. Non verbal communication A. Mongol #2
70. Which of the following is NOT a B. Permanent Ink
characteristic of an effective Nurse- C. A felt or fountain pen
Client relationship? D. Pilot Pentel Pen marker
76. The client has an allergy to
A. Focused on the patient Iodine based dye. Where should you
B. Based on mutual trust put this vital information in the
C. Conveys acceptance client’s chart?
D. Discourages emotional bond
71. A type of record wherein , each A. In the first page of the client’s
person or department makes chart
notation in separate records. A nurse B. At the last page of the client’s
will use the nursing notes, The chart
doctor will use the Physician’s order C. At the front metal plate of
sheet etc. Data is arranged the chart
according to information source. D. In the Kardex
77. Which of the following is NOT
A. POMR TRUE about the Kardex
B. POR
A. It provides readily available D. With one half dose
information 84. Which of the following indicates
B. It is a tool of end of shift reports that learning has been achieved?
C. The primary basis of
endorsement A. Matuts starts exercising
D. Where Allergies information are every morning and eating a
written balance diet after you taught
78. Which of the following, if seen on her mag HL tayo program
the Nurses notes, violates B. Donya Delilah has been able to
characteristic of good recording? repeat the steps of insulin
administration after you taught it
A. The client has a blood to her
pressure of 120/80, C. Marsha said “ I understand “
Temperature of 36.6 C Pulse after you a health teaching about
rate of 120 and Respiratory family planning
rate of 22 D. John rated 100% on your given
B. Ate 50% of food served quiz about smoking and
C. Refused administration of alcoholism
betaxolol
D. Visited and seen By Dr. Santiago Answer: A. Matuts starts exercising
79. The physician ordered : Mannerix every morning and eating a balance
a.c , what does a.c means? diet after you taught her mag HL
tayo program
A. As desired
B. Before meals 85. In his theory of learning as a
C. After meals BEHAVIORISM, he stated that
D. Before bed time transfer of knowledge occurs if a
80. The physician ordered, Maalox, 2 new situation closely resembles an
hours p.c, what does p.c means? old one.

A. As desired A. Bloom
B. Before meals B. Lewin
C. After meals C. Thorndike
D. Before bed time D. Skinner
81. The physician ordered, Maxitrol, 86. Which of the following is TRUE
Od. What does Od means? with regards to learning?

A. Left eye A. Start from complex to simple


B. Right eye B. Goals should be hard to achieve
C. Both eye so patient can strive to attain
D. Once a day unrealistic goals
82. The physician orderd, C. Visual learning is the best for
Magnesium Hydroxide cc Aluminum every individual
Hydroxide. What does cc means? D. Do not teach a client when he
is in pain
A. without 87. According to Bloom, there are 3
B. with domains in learning. Which of these
C. one half domains is responsible for the ability
D. With one half dose of Donya Delilah to inject insulin?
83. Physician ordered, Paracetamol
tablet ss. What does ss means? A. Cognitive
B. Affective
A. without C. Psychomotor
B. with D. Motivative
C. one half
88. Which domains of learning is in dealing with covert
responsible for making John and communication?
Marsha understand the different
kinds of family planning methods? A. Validation
B. Listening
A. Cognitive C. Evaluation
B. Affective D. Clarification
C. Psychomotor 92. Which of the following are
D. Motivative qualities of a good recording?
89. Which of the following statement
clearly defines therapeutic 1. Brevity
communication? 2. Completeness and chronology
3. Appropriateness
A. Therapeutic communication is an 4. Accuracy
interaction process which is A. 1,2
primarily directed by the nurse B. 3,4
B. It conveys feeling of warmth, C. 1,2,3
acceptance and empathy from D. 1,2,3,4
the nurse to a patient in relaxed 93. All of the following chart entries
atmosphere are correct except
C. Therapeutic communication
is a reciprocal interaction A. V/S 36.8 C,80,16,120/80
based on trust and aimed at B. Complained of chest pain
identifying patient needs and C. Seems agitated
developing mutual goals D. Able to ambulate without
D. Therapeutic communication is an assistance
assessment component of the 94. Which of the following teaching
nursing process method is effective in client who
needs to be educated about self
Answer:C. Therapeutic injection of insulin?
communication is a reciprocal
interaction based on trust and A. Detailed explanation
aimed at identifying patient needs B. Demonstration
and developing mutual goals C. Use of pamphlets
D. Film showing
90. Which of the following concept is 95. What is the most important
most important in establishing a characteristic of a nurse patient
therapeutic nurse patient relationship?
relationship?
A. It is growth facilitating
A. The nurse must fully understand B. Based on mutual understanding
the patient’s feelings, perception C. Fosters hope and confidence
and reactions before goals can D. Involves primarily emotional
be established bond
B. The nurse must be a role model 96. Which of the following nursing
for health fostering behavior intervention is needed before
C. The nurse must recognize that teaching a client post spleenectomy
the patient may manifest deep breathing and coughing
maladaptive behavior after exercises?
illness
D. The nurse should understand A. Tell the patient that deep
that patients might test her breathing and coughing
before trust is established exercises is needed to
91. Which of the following promote good breathing,
communication skill is most effective
circulation and prevent PNLE: FON Practice Exam for
complication Nursing Process, Physical and
B. Tell the client that deep breathing
and coughing exercises is needed
Health Assessment and
to prevent Thrombophlebitis, Routine Procedures
hydrostatic pneumonia and 1. She is the first one to coin the
atelectasis term “NURSING PROCESS” She
C. Medicate client for pain introduced 3 steps of nursing
D. Tell client that cooperation is vital process which are Observation,
to improve recovery Ministration and Validation.
97. The client has an allergy with
penicillin. What is the best way to A. Nightingale
communicate this information? B. Johnson
C. Rogers
A. Place an allergy alert in the D. Hall
Kardex 2. The American Nurses association
B. Notify the attending formulated an innovation of the
physician Nursing process. Today, how many
C. Write it on the patient’s chart distinct steps are there in the
D. Take note when giving nursing process?
medications
98. An adult client is on extreme A. APIE – 4
pain. He is moaning and grimacing. B. ADPIE – 5
What is the best way to assess the C. ADOPIE – 6
client’s pain? D. ADOPIER – 7
3. They are the first one to suggest a
A. Perform physical assessment 4 step nursing process which are :
B. Have the client rate his pain on APIE , or assessment, planning,
the smiley pain rating scale implementation and evaluation.
C. Active listening on what the
patient says 1. Yura
D. Observe the client’s behavior 2. Walsh
99. Therapeutic communication 3. Roy
begins with? 4. Knowles
A. 1,2
A. Knowing your client B. 1,3
B. Knowing yourself C. 3,4
C. Showing empathy D. 2,3
D. Encoding 4. Which characteristic of nursing
100. The PCS gave new guidelines process is responsible for proper
including leaflets to educate cancer utilization of human resources, time
patients. As a nurse, When using and cost resources?
materials like this, what is your
responsibility? A. Organized and Systematic
B. Humanistic
A. Read it for the patient C. Efficient
B. Give it for the patient to read D. Effective
himself 5. Which characteristic of nursing
C. Let the family member read the process addresses the
material for the patient INDIVIDUALIZED care a client must
D. Read it yourself then, Have receive?
the client read the material
A. Organized and Systematic
B. Humanistic
C. Efficient
D. Effective POWERLESSNESS R/T DIFFICULTY
6. A characteristic of the nursing ACCEPTING LOSS OF LOVED ONE.
process that is essential to promote She then focus on gathering data to
client satisfaction and progress. The refute or prove her diagnosis but her
care should also be relevant with the plans and interventions are already
client’s needs. ongoing for the diagnosis. Which
type of Diagnosis is this?
A. Organized and Systematic
B. Humanistic A. Actual
C. Efficient B. Probable
D. Effective C. Possible
7. Rhina, who has Menieres disease, D. Risk
said that her environment is moving. 11. Nurse Angela knew that Stephen
Which of the following is a valid Lee Mu Chin, has just undergone an
assessment? operation with an incision near the
diaphragm. She knew that this will
1. Rhina is giving an objective data contribute to some complications
2. Rhina is giving a subjective data later on. She then should develop
3. The source of the data is primary what type of Nursing diagnosis?
4. The source of the data is
secondary A. Actual
A. 1,3 B. Probable
B. 2,3 C. Possible
C. 2.4 D. Risk
D. 1,4 12. Which of the following Nursing
8. Nurse Angela, observe Joel who is diagnosis is INCORRECT?
very apprehensive over the
impending operation. The client is A. Fluid volume deficit R/T Diarrhea
experiencing dyspnea, diaphoresis B. High risk for injury R/T
and asks lots of questions. Angela Absence of side rails
made a diagnosis of ANXIETY R/T C. Possible ineffective coping R/T
INTRUSIVE PROCEDURE. This is what Loss of loved one
type of Nursing Diagnosis? D. Self esteem disturbance R/T
Effects of surgical removal of the
A. Actual leg
B. Probable 13. Among the following statements,
C. Possible which should be given the HIGHEST
D. Risk priority?
9. Nurse Angela diagnosed Mrs.
Delgado, who have undergone a A. Client is in extreme pain
BKA. Her diagnosis is SELF ESTEEM B. Client’s blood pressure is 60/40
DISTURBANCE R/T CHANGE IN BODY C. Client’s temperature is 40 deg.
IMAGE. Although the client has not Centigrade
yet seen her lost leg, Angela already D. Client is cyanotic
anticipated the diagnosis. This is 14. Which of the following need is
what type of Diagnosis? given a higher priority among
others?
A. Actual
B. Probable A. The client has attempted suicide
C. Possible and safety precaution is needed
D. Risk B. The client has disturbance in his
10. Nurse Angela is about to make a body image because of the
diagnosis but very unsure because recent operation
the S/S the client is experiencing is C. The client is depressed because
not specific with her diagnosis of her boyfriend left her all alone
D. The client is thirsty and A. It is nursing centered
dehydrated B. Rationales are supported by
15. Which of the following is TRUE interventions
with regards to Client Goals? C. Verbal
D. Atleast 2 goals are needed for
A. They are specific, measurable, every nursing diagnosis
attainable and time bounded 21. A framework for health
B. They are general and broadly assessment that evaluates the
stated effects of stressors to the mind,
C. They should answer for WHO, body and environment in relation
WHAT ACTIONS, WHAT with the ability of the client to
CIRCUMSTANCES, HOW WELL and perform ADL.
WHEN.
D. Example is : After discharge A. Functional health framework
planning, Client demonstrated B. Head to toe framework
the proper psychomotor skills for C. Body system framework
insulin injection. D. Cephalocaudal framework
16. Which of the following is a NOT a 22. Client has undergone Upper GI
correct statement of an Outcome and Lower GI series. Which type of
criteria? health assessment framework is
used in this situation?
A. Ambulates 30 feet with a cane
before discharge A. Functional health framework
B. Discusses fears and concerns B. Head to toe framework
regarding the surgical procedure C. Body system framework
C. Demonstrates proper coughing D. Cephalocaudal framework
and breathing technique after a 23. Which of the following statement
teaching session is true regarding temperature?
D. Reestablishes a normal
pattern of elimination A. Oral temperature is more
17. Which of the following is a accurate than rectal temperature
OBJECTIVE data? B. The bulb used in Rectal
temperature reading is pear
A. Dizziness shaped or round
B. Chest pain C. The older the person, the higher
C. Anxiety his BMR
D. Blue nails D. When the client is swimming,
18. A patient’s chart is what type of BMR Decreases
data source? 24. A type of heat loss that occurs
when the heat is dissipated by air
A. Primary current
B. Secondary
C. Tertiary A. Convection
D. Can be A and B B. Conduction
19. All of the following are C. Radiation
characteristic of the Nursing process D. Evaporation
except 25. Which of the following is TRUE
about temperature?
A. Dynamic
B. Cyclical A. The highest temperature
C. Universal usually occurs later in a day,
D. Intrapersonal around 8 P.M to 12 M.N
20. Which of the following is true B. The lowest temperature is
about the NURSING CARE PLAN? usually in the Afternoon, Around
12 P.M
C. Thyroxin decreases body now, his temperature is back to
temperature normal. Which of the following best
D. Elderly people are risk for describe the fever john is having?
hyperthermia due to the absence
of fats, Decreased A. Relapsing
thermoregulatory control and B. Intermittent
sedentary lifestyle. C. Remittent
26. Hyperpyrexia is a condition in D. Constant
which the temperature is greater 31. The characteristic fever in
than Dengue Virus is characterized as:

A. 40 degree Celsius A. Tricyclic


B. 39 degree Celsius B. Bicyclic
C. 100 degree Fahrenheit C. Biphasic
D. 105.8 degree Fahrenheit D. Triphasic
27. Tympanic temperature is taken 32. When John has been given
from John, A client who was brought paracetamol, his fever was brought
recently into the ER due to frequent down dramatically from 40 degrees
barking cough. The temperature Celsius to 36.7 degrees in a matter
reads 37.9 Degrees Celsius. As a of 10 minutes. The nurse would
nurse, you conclude that this assess this event as:
temperature is
A. The goal of reducing john’s fever
A. High has been met with full
B. Low satisfaction of the outcome
C. At the low end of the normal criteria
range B. The desired goal has been
D. At the high end of the normal partially met
range C. The goal is not completely met
28. John has a fever of 38.5 Deg. D. The goal has been met but
Celsius. It surges at around 40 not with the desired outcome
Degrees and go back to 38.5 criteria
degrees 6 times today in a typical 33. What can you expect from
pattern. What kind of fever is John Marianne, who is currently at the
having? ONSET stage of fever?

A. Relapsing A. Hot, flushed skin


B. Intermittent B. Increase thirst
C. Remittent C. Convulsion
D. Constant D. Pale,cold skin
29. John has a fever of 39.5 degrees 34. Marianne is now at the
2 days ago, But yesterday, he has a Defervescence stage of the fever,
normal temperature of 36.5 degrees. which of the following is expected?
Today, his temperature surges to 40
degrees. What type of fever is John A. Delirium
having? B. Goose flesh
C. Cyanotic nail beds
A. Relapsing D. Sweating
B. Intermittent 35. Considered as the most
C. Remittent accessible and convenient method
D. Constant for temperature taking
30. John’s temperature 10 hours ago
is a normal 36.5 degrees. 4 hours A. Oral
ago, He has a fever with a B. Rectal
temperature of 38.9 Degrees. Right C. Tympanic
D. Axillary B. Males have higher pulse rate
36. Considered as Safest and most than females after puberty
non invasive method of temperature C. Digitalis has a positive
taking chronotropic effect
D. In lying position, Pulse rate is
A. Oral higher
B. Rectal 43. The following are correct actions
C. Tympanic when taking radial pulse except:
D. Axillary
37. Which of the following is NOT a A. Put the palms downward
contraindication in taking ORAL B. Use the thumb to palpate the
temperature? artery
C. Use two or three fingers to
A. Quadriplegic palpate the pulse at the inner
B. Presence of NGT wrist
C. Dyspnea D. Assess the pulse rate, rhythm,
D. Nausea and Vomitting volume and bilateral quality
38. Which of the following is a 44. The difference between the
contraindication in taking RECTAL systolic and diastolic pressure is
temperature? termed as

A. Unconscious A. Apical rate


B. Neutropenic B. Cardiac rate
C. NPO C. Pulse deficit
D. Very young children D. Pulse pressure
39. How long should the Rectal 45. Which of the following
Thermometer be inserted to the completely describes PULSUS
clients anus? PARADOXICUS?

A. 1 to 2 inches A. A greater-than-normal increase in


B. 5 to 1.5 inches systolic blood pressure with
C. 3 to 5 inches inspiration
D. 2 to 3 inches B. A greater-than-normal
40. In cleaning the thermometer decrease in systolic blood
after use, The direction of the pressure with inspiration
cleaning to follow Medical Asepsis C. Pulse is paradoxically low when
is : client is in standing position and
high when supine.
A. From bulb to stem D. Pulse is paradoxically high when
B. From stem to bulb client is in standing position and
C. From stem to stem low when supine.
D. From bulb to bulb 46. Which of the following is TRUE
41. How long should the about respiration?
thermometer stay in the Client’s
Axilla? A. I:E 2:1
B. I:E : 4:3
A. 3 minutes C. I:E 1:1
B. 4 minutes D. I:E 1:2
C. 7 minutes 47. Contains the pneumotaxic and
D. 10 minutes the apneutic centers
42. Which of the following statement
is TRUE about pulse? A. Medulla oblongata
B. Pons
A. Young person have higher C. Carotid bodies
pulse than older persons D. Aortic bodies
48. Which of the following is B. BP = 160/120
responsible for deep and prolonged C. BP = 90/60
inspiration D. BP = 80/50
55. Refers to the pressure when the
A. Medulla oblongata ventricles are at rest
B. Pons
C. Carotid bodies A. Diastole
D. Aortic bodies B. Systole
49. Which of the following is C. Preload
responsible for the rhythm and D. Pulse pressure
quality of breathing? 56. Which of the following is TRUE
about the blood pressure
A. Medulla oblongata determinants?
B. Pons
C. Carotid bodies A. Hypervolemia lowers BP
D. Aortic bodies B. Hypervolemia increases GFR
50. The primary respiratory center C. HCT of 70% might decrease or
increase BP
A. Medulla oblongata D. Epinephrine decreases BP
B. Pons 57. Which of the following do not
C. Carotid bodies correctly correlates the increase BP
D. Aortic bodies of Ms. Aida, a 70 year old diabetic?
51. Which of the following is TRUE
about the mechanism of action of A. Females, after the age 65
the Aortic and Carotid bodies? tends to have lower BP than
males
A. If the BP is elevated, the RR B. Disease process like Diabetes
increases increase BP
B. If the BP is elevated, the RR C. BP is highest in the morning, and
decreases lowest during the night
C. Elevated BP leads to Metabolic D. Africans, have a greater risk of
alkalosis hypertension than Caucasian and
D. Low BP leads to Metabolic Asians.
acidosis 58. How many minutes are allowed
52. All of the following factors to pass if the client had engaged in
correctly influence respiration except strenuous activities, smoked or
one. Which of the following is ingested caffeine before taking
incorrect? his/her BP?

A. Hydrocodone decreases RR A. 5
B. Stress increases RR B. 10
C. Increase temperature of the C. 15
environment, Increase RR D. 30
D. Increase altitude, Increase RR 59. Too narrow cuff will cause what
53. When does the heart receives change in the Client’s BP?
blood from the coronary artery?
A. True high reading
A. Systole B. True low reading
B. Diastole C. False high reading
C. When the valves opens D. False low reading
D. When the valves closes 60. Which is a preferable arm for BP
54. Which of the following is more taking?
life threatening?
A. An arm with the most
A. BP = 180/100 contraptions
B. The left arm of the client with a D. Pulse +1 is considered as
CVA affecting the right brain NORMAL
C. The right arm 65. In assessing the abdomen,
D. The left arm Which of the following is the correct
61. Which of the following is sequence of the physical
INCORRECT in assessing client’s BP? assessment?

A. Read the mercury at the A. Inspection, Auscultation,


upper meniscus, preferably Percussion, Palpation
at the eye level to prevent B. Palpation, Auscultation,
error of parallax Percussion, Inspection
B. Inflate and deflate slowly, 2-3 C. Inspection, Palpation,
mmHg at a time Auscultation, Percussion
C. The sound heard during taking D. Inspection, Auscultation,
BP is known as KOROTKOFF Palpation, Percussion
sound 66. The sequence in examining the
D. If the BP is taken on the left leg quadrants of the abdomen is:
using the popliteal artery
pressure, a BP of 160/80 is A. RUQ,RLQ,LUQ,LLQ
normal. B. RLQ,RUQ,LLQ,LUQ
62. Which of the following is the C. RUQ,RLQ,LLQ,LUQ
correct interpretation of the ERROR D. RLQ,RUQ,LUQ,LLQ
OF PARALLAX 67. In inspecting the abdomen,
which of the following is NOT DONE?
A. If the eye level is higher than the
level of the meniscus, it will A. Ask the client to void first
cause a false high reading B. Knees and legs are
B. If the eye level is higher than straighten to relax the
the level of the meniscus, it abdomen
will cause a false low reading C. The best position in assessing
C. If the eye level is lower than the the abdomen is Dorsal
level of the meniscus, it will recumbent
cause a false low reading D. The knees and legs are externally
D. If the eye level is equal to that of rotated
the level of the upper meniscus, 68. Dr. Fabian De Las Santas, is
the reading is accurate about to conduct an
63. How many minute/s is/are ophthalmoscope examination. Which
allowed to pass before making a re- of the following, if done by a nurse,
reading after the first one? is a Correct preparation before the
procedure?
A. 1
B. 5 A. Provide the necessary draping to
C. 15 ensure privacy
D. 30 B. Open the windows, curtains and
64. Which of the following is TRUE light to allow better illumination
about the auscultation of blood C. Pour warm water over the
pressure? ophthalmoscope to ensure
comfort
A. Pulse + 4 is considered as FULL D. Darken the room to provide
B. The bell of the stethoscope is better illumination
use in auscultating BP 69. If the client is female, and the
C. Sound produced by BP is doctor is a male and the patient is
considered as HIGH frequency about to undergo a vaginal and
sound cervical examination, why is it
necessary to have a female nurse in D. The nurse added preservatives
attendance? as per protocol and refrigerates
the specimen
A. To ensure that the doctor 74. This specimen is required to
performs the procedure safely assess glucose levels and for the
B. To assist the doctor presence of albumin the the urine
C. To assess the client’s response to
examination A. Midstream clean catch urine
D. To ensure that the procedure B. 24 hours urine collection
is done in an ethical manner C. Postprandial urine collection
70. In palpating the client’s breast, D. Second voided urine
Which of the following position is 75. When should the client test his
necessary for the patient to assume blood sugar levels for greater
before the start of the procedure? accuracy?

A. Supine A. During meals


B. Dorsal recumbent B. In between meals
C. Sitting C. Before meals
D. Lithotomy D. 2 Hours after meals
71. When is the best time to collect 76. In collecting a urine from a
urine specimen for routine urinalysis catheterized patient, Which of the
and C/S? following statement indicates an
accurate performance of the
A. Early morning procedure?
B. Later afternoon
C. Midnight A. Clamp above the port for 30 to
D. Before breakfast 60 minutes before drawing the
72. Which of the following is among urine from the port
an ideal way of collecting a urine B. Clamp below the port for 30
specimen for culture and sensitivity? to 60 minutes before drawing
the urine from the port
A. Use a clean container C. Clamp above the port for 5 to 10
B. Discard the first flow of urine minutes before drawing the urine
to ensure that the urine is from the port
not contaminated D. Clamp below the port for 5 to 10
C. Collect around 30-50 ml of urine minutes before drawing the urine
D. Add preservatives, refrigerate the from the port
specimen or add ice according to 77. A community health nurse
the agency’s protocol should be resourceful and meet the
73. In a 24 hour urine specimen needs of the client. A villager ask
started Friday, 9:00 A.M, which of him, Can you test my urine for
the following if done by a Nurse glucose? Which of the following
indicate a NEED for further technique allows the nurse to test a
procedural debriefing? client’s urine for glucose without the
need for intricate instruments.
A. The nurse ask the client to
urinate at 9:00 A.M, Friday A. Acetic Acid test
and she included the urine in B. Nitrazine paper test
the 24 hour urine specimen C. Benedict’s test
B. The nurse discards the Friday D. Litmus paper test
9:00 A M urine of the client 78. A community health nurse is
C. The nurse included the Saturday assessing client’s urine using the
9:00 A.M urine of the client to the Acetic Acid solution. Which of the
specimen collection following, if done by a nurse,
indicates lack of correct knowledge B. Continue iron preparation to
with the procedure? prevent further loss of Iron
C. Do not eat read meat 12 hours
A. The nurse added the Urine as the before procedure
2/3 part of the solution D. Encourage caffeine and dark
B. The nurse heats the test tube colored foods to produce
after adding 1/3 part acetic accurate results
acid 83. In collecting a routine specimen
C. The nurse heats the test tube for fecalysis, Which of the following,
after adding 2/3 part of Urine if done by a nurse, indicates
D. The nurse determines abnormal inadequate knowledge and skills
result if she noticed that the test about the procedure?
tube becomes cloudy
79. Which of the following is A. The nurse scoop the specimen
incorrect with regards to proper specifically at the site with blood
urine testing using Benedict’s and mucus
Solution? B. She took around 1 inch of
specimen or a teaspoonful
A. Heat around 5ml of Benedict’s C. Ask the client to call her for
solution together with the urine the specimen after the client
in a test tube wiped off his anus with a
B. Add 8 to 10 drops of urine tissue
C. Heat the Benedict’s solution D. Ask the client to defecate in a
without the urine to check if the bedpan, Secure a sterile
solution is contaminated container
D. If the color remains BLUE, the 84. In a routine sputum analysis,
result is POSITIVE Which of the following indicates
80. +++ Positive result after proper nursing action before sputum
Benedicts test is depicted by what collection?
color?
A. Secure a clean container
A. Blue B. Discard the container if the
B. Green outside becomes contaminated
C. Yellow with the sputum
D. Orange C. Rinse the client’s mouth with
81. Clinitest is used in testing the Listerine after collection
urine of a client for glucose. Which D. Tell the client that 4 tablespoon
of the following, If committed by a of sputum is needed for each
nurse indicates error? specimen for a routine sputum
analysis
A. Specimen is collected after 85. Who collects Blood specimen?
meals
B. The nurse puts 1 clinitest tablet A. The nurse
into a test tube B. Medical technologist
C. She added 5 drops of urine and C. Physician
10 drops of water D. Physical therapist
D. If the color becomes orange or 86. David, 68 year old male client is
red, It is considered postitive scheduled for Serum Lipid analysis.
82. Which of the following nursing Which of the following health
intervention is important for a client teaching is important to ensure
scheduled to have a Guaiac Test? accurate reading?

A. Avoid turnips, radish and A. Tell the patient to eat fatty meals
horseradish 3 days before 3 days prior to the procedure
procedure
B. NPO for 12 hours pre A. Dorsal recumbent
procedure B. Sitting
C. Ask the client to drink 1 glass of C. Standing
water 1 hour prior to the D. Supine
procedure 94. In palpating the client’s
D. Tell the client that the normal abdomen, Which of the following is
serum lipase level is 50 to 140 the best position for the client to
U/L assume?
87. The primary factor responsible
for body heat production is the A. Dorsal recumbent
B. Side lying
A. Metabolism C. Supine
B. Release of thyroxin D. Lithotomy
C. Muscle activity 95. Rectal examination is done with
D. Stress a client in what position?
88. The heat regulating center is
found in the A. Dorsal recumbent
B. Sims position
A. Medulla oblongata C. Supine
B. Thalamus D. Lithotomy
C. Hypothalamus 96. Which of the following is a
D. Pons correct nursing action when
89. A process of heat loss which collecting urine specimen from a
involves the transfer of heat from client with an Indwelling catheter?
one surface to another is
A. Collect urine specimen from the
A. Radiation drainage bag
B. Conduction B. Detach catheter from the
C. Convection connecting tube and draw the
D. Evaporation specimen from the port
90. Which of the following is a C. Use sterile syringe to
primary factor that affects the BP? aspirate urine specimen from
the drainage port
A. Obesity D. Insert the syringe straight to the
B. Age port to allow self sealing of the
C. Stress port
D. Gender 97. Which of the following is
91. The following are social data inappropriate in collecting mid
about the client except stream clean catch urine specimen
for urine analysis?
A. Patient’s lifestyle
B. Religious practices A. Collect early in the morning, First
C. Family home situation voided specimen
D. Usual health status B. Do perineal care before specimen
92. The best position for any collection
procedure that involves vaginal and C. Collect 5 to 10 ml for urine
cervical examination is D. Discard the first flow of the urine
98. When palpating the client’s neck
A. Dorsal recumbent for lymphadenopathy, where should
B. Side lying the nurse position himself?
C. Supine
D. Lithotomy A. At the client’s back
93. Measure the leg circumference B. At the client’s right side
of a client with bipedal edema is C. At the client’s left side
best done in what position? D. In front of a sitting client
99. Which of the following is the best Sphenoid
position for the client to assume if Maxillary
the back is to be examined by the Frontal
nurse? 4. Which paranasal sinus is found
over the eyebrow?
A. Standing
B. Sitting Ehtmoid
C. Side lying Sphenoid
D. Prone Maxillary
100. In assessing the client’s chest, Frontal
which position best show chest 5. Gene De Vonne
expansion as well as its movements? Katrouchuacheulujiki wants to
change her surname to something
A. Sitting shorter, The court denied her
B. Prone request which depresses her and
C. Sidelying find herself binge eating. She
D. Supine accidentally aspirate a large piece of
nut and it passes the carina.
Probabilty wise, Where will the nut
PNLE: FON go?

Right main stem bronchus


Practice Exam for Left main stem bronchus
Be dislodged in between the carina

Oxygenation and Be blocked by the closed epiglottis


6. Which cell secretes mucus that
help protect the lungs by trapping
Nutrition debris in the respiratory tract?

Type I pneumocytes
1. Which one of the following is NOT Type II pneumocytes
a function of the Upper airway? Goblet cells
Adipose cells
For clearance mechanism such 7. How many lobes are there in the
as coughing RIGHT LUNG?
Transport gases to the lower
airways One
Warming, Filtration and Two
Humidification of inspired air Three
Protect the lower airway from Four
foreign mater 8. The presence of the liver causes
2. It is the hair the lines the which anatomical difference of the
vestibule which function as a Kidneys and the Lungs?
filtering mechanism for foreign
objects Left kidney slightly lower, Left
lung slightly shorter
Cilia Left kidney slightly higher, Left lung
Nares slightly shorter
Carina Right kidney lower, Right lung
Vibrissae shorter
3. This is the paranasal sinus found Right kidney higher, Right lung
between the eyes and the nose that shorter
extends backward into the skull 9. Surfactant is produced by what
cells in the alveoli?
Ehtmoid
Type I pneumocytes God is good, Man requires 21% of
Type II pneumocytes oxygen and we have 21%
Goblet cells available in our air
Adipose cells Man requires 16% of oxygen and we
10. The normal L:S Ratio to consider have 35% available in our air
the newborn baby viable is Man requires 10% of oxygen and we
have 50% available in our air
1:2 Human requires 21% of oxygen
2:1 and we have 21% available in
3:1 our air
1:3 16. Which of the following is TRUE
11. Refers to the extra air that can about Expiration?
be inhaled beyond the normal tidal
volume A passive process
The length of which is half of the
Inspiratory reserve volume length of Inspiration
Expiratory reserve volume Stridor is commonly heard during
Functional residual capacity expiration
Residual volume Requires energy to be carried out
12. This is the amount of air 17. Which of the following is TRUE in
remained in the lungs after a postural drainage?
forceful expiration
Patient assumes position for 10
Inspiratory reserve volume to 15 minutes
Expiratory reserve volume Should last only for 60 minutes
Functional residual capacity Done best P.C
Residual volume An independent nursing action
13. Casssandra, A 22 year old grade
Agnostic, Asked you, how many 18. All but one of the following is a
spikes of bones are there in my ribs? purpose of steam inhalation
Your best response is which of the
following? Mucolytic
Warm and humidify air
We have 13 pairs of ribs Cassandra Administer medications
We have 12 pairs of ribs Promote bronchoconstriction
Cassandra
Humans have 16 pairs of ribs, and 19. Which of the following is NOT
that was noted by Vesalius in TRUE in steam inhalation?
1543
Humans have 8 pairs of ribs. 4 of It is a dependent nursing action
which are floating Spout is put 12-18 inches away
14. Which of the following is from the nose
considered as the main muscle of Render steam inhalation for
respiration? atleast 60 minutes
Cover the client’s eye with wash
Lungs cloth to prevent irritation
Intercostal Muscles 20. When should a nurse suction a
Diaphragm client?
Pectoralis major
15. Cassandra asked you : How As desired
many air is there in the oxygen and As needed
how many does human requires? Every 1 hour
Which of the following is the best Every 4 hours
response : 21. Ernest Arnold Hamilton, a 60
year old American client was
mobbed by teen gangsters near New Use KY Jelly if suctioning
york, Cubao. He was rushed to John nasopharyngeal secretion
John Hopio Medical Center and was The maximum time of suctioning
Unconscious. You are his nurse and should not exceed 15 seconds
you are to suction his secretions. In Allow 30 seconds interval between
which position should you place Mr. suctioning
Hamilton?
26. Which of the following is the
High fowlers initial sign of hypoxemia in an adult
Semi fowlers client?
Prone
Side lying Tachypnea
22. You are about to set the suction Tachycardia
pressure to be used to Mr. Hamilton. Cyanosis
You are using a Wall unit suction Pallor
machine. How much pressure should Irritability
you set the valve before suctioning Flaring of NaresA. 1,2
Mr. Hamilton? B. 2,5
C. 2,6
50-95 mmHg D. 3,4
200-350 mmHg 27. Which method of oxygenation
100-120 mmHg least likely produces anxiety and
10-15 mmHg apprehension?
23. The wall unit is not functioning;
You then try to use the portable Nasal Cannula
suction equipment available. How Simple Face mask
much pressure of suction equipment Non Rebreather mask
is needed to prevent trauma to Partial Rebreather mask
mucus membrane and air ways in
case of portable suction units? 28. Which of the following oxygen
delivery method can deliver 100%
2-5 mmHg Oxygen at 15 LPM?
5-10 mmHg
10-15 mmHg Nasal Cannula
15-25 mmHg Simple Face mask
24. There are four catheter sizes Non Rebreather mask
available for use, which one of these Partial Rebreather mask
should you use for Mr. Hamilton? 29. Which of the following is not true
about OXYGEN?
Fr. 18
Fr. 12 Oxygen is odorless, tasteless and
Fr. 10 colorless gas.
Fr, 5 Oxygen can irritate mucus
25. Which of the following, if done membrane
by the nurse, indicates Oxygen supports combustion
incompetence during suctioning an Excessive oxygen
unconscious client? administration results in
respiratory acidosis
Measure the length of the
suction catheter to be 30. Roberto San Andres, A new
inserted by measuring from nurse in the hospital is about to
the tip of the nose, to the administer oxygen on patient with
earlobe, to the xiphoid Respiratory distress. As his senior
process nurse, you should intervene if
Roberto will:
Uses venture mask in oxygen 36. There is a continuous bubbling in
administration the water sealed drainage system
Put a non rebreather mask in with suction. And oscillation is
the patient before opening observed. As a nurse, what should
the oxygen source you do?
Use a partial rebreather mask to
deliver oxygen Consider this as normal findings
Check for the doctor’s order for Notify the physician
Oxygen administration Check for tube leak
31. Which of the following will alert Prepare a petrolatum gauze
the nurse as an early sign of dressing
hypoxia? 37. Which of the following is true
about nutrition?
Client is tired and dyspneic
The client is coughing out blood It is the process in which food are
The client’s heart rate is 50 BPM broken down, for the body to use
Client is frequently turning in growth and development
from side to side It is a process in which digested
32. Miguelito de balboa, An OFW proteins, fats, minerals, vitamins
presents at the admission with an and carbohydrates are
A:P Diameter ratio of 2:1, Which of transported into the circulation
the following associated finding It is a chemical process that occurs
should the nurse expect? in the cell that allows for energy
production, energy use, growth
Pancytopenia and tissue repair
Anemia It is the study of nutrients and
Fingers are Club-like the process in which they are
Hematocrit of client is decreased use by the body
33. The best method of oxygen 38. The majority of the digestion
administration for client with COPD processes take place in the
uses:
Mouth
Cannula Small intestine
Simple Face mask Large intestine
Non rebreather mask Stomach
Venturi mask 39. All of the following is true about
34. Mang dagul, a 50 year old digestion that occurs in the Mouth
chronic smoker was brought to the except
E.R because of difficulty in
breathing. Pleural effusion was the It is where the digestion process
diagnosis and CTT was ordered. starts
What does C.T.T Stands for? Mechanical digestion is brought
about by mastication
Chest tube thoracotomy The action of ptyalin or the
Chest tube thoracostomy salivary tyrpsin breaks down
Closed tube thoracotomy starches into maltose
Closed tube thoracostmy Deglutition occurs after food is
35. Where will the CTT be inserted if broken down into small pieces
we are to drain fluids accumulated in and well mixed with saliva
Mang dagul’s pleura? 40. Which of the following foods
lowers the cardiac sphincter
2nd ICS pressure?
4th ICS
5th ICS Roast beef, Steamed cauliflower
8th ICS and Rice
Orange juice, Non fat milk, Dry Glucose + Fructose
crackers Glucose + Galactose
Decaffeinated coffee, Sky flakes Fructose + Fructose
crackers, Suman 47. This is the enzyme secreted by
Coffee with coffee mate, Bacon the pancrease that completes the
and Egg protein digestion
41. Where does the digestion of
carbohydrates start? Trypsin
Enterokinase
Mouth Enterogastrone
Esophagus Amylase
Small intestine 48. The end product of protein
Stomach digestion or the “Building blocks of
42. Protein and Fat digestion begins Protein” is what we call
where?
Nucleotides
Mouth Fatty acids
Esophagus Glucose
Small intestine Amino Acids
Stomach 49. Enzyme secreted by the small
43. All but one is true about intestine after it detects a bolus of
digestion that occurs in the Stomach fatty food. This will contract the
gallbladder to secrete bile and relax
Carbohydrates are the fastest to be the sphincter of Oddi to aid in the
digested, in about an hour emulsification of fats and its
Fat is the slowest to be digested, in digestion.
about 5 hours
HCl inhibits absorption of Lipase
Calcium in the gastric mucosa Amylase
HCl converts pepsinogen to pepsin, Cholecystokinin
which starts the complex process Pancreozymin
of protein digestion 50. Which of the following is not true
44. Which of the following is NOT an about the Large Intestine?
enzyme secreted by the small
intestine? It absorbs around 1 L of water
making the feces around 75%
Sucrase water and 25% solid
Enterokinase The stool formed in the transverse
Amylase colon is not yet well formed
Enterokinase It is a sterile body cavity
45. The hormone secreted by the It is called large intestine because it
Small intestine that stimulates the is longer than the small intestine
production of pancreatic juice which 51. This is the amount of heat
primarily aids in buffering the acidic required to raise the temperature of
bolus passed by the Stomach 1 kg water to 1 degree Celsius

Enterogastrone Calorie
Cholecystokinin Joules
Pancreozymin Metabolism
Enterokinase Basal metabolic rate
46. When the duodenal enzyme 52. Assuming a cup of rice provides
sucrase acts on SUCROSE, which 2 50 grams of carbohydrates. How
monosaccharides are formed? many calories are there in that cup
of rice?
Galactose + Galactose
150 calories Vitamin B1
200 calories Vitamin B2
250 calories Vitamin B3
400 calories Vitamin B6
53. An average adult filipino requires 59. The inflammation of the Lips,
how many calories in a day? Palate and Tongue is associated in
the deficiency of this vitamin
1,000 calories
1,500 calories Vitamin B1
2,000 calories Vitamin B2
2,500 calories Vitamin B3
54. Which of the following is true Vitamin B6
about an individual’s caloric needs? 60. Beri beri is caused by the
deficiency of which Vitamin?
All individual have the same caloric
needs Vitamin B1
Females in general have higher Vitamin B2
BMR and therefore, require more Vitamin B3
calories Vitamin C
During cold weather, people 61. Which of the following is the best
need more calories due to source of Vitamin E?
increase BMR
Dinner should be the heaviest meal Green leafy vegetables
of the day Vegetable oil
55. Among the following people, who Fortified Milk
requires the greatest caloric intake? Fish liver oil

An individual in a long state of 62. Among the following foods,


gluconeogenesis which food should you emphasize
An individual in a long state of giving on an Alcoholic client?
glycogenolysis
A pregnant individual Pork liver and organ meats,
An adolescent with a BMI of 25 Pork
56. Which nutrient deficiency is Red meat, Eggs and Dairy
associated with the development of products
Pellagra, Dermatitis and Diarrhea? Green leafy vegetables, Yellow
vegetables, Cantaloupe and
Vitamin B1 Dairy products
Vitamin B2 Chicken, Peanuts, Bananas, Wheat
Vitamin B3 germs and yeasts
Vitamin B6 63. Which food group should you
57. Which Vitamin is not given in emphasize giving on a pregnant
conjunction with the intake of mother in first trimester to prevent
LEVODOPA in cases of Parkinson’s neural tube defects?
Disease due to the fact that
levodopa increases its level in the Broccoli, Guava, Citrus fruits,
body? Tomatoes
Butter, Sardines, Tuna, Salmon, Egg
Vitamin B1 yolk
Vitamin B2 Wheat germ, Vegetable Oil,
Vitamin B3 soybeans, corn, peanuts
Vitamin B6 Organ meats, Green leafy
58. A vitamin taken in conjunction vegetables, Liver, Eggs
with ISONIAZID to prevent peripheral 64. A client taking Coumadin is to be
neuritis educated on his diet. As a nurse,
which of the following food should Raisin
you instruct the client to avoid? Banana
70. A client has HEMOSIDEROSIS.
Spinach, Green leafy Which of the following drug would
vegetables, Cabbage, Liver you expect to be given to the client?
Salmon, Sardines, Tuna
Butter, Egg yolk, breakfast cereals Acetazolamide
Banana, Yeast, Wheat germ, Deferoxamine
Chicken Calcium EDTA
Activated charcoal
65. Vitamin E plus this mineral works 71. Which of the following provides
as one of the best anti oxidant in the the richest source of Iron per area of
body according to the latest their meat?
research. They are combined with 5
Alpha reductase inhibitor to reduce Pork meat
the risk of acquiring prostate cancer Lean read meat
Pork liver
Zinc Green mongo
Iron
Selenium 72. Which of the following is
Vanadium considered the best indicator of
66. Incident of prostate cancer is nutritional status of an individual?
found to have been reduced on a
population exposed in tolerable Height
amount of sunlight. Which vitamin is Weight
associated with this phenomenon? Arm muscle circumference
BMI
Vitamin A 73. Jose Miguel, a 50 year old
Vitamin B business man is 6’0 Tall and weights
Vitamin C 179 lbs. As a nurse, you know that
Vitamin D Jose Miguel is :
67. Micronutrients are those
nutrients needed by the body in a Overweight
very minute amount. Which of the Underweight
following vitamin is considered as a Normal
MICRONUTRIENT Obese
74. Jose Miguel is a little bit
Phosphorous nauseous. Among the following
Iron beverages, Which could help relieve
Calcium JM’s nausea?
Sodium
68. Deficiency of this mineral results Coke
in tetany, osteomalacia, Sprite
osteoporosis and rickets. Mirinda
Orange Juice or Lemon Juice
Vitamin D 75. Which of the following is the first
Iron sign of dehydration?
Calcium
Sodium Tachycardia
69. Among the following foods, Restlessness
which has the highest amount of Thirst
potassium per area of their meat? Poor skin turgor
76. What Specific gravity lab result
Cantaloupe is compatible with a dehydrated
Avocado client?
1.007 Measure the amount of the
1.020 tube to be inserted from the
1.039 Tip of the nose, to the
1.029 earlobe, to the xiphoid
77. Which hematocrit value is process
expected in a dehydrated male Soak the NGT in a basin of ice water
client? to facilitate easy insertion
Check the placement of the tube by
67% introducing 10 cc of sterile water
50% and auscultating for bubbling
36% sound
45% 84. Which of the following is the
78. Which of the following statement BEST method in assessing for the
by a client with prolonged vomiting correct placement of the NGT?
indicates the initial onset of
hypokalemia? X-Ray
Immerse tip of the tube in water to
My arm feels so weak check for bubbles produced
I felt my heart beat just right now Aspirating gastric content to check
My face muscle is twitching if the content is acidic
Nurse, help! My legs are Instilling air in the NGT and listening
cramping for a gurgling sound at the
79. Which of the following is not an epigastric area
anti-emetic?
85. A terminally ill cancer patient is
Marinol scheduled for an NGT feeding today.
Dramamine How should you position the patient?
Benadryl
Alevaire Semi fowlers in bed
Bring the client into a chair
80. Which is not a clear liquid diet? Slightly elevated right side lying
position
Hard candy Supine in bed
Gelatin 86. A client is scheduled for NGT
Coffee with Coffee mate Feeding. Checking the residual
Bouillon volume, you determined that he has
81. Which of the following is 40 cc residual from the last feeding.
included in a full liquid diet? You reinstill the 40 cc of residual
volume and added the 250 cc of
Popsicles feeding ordered by the doctor. You
Pureed vegetable meat then instill 60 cc of water to clear
Pineapple juice with pulps the lumen and the tube. How much
Mashed potato will you put in the client’s chart as
82. Which food is included in a input?
BLAND DIET?
250 cc
Steamed broccoli 290 cc
Creamed potato 350 cc
Spinach in garlic 310 cc
Sweet potato 87. Which of the following if done by
83. Which of the following if done by a nurse indicates deviation from the
the nurse, is correct during NGT standards of NGT feeding?
Insertion?
Do not give the feeding and notify
Use an oil based lubricant the doctor of residual of the last
feeding is greater than or equal 3 day diet recall
to 50 ml Eating style and habits
Height of the feeding should be 12
inches about the tube point of 92. The vomiting center is found in
insertion to allow slow the
introduction of feeding
Ask the client to position in Medulla Oblongata
supine position immediately Pons
after feeding to prevent Hypothalamus
dumping syndrome Cerebellum
Clamp the NGT before all of the 93. The most threatening
water is instilled to prevent air complication of vomiting in client’s
entry in the stomach with stroke is
88. What is the most common
problem in TUBE FEEDING? Aspiration
Dehydration
Diarrhea Fluid and electrolyte imbalance
Infection Malnutrition
Hyperglycemia 94. Which among this food is the
Vomiting richest source of Iron?
89. Which of the following is TRUE in
colostomy feeding? Ampalaya
Broccoli
Hold the syringe 18 inches above Mongo
the stoma and administer the Malunggay leaves
feeding slowly 95. Which of the following is a good
Pour 30 ml of water before and after source of Vitamin A?
feeding administration
Insert the ostomy feeding tube 1 Egg yolk
inch towards the stoma Liver
A Pink stoma means that circulation Fish
towards the stoma is all well Peanuts
90. A client with TPN suddenly 96. The most important nursing
develops tremors, dizziness, action before gastrostomy feeding is
weakness and diaphoresis. The
client said “I feel weak” You saw that Check V/S
his TPN is already empty and Assess for patency of the tube
another TPN is scheduled to replace Measure residual feeding
the previous one but its provision is Check the placement of the tube
already 3 hours late. Which of the 97. The primary advantage of
following is the probable gastrostomy feeding is
complication being experienced by
the client? Ensures adequate nutrition
It prevents aspiration
Hyperglycemia Maintains Gastro esophageal
Hypoglycemia sphincter integrity
Infection Minimizes fluid-electrolyte
Fluid overload imbalance
91. To assess the adequacy of food 98. What is the BMI Of Budek,
intake, which of the following weighing 120 lbs and has a height of
assessment parameters is best 5 feet 7 inches.
used?
20
Food likes and dislikes 19
Regularity of meal times 15
25 musical sound. The nurse
99. Which finding is consistent with documents this as:
PERNICIOUS ANEMIA?
Wheezes
Strawberry tongue Rhonchi
Currant Jelly stool Gurgles
Beefy red tongue Vesicular
Pale [ HYPOCHROMIC ] RBC
100. The nurse is browsing the chart (A) Wheezes. Wheezes are
of the patient and notes a normal indicated by continuous, lengthy,
serum lipase level. Which of the musical; heard during inspiration
following is a normal serum lipase or expiration. Rhonchi are usually
value? coarse breath sounds. Gurgles
are loud gurgling, bubbling
10 U/L sound. Vesicular breath sounds
100 U/L are low pitch, soft intensity on
200 U/L expiration
350 U/L
3. The nurse in charge measures a
patient’s temperature at 101
PNLE degrees F. What is the equivalent
centigrade temperature?

Fundamentals in 36.3 degrees C


37.95 degrees C
Nursing Exam 1 40.03 degrees C
38.01 degrees C
1. Jake is complaining of shortness of
breath. The nurse assesses his (B) 37.95 degrees C. To convert °F
respiratory rate to be 30 breaths per to °C use this formula, ( °F – 32 )
minute and documents that Jake is (0.55). While when converting °C
tachypneic. The nurse understands to °F use this formula, ( °C x 1.8)
that tachypnea means: + 32. Note that 0.55 is 5/9 and
1.8 is 9/5.
Pulse rate greater than 100 beats
per minute 4. Which approach to problem
Blood pressure of 140/90 solving tests any number of
Respiratory rate greater than solutions until one is found that
20 breaths per minute works for that particular problem?
Frequent bowel sounds
Intuition
(C) Respiratory rate greater than Routine
20 breaths per minute. A Scientific method
respiratory rate of greater than Trial and error
20 breaths per minute is
tachypnea. A blood pressure of (D) Trial and error. The trial and
140/90 is considered error method of problem solving
hypertension. Pulse greater isn’t systematic (as in the
than 100 beats per minute is scientific method of problem
tachycardia. Frequent bowel solving) routine, or based on
sounds refer to hyper-active inner prompting (as in the
bowel sounds. intuitive method of problem
solving).
2. The nurse listens to Mrs. Sullen’s
lungs and notes a hissing sound or
5. What is the order of the nursing “The patient will feel less nauseated
process? in 24 hours.”
“The patient will eat the right
Assessing, diagnosing, amount of food daily.”
implementing, evaluating, “The patient will identify all the
planning high-salt food from a
Diagnosing, assessing, planning, prepared list by discharge.”
implementing, evaluating
Assessing, diagnosing, (C) “The patient will identify all the
planning, implementing, high-salt food from a prepared
evaluating list by discharge.”. Expected
Planning, evaluating, diagnosing, outcomes are specific,
assessing, implementing measurable, realistic statements
of goal attainment. The phrases
(C) Assessing, diagnosing, planning, “right amount”, “less nauseated”
implementing, evaluating. The and “enough sleep” are vague
correct order of the nursing and not measurable.
process is assessing, diagnosing,
planning, implementing, “The patient will have enough
evaluating. sleep.”
9. Which of the following behaviors
6. During the planning phase of the by Nurse Jane Robles demonstrates
nursing process, which of the that she understands well th
following is the outcome? elements of effecting charting?

Nursing history She writes in the chart using a no. 2


Nursing notes pencil.
Nursing care plan She noted: appetite is good this
Nursing diagnosis afternoon.
She signs on the medication
(C) Nursing care plan. The outcome, sheet after administering the
or the product of the planning medication.
phase of the nursing process is a She signs her charting as follow: J.R
Nursing care plan.
(C) She signs on the medication
7. What is an example of a sheet after administering the
subjective data? medication.A nurse should record
a nursing intervention (ex. Giving
Heart rate of 68 beats per minute medications) after performing
Yellowish sputum the nursing intervention (not
Client verbalized, “I feel pain before). Recording should also be
when urinating.” done using a pen, be complete,
Noisy breathing and signed with the nurse’s full
name and title.
(C) Client verbalized, “I feel pain
when urinating.”. Subjective data 10. What is the disadvantage of
are those that can be described computerized documentation of the
only by the person experiencing nursing process?
it. Therefore, only the patient can
describe or verify whether he is Accuracy
experiencing pain or not. Legibility
Concern for privacy
8. Which expected outcome is Rapid communication
correctly written?
(C) Concern for privacy. A patient’s since 10 years ago. Since then, she
privacy may be violated if had maintained low sodium, low fat
security measures aren’t used diet, to control her blood pressure.
properly or if policies and This practice is viewed as:
procedures aren’t in place that
determines what type of Cultural belief
information can be retrieved, by Personal belief
whom, and for what purpose. Health belief
Superstitious belief
11. The theorist who believes that
adaptation and manipulation of (C) Health belief. Health belief of an
stressors are related to foster individual influences his/her
change is: preventive health behavior.

Dorothea Orem 14. Becky is on NPO since midnight


Sister Callista Roy as preparation for blood test.
Imogene King Adreno-cortical response is
Virginia Henderson activated. Which of the following is
an expected response?
(B) Sister Callista Roy. Sister Roy’s
theory is called the adaptation Low blood pressure
theory and she viewed each Warm, dry skin
person as a unified biophysical Decreased serum sodium levels
system in constant interaction Decreased urine output
with a changing environment.
Orem’s theory is called self-care (D) Decreased urine output. Adreno-
deficit theory and is based on the cortical response involves release
belief that individual has a need of aldosterone that leads to
for self-care actions. King’s retention of sodium and water.
theory is the Goal attainment This results to decreased urine
theory and described nursing as output.
a helping profession that assists
individuals and groups in society 15. What nursing action is
to attain, maintain, and restore appropriate when obtaining a sterile
health. Henderson introduced the urine specimen from an indwelling
nature of nursing model and catheter to prevent infection?
identified the 14 basic needs.
Use sterile gloves when obtaining
12. Formulating a nursing diagnosis urine.
is a joint function of: Open the drainage bag and pour
out the urine.
Patient and relatives Disconnect the catheter from the
Nurse and patient tubing and get urine.
Doctor and family Aspirate urine from the tubing
Nurse and doctor port using a sterile syringe.

(B) Nurse and patient. Although (D) Aspirate urine from the tubing
diagnosing is basically the port using a sterile syringe. The
nurse’s responsibility, input from nurse should aspirate the urine
the patient is essential to from the port using a sterile
formulate the correct nursing syringe to obtain a urine
diagnosis. specimen. Opening a closed
drainage system increase the risk
13. Mrs. Caperlac has been of urinary tract infection.
diagnosed to have hypertension
16. A client is receiving 115 ml/hr of Assist the patient in fowler’s
continuous IVF. The nurse notices position.
that the venipuncture site is red and
swollen. Which of the following (A) Place the feeding 20 inches
interventions would the nurse above the pint if insertion of
perform first? NGT. The height of the feeding is
above 12 inches above the point
Stop the infusion of insertion, bot 20 inches. If the
Call the attending physician height of feeding is too high, this
Slow that infusion to 20 ml/hr results to very rapid introduction
Place a clod towel on the site of feeding. This may trigger
nausea and vomiting.
(A) Stop the infusion. The sign and
symptoms indicate extravasation 19. A female patient is being
so the IVF should be stopped discharged after thyroidectomy.
immediately and put warm not After providing the medication
cold towel on the affected site. teaching. The nurse asks the patient
to repeat the instructions. The nurse
17. The nurse enters the room to is performing which professional
give a prescribed medication but the role?
patient is inside the bathroom. What
should the nurse do? Manager
Caregiver
Leave the medication at the Patient advocate
bedside and leave the room. Educator
After few minutes, return to
that patient’s room and do (D) Educator. When teaching a
not leave until the patient patient about medications before
takes the medication. discharge, the nurse is acting as
Instruct the patient to take the an educator. A caregiver provides
medication and leave it at the direct care to the patient. The
bedside. nurse acts as s patient advocate
Wait for the patient to return to bed when making the patient’s
and just leave the medication at wishes known to the doctor.
the bedside.
20. Which data would be of greatest
(B) After few minutes, return to that concern to the nurse when
patient’s room and do not leave completing the nursing assessment
until the patient takes the of a 68-year-old woman hospitalized
medication. This is to verify or to due to Pneumonia?
make sure that the medication
was taken by the patient as Oriented to date, time and place
directed. Clear breath sounds
Capillary refill greater than 3
18. Which of the following is seconds and buccal cyanosis
inappropriate nursing action when Hemoglobin of 13 g/dl
administering NGT feeding?
(C) Capillary refill greater than 3
Place the feeding 20 inches seconds and buccal
above the pint if insertion of cyanosis. Capillary refill greater
NGT. than 3 seconds and buccal
Introduce the feeding slowly. cyanosis indicate decreased
Instill 60ml of water into the NGT oxygen to the tissues which
after feeding. requires immediate
attention/intervention. Oriented
to date, time and place,
hemoglobin of 13 g/dl are normal 23. Which of the following is the
data. most important purpose of planning
care with this patient?
21. During a change-of-shift report,
it would be important for the nurse Development of a standardized NCP.
relinquishing responsibility for care Expansion of the current taxonomy
of the patient to communicate. of nursing diagnosis
Which of the following facts to the Making of individualized patient
nurse assuming responsibility for care
care of the patient? Incorporation of both nursing and
medical diagnoses in patient care
That the patient verbalized, “My
headache is gone.” (C) Making of individualized patient
That the patient’s barium enema care. To be effective, the nursing
performed 3 days ago was care plan developed in the
negative planning phase of the nursing
Patient’s NGT was removed 2 process must reflect the
hours ago individualized needs of the
Patient’s family came for a visit this patient.
morning.
24. Using Maslow’s hierarchy of
(C) Patient’s NGT was removed 2 basic human needs, which of the
hours ago. The change-of-shift following nursing diagnoses has the
report should indicate significant highest priority?
recent changes in the patient’s
condition that the nurse Ineffective breathing pattern
assuming responsibility for care related to pain, as evidenced
of the patient will need to by shortness of breath.
monitor. The other options are Anxiety related to impending
not critical enough to include in surgery, as evidenced by
the report. insomnia.
Risk of injury related to
22. Which statement is the most autoimmune dysfunction
appropriate goal for a nursing Impaired verbal communication
diagnosis of diarrhea? related to tracheostomy, as
evidenced by inability to speak.
“The patient will experience
decreased frequency of (A) Ineffective breathing pattern
bowel elimination.” related to pain, as evidenced by
“The patient will take anti-diarrheal shortness of breath.. Physiologic
medication.” needs (ex. Oxygen, fluids,
“The patient will give a stool nutrition) must be met before
specimen for laboratory lower needs (such as safety and
examinations.” security, love and belongingness,
“The patient will save urine for self-esteem and self-
inspection by the nurse. actualization) can be met.
Therefore, physiologic needs
(A) “The patient will experience have the highest priority.
decreased frequency of bowel
elimination.” The goal is the 25. When performing an abdominal
opposite, healthy response of the examination, the patient should be
problem statement of the nursing in a supine position with the head of
diagnosis. In this situation, the the bed at what position?
problem statement is diarrhea.
30 degrees (B) sardines. The normal serum
90 degrees sodium level is 135 to 145
45 degrees mEq/L, the client is having
0 degree hypernatremia. Pia should avoid
food high in sodium like
(D) 0 degree. The patient should be processed food. Broccoli,
positioned with the head of the bed cabbage and tomatoes are good
completely flattened to perform an source of Vitamin C.
abdominal examination. If the head
of the bed is elevated, the 3. Jason, 3 years old vomited. His
abdominal muscles and organs can mom stated, “He vomited 6 ounces
of his formula this morning.” This
be bunched up, altering the findings
statement is an example of:

PNLE objective data from a secondary


source
objective data from a primary
Fundamentals in source
subjective data from a primary
Nursing Exam 2 source
subjective data from a secondary
source
1. A patient is wearing a soft wrist-
safety device. Which of the following (A) objective data from a secondary
nursing assessment is considered source. Jason is the primary
abnormal? source; his mother is a secondary
source. The data is objective
Palpable radial pulse because it can be perceived by
Palpable ulnar pulse the senses, verified by another
Capillary refill within 3 seconds person observing the same
Bluish fingernails, cool and pale patient, and tested against
fingers accepted standards or norms.

(D) Bluish fingernails, cool and pale 4. Which of the following is a nursing
fingers. A safety device on the diagnosis?
wrist may impair blood
circulation. Therefore, the nurse Hypethermia
should assess the patient for Diabetes Mellitus
signs of impaired circulation such Angina
as bluish fingernails, cool and Chronic Renal Failure
pale fingers. Palpable radial and
ulnar pulses, capillary refill within A) Hypethermia. Hyperthermia is a
3 seconds are all normal findings. NANDA-approved nursing
diagnosis. Diabetes Mellitus,
2. Pia’s serum sodium level is 150 Angina and Chronic Renal Failure
mEq/L. Which of the following food are medical diagnoses.
items does the nurse instruct Pia to
avoid? 5. What is the characteristic of the
nursing process?
broccoli
sardines stagnant
cabbage inflexible
Tomatoes asystematic
goal-oriented
(D) goal-oriented. The nursing Discourage the client in expressing
process is goal-oriented. It is also her emotions.
systematic, patient-centered, and
dynamic. (B) Provide opportunity to the client
to tell their story. Providing a
6. A skin lesion which is fluid-filled, grieving person an opportunity to
less than 1 cm in size is called: tell their story allows the person
to express feelings. This is
papule therapeutic in assisting the client
vesicle resolve grief.
bulla
Macule 9. It is the gradual decrease of the
body’s temperature after death.
(B) vesicle. Vesicle is a
circumscribed circulation livor mortis
containing serous fluid or blood rigor mortis
and less than 1 cm (ex. Blister, algor mortis
chicken pox). none of the above

7. During application of medication (C) algor mortis. Algor mortis is the


into the ear, which of the following is decrease of the body’s
inappropriate nursing action? temperature after death. Livor
mortis is the discoloration of the
In an adult, pull the pinna upward. skin after death. Rigor mortis is
Instill the medication directly the stiffening of the body that
into the tympanic membrane. occurs about 2-4 hours after
Warm the medication at room or death.
body temperature.
Press the tragus of the ear a few 10. When performing an admission
times to assist flow of medication assessment on a newly admitted
into the ear canal. patient, the nurse percusses
resonance. The nurse knows that
(B) Instill the medication directly resonance heard on percussion is
into the tympanic most commonly heard over which
membrane. During the organ?
application of medication it is
inappropriate to instill the thigh
medication directly into the liver
tympanic membrane. The right intestine
thing to do is instill the Lung
medication along the lateral wall
of the auditory canal. (D) lung. Resonance is loud, low-
pitched and long duration that’s
8. Which of the following is heard most commonly over an
appropriate nursing intervention for air-filled tissue such as a normal
a client who is grieving over the lung.
death of her child?
11. The nurse is aware that Bell’s
Tell her not to cry and it will be palsy affects which cranial nerve?
better.
Provide opportunity to the 2nd CN (Optic)
client to tell their story. 3rd CN (Occulomotor)
Encourage her to accept or to 4th CN (Trochlear)
replace the lost person. 7th CN (Facial)
(D) 7th CN (Facial). Bells’ palsy is nursing intervention is the nurse
the paralysis of the motor performing?
component of the 7th caranial
nerve, resulting in facial sag, Independent
inability to close the eyelid or the Dependent
mouth, drooling, flat nasolabial Collaborative
fold and loss of taste on the Professional
affected side of the face.
(A) Independent. Independent
12. Prolonged deficiency of Vitamin nursing interventions involve
B9 leads to: actions that nurses initiate based
on their own knowledge and
scurvy skills without the direction or
pellagra supervision of another member
megaloblastic anemia of the health care team.
pernicious anemia
15. Claire is admitted with a
(C) megaloblastic diagnosis of chronic shoulder pain.
anemia. Prolonged Vitamin B9 By definition, the nurse understands
deficiency will lead to that the patient has had pain for
megaloblastic anemia while more than:
pernicious anemia results in
deficiency in Vitamin B12. 3 months
Prolonged deficiency of Vitamin C 6 months
leads to scurvy and Pellagra 9 months
results in deficiency in Vitamin 1 year
B3.
(B) 6 months. Chronic pain s usually
13. Nurse Cherry is teaching a 72 defined as pain lasting longer
year old patient about a newly than 6 months.
prescribed medication. What could
cause a geriatric patient to have 16. Which of the following
difficulty retaining knowledge about statements regarding the nursing
the newly prescribed medication? process is true?

Absence of family support It is useful on outpatient settings.


Decreased sensory functions It progresses in separate, unrelated
Patient has no interest on learning steps.
Decreased plasma drug levels It focuses on the patient, not
the nurse.
(B) Decreased sensory It provides the solution to all patient
functions. Decreased in sensory health problems.
functions could cause a geriatric
patient to have difficulty (C) It focuses on the patient, not the
retaining knowledge about the nurse. The nursing process is
newly prescribed medications. patient-centered, not nurse-
Absence of family support and no centered. It can be use in any
interest on learning may affect setting, and the steps are
compliance, not knowledge related. The nursing process
retention. Decreased plasma can’t solve all patient health
levels do not alter patient’s problems.
knowledge about the drug.
17. Which of the following is
14. When assessing a patient’s level considered significant enough to
of consciousness, which type of require immediate communication to
another member of the health care
team? 20. When providing a continuous
enteral feeding, which of the
Weight loss of 3 lbs in a 120 lb following action is essential for the
female patient. nurse to do?
Diminished breath sounds in
patient with previously Place the client on the left side of
normal breath sounds the bed.
Patient stated, “I feel less Attach the feeding bag to the
nauseated.” current tubing.
Change of heart rate from 70 to 83 Elevate the head of the bed.
beats per minute. Cold the formula before
administering it
(B) Diminished breath sounds in
patient with previously normal (C) Elevate the head of the
breath sounds. Diminished bed. Elevating the head of the
breath sound is a life threatening bed during an enteral feeding
problem therefore it is highly prevents aspiration. The patient
priority because they pose the may be placed on the right side
greatest threat to the patient’s to prevent aspiration. Enteral
well-being. feedings are given at room
temperature to lessen GI
18. To assess the adequacy of food distress. The enteral tubing
intake, which of the following should be changed every 24
assessment parameters is best hours to limit microbial growth.
used? .
21. Kussmaul’s breathing is;
food preferences
regularity of meal times Shallow breaths interrupted by
3-day diet recall apnea.
eating style and habits Prolonged gasping inspiration
followed by a very short, usually
(C) 3-day diet recall. 3-day diet inefficient expiration.
recall is an example of dietary Marked rhythmic waxing and
history. This is used to indicate waning of respirations from very
the adequacy of food intake of deep to very shallow breathing
the client. and temporary apnea.
Increased rate and depth of
19. Van Fajardo is a 55 year old who respiration.
was admitted to the hospital with
newly diagnosed hepatitis. The (D) Increased rate and depth of
nurse is doing a patient teaching respiration. Kussmaul breathing
with Mr. Fajardo. What kind of role is also called as hyperventilation.
does the nurse assume? Seen in metabolic acidosis and
renal failure. Option A refers to
talker Biot’s breathing. Option B is
teacher apneustic breathing and option C
thinker is the Cheyne-stokes breathing.
Doer
22. Presty has terminal cancer and
(B) teacher. The nurse will assume she refuses to believe that loss is
the role of a teacher in this happening ans she assumes artificial
therapeutic relationship. The cheerfulness. What stage of grieving
other roles are inappropriate in is she in?
this situation.
depression (C) Thyroid. The thyroid is part of
bargaining the endocrine system. Heart,
denial sinus and thymus are not.
Acceptance .

(C) denial. The client is in denial


stage because she is unready to
PNLE
face the reality that loss is
happening and she assumes Fundamentals in
artificial cheerfulness.

23. Immunization for healthy babies


Nursing Exam 3
and preschool children is an 1. Nurse Brenda is teaching a
example of what level of preventive patient about a newly prescribed
health care? drug. What could cause a geriatric
patient to have difficulty retaining
Primary knowledge about prescribed
Secondary medications?
Tertiary
Curative Decreased plasma drug levels
Sensory deficits
(A) Primary. The primary level Lack of family support
focuses on health promotion. History of Tourette syndrome
Secondary level focuses on
health maintenance. Tertiary (B) Sensory deficits. Sensory
focuses on rehabilitation. There is deficits could cause a geriatric
n Curative level of preventive patient to have difficulty
health care problems. retaining knowledge about
prescribed medications.
24. Which is an example of a Decreased plasma drug levels do
subjective data? not alter the patient’s knowledge
about the drug. A lack of family
Temperature of 38 0C support may affect compliance,
Vomiting for 3 days not knowledge retention. Toilette
Productive cough syndrome is unrelated to
Patient stated, “My arms still knowledge retention.
hurt.”
2. When examining a patient with
(D) Patient stated, “My arms still abdominal pain the nurse in charge
hurt.”. Subjective data are should assess:
apparent only to the person
affected and can or verified only Any quadrant first
by that person. The symptomatic quadrant first
The symptomatic quadrant last
25. The nurse is assessing the The symptomatic quadrant either
endocrine system. Which organ is second or third
part of the endocrine system?
(C) The symptomatic quadrant
Heart last. The nurse should
Sinus systematically assess all areas of
Thyroid the abdomen, if time and the
Thymus patient’s condition permit,
concluding with the symptomatic
area. Otherwise, the nurse may
elicit pain in the symptomatic
area, causing the muscles in Transverse plane
other areas to tighten. This would
interfere with further (A) Frontal plane. Frontal or coronal
assessment. plane runs longitudinally at a
right angle to a sagittal plane
3. The nurse is assessing a dividing the body in anterior and
postoperative adult patient. Which posterior regions. A sagittal plane
of the following should the nurse runs longitudinally dividing the
document as subjective data? body into right and left regions; if
exactly midline, it is called a
Vital signs midsagittal plane. A transverse
Laboratory test result plane runs horizontally at a right
Patient’s description of pain angle to the vertical axis,
Electrocardiographic (ECG) dividing the structure into
waveforms superior and inferior regions.

(C) Patient’s description of 6. A female patient with a terminal


pain. Subjective data come illness is in denial. Indicators of
directly from the patient and denial include:
usually are recorded as direct
quotations that reflect the Shock dismay
patient’s opinions or feelings Numbness
about a situation. Vital signs, Stoicism
laboratory test result, and ECG Preparatory grief
waveforms are examples of
objective data. (A) Shock dismay. Shock and
dismay are early signs of denial-
4. A male patient has a soft wrist- the first stage of grief. The other
safety device. Which assessment options are associated with
finding should the nurse consider depression—a later stage of grief.
abnormal?
7. The nurse in charge is transferring
A palpable radial pulse a patient from the bed to a chair.
A palpable ulnar pulse Which action does the nurse take
Cool, pale fingers during this patient transfer?
Pink nail beds
Position the head of the bed flat
(C) Cool, pale fingers. A safety Helps the patient dangle the
device on the wrist may impair legs
circulation and restrict blood Stands behind the patient
supply to body tissues. Places the chair facing away from
Therefore, the nurse should the bed
assess the patient for signs of
impaired circulation, such as (B) Helps the patient dangle the
cool, pale fingers. A palpable legs. After placing the patient in
radial or lunar pulse and pink nail high Fowler’s position and
beds are normal findings. moving the patient to the side of
the bed, the nurse helps the
5. Which of the following planes patient sit on the edge of the bed
divides the body longitudinally into and dangle the legs; the nurse
anterior and posterior regions? then faces the patient and places
the chair next to and facing the
Frontal plane head of the bed.
Sagittal plane
Midsagittal plane
8. A female patient who speaks a options are considered unsafe
little English has emergency because they promote error.
gallbladder surgery, during
discharge preparation, which nursing 10. When administering drug
action would best help this patient therapy to a male geriatric patient,
understand wound care instruction? the nurse must stay especially alert
for adverse effects. Which factor
Asking frequently if the patient makes geriatric patients to adverse
understands the instruction drug effects?
Asking an interpreter to replay the
instructions to the patient. Faster drug clearance
Writing out the instructions and Aging-related physiological
having a family member read changes
them to the patient Increased amount of neurons
Demonstrating the procedure Enhanced blood flow to the GI tract
and having the patient return
the demonstration (B) Aging-related physiological
changes. Aging-related
(D) Demonstrating the procedure physiological changes account
and having the patient return the for the increased frequency of
demonstration. Demonstrating adverse drug reactions in
by the nurse with a return geriatric patients. Renal and
demonstration by the patient hepatic changes cause drugs to
ensures that the patient can clear more slowly in these
perform wound care correctly. patients. With increasing age,
Patients may claim to understand neurons are lost and blood flow
discharge instruction when they to the GI tract decreases.
do not. An interpreter of family
member may communicate 11. A female patient is being
verbal or written instructions discharged after cataract surgery.
inaccurately. After providing medication teaching,
the nurse asks the patient to repeat
9. Before administering the evening the instructions. The nurse is
dose of a prescribed medication, the performing which professional role?
nurse on the evening shift finds an
unlabeled, filled syringe in the Manager
patient’s medication drawer. What Educator
should the nurse in charge do? Caregiver
Patient advocate
Discard the syringe to avoid a
medication error (B) Educator. When teaching a
Obtain a label for the syringe from patient about medications before
the pharmacy discharge, the nurse is acting as
Use the syringe because it looks like an educator. The nurse acts as a
it contains the same medication manager when performing such
the nurse was prepared to give activities as scheduling and
Call the day nurse to verify the making patient care
contents of the syringe assignments. The nurse performs
the care giving role when
(A) Discard the syringe to avoid a providing direct care, including
medication error. As a safety bathing patients and
precaution, the nurse should administering medications and
discard an unlabeled syringe that prescribed treatments. The nurse
contains medication. The other acts as a patient advocate when
making the patient’s wishes and provides the surgeon and
known to the doctor. scrub nurse with supplies.

12. A female patient exhibits signs of 14. A patient is in the bathroom


heightened anxiety. Which response when the nurse enters to give a
by the nurse is most likely to reduce prescribed medication. What should
the patient’s anxiety? the nurse in charge do?

“Everything will be fine. Don’t Leave the medication at the


worry.” patient’s bedside
“Read this manual and then ask me Tell the patient to be sure to take
any questions you may have.” the medication. And then leave it
“Why don’t you listen to the radio?” at the bedside
“Let’s talk about what’s Return shortly to the patient’s
bothering you.” room and remain there until
the patient takes the
(D) “Let’s talk about what’s medication
bothering you.” Anxiety may Wait for the patient to return to bed,
result from feeling of and then leave the medication at
helplessness, isolation, or the bedside
insecurity. This response helps
reduce anxiety by encouraging (C) Return shortly to the patient’s
the patient to express feelings. room and remain there until the
The nurse should be supportive patient takes the medication. The
and develop goals together with nurse should return shortly to the
the patient to give the patient patient’s room and remain there
some control over an anxiety- until the patient takes the
inducing situation. Because the medication to verify that it was
other options ignore the patient’s taken as directed. The nurse
feeling and block communication, should never leave medication at
they would not reduce anxiety. the patient’s bedside unless
specifically requested to do so.
13. A scrub nurse in the operating
room has which responsibility? 15. The physician orders heparin,
7,500 units, to be administered
Positioning the patient subcutaneously every 6 hours. The
Assisting with gowning and gloving vial reads 10,000 units per millilitre.
Handling surgical instruments The nurse should anticipate giving
to the surgeon how much heparin for each dose?
Applying surgical drapes
¼ ml
(C) Handling surgical instruments to ½ ml
the surgeon. The scrub nurse ¾ ml
assist the surgeon by providing 1 ¼ ml
appropriate surgical instruments
and supplies, maintaining strict (C) ¾ ml. The nurse solves the
surgical asepsis and, with the problem as follows: 10,000
circulating nurse, accounting for units/7,500 units = 1 ml/X 10,000
all gauze, sponges, needles, and X = 7,500 X= 7,500/10,000 or ¾
instruments. The circulating ml
nurse assists the surgeon and
scrub nurse, positions the 16. The nurse in charge measures a
patient, applies appropriate patient’s temperature at 102
equipment and surgical drapes, degrees F. what is the equivalent
assists with gowning and gloving, Centigrade temperature?
39 degrees C 19. A male patient is to be
47 degrees C discharged with a prescription for an
38.9 degrees C analgesic that is a controlled
40.1 degrees C substance. During discharge
teaching, the nurse should explain
(C) 38.9 degrees C. To convert that the patient must fill this
Fahrenheit degrees to centigrade, prescription how soon after the date
use this formula: C degrees = (F on which it was written?
degrees – 32) x 5/9 C degrees =
(102 – 32) 5/9 + 70 x 5/9 38.9 Within 1 month
degrees C Within 3 months
Within 6 months
17. To evaluate a patient for Within 12 months
hypoxia, the physician is most likely
to order which laboratory test? (C) Within 6 months. In most cases,
an outpatient must fill a
Red blood cell count prescription for a controlled
Sputum culture substance within 6 months of the
Total hemoglobin date on which the prescription
Arterial blood gas (ABG) was written.
analysis
20. Which human element
(D) Arterial blood gas (ABG) considered by the nurse in charge
analysis. All of these test help during assessment can affect drug
evaluate a patient with administration?
respiratory problems. However,
ABG analysis is the only test The patient’s ability to recover
evaluates gas exchange in the The patient’s occupational hazards
lungs, providing information The patient’s socioeconomic status
about patient’s oxygenation The patient’s cognitive abilities
status.
(D) The patient’s cognitive
18. The nurse uses a stethoscope to abilities. The nurse must consider
auscultate a male patient’s chest. the patient’s cognitive abilities to
Which statement about a understand drug instructions. If
stethoscope with a bell and not, the nurse must find a family
diaphragm is true? member or significant other to
take on the responsibility of
The bell detects high-pitched administering medications in the
sounds best home setting. The patient’s
The diaphragm detects high- ability to recover, occupational
pitched sounds best hazards, and socioeconomic
The bell detects thrills best status do not affect drug
The diaphragm detects low-pitched administration.
sounds best
21. When explaining the initiation of
(B) The diaphragm detects high- I.V. therapy to a 2-year-old child, the
pitched sounds best. The nurse should:
diaphragm of a stethoscope
detects high-pitched sound best; Ask the child, “Do you want me to
the bell detects low pitched start the I.V. now?”
sounds best. Palpation detects Give simple directions shortly
thrills best. before the I.V. therapy is to
start
Tell the child, “This treatment is for the plunger, and the needle tip
your own good” must remain sterile until after the
Inform the child that the needle will injection.
be in place for 10 days
23. The best way to instill eye drops
(B) Give simple directions shortly is to:
before the I.V. therapy is to
start. Because a 2-year-old child Instruct the patient to lock
has limited understanding, the upward, and drop the
nurse should give simple medication into the center of
directions and explanations of the lower lid
what will occur shortly before the Instruct the patient to look ahead,
procedure. She should try to and drop the medication into the
avoid frightening the child with center of the lower lid
the explanation and allow the Drop the medication into the inner
child to make simple choices, canthus regardless of eye
such as choosing the I.V. position
insertion site, if possible. Drop the medication into the center
However, she shouldn’t ask the of the canthus regardless of eye
child if he wants the therapy, position
because the answer may be
“No!” Telling the child that the (A) Instruct the patient to lock
treatment is for his own good is upward, and drop the medication
ineffective because a 2-year-old into the center of the lower
perceives pain as a negative lid. Having the patient look
sensation and cannot understand upward reduces blinking and
that a painful procedure can protects the cornea. Instilling
have position results. Telling the drops in the center of the lower
child how long the therapy will lid promotes absorption because
last is ineffective because the 2- the drops are less likely to run
year-old doesn’t have a good into the nasolacrimal duct or out
understanding of time. of the eye.

22. All of the following parts of the


syringe are sterile except the: 24. The difference between an 18G
needle and a 25G needle is the
Barrel needle’s:
Inside of the plunger Length
Needle tip Bevel angle
Barrel tip Thickness
Sharpness
(A) Barrel. All syringes have three
parts: a tip, which connects the (C) Thickness. Gauge is a measure
needle to the syringe; a barrel, of the needle’s thickness: The
the outer part on which the higher the number the thinner
measurement scales are printed; the shaft. Therefore, an 18G
and a plunger, which fits inside needle is considerably thicker
the barrel to expel the than a 25G needle.
medication. The external part of
the barrel and the plunger and 25. A patient receiving an
(flange) must be handled during anticoagulant should be assessed for
the preparation and signs of:
administration of the injection.
However, the inside and trip of Hypotension
the barrel, the inside (shaft) of Hypertension
An elevated hemoglobin count (D) 18th week of pregnancy. On the 8th
An increased number of week of pregnancy, the uterus is still
within the pelvic area. On the 10th week,
erythrocytes the uterus is still within the pelvic area.
On the 12th week, the uterus and
Maternal and Child placenta have grown, expanding into the
abdominal cavity. On the 18th week, the
Nursing uterus has already risen out of the pelvis
and is expanding into the abdominal
area.

PNLE Maternal 4. Which of the following urinary symptoms


does the pregnant woman most frequently

and Child Health experience during the first trimester:

Nursing Exam 1 a) frequency


b) dysuria
c) incontinence
1. A client asks the nurse what a third degree d) Burning
laceration is. She was informed that she had
one. The nurse explains that this is: (A) frequency. Pressure and irritation of the
bladder by the growing uterus during the
a) that extended their anal sphincter first trimester is responsible for causing
b) through the skin and into the muscles urinary frequency. Dysuria, incontinence
c) that involves anterior rectal wall and burning are symptoms associated
d) that extends through the perineal with urinary tract infection.
muscle.

Answer. (A) that extended their anal 5. Mrs. Jimenez went to the health center for
sphincter. Third degree laceration pre-natal check-up. the student nurse took
involves all in the second degree her weight and revealed 142 lbs. She asked
laceration and the external sphincter of the student nurse how much should she gain
the rectum. Options B, C and D are under weight in her pregnancy.
the second degree laceration.
a. 20-30 lbs
2. Betina 30 weeks AOG discharged with a b. 25-35 lbs
diagnosis of placenta previa. The nurse c. 30- 40 lbs
knows that the client understands her care at d. 10-15 lbs
home when she says:
(B) 25-35 lbs. A weight gain of 11. 2 to 15.9
kg (25 to 35 lbs) is currently
A. I am happy to note that we can have sex recommended as an average weight
occasionally when I have no bleeding. gain in pregnancy. This weight gain
B. I am afraid I might have an operation consists of the following: fetus- 7.5 lb;
when my due comes placenta- 1.5 lb; amniotic fluid- 2 lb;
C. I will have to remain in bed until my uterus- 2.5 lb; breasts- 1.5 to 3 lb; blood
due date comes volume- 4 lb; body fat- 7 lb; body fluid- 7
D/ I may go back to work since I stay only at lb.
the office.
6. The nurse is preparing Mrs. Jordan for
Answer. (C) I will have to remain in bed until cesarean delivery. Which of the following key
my due date comes. Placenta previa means concept should the nurse consider when
that the placenta is the presenting part. On implementing nursing care?
the first and second trimester there is
spotting. On the third trimester there is
bleeding that is sudden, profuse and painless a) Explain the surgery, expected outcome
and kind of anesthetics.
3. The uterus has already risen out of the b) Modify preoperative teaching to
pelvis and is experiencing farther into the meet the needs of either a planned
abdominal area at about the: or emergency cesarean birth.
c) Arrange for a staff member of the
anesthesia department to explain what
a. 8th week of pregnancy to expect post-operatively.
b. 10th week of pregnancy d) Instruct the mother’s support person to
c. 12th week of pregnancy remain in the family lounge until after
d. 18th week of pregnancy the delivery.
(B) Modify preoperative teaching to meet (D) assume Sim’s position. When the
the needs of either a planned or woman is in Sim’s position, this puts the
emergency cesarean birth. A key point to weight of the fetus on bed, not on the
consider when preparing the client for a woman and allows good circulation in the
cesarean delivery is to modify the lower extremities.
preoperative teaching to meet the needs
of either planned or emergency cesarean 10. Which is true regarding the fontanels of
birth, the depth and breadth of the newborn?
instruction will depend on circumstances
and time available. a) The anterior is large in shape when
compared to the posterior fontanel.
7. Bettine Gonzales is hospitalized for the b) The anterior is triangular shaped; the
treatment of severe preecplampsia. Which of posterior is diamond shaped.
the following represents an unusual finding c) The anterior is bulging; the posterior
for this condition? appears sunken.
d) The posterior closes at 18 months; the
a. generalized edema anterior closes at 8 to 12 months.
b. proteinuria 4+
c. blood pressure of 160/110 (A) The anterior is large in shape when
d. Convulsions compared to the posterior fontanel.. The
anterior fontanel is larger in size than
(D) convulsions. Options A, B and C are the posterior fontanel. Additionally, the
findings of severe preeclampsia. anterior fontanel, which is diamond
Convulsions is a finding of eclampsia—an shaped closes at 18 month, whereas the
obstetrical emergency. posterior fontanel, which is triangular in
shape closes at 8 to 12 weeks. Neither
fontanel should appear bulging, which
8. Nurse Geli explains to the client who is 33 may indicate increases ICP or sunken,
weeks pregnant and is experiencing vaginal which may indicate hydration.
bleeding that coitus:

a) Need to be modified in any way by either 11. Mrs. Quijones gave birth by spontaneous
partner delivery to a full term baby boy. After a
b) Is permitted if penile penetration is not minute after birth, he is crying and moving
deep. actively. His birth weight is 6.8 lbs. What do
c) Should be restricted because it may you expect baby Quijones to weigh at 6
stimulate uterine activity. months?
d) Is safe as long as she is in side-lying
position. a. 13 -14 lbs
(C) Should be restricted because it may b. 16 -17 lbs
stimulate uterine activity.. Coitus is c. 22 -23 lbs
restricted when there is watery d. 27 -28 lbs
discharge, uterine contraction and
vaginal bleeding. Also those women with A) 13 -14 lbs. The birth weight of an infant is
a history of spontaneous miscarriage doubled at 6 months and is tripled at 12
may be advised to avoid coitus during months
the time of pregnancy when a previous
miscarriage occurred. 12. During the first hours following delivery,
the post partum client is given IVF with
9. Mrs. Precilla Abuel, a 32 year old mulripara oxytocin added to them. The nurse
is admitted to labor and delivery. Her last 3 understands the primary reason for this is:
pregnancies in short stage one of labor. The
nurses decide to observe her closely. The a) To facilitate elimination
physician determines that Mrs. Abuel’s cervix b) To promote uterine contraction
is dilated to 6 cm. Mrs. Abuel states that she c) To promote analgesia
is extremely uncomfortable. To lessen Mrs. d) To prevent infection
Abuel’s discomfort, the nurse can advise her B To promote uterine contraction. Oxytocin
to: is a hormone produced by the pituitary
gland that produces intermittent uterine
a. lie face down contractions, helping to promote uterine
b. not drink fluids involution.
c. practice holding breaths between
contractions 13. Nurse Luis is assessing the newborn’s
d. assume Sim’s position heart rate. Which of the following would be
considered normal if the newborn is
sleeping?
a. 80 beats per minute b) Using gesture to communicate at 18
b. 100 beats per minute months.
c. 120 beats per minute c) Cooing at 3 months.
d. 140 beats per minute d) Saying “mama” or “dada” for the
first time at 18 months of age.
(B) 100 beats per minute. The normal heart
rate for a newborn that is sleeping is (D) Saying “mama” or “dada” for the first
approximately 100 beats per minute. If time at 18 months of age.. A child should
the newborn was awake, the normal say “mama” or “dada” during 10 to 12
heart rate would range from 120 to 160 months of age. Options A, B and C are all
beats per minute. normal assessments of language
development of a child.
14. The infant with Down Syndrome should
go through which of the Erikson’s
developmental stages first? 18. Isabelle, a 2 year old girl loves to move
around and oftentimes manifests negativism
a) Initiative vs. Self doubt and temper tantrums. What is the best way
b) Industry vs. Inferiority to deal with her behavior?
c) Autonomy vs. Shame and doubt
d) Trust vs. Mistrust a. Tell her that she would not be loved by
others is she behaves that way..
D) Trust vs. Mistrust. The child with Down b. Withholding giving her toys until she
syndrome will go through the same first behaves properly.
stage, trust vs. mistrust, only at a slow c. Ignore her behavior as long as she does
rate. Therefore, the nurse should not hurt herself and others.
concentrate on developing on bond d. Ask her what she wants and give it to
between the primary caregiver and the pacify her.
child. (B) Ignore her behavior as long as she
does not hurt herself and others.. If a
15. The child with phenylketonuria (PKU) child is trying to get attention or trying
must maintain a low phenylalanine diet to to get something through tantrums—
prevent which of the following ignore his/her behavio
complications?
19. Baby boy Villanueva, 4 months old, was
a. Irreversible brain damage seen at the pediatric clinic for his scheduled
b. Kidney failure check-up. By this period, baby Villanueva has
c. Blindness already increased his height by how many
d. Neutropenia inches?

(A) Irreversible brain damage. The child a) 3 inches


with PKU must maintain a strict low b) 4 inches
phenylalanine diet to prevent central c) 5 inches
nervous system damage, seizures and d) 6 inches
eventual dea (B) 4 inches. From birth to 6 months, the
infant grows 1 inch (2.5 cm) per month.
16. Which age group is with imaginative From 6 to 12 months, the infant grows ½
minds and creates imaginary friends? inch (1.25 cm) per month.

a) Toddler 20. Alice, 10 years old was brought to the ER


b) Preschool because of Asthma. She was immediately
c) School put under aerosol administration of
d) Adolescence Terbutaline. After sometime, you observe
that the child does not show any relief from
(B) Preschool. During preschool, this is the the treatment given. Upon assessment, you
time when children do imitative play, noticed that both the heart and respiratory
imaginative play—the occurrence of rate are still elevated and the child shows
imaginative playmates, dramatic play difficulty of exhaling. You suspect:
where children like to act, dance and
sing. a. Bronchiectasis
b. Atelectasis
17. Which of the following situations would c. Epiglotitis
alert you to a potentially developmental d. Status Asthmaticus
problem with a child? (D) Status Asthmaticus. Status asthmaticus
leads to respiratory distress and
a) Pointing to body parts at 15 months of bronchospasm despite of treatment and
age.
interventions. Mechanical ventilation 24. Which of the following is the most
maybe needed due to respiratory failure. appropriate intervention to reduce stress in a
preterm infant at 33 weeks gestation?
21. Nurse Jonas assesses a 2 year old boy
with a tentative diagnosis of a. Sensory stimulation including several
nephroblastoma. Symptoms the nurse senses at a time
observes that suggest this problem include: b. tactile stimulation until signs of over
stimulation develop
a) Lymphedema and nerve palsy c. An attitude of extension when prone or
b) Hearing loss and ataxia side lying
c) Headaches and vomiting d. Kangaroo care
d) Abdominal mass and weakness
(D) Kangaroo care. Kangaroo care is the use
(D)Abdominal mass and of skin-to-skin contact to maintain body
weakness. Nephroblastoma or Wilm’s tumor heat. This method of care not only
is caused by chromosomal abnormalities, supplies heat but also encourages
most common kidney cancer among children parent-child interaction.
characterized by abdominal mass,
hematuria, hypertension and fever 25. The parent of a client with albinism
would need to be taught which preventive
22. Which of the following danger sings healthcare measure by the nurse:
should be reported immediately during the
antepartum period? a) Ulcerative colitis diet
b) Use of a high-SPF sunblock
a. blurred vision c) Hair loss monitoring
b. nasal stuffiness d) Monitor for growth retardation
c. breast tenderness (B) Use of a high-SPF sunblock. Without
d. Constipation melanin production, the child with
(A) blurred vision. Danger signs that require albinism is at risk for severe sunburns.
prompt reporting are leaking of amniotic Maximum sun protection should be
fluid, blurred vision, vaginal bleeding, taken, including use of hats, long
rapid weight gain and elevated blood sleeves, minimal time in the sun and
pressure. Nasal stuffiness, breast high-SPF sunblock, to prevent any
tenderness, and constipation are problems.
common discomforts associated with
pregnancy.

23. Nurse Jacob is assessing a 15 month old


child with acute otitis media. Which of the
following symptoms would the nurse
anticipate finding?

a) periorbital edema, absent light reflex and


translucent tympanic membrane
b) irritability, purulent drainage in
middle ear, nasal congestion and
cough
c) diarrhea, retracted tympanic membrane
and enlarged parotid gland
d) Vomiting, pulling at ears and pearly white
tympanic membrane
(B) irritability, purulent drainage in middle
ear, nasal congestion and
cough. Irritability, purulent drainage in
PNLE Maternal and
middle ear, nasal congestion and cough, Child Health Nursing
fever, loss of appetite, vomiting and
diarrhea are clinical manifestations of Exam 2
otitis media. Acute otitis media is
common in children 6 months to 3 years
old and 8 years old and above. Breast 1. Nurse Bella explains to a 28 year old
fed infants have higher resistance due to pregnant woman undergoing a non-stress
protection of Eustachian tubes and test that the test is a way of evaluating the
middle ear from breast milk. condition of the fetus by comparing the fetal
heart rate with:

a. Fetal lie
b. Fetal movement nurse would explain that this is most
c. Maternal blood pressure probably the result of which of the following:
d. Maternal uterine contractions
a) Thrombophlebitis
(B) Fetal movement. Non-stress test b) PIH
measures response of the FHR to the c) Pressure on blood vessels from the
fetal movement. With fetal movement, enlarging uterus
FHR increase by 15 beats and remain for d) The force of gravity pulling down on the
15 seconds then decrease to average uterus
rate. No increase means poor
oxygenation perfusion to fetus. (C) Pressure on blood vessels from the
enlarging uterus. Pressure of the growing
2. During a 2 hour childbirth focusing on fetus on blood vessels results in an
labor and delivery process for primigravida. increase risk for venous stasis in the
The nurse describes the second maneuver lower extremities. Subsequently, edema
that the fetus goes through during labor and varicose vein formation may occur.
progress when the head is the presenting
part as which of the following: 6. Mrs. Ella Santoros is a 25 year old
primigravida who has Rheumatic heart
a. Flexion disease lesion. Her pregnancy has just been
b. Internal rotation diagnosed. Her heart disease has not caused
c. Descent her to limit physical activity in the past. Her
d. External rotation cardiac disease and functional capacity
(A) Flexion. The 6 cardinal movements of classification is:
labor are descent, flexion, internal
rotation, extension, external rotation and a. Class I
expulsion. b. Class II
c. Class III
3. Mrs. Jovel Diaz went to the hospital to d. class IV
have her serum blood test for alpha-
fetoprotein. The nurse informed her about (A) Class I. Clients under class I has no
the result of the elevation of serum AFP. The physical activity limitation. There is a
patient asked her what was the test for: slight limitation of physical activity in
class II, ordinary activity causes
a) Congenital Adrenal Hyperplasia fatigue, palpitation, dyspnea or
b) PKU angina. Class III is moderate limitation
c) Down Syndrome of physical activity; less than ordinary
d) Neural tube defects activity causes fatigue. Unable to
carry on any activity without
(D)Neural tube defects. Alpha-fetoprotein is a experiencing discomfort is under class
substance produces by the fetal liver that is IV
present in amniotic fluid and maternal
serum. The level is abnormally high in the 7. The client asks the nurse, “When will this
maternal serum if the fetus has an open soft spot at the top of the head of my baby
spinal or abdominal defect because the open will close?” The nurse should instruct the
defect allows more AFP to appear mother that the neonate’s anterior fontanel
will normally close by age:
4. Fetal heart rate can be auscultated with a
fetoscope as early as: a) 2-3 months
b) 6-8 months
a. 5 weeks of gestation c) 10-12 months
b. 10 weeks of gestation d) 12-18 months
c. 15 weeks of gestation
d. 20 weeks of gestation (D) 12-18 months. Anterior fontanel closes
at 12-18 months while posterior fontanel
(D) 20 weeks of gestation. The FHR can be closes at birth until 2 months.
auscultated with a fetoscope at about 20
weeks of gestation. FHR is usually 8. When a mother bleeds and the uterus is
auscultated at the midline suprapubic relaxed, soft and non-tender, you can
region with Doppler ultrasound at 10 to account the cause to:
12 weeks of gestation. FHR cannot be
heard any earlier than 10 weeks of a. Atony of the uterus
gestation. b. Presence of uterine scar
5. Mrs. Bendivin states that she is c. Laceration of the birth canal
experiencing aching swollen, leg veins. The d. Presence of retained placenta fragments
A) Atony of the uterus. Uterine atony, or 12. Mrs. Grace Evangelista is admitted with
relaxation of the uterus is the most frequent severe preeclampsia. What type of room
cause of postpartal hemorrhage. It is the should the nurse select this patient?
inability to maintain the uterus in contracted
state. a. A room next to the elevator.
b. The room farthest from the nursing
9. Mrs. Pichie Gonzales’s LMP began April 4, station.
2010. Her EDD should be which of the c. The quietest room on the floor.
following: d. The labor suite.
(C) The quietest room on the floor.A loud
a) February 11, 2011 noise such as a crying baby, or a
b) January 11, 20111 dropped tray of equipment may be
c) December 12, 2010 sufficient to trigger a seizure initiating
d) Nowember 14, 2010 eclampsia, a woman with severe
preeclampsia should be admiotted to a
(B) January 11, 20111. Using the Nagel’s private room so she can rest as
rule, he use this formula ( -3 calendar undisturbed as possible. Darken the
months + 7 days). room if possible because bright light can
trigger seizures.

10. Which of the following prenatal 13. During a prenatal check-up, the nurse
laboratory test values would the nurse explains to a client who is Rh negative that
consider as significant? RhoGAM will be given:

a. Hematocrit 33.5% a) Weekly during the 8th month because


b. WBC 8,000/mm3 this is her third pregnancy.
c. Rubella titer less than 1:8 b) During the second trimester, if
d. One hour glucose challenge test 110 g/dL amniocentesis indicates a problem.
c) To her infant immediately after delivery if
C Rubella titer less than 1:8. A rubella titer the Coomb’s test is positive.
should be 1:8 or greater. Thus, a finding of a d) Within 72 hours after delivery if
titer less than 1:8 is significant, indicating infant is found to be Rh positive.
that the client may not possess immunity to
rubella. A hematocrit of 33.5%, WBC of (D) Within 72 hours after delivery if infant is
8,000/mm3, and a 1 hour glucose challenge found to be Rh positive. RhoGAM is given
test of 110 g/dL are within normal to Rh-negative mothers within 72 hours
parameters. after birth of Rh-positive baby to prevent
development of antibodies in the
maternal blood stream, which will be fata
11. Aling Patricia is a patient with to succeeding Rh-positive offspring.
preeclampsia. You advise her about her
condition, which would tell you that she has
not really understood your instructions? 14. A baby boy was born at 8:50pm. At
8:55pm, the heart rate was 99 bpm. She has
a) “I will restrict my fat in my diet.” a weak cry, irregular respiration. She was
b) “I will limit my activities and rest moving all extremities and only her hands
more frequently throughout the and feet were still slightly blue. The nurse
day.” should enter the APGAR score as:
c) “I will avoid salty foods in my diet.”
d) “I will come more regularly for check-up.” a. 5
b. 6
(B) “I will limit my activities and rest more c. 7
frequently throughout the day.”Pregnant d. 8
woman with preeclampsia should be in a
complete bed rest. When body is in (B) 6. Heart rate of 99 bpm-1; weak cry-1;
recumbent position, sodium tends to be irregular respiration-1; moving all
excreted at a faster rate. It is the best extremities-2; extremities are slightly
method of aiding increased excretion of blue-1; with a total score of 6.
sodium and encouraging diuresis. Rest
should always be in a lateral recumbent 15. Billy is a 4 year old boy who has an IQ of
position to avoid uterine pressure on the 140 which means:
vena cava and prevent supine
hypotension. a. average normal
b. very superior
c. above average
d. Genius
would be the nurse’s most accurate analysis
(D) genius. IQ= mental age/chronological of the mother’s comment?
age x 100. Mental age refers to the
typical intelligence level found for people a. The child has not experienced limit-
at a give chronological age. OQ of 140 setting or structure.
and above is considered genius. b. The child is expressing a physical need,
such as hunger.
c. The mother has nurtured
16. A newborn is brought to the nursery. overdependence in the child.
Upon assessment, the nurse finds that the d. The mother is describing her child’s
child has short palpebral fissures, thinned separation anxiety.
upper lip. Based on this data, the nurse (D) The mother is describing her child’s
suspects that the newborn is MOST likely separation anxiety. Before coming to any
showing the effects of: conclusion, the nurse should ask the
mother focused questions; however,
a) Chronic toxoplasmosis based on initial information, the analysis
b) Lead poisoning of separation anxiety would be most
c) Congenital anomalies valid. Separation anxiety is a normal
d) Fetal alcohol syndrome toddler response. When the child senses
he is being sent away from those who
(D) Fetal alcohol syndrome. The newborn most provide him with love and security.
with fetal alcohol syndrome has a Crying is one way a child expresses a
number of possible problems at birth. physical need; however, the nurse would
Characteristics that mark the syndrome be hasty in drawing this as first
include pre and postnatal growth conclusion based on what the mother
retardation; CNS involvement such as has said. Nurturing overdependence or
cognitive challenge, microcephally and not providing structure for the toddler
cerebral palsy; and a distinctive facial are inaccurate conclusions based on the
feature of a short palpebral fissure and information provided.
thin upper lip.
20. Mylene Lopez, a 16 year old girl with
17. A priority nursing intervention for the scoliosis has recently received an invitation
infant with cleft lip is which of the following: to a pool party. She asks the nurse how she
can disguise her impairment when dressed in
a. Monitoring for adequate nutritional a bathing suit. Which nursing diagnosis can
intake be justified by Mylene’s statement?
b. Teaching high-risk newborn care
c. Assessing for respiratory distress a) Anxiety
d. Preventing injury b) Body image disturbance
c) Ineffective individual coping
(A) Monitoring for adequate nutritional d) Social isolation
intake. The infant with cleft lip is unable
to create an adequate seal for sucking. (B) Body image disturbance. Mylene is
The child is at risk for inadequate experiencing uneasiness about the
nutritional intake as well as aspiration. curvative of her spine, which will be
more evident when she wears a bathing
18. Nurse Jacob is assessing a 12 year old suit. This data suggests a body image
who has hemophilia A. Which of the following disturbance. There is no evidence of
assessment findings would the nurse anxiety or ineffective coping. The fact
anticipate? that Mylene is planning to attend a pool
party dispels a diagnosis of social
a) an excess of RBC isolation.
b) an excess of WBC
c) a deficiency of clotting factor VIII 21. The foul-smelling, frothy characteristic of
d) a deficiency of clotting factor IX the stool in cystic fibrosis results from the
presence of large amounts of which of the
C) a deficiency of clotting factor following:
VIII. Hemophillia A (classic hemophilia) is
a deficiency in factor VIII (an alpha a. sodium and chloride
globulin that stabilizes fibrin clots). b. undigested fat
c. semi-digested carbohydrates
d. lipase, trypsin and amylase
19. Celine, a mother of a 2 year old tells the
nurse that her child “cries and has a fit when (B) undigested fat. The client with cystic
I have to leave him with a sitter or someone fibrosis absorbs fat poorly because of the
else.” Which of the following statements think secretions blocking the pancreatic
duct. The lack of natural pancreatic d) Temporary separation from the
enzyme leads to poor absorption of loved one.
predominantly fats in the duodenum.
Foul-smelling, frothy stool is termed (D) Temporary separation from the loved
steatorrhea. one. The predominant perception of
death by preschool age children is that
22. Which of the following would be a death is temporary separation. Because
disadvantage of breast feeding? that child is losing someone significant
and will not see that person again, it’s
a) involution occurs rapidly inaccurate to infer death is insignificant,
b) the incidence of allergies increases due regardless of the child’s response.
to maternal antibodies
c) the father may resent the infant’s 25. Catherine Diaz is a 14 year old patient on
demands on the mother’s body a hematology unit who is being treated for
d) there is a greater chance of error during sickle cell crisis. During a crisis such as that
preparation seen in sickle cell anemia, aldosterone
release is stimulated. In what way might this
(C) the father may resent the infant’s influence Catherine’s fluid and electrolyte
demands on the mother’s body. With balance?
breast feeding, the father’s body is not
capable of providing the milk for the a. sodium loss, water loss and potassium
newborn, which may interfere with retention
feeding the newborn, providing fewer b. sodium loss, water los and potassium
chances for bonding, or he may be loss
jealous of the infant’s demands on his c. sodium retention, water loss and
wife time and body. Breast feeding is potassium retention
advantageous because uterine involution d. sodium retention, water retention
occurs more rapidly, thus minimizing and potassium loss
blood loss. The presence of maternal
antibodies in breast milk helps decrease (D) sodium retention, water retention and
the incidence of allergies in the newborn. potassium loss. Stress stimulates the adrenal
A greater chance for error is associated cortex to increase the release of aldosterone.
with bottle feeding. No preparation Aldosterone promotes the resorption of
required for breast feeding. sodium, the retention of water and the loss
of potassium
23. A client is noted to have lymphedema,
webbed neck and low posterior hairline.
Which of the following diagnoses is most
appropriate?
PNLE Maternal
a. Turner’s syndrome
b.
c.
Down’s syndrome
Marfan’s syndrome and Child Health
d. Klinefelter’s syndrome

(A) Turner’s syndrome. Lymphedema,


webbed neck and low posterior hairline,
Nursing Exam 3
these are the 3 key assessment features
in Turner’s syndrome. If the child is 1. A pregnant woman who is at term is
diagnosed early in age, proper treatment admitted to the birthing unit in active
can be offered to the family. All
labor. The client has only progressed
newborns should be screened for
from 2cm to 3 cm in 8 hours. She is
possible congenital defects.
diagnosed with hypotonic dystocia and
the physician ordered Oxytocin (Pitocin)
24. A 4 year old boy most likely perceives to augment her contractions. Which of
death in which way: the following is the most important
aspect of nursing intervention at this
a) An insignificant event unless taught time?
otherwise
b) Punishment for something the individual a) Timing and recording length of
did contractions.
c) Something that just happens to older b) Monitoring.
people c) Preparing for an emergency cesarean
birth.
d) Checking the perineum for bulging. A. Phenylephrine, with frequent and
continued use, can cause rebound
A. The oxytocic effect of Pitocin congestion of mucous membranes.
increases the intensity and durations
of contractions; prolonged 5. A client with tuberculosis is to be
contractions will jeopardize the admitted in the hospital. The nurse who
safetyof the fetus and necessitate will be assigned to care for the client
discontinuing the drug. must institute appropriate precautions.
The nurse should:
2. A client who hallucinates is not in
touch with reality. It is important for the a) Place the client in a private room.
nurse to: b) Wear an N 95 respirator when
caring for the client.
a. Isolate the client from other patients. c) Put on a gown every time when
b. Maintain a safe environment. entering the room.
c. Orient the client to time, place, and d) Don a surgical mask with a face
person. shield when entering the room.
d. Establish a trusting relationship.
B. The N 95 respirator is a high-
B. It is of paramount importance to particulate filtration mask that meets
prevent the client from hurting the CDC performance criteria for a
himself or herself or others tuberculosis respirator.

3. The nurse is caring to a child client 6. Which of the following is the most
who has had a tonsillectomy. The child frequent cause of noncompliance to the
complains of having dryness of the medical treatment of open-angle
throat. Which of the following would the glaucoma?
nurse give to the child?
a. The frequent nausea and vomiting
a) Cola with ice accompanying use of miotic drug.
b) Yellow noncitrus Jello b. Loss of mobility due to severe driving
c) Cool cherry Kool-Aid restrictions.
d) A glass of milk c. Decreased light and near-vision
accommodation due to miotic
B. After tonsillectomy, clear, cool liquids effects of pilocarpine.
should be given. Citrus, carbonated, d. The painful and insidious progression
and hot or cold liquids should be of this type of glaucoma.
avoided because they may irritate
the throat. Red liquids should be C. The most frequent cause of
avoided because they give the noncompliance to the treatment of
appearance of blood if the child chronic, or open-angle glaucoma is
vomits. Milk and milk products the miotic effects of pilocarpine.
including pudding are avoided Pupillary constriction impedes normal
because they coat the throat, cause accommodation, making night driving
the child to clear the throat, and difficult and hazardous, reducing the
increase the risk of bleeding. client’s ability to read for extended
periods and making participation in
4. The physician ordered Phenylephrine games with fast-moving objects
(Neo-Synephrine) nasal spray to a 13- impossible.
year-old client. The nurse caring to the
client provides instructions that the nasal 7. In the morning shift, the nurse is
spray must be used exactly as directed making rounds in the nursing care units.
to prevent the development of: The nurse enters in a client’s room and
notes that the client’s tube has become
a. Increased nasal congestion. disconnected from the Pleurovac. What
b. Nasal polyps. would be the initial nursing action?
c. Bleeding tendencies.
d. Tinnitus and diplopia. a) Apply pressure directly over the
incision site.
b) Clamp the chest tube near the
incision site.
c) Clamp the chest tube closer to the weeks of life. In hypothyroidism, the
drainage system. infant’s muscle tone would be poor
d) Reconnect the chest tube to the and the infant would not be able to
Pleurovac. achieve this milestone.

B. This stops the sucking of air through


the tube and prevents the entry of 11. The physician calls the nursing unit to
contaminants. In addition, clamping leave an order. The senior nurse had
near the chest wall provides for some conversation with the other staff. The
stability and may prevent the clamp newly hired nurse answers the phone so
from pulling on the chest tube. that the senior nurses may continue their
conversation. The new nurse does not
8. Which of the following complications knowthe physician or the client to whom
during a breech birth the nurse needs to the order pertains. The nurse should:
be alarmed?
a) Ask the physician to call back after
a. Abruption placenta. the nurse has read the hospital policy
b. Caput succedaneum. manual.
c. Pathological hyperbilirubinemia. b) Take the telephone order.
d. Umbilical cord prolapse. c) Refuse to take the telephone order.
d) Ask the charge nurse or one of
D. Because umbilical cord’s insertion the other senior staff nurses to
site is born before the fetal head, the take the telephone order.
cord may be compressed by the D. Get a senior nurse who know s the
after-coming head in a breech birth. policies, the client, and the doctor.
Generally speaking, a nurse should
not accept telephone orders.
9. The nurse is caring to a client However, if it is necessary to take
diagnosed with severe depression. Which one, follow the hospital’s policy
of the following nursing approach is regarding telephone orders. Failure to
important in depression? followhospital policy could be
considered negligence. In this case,
a) Protect the client against harm to the nurse was new and did not know
others. the hospital’s policy concerning
b) Provide the client with motor telephone orders. The nurse was also
outlets for aggressive, hostile unfamiliar with the doctor and the
feelings. client. Therefore the nurse should not
c) Reduce interpersonal contacts. take the order unless a) no one else
d) Deemphasizing preoccupation with is available and b) it is an emergency
elimination, nourishment, and sleep. situation.

B. It is important to externalize the 12. The staff nurse on the labor and
anger away from self. delivery unit is assigned to care to a
primigravida in transition complicated by
hypertension. A new pregnant woman in
10. A 3-month-old client is in the active labor is admitted in the same unit.
pediatric unit. During assessment, the The nurse manager assigned the same
nurse is suspecting that the baby may nurse to the second client. The nurse
have hypothyroidism when mother states feels that the client with hypertension
that her baby does not: requires one-to-one care. What would be
the initial actionof the nurse?
a. Sit up.
b. Pick up and hold a rattle. a. Accept the new assignment and
c. Roll over. complete an incident report
d. Hold the head up. describing a shortage of nursing staff.
b. Report the incident to the nursing
D. Development normally proceeds supervisor and request to be floated.
cephalocaudally; so the first major c. Report the nursing assessment of
developmental milestone that the the client in transitional labor to
infant achieves is the ability to hold the nurse manager and discuss
the head up within the first 8-12
misgivings about the new b) “Has he been taking diuretics at
assignment. home?”
Accept the new assignment and provide c) “Do any of his brothers and sisters
the best care. have history of cardiac problems?”
d) “Has he been going to school
C. The nurse is obligated to inform the regularly?”
nurse manager about changes in the
condition of the client, which may B. The child who is concurrently taking
change the decision made by the digoxin and diuretics is at increased
nurse manager. risk for digoxin toxicity due to the
loss of potassium. The child and
13. A newborn infant with Down parents should be taught what foods
syndrome is to be discharged today. The are high in potassium, and the child
nurse is preparing to give the discharge should be encouraged to eat a high-
teaching regarding the proper care at potassium diet. In addition, the
home. The nurse would anticipate that child’s serum potassium level should
the mother is probably at the: be carefully monitored.

a) 40 years of age. 16. The nurse noticed that the signed


b) 20 years of age. consent form has an error. The form
c) 35 years of age. states, “Amputation of the right leg”
d) 20 years of age. instead of the left leg that is to be
amputated. The nurse has administered
A. Perinatal risk factors for the already the preoperative medications.
development of Down syndrome What should the nurse do?
include advanced maternal age,
especially with the first pregnancy. a. Call the physician to reschedule
the surgery.
14. The emergency department has b. Call the nearest relative to come in to
shortage of staff. The nurse manager sign a new form.
informs the staff nurse in the critical care c. Cross out the error and initial the
unit that she has to float to the form.
emergency department. What should the d. Have the client sign another form.
staff nurse expect under these
conditions? A. The responsible for an accurate
informed consent is the physician. An
a. The float staff nurse will be informed exception to this answer would be a life-
of the situation before the shift threatening emergency, but there are no
begins. data to support another response
b. The staff nurse will be able to
negotiate the assignments in the 17. The nurse in the nursing care unit
emergency department. checks the fluctuation in the water-seal
c. Cross training will be available for the compartment of a closed chest drainage
staff nurse. system. The fluctuation has stopped, the
d. Client assignments will be equally nurse would:
divided among the nurses.
B. Assignments should be based on a) Vigorously strip the tube to dislodge a
scope of practice and expertise. clot.
b) Raise the apparatus above the chest
15. The nurse is assigned to care for a to move fluid.
child client admitted in the pediatrics c) Increase wall suction above 20 cm
unit. The client is receiving digoxin. H2O pressure.
Which of the following questions will be d) Ask the client to cough and take
asked by the nurse to the parents of the a deep breath.
child in order to assess the client’s risk
for digoxin toxicity? D. Asking the client to cough and take a
deep breath will help determine if the
a) “Has he been exposed to any chest tube is kinked or if the lungs has
childhood communicable diseases in reexpanded
the past 2-3 weeks?”
18. The pediatric nurse in the neonatal discharge on prednisone therapy, the
unit was informed that the baby that is nurse should advise the client to:
brought to the mother in the hospital
room is wrong. The nurse determines a) Wear sunglasses if exposed to bright
that two babies were placed in the wrong light for an extended period of time.
cribs. The most appropriate nursing b) Take oral preparations of prednisone
action would be to: before meals.
c) Have periodic complete blood counts
a. Determine who is responsible for the while on the medication.
mistake and terminate his or her d) Never stop or change the amount
employment. of the medication without
b. Record the event in an medical advice.
incident/variance report and
notify the nursing supervisor. D. In preparing the client for discharge
c. Reassure both mothers, report to the that is receiving prednisone, the nurse
charge nurse, and do not record. should caution the client to (a) take oral
d. Record detailed notes of the event on preparations after meals; (b) remember
the mother’s medical record. that routine checks of vital signs, weight,
and lab studies are critical; (c) NEVER
B. Every event that exposes a client to STOP OR CHANGE THE AMOUNT OF
harm should be recorded in an MEDICATION WITHOUT MEDICAL ADVICE;
incident report, as well as reported to (d) store the medication in a light-
the appropriate supervisors in order resistant container.
to resolve the current problems and
permit the institution to prevent the 22. A pregnant client tells the nurse that
problem from happening again. she is worried about having urinary
frequency. What will be the most
19. Before the administration of digoxin, appropriate nursing response?
the nurse completes an assessment to a
toddler client for signs and symptoms of a. “Try using Kegel (perineal)
digoxin toxicity. Which of the following is exercises and limiting fluids
the earliest and most significant sign of before bedtime. If you have
digoxin toxicity? frequency associated with fever,
pain on voiding, or blood in the
a) Tinnitus urine, call your doctor/nurse-
b) Nausea and vomiting midwife.
c) Vision problem b. “Placental progesterone causes
d) Slowing in the heart rate irritability of the bladder sphincter.
Your symptoms will go away after the
D. One of the earliest signs of digoxin baby comes.”
toxicity is Bradycardia. For a toddler, c. “Pregnant women urinate frequently
any heart rate that falls below the to get rid of fetal wastes. Limit fluids
norm of about 100-120 bpm would to 1L/daily.”
indicate Bradycardia and would d. “Frequency is due to bladder irritation
necessitate holding the medication from concentrate urine and is normal
and notifying the physician. in pregnancy. Increase your daily fluid
intake to 3L.”
20. Which of the following treatment
modality is appropriate for a client with A. Progesterone also reduces smooth
paranoid tendency? muscle motility in the urinary tract
and predisposes the pregnant woman
a. Activity therapy. to urinary tract infections. Women
b. Individual therapy. should contact their doctors if they
c. Group therapy. exhibit signs of infection. Kegel
d. Family therapy. exercise will help strengthen the
perineal muscles; limiting fluids at
B. This option is least threatening. bedtime reduces the possibility of
being awakened by the necessity of
21. The client with rheumatoid arthritis is voiding.
for discharge. In preparing the client for
23. Which of the following will help the c) Plan to give phenytoin over 30-60
nurse determine that the expression of minutes, using an in-line filter.
hostility is useful? d) Flush the IV tubing with normal saline
before starting phenytoin.
a) Expression of anger dissipates the
energy. A. Phenytoin should be infused or
b) Energy from anger is used to injected into larger veins to avoid the
accomplish what needs to be discoloration know as purple glove
done. syndrome; infusing into a smaller
c) Expression intimidates others. vein is not appropriate.
d) Degree of hostility is less than the
provocation.
26. The pregnant woman visits the clinic
B. This is the proper use of anger. for check –up. Which assessment findings
will help the nurse determine that the
client is in 8-week gestation?
24. The nurse is providing an orientation
regarding case management to the a. Leopold maneuvers.
nursing students. Which characteristics b. Fundal height.
should the nurse include in the c. Positive radioimmunoassay test
discussion in understanding case (RIA test).
management? d. Auscultation of fetal heart tones.

a. Main objective is a written plan that C. Serum radioimmunoassay (RIA) is


combines discipline-specific accurate within 7days of conception.
processes used to measure outcomes This test is specific for HCG, and
of care. accuracy is not compromised by
b. Main purpose is to identify expected confusion with LH.
client, family and staff performance
against the timeline for clients with
the same diagnosis. 27. Which of the following nursing
c. Main focus is comprehensive intervention is essential for the client
coordination of client care, avoid who had pneumonectomy?
unnecessary duplication of
services, improve resource a) Medicate for pain only when needed.
utilization and decrease cost. b) Connect the chest tube to water-seal
d. Primary goal is to understand why drainage.
predicted outcomes have not been c) Notify the physician if the chest
met and the correction of identified drainage exceeds 100mL/hr.
problems. d) Encourage deep breathing and
coughing.
C. There are several models of case
management, but the commonality is D. Surgery and anesthesia can increase
comprehensive coordination of care mucus production. Deep breathing
to better predict needs of high-risk and coughing are essential to prevent
clients, decrease exacerbations and atelectasis and pneumonia in the
continually monitor progress client’s only remaining lung.
overtime.
28. The nurse is providing a health
25. The physician orders a dose of IV teaching to a group of parents regarding
phenytoin to a child client. In preparing in Chlamydia trachomatis. The nurse is
the administration of the drug, which correct in the statement, “Chlamydia
nursing action is not correct? trachomatis is not only an intracellular
bacterium that causes neonatal
a) Infuse the phenytoin into a conjunctivitis, but it also can cause:
smaller vein to prevent purple
glove syndrome. a. Discoloration of baby and adult teeth.
b) Check the phenytoin solution to be b. Pneumonia in the newborn.
sure it is clear or light yellow in color, c. Snuffles and rhagades in the
never cloudy. newborn.
d. Central hearing defects in infancy.
A. Contraction of the milk ducts and
B. Newborns can get pneumonia let-down reflex occur under the
(tachypnea, mild hypoxia, cough, stimulation of oxytocin released by
eosinophilia) and conjunctivitis from the posterior pituitary gland.
Chlamydia.
32. One staff nurse is assigned to a group
29. The nurse is assigned to care to a 17- of 5 patients for the 12-hour shift. The
year-old male client with a history of nurse is responsible for the overall
substance abuse. The client asks the planning, giving and evaluating care
nurse, “Have you ever tried or used during the entire shift. After the shift,
drugs?” The most correct response of the same responsibility will be endorsed to
nurse would be: the next nurse in charge. This describes
nursing care delivered via the:
a) “Yes, once I tried grass.”
b) “No, I don’t think so.” a) Primary nursing method.
c) “Why do you want to know that?” b) Case method.
d) “How will my answer help you?” c) Functional method.
d) Team method.
D. The client may perceive this as
avoidance, but it is more important to B. In case management, the nurse
redirect back to the client, especially assumes total responsibility for
in light of the manipulative behavior meeting the needs of the client
of drug abusers and adolescents. during the entire time on duty.

30. Which of the following describes a 33. The ambulance team calls the
health care team with the principles of emergency department that they are
participative leadership? going to bring a client who sustained
burns in a house fire. While waiting for
a. Each member of the team can the ambulance, the nurse will anticipate
independently make decisions emergency care to include assessment
regarding the client’s care without for:
necessarily consulting the other
members. a. Gas exchange impairment.
b. The physician makes most of the b. Hypoglycemia.
decisions regarding the client’s care. c. Hyperthermia.
c. The team uses the expertise of d. Fluid volume excess.
its members to influence the
decisions regarding the client’s A. Smoke inhalation affects gas
care. exchange.
d. Nurses decide nursing care;
physicians decide medical and other
treatment for the client. 34. Most couples are using “natural”
family planning methods. Most accidental
C. It describes a democratic process in pregnancies in couples preferred to use
which all members have input in the this method have been related to
client’s care. unprotected intercourse before ovulation.
Which of the following factor explains
why pregnancy may be achieved by
31. A nurse is giving a health teaching to unprotected intercourse during the
a woman who wants to breastfeed her preovulatory period?
newborn baby. Which hormone, normally
secreted during the postpartum period, a) Ovum viability.
influences both the milk ejection reflex b) Tubal motility.
and uterine involution? c) Spermatozoal viability.
d) Secretory endometrium.
a) Oxytocin.
b) Estrogen. C. Sperm deposited during intercourse
c) Progesterone. may remain viable for about 3 days.
d) Relaxin. If ovulation occurs during this period,
conception may result.
vehicle accident. While monitoring the
35. An older adult client wakes up at 2 client, the nurse suspects increasing
o’clock in the morning and comes to the intracranial pressure when:
nurse’s station saying, “I am having
difficulty in sleeping.” What is the best a) Client is oriented when aroused
nursing response to the client? from sleep, and goes back to
sleep immediately.
a. “I’ll give you a sleeping pill to help b) Blood pressure is decreased from
you get more sleep now.” 160/90 to 110/70.
b. “Perhaps you’d like to sit here at c) Client refuses dinner because of
the nurse’s station for a while.” anorexia.
c. “Would you like me to show you d) Pulse is increased from 88-96 with
where the bathroom is?” occasional skipped beat.
d. “What woke you up?”
A. This suggests that the level of
B. This option shows acceptance (key consciousness is decreasing.
concept) of this age-typical sleep
pattern (that of waking in the early 39. The nurse is conducting a lecture to a
morning). class of nursing students about advance
directives to preoperative clients. Which
36. The nurse is taking care of a of the following statement by the nurse js
multipara who is at 42 weeks of gestation correct?
and in active labor, her membranes
ruptured spontaneously 2 hours ago. a. “The spouse, but not the rest of the
While auscultating for the point of family, may override the advance
maximum intensity of fetal heart tones directive.”
before applying an external fetal monitor, b. “An advance directive is required for a
the nurse counts 100 beats per minute. “do not resuscitate” order.”
The immediate nursing action is to: c. “A durable power of attorney, a form
of advance directive, may only be
a) Start oxygen by mask to reduce fetal held by a blood relative.”
distress. d. “The advance directive may be
b) Examine the woman for signs of a enforced even in the face of
prolapsed cord. opposition by the spouse.”
c) Turn the woman on her left side to
increase placental perfusion. D. An advance directive is a form of
d) Take the woman’s radial pulse informed consent, and only a
while still auscultating the FHR. competent adult or the holder of a
durable power of attorney has the
D. Taking the mother’s pulse while right to consent or refuse treatment.
listening to the FHR will differentiate If the spouse does not hold the power
between the maternal and fetal heart of attorney, the decisions of the
rates and rule out fetal Bradycardia. holder, even if opposed by the
spouse, are enforced.

37. The nurse must instruct a client with 40. A client diagnosed with schizophrenia
glaucoma to avoid taking over-the- is shouting and banging on the door
counter medications like: leading to the outside, saying, “I need to
go to an appointment.” What is the
a. Antihistamines. appropriate nursing intervention?
b. NSAIDs.
c. Antacids. a) Tell the client that he cannot bang on
d. Salicylates. the door.
b) Ignore this behavior.
A. Antihistamines cause pupil dilation c) Escort the client going back into
and should be avoided with the room.
glaucoma. d) Ask the client to move away from the
door.

38. A male client is brought to the C. Gentle but firm guidance and
emergency department due to motor nonverbal direction is needed to
intervene when a client with Monitor clotting times and signs of
schizophrenic symptoms is being any gastrointestinal or internal
disruptive. bleeding.

41. Which of the following action is an 45. A client who undergone left
accurate tracheal suctioning technique? nephrectomy has a large flank incision.
Which of the following nursing action will
a. 25 seconds of continuous suction facilitate deep breathing and coughing?
during catheter insertion.
b. 20 seconds of continuous suction a. Push fluid administration to loosen
during catheter insertion. respiratory secretions.
c. 10 seconds of intermittent b. Have the client lie on the unaffected
suction during catheter side.
withdrawal. c. Maintain the client in high Fowler’s
d. 15 seconds of intermittent suction position.
during catheter withdrawal. d. Coordinate breathing and
C. Suctioning is only done for 10 coughing exercise with
seconds, intermittently, as the administration of analgesics.
catheter is being withdrawn.
D. Because flank incision in
42. The client’s jaw and cheekbone is nephrectomy is directly below the
sutured and wired. The nurse anticipates diaphragm, deep breathing is painful.
that the most important thing that must Additionally, there is a greater
be ready at the bedside is: incisional pull each time the person
moves than there is with abdominal
a) Suture set. surgery. Incisional pain following
b) Tracheostomy set. nephrectomy generally requires
c) Suction equipment. analgesics administration every 3-4
d) Wire cutters. hours for 24-48 hours after surgery.
Therefore, turning, coughing and
D. The priority for this client is being deep-breathing exercises should be
able to establish an airway. planned to maximize the analgesic
effects.
43. A mother is in the third stage of labor.
Which of the following signs will help the 46. The community nurse is teaching the
nurse determine the signs of placental group of mothers about the cervical
separation? mucus method of natural family
planning. Which characteristics are
a. The uterus becomes globular. typical of the cervical mucus during the
b. The umbilical cord is shortened. “fertile” period of the menstrual cycle?
c. The fundus appears at the introitus.
d. Mucoid discharge is increased. a) Absence of ferning.
b) Thin, clear, good spinnbarkeit.
A. Signs of placental separation include c) Thick, cloudy.
a change in the shape of the uterus d) Yellow and sticky.
from ovoid to globular.
B. Under high estrogen levels, during
44. After therapy with the thrombolytic the period surrounding ovulation, the
alteplase (t-PA), what observation will the cervical mucus becomes thin, clear,
nurse report to the physician? and elastic (spinnbarkeit), facilitating
sperm passage.
a) 3+ peripheral pulses.
b) Change in level of consciousness 47. A client with ruptured appendix had
and headache. surgery an hour ago and is transferred to
c) Occasional dysrhythmias. the nursing care unit. The nurse placed
d) Heart rate of 100/bpm. the client in a semi-Fowler’s position
primarily to:
B. This could indicate intracranial
bleeding. Alteplase is a thrombolytic a. Facilitate movement and reduce
enzyme that lyses thrombi and complications from immobility.
emboli. Bleeding is an adverse effect. b. Fully aerate the lungs.
c. Splint the wound.
d. Promote drainage and prevent
subdiaphragmatic abscesses. 51. The nurse is completing an
assessment to a newborn baby boy. The
D. After surgery for a ruptured appendix, nurse observes that the skin of the
the client should be placed in a semi- newborn is dry and flaking and there are
Fowler’s position to promote drainage several areas of an apparent macular
and to prevent possible rash. The nurse charts this as:
complications.
a. Icterus neonatorum
b. Multiple hemangiomas
48. Which of the following will best c. Erythema toxicum
describe a management function? d. Milia

a) Writing a letter to the editor of a C. Erythema toxicum is the normal,


nursing journal. nonpathological macular newborn
b) Negotiating labor contracts. rash
c) Directing and evaluating nursing
staff members. 52. The client is brought to the
d) Explaining medication side effects to emergency department because of
a client. serious vehicle accident. After an hour,
the client has been declared brain dead.
C. Directing and evaluation of staff is a The nurse who has been with the client
major responsibility of a nursing manager must now talk to the family about organ
donation. Which of the following
49. The parents of an infant client ask consideration is necessary?
the nurse to teach them how to
administer Cortisporin eye drops. The a) Include as many family members as
nurse is correct in advising the parents to possible.
place the drops: b) Take the family to the chapel.
c) Discuss life support systems.
a. In the middle of the lower d) Clarify the family’s
conjunctival sac of the infant’s understanding of brain death.
eye.
b. Directly onto the infant’s sclera. D. The family needs to understand what
c. In the outer canthus of the infant’s brain death is before talking about
eye. organ donation. They need time to
d. In the inner canthus of the infant’s accept the death of their family
eye. member. An environment conducive
to discussing an emotional issue is
A. The recommended procedure for needed.
administering eyedrops to any client
calls for the drops to be placed in the 53. The nurse is teaching exercises that
middle of the lower conjunctival sac. are good for pregnant women increasing
tone and fitness and decreasing lower
backache. Which of the following should
50. The nurse is assessing on the client the nurse exclude in the exercise
who is admitted due to vehicle accident. program?
Which of the following findings will help
the nurse that there is internal bleeding? a. Stand with legs apart and touch
hands to floor three times per
a) Frank blood on the clothing. day.
b) Thirst and restlessness. b. Ten minutes of walking per day with
c) Abdominal pain. an emphasis on good posture.
d) Confusion and altered of c. Ten minutes of swimming or leg
consciousness. kicking in pool per day.
d. Pelvic rock exercise and squats three
B. Thirst and restlessness indicate times a day.
hypovolemia and hypoxemia. Internal
bleeding is difficult to recognized and A. Bending from the waist in pregnancy
evaluate because it is not apparent. tends to make backache worse.
would be approximately between day
54. A client with obsessive-compulsive 13 and 17.
behavior is admitted in the psychiatric d) In a 28-day cycle, ovulation occurs 8
unit. The nurse taking care of the client days before the next period or at
knows that the primary treatment goal is about day 20. The fertile period is
to: between day 20 and the beginning of
the next period.
a) Provide distraction.
b) Support but limit the behavior. B. It is the most accurate statement of
c) Prohibit the behavior. physiological facts for a 28-day
d) Point out the behavior. menstrual cycle: ovulation at day 14,
egg life span 24 hours, sperm life
B. Support and limit setting decrease span of 72 hours. Fertilization could
anxiety and provide external control. occur from sperm deposited before
ovulation.
55. After ileostomy, the nurse expects
that the drainage appliance will be 57. Which of the following statement
applied to the stoma: describes the role of a nurse as a client
advocate?
a. When the client is able to begin self-
care procedures. a. A nurse may override clients’ wishes
b. 24 hours later, when the swelling for their own good.
subsided. b. A nurse has the moral obligation to
c. In the operating room after the prevent harm and do well for clients.
ileostomy procedure. c. A nurse helps clients gain greater
d. After the ileostomy begins to independence and self-
function. determination.
d. A nurse measures the risk and
C. The stoma drainage bag is applied in benefits of various health situations
the operating room. Drainage from while factoring in cost.
the ileostomy contains secretions
that are rich in digestive enzymes C. An advocate role encourage freedom
and highly irritating to the skin. of choice, includes speaking out for
Protection of the skin from the effects the client, and supports the client’s
of these enzymes is begun at once. best interests.
Skin exposed to these enzymes even
for a short time becomes reddened, 58. A community health nurse is
painful and excoriated. providing a health teaching to a woman
infected with herpes simplex 2. Which of
56. A female client who has a 28-day the following health teaching must the
menstrual cycle asks the community nurse include to reduce the chances of
health nurse when she get pregnant transmission of herpes simplex 2?
during her cycle. What will be the best
nursing response? a) “Abstain from intercourse until
lesions heal.”
a) It is impossible to determine the b) “Therapy is curative.”
fertile period reliably. So it is best to c) “Penicillin is the drug of choice for
assume that a woman is always treatment.”
fertile. d) “The organism is associated with later
b) In a 28-day cycle, ovulation development of hydatidiform mole.
occurs at or about day 14. The
egg lives for about 24 hours and A. Abstinence will eliminate any
the sperm live for about 72 unnecessary pain during intercourse
hours. The fertile period would and will reduce the possibility of
be approximately between day 11 transmitting infection to one’s sexual
and day 15. partner.
c) In a 28- day cycle, ovulation occurs at
or about day 14. The egg lives for 59. The nurse in the psychiatric ward
about 72 hours and the sperm live for informed the male client that he will be
about 24 hours. The fertile period attending the 9:00 AM group therapy
sessions. The client tells the nurse that
he must wash his hands from 9:00 to
9:30 AM each day and therefore he B. The registered nurse cannot delegate
cannot attend. Which concept does the the responsibility for assessment and
nursing staff need to keep in mind in evaluation of clients. The status of
planning nursing intervention for this the client in restraint requires further
client? assessment to determine if there are
additional causes for the behavior.
a. Depression underlines ritualistic
behavior.
b. Fear and tensions are often
expressed in disguised form 62. In the admission care unit, which of
through symbolic processes. the following client would the nurse give
c. Ritualistic behavior makes others immediate attention?
uncomfortable.
d. Unmet needs are discharged through a) A client who is 3 days postoperative
ritualistic behavior. with left calf pain.
b) A client who is postoperative hip
B. Anxiety is generated by group pinning who is complaining of pain.
therapy at 9:00 AM. The ritualistic c) New admitted client with chest
behavioral defense of hand washing pain.
decreases anxiety by avoiding group d) A client with diabetes who has a
therapy. glucoscan reading of 180.

60. The nurse assesses the health C. The client with chest pain may be
condition of the female client. The client having a myocardial infarction, and
tells the nurse that she discovered a immediate assessment and
lump in the breast last year and intervention is a priority.
hesitated to seek medical advice. The
nurse understands that, women who tend 63. A couple seeks medical advice in the
to delay seeking medical advice after community health care unit. A couple has
discovering the disease are displaying been unable to conceive; the man is
what common defense mechanism? being evaluated for possible problems.
The physician ordered semen analysis.
a) Intellectualization. Which of the following instructions is
b) Suppression. correct regarding collection of a sperm
c) Repression. specimen?
d) Denial.
a. Collect a specimen at the clinic, place
D. Denial is a very strong defense in iced container, and give to
mechanism used to allay the laboratory personnel immediately.
emotional effects of discovering a b. Collect specimen after 48-72
potential threat. Although denial has hours of abstinence and bring to
been found to be an effective clinic within 2 hours.
mechanism for survival in some c. Collect specimen in the morning after
instances, such as during natural 24 hours of abstinence and bring to
disasters, it may in greater pathology clinic immediately.
in a woman with potential breast d. Collect specimen at night, refrigerate,
carcinoma. and bring to clinic the next morning.

61. Which of the following situations B. Is correct because semen analysis


cannot be delegated by the registered requires that a freshly masturbated
nurse to the nursing assistant? specimen be obtained after a rest
(abstinence) period of 48-72 hours.
a. A postoperative client who is stable
needs to ambulate. 64. The physician ordered
b. Client in soft restraint who is Betamethasone to a pregnant woman at
very agitated and crying. 34 weeks of gestation with sign of
c. A confused elderly woman who needs preterm labor. The nurse expects that the
assistance with eating. drug will:
d. Routine temperature check that must
be done for a client at end of shift. a) Treat infection.
b) Suppress labor contraction. wants to know the condition of his wife.
c) Stimulate the production of How should the nurse respond to the
surfactant. husband?
d) Reduce the risk of hypertension.
a. Find out what information he
C. Betamethasone, a form of cortisone, already has.
acts on the fetal lungs to produce b. Suggest that he discuss it with his
surfactant. wife.
c. Refer him to the doctor.
65. A tracheostomy cuff is to be deflated, d. Refer him to the nurse in charge.
which of the following nursing
intervention should be implemented A. It is best to establish baseline
before starting the procedures? information first.

a. Suction the trachea and mouth. 68. A hospitalized client cannot find his
b. Have the obdurator available. handkerchief and accuses other cient in
c. Encourage deep breathing and the room and the nurse of stealing them.
coughing. Which is the most therapeutic approach
d. Do a pulse oximetry reading. to this client?

A. Secretions may have pooled above a) Divert the client’s attention.


the tracheostomy cuff. If these are b) Listen without reinforcing the
not suctioned before deflation, the client’s belief.
secretions may be aspirated. c) Inject humor to defuse the intensity.
d) Logically point out that the client is
66. A client is diagnosed with jumping to conclusions.
Tuberculosis and respiratory isolation is
initiated. This means that: C. Listening is probably the most
effective response of the four choices
a) Gloves are worn when handling the
client’s tissue, excretions, and linen. 69. After a cystectomy and formation of
b) Both client and attending nurse must an ileal conduit, the nurse provides
wear masks at all times. instruction regarding prevention of
c) Nurse and visitors must wear leakage of the pouch and backflow of the
masks until chemotherapy is urine. The nurse is correct to include in
begun. Client is instructed in the instruction to empty the urine pouch:
cough and tissue techniques.
d) Full isolation; that is, caps and gowns a. Every 3-4 hours.
are required during the period of b. Every hour.
contagion. c. Twice a day.
d. Once before bedtime.
C. Proper handling of sputum is essential
to allay droplet transference of bacilli A. Urine flow is continuous. The pouch
in the air. Clients need to be taught to has an outlet valve for easy drainage
cover their nose and mouth with every 3-4 hours. (the pouch should
tissues when sneezing or coughing. be changed every 3-5 days, or sooner
Chemotherapy generally renders the if the adhesive is loose).
client noninfectious within days to a
few weeks, usually before cultures for 70. Which telephone call from a student’s
tubercle bacilli are negative. Until mother should the school nurse take care
chemical isolation is established, of at once?
many institutions require the client to
wear a mask when visitors are in the a) A telephone call notifying the
room or when the nurse is in school nurse that the child’
attendance. Client should be in a pediatrician has informed the
well-ventilated room, without air mother that the child will need
recirculation, to prevent air cardiac repair surgery within the
contamination. next few weeks.
b) A telephone call notifying the school
67. A client with lung cancer is admitted nurse that the child’s pediatrician has
in the nursing care unit. The husband
informed the mother that the child D. Gentle aspiration of mucus helps
has head lice. maintain a patent airway, required
c) A telephone call notifying the school for effective gas exchange.
nurse that a child has a temperature
of 102ºF and a rash covering the 73. The nurse is formulating a plan of
trunk and upper extremities of the care to a client with a somatoform
body. disorder. The nurse needs to have
d) A telephone call notifying the school knowledge of which psychodynamic
nurse that a child underwent an principle?
emergency appendectomy during the
previous night. a. The symptoms of a somatoform
disorder are an attempt to adjust
C. A high fever accompanied by a body to painful life situations or to
rash could indicate that the child has cope with conflicting sexual,
a communicable disease and would aggressive, or dependent
have exposed other students to the feelings.
infection. The school nurse would b. The major fundamental mechanism is
want to investigate this telephone regression.
call immediately so that plans could c. The client’s symptoms are imaginary
be instituted to control the spread of and the suffering is faked.
such infection. d. An extensive, prolonged study of the
symptoms will be reassuring to the
71. Which of the following signs and client, who seeks sympathy, attention
symptoms that require immediate and love.
attention and may indicate most serious
complications during pregnancy? A. Somatoform disorders provide a way
of coping with conflicts.
a. Severe abdominal pain or fluid
discharge from the vagina. 74. An infant is brought to the health
b. Excessive saliva, “bumps around the care clinic for three immunizations at the
areolae, and increased vaginal same time. The nurse knows that
mucus. hepatitis B, DPT, and Haemophilus
c. Fatigue, nausea, and urinary influenzae type B immunizations should:
frequency at any time during
pregnancy. a) Be drawn in the same syringe and
d. Ankle edema, enlarging varicosities, given in one injection.
and heartburn. b) Be mixed and inject in the same sites.
c) Not be mixed and the nurse must
A. Severe abdominal pain may indicate give three injections in three
complications of pregnancy such as sites.
abortion, ectopic pregnancy, or d) Be mixed and the nurse must give the
abruption placenta; fluid discharge injection in three sites.
from the vagina may indicate
premature rupture of the membrane. C. Immunization should never be mixed
together in a syringe, thus
72. The nurse is assessing the newborn necessitating three separate
boy. Apgar scores are 7 and 9. The injections in three sites. Note: some
newborn becomes slightly cyanotic. What manufacturers make a premixed
is the initial nursing action? combination of immunization that is
safe and effective.
a) Elevate his head to promote gravity
drainage of secretions. 75. A female client with cancer has
b) Wrap him in another blanket, to radium implants. The nurse wants to
reduce heat loss. maintain the implants in the correct
c) Stimulate him to cry,, to increase position. The nurse should position the
oxygenation. client:
d) Aspirate his mouth and nose with
bulb syringe. a. Flat in bed.
b. On the side only.
c. With the foot of the bed elevated.
d. With the head elevated 45-degrees a. Assess the child’s injuries.
(semi-Fowler’s). b. Report the incident to protective
agencies.
A. Clients with radioactive implants c. Refer the family to appropriate
should be positioned flat in bed to support group.
prevent dislodgement of the vaginal d. Assist the family to identify stressors
packing. The client may roll to the and use of other coping mechanisms
side for meals but the upper body to prevent further incidents.
should not be raised more than 20
degrees. A. Assessment of physical injuries (like
bruises, lacerations, bleeding and
76. The nurse wants to know if the fractures) is the first priority.
mother of a toddler understands the
instructions regarding the administration 79. The nurse in the neonatal care unit is
of syrup of ipecac. Which of the following supervising the actions of a certified
statement will help the nurse to know nursing assistant in giving care to the
that the mother needs additional newborns. The nursing assistant
teaching? mistakenly gives a formula feeding to a
newborn that is on water feeding only.
a) “I’ll give the medicine if my child The nurse is responsible for the mistake
gets into some toilet bowl of the nursing assistant:
cleaner.”
b) “I’ll give the medicine if my child gets a) Always, as a representative of the
into some aspirin.” institution.
c) “I’ll give the medicine if my child gets b) Always, because nurses who
into some plant bulbs.” supervise less-trained individuals are
d) “I’ll give the medicine if my child gets responsible for their mistakes.
into some vitamin pills.” c) If the nurse failed to determine
whether the nursing assistant
A. Syrup of ipecac is not administered was competent to take care of
when the ingested substances is the client.
corrosive in nature. Toilet bowl d) Only if the nurse agreed that the
cleaners, as a collective whole, are newborn could be fed formula.
highly corrosive substances. If the
ingested substance “burned” the C. The nurse who is supervising others
esophagus going down, it will “burn” has a legal obligation to determine
the esophagus coming back up when that they are competent to perform
the child begins to vomit after the assignment, as well as legal
administration of syrup of ipecac. obligation to provide adequate
supervision.
77. To assess if the cranial nerve VII of
the client was damaged, which changes
would not be expected? 80. The nurse is assigned to care for a
client with urinary calculi. Fluid intake of
a. Drooling and drooping of the mouth. 2L/day is encouraged to the client. the
b. Inability to open eyelids on primary reason for this is to:
operative side.
c. Sagging of the face on the operative a. Reduce the size of existing stones.
side. b. Prevent crystalline irritation to the
d. Inability to close eyelid on operative ureter.
side. c. Reduce the size of existing stones
d. Increase the hydrostatic pressure
B. Inability to open eyelids on operative in the urinary tract.
side is seen with cranial nerve III
damage. D. Increasing hydrostatic pressure in the
urinary tract will facilitate passage of
78. The community health nurse makes a the calculi.
home visit to a family. During the visit,
the nurse observes that the mother is 81. The nurse is counseling a couple in
beating her child. What is the priority their mid 30’s who have been unable to
nursing intervention in this situation? conceive for about 6 months. They are
concerned that one or both of them may d) “Try to sleep. You need the rest before
be infertile. What is the best advice the tomorrow’s test.”
nurse could give to the couple?
A. Acknowledging a feeling tone is the
a) “it is no unusual to take 6-12 most therapeutic response and
months to get pregnant, provides a broad opening for the
especially when the partners are client to elaborate feelings.
in their mid-30s. Eat well,
exercise, and avoid stress.” 84. In the hospital lobby, the registered
b) “Start planning adoption. Many nurse overhears a two staff members
couples get pregnant when they are discussing about the health condition of
trying to adopt.” her client. What would be the appropriate
c) “Consult a fertility specialist and start action for the registered nurse to take?
testing before you get any older.”
d) “Have sex as often as you can, a. Join in the conversation, giving her
especially around the time of input about the case.
ovulation, to increase your chances of b. Ignore them, because they have the
pregnancy.” right to discuss anything they want
to.
A. Infertility is not diagnosed until c. Tell them it is not appropriate to
atleast 12months of unprotected discuss such things.
intercourse has failed to produce a d. Report this incident to the nursing
pregnancy. Older couples will supervisor.
experience a longer time to get
pregnant. C. The behavior should be stopped. The
first is to remind the staff that
82. The nurse is caring for a cient who Is confidentiality maybe violated.
a retired nurse. A 24-hour urine collection
for Creatinine clearance is to be done. 85. The client has had a right-sided
The client tells the nurse, “I can’t cerebrovascular accident. In transferring
remember what this test is for.” The best the client from the wheelchair to bed, in
response by the nurse is: what position should a client be placed to
facilitate safe transfer?
a. “It provides a way to see if you are
passing any protein in your urine.” a) Weakened (L) side of the cient next to
b. “It tells how well the kidneys bed.
filter wastes from the blood.” b) Weakened (R) side of the client next
c. “It tells if your renal insufficiency has to bed.
affected your heart.” c) Weakened (L) side of the client
d. “The test measures the number of away from bed.
particles the kidney filters.” d) Weakened (R) side of the cient away
from bed.
B. Determining how well the kidneys
filter wastes states the purpose of a C. With a right-sided cerebrovascular
Creatinine clearance test. accident the client would have left-
sided hemiplegia or weakness. The
client’s good side should be closest to
83. The nurse observes the female client the bed to facilitate the transfer.
in the psychiatric ward that she is having
a hard time sleeping at night. The nurse 86. The child client has undergone hip
asks the client about it and the client surgery and is in a spica cast. Which of
says, “I can’t sleep at night because of the following toy should be avoided to be
fear of dying.” What is the best initial in the child’s bed?
nursing response?
a. A toy gun.
a) “It must be frightening for you to b. A stuffed animal.
feel that way. Tell me more about c. A ball.
it.” d. Legos.
b) “Don’t worry, you won’t die. You are
just here for some test.” D. Legos are small plastic building
c) “Why are you afraid of dying?” blocks that could easily slip under the
child’s cast and lead to a break in
skin integrity and even infection. C. The recommended dosage of
Pencils, backscratchers, and marbles tetracycline is 25-50mg/kg/day. If the
are some other narrow or small items child weighs 20kg and the maximum
that could easily slip under the child’s dose is 50mg/kg, this would indicate
cast and lead to a break in skin a total daily dose of 1000mg of
integrity and infection. tetracycline. In this case, the child is
being given this medication four
87. The LPN/LVN asks the registered times a day. Therefore the maximum
nurse why oxytocin (Pitocin), 10 units (IV single dose that can be given is
or IM) must be given to a client after 250mg (1000 mg of tetracycline
birth fo the fetus. The nurse is correct to divided by four doses.)
explain that oxytocin:
90. The nurse is completing an obstetric
a) Minimizes discomfort from history of a woman in labor. Which event
“afterpains.” in the obstetric history will help the nurse
b) Suppresses lactation. suspects dysfunctional labor in the
c) Promotes lactation. current pregnancy?
d) Maintains uterine tone.
a. Total time of ruptured membranes
D. Oxytocin (Pitocin) is used to maintain was 24 hours with the second birth.
uterine tone. b. First labor lasting 24 hours.
c. Uterine fibroid noted at time of
88. The nurse in the nursing care unit is cesarean delivery.
aware that one of the medical staff d. Second birth by cesarean for face
displays unlikely behaviors like confusion, presentation.
agitation, lethargy and unkempt
appearance. This behavior has been C. An abnormality in the uterine muscle
reported to the nurse manager several could reduce the effectiveness of
times, but no changes observed. The uterine contractions and lengthen the
nurse should: duration of subsequent labors.

a. Continue to report observations of 91. The nurse is planning to talk to the


unusual behavior until the problem is client with an antisocial personality
resolved. disorder. What would be the most
b. Consider that the obligation to therapeutic approach?
protect the patient from harm
has been met by the prior reports a) Provide external controls.
and do nothing further. b) Reinforce the client’s self-concept.
c. Discuss the situation with friends who c) Give the client opportunities to test
are also nurses to get ideas . reality.
d. Approach the partner of this medical d) Gratify the client’s inner needs.
staff member with these concerns.
A. Personality disorders stem from a
B. The submission of reports about weak superego, implying a lack of
incidents that expose clients to harm adequate controls.
does not remove the obligation to
report ongoing behavior as long as 92. The nurse is teaching a group of
the risk to the client continues. women about fertility awareness, the
nurse should emphasize that basal body
89. The physician ordered tetracycline PO temperature:
qid to a child client who weights 20kg.
The recommended PO tetracycline dose a. Can be done with a mercury
is 25-50 mg/kg/day. What is the thermometer but no a digital one.
maximum single dose that can be safely b. The average temperature taken each
administered to this child? morning.
c. Should be recorded each morning
a) 1g before any activity.
b) 500 mg d. Has a lower degree of accuracy in
c) 250 mg predicting ovulation than the cervical
d) 125 mg mucus test.
95. A male client tells the nurse that
C. The basal body temperature is the there is a big bug in his bed. The most
lowest body temperature of a healthy therapeutic nursing response would be:
person that is taken immediately
after waking and before getting out A. Silence.
of bed. The BBT usually varies from B. “Where’s the bug? I’ll kill it for you.”
36.2 ºC to 36.3ºC during menses and C. “I don’t see a bug in your bed,
for about 5-7 days afterward. About but you seem afraid.”
the time of ovulation, a slight drop in D. “You must be seeing things.”
temperature may be seen, after
ovulation in concert with the C. This response does not contradict the
increasing progesterone levels of the client’s perception, is honest, and
early luteal phase, the BBT rises 0.2- shows empathy.
0.4 ºC. This elevation remains until 2-
3 days before menstruation, or if 96. A pregnant client in late pregnancy is
pregnancy has occurred. complaining of groin pain that seems
worse on the right side. Which of the
93. The nursing applicant has given the following is the most likely cause of it?
chance to ask questions during a job
interview at a local hospital. What should a. Beginning of labor.
be the most important question to ask b. Bladder infection.
that can increase chances of securing a c. Constipation.
job offer? d. Tension on the round ligament.

a) Begin with questions about client D. Tension on round ligament occurs


care assignments, advancement because of the erect human posture
opportunities, and continuing and pressure exerted by the growing
education. fetus.
b) Decline to ask questions, because 97. The nurse is conducting a lecture to a
that is the responsibility of the group of volunteer nurses. The nurse is
interviewer. correct in imparting the idea that the
c) Ask as many questions about the Good Samaritan law protects the nurse
facility as possible. from a suit for malpractice when:
d) Clarify information regarding salary,
benefits, and working hours first, a) The nurse stops to render emergency
because this will help in deciding aid and leaves before the ambulance
whether or not to take the job. arrives.
b) The nurse acts in an emergency at his
A. This choice implies concern for client or her place of employment.
care and self-improvement. c) The nurse refuses to stop for an
emergency outside of the scope of
94. The nurse advised the pregnant employment.
woman that smoking and alcohol should d) The nurse is grossly negligent at
be avoided during pregnancy. The nurse the scene of an emergency.
takes into account that the developing
fetus is most vulnerable to environment D. The Good Samaritan Law does not
teratogens that cause malformation impose a duty to stop at the scene of
during: an emergency outside of the scope of
employment, therefore nurses who
a. The entire pregnancy. do not stop are not liable for suit.
b. The third trimester.
c. The first trimester. 98. A woman is hospitalized with mild
d. The second trimester. preeclampsia. The nurse is formulating a
plan of care for this client, which nursing
C. The first trimester is the period of care is least likely to be done?
organogenesis, that is, cell
differentiation into the various a. Deep-tendon reflexes once per shift.
organs, tissues, and structures. b. Vital signs and FHR and rhythm q4h
while awake.
c. Absolute bed rest.
d. Daily weight.
C. Although reducing environment
stimuli and activity is necessary for a
woman with mild preeclampsia, she
will most probably have bathroom
privileges.

99. While feeding a newborn with an


unrepaired cardiac defect, the nurse
keeps on assessing the condition of the
client. The nurse notes that the
newborn’s respiration is 72 breaths per
minute. What would be the initial nursing
action?

a) Burp the newborn.


b) Stop the feeding.
c)
d)
Continue the feeding.
Notify the physician.
PNLE Pediatric
B. A normal respiratory rate for a Nursing Exam
newborn is 30-40 breaths per minute.
Situation 1: Raphael, a 6 year’s
100. A client who undergone old prep pupil is seen at the
appendectomy 3 days ago is scheduled school clinic for growth and
for discharge today. The nurse notes that development monitoring
the client is restless, picking at
bedclothes and saying, “I am late on my (Questions 1-5)
appointment,” and calling the nurse by
1. Which of the following is characterized
the wrong name. The nurse suspects:
the rate of growth during this period?
a. Panic reaction.
most rapid period of growth
b. Medication overdose.
a decline in growth rate
c. Toxic reaction to an antibiotic.
growth spurt
d. Delirium tremens.
slow uniform growth rate
D. The behavior described is likely to be
B. a decline in growth rate. During the
symptoms of delirium tremens, or
Preschooler stage growth is very
alcohol withdrawal (often
minimal. Weight gain is only 4.5lbs
unsuspected on a surgical unit.)
(2kgs) per year and Height is 3.5in
(6-8cm) per year.
Review:
Most rapid growth and
development- Infancy
Slow growth- Toddler hood and
Preschooler
Slower growth- School age
Rapid growth- Adolescence

2. In assessing Raphael’s growth and


development, the nurse is guided by
principles of growth and development.
Which is not included?

All individuals follow cephalo-caudal and


proximo-distal
Different parts of the body grows at
different rate
All individual follow standard growth rate
Rate and pattern of growth can be
modified
5. Based on Kohlberg’s theory, what is
D. Rate and pattern of growth can be the stage of moral development of
modified. Growth and development Raphael?
occurs in cephalo-caudal meaning
development occurs through out the Punishment-obedience
body’s axis. Example: the child must “good boy-Nice girl”
be able to lift the head before he is naïve instrumental orientation
able to lift his chest. Proximo-distal is social contact
development that progresses from
center of the body to the extremities. C. naïve instrumental
Example: a child first develops arm orientation. According to Kohlber, a
movement before fine-finger preschooler is under Pre-conventional
movement. Different parts of the where a child learns about
body grows at different range instrumental purpose and exchange,
because some body tissue mature that is they will something do for
faster than the other such as the another if that that person does
neurologic tissues peaks its growth something with the child in return.
during the first years of life while the Letter A is applicable for Toddlers and
genital tissue doesn’t till puberty. letter B is for a School age child
Also G&D is predictable in the
sequence which a child normally Situation 2 Baby boy Lacson
precedes such as motor skills and
delivered at 36 weeks
behavior. Lastly G&D can never be
modified . gestation weighs 3,400 gm and
height of 59 cm (6-10)
3. What type of play will be ideal for
Raphael at this period? 6. Baby boy Lacson’s height is

Make believe Long


Hide and seek Short
Peek-a-boo Average
Building blocks Too short

A. Make believe. Make believe is most A. Long. The average length of full-term
appropriate because it enhances the babies at birth is 20 in. (51 cm),
imitative play and imagination of the although the normal range is 46 cm
preschooler. C and D are for infants (18 in.) to 56 cm (22 in.).
while letter A is B is recommended
for schoolers because it enhances 7. Growth and development in a child
competitive play. progresses in the following ways EXCEPT

4. Which of the following information From cognitive to psychosexual


indicate that Raphael is normal for his From trunk to the tip of the extremities
age? From head to toe
From general to specific
Determine own sense self
Develop sense of whether he can trust A. From cognitive to
the world psychosexual. Growth and
Has the ability to try new things development occurs in cephalo-
Learn basic skills within his culture caudal (head to toe), proximo-distal
(trunk to tips of the extremities and
C. Has the ability to try new general to specific, but it doesn’t
things. Erickson defines the occurs in cognitive to psychosexual
developmental task of a preschool because they can develop at the
period is learning Initiative vs. Guilt. same time.
Children can initiate motor activities
of various sorts on their own and no 8. As described by Erikson, the major
longer responds to or imitate the psychosexual conflict of the above
actions of other children or of their situation is
parents.
Autonomy vs. Shame and doubt
Industry vs. Inferiority
Trust vs. mistrust A. Mother’s breast. Place it at the
Initiation vs. guilt mother’s breast for latch-on. (Note:
for NSD breast feed ASAP while for CS
C. Trust vs. mistrust. According to delivery, breast feed after 4 hours)
Erikson, children 0-18 months are
under the developmental task of 12. The baby’s mother is RH(-). Which of
Trust vs. Mistrust. the following laboratory tests will
probably be ordered for the newborn?
9. Which of the following is true about
Mongolian Spots? Direct Coomb’s
Indirect Coomb’s
Disappears in about a year Blood culture
Are linked to pathologic conditions Platelet count
Are managed by tropical steroids
Are indicative of parental abuse A. Direct Coomb’s. Coomb’s test is the
test to determine if RH antibodies are
A. Disappears in about a year. Mongolian present. Indirect Coomb’s is done to
spots are stale grey or bluish patches the mother and Direct Coomb’s is the
of discoloration commonly seen one don’t to the baby. Blood culture
across the sacrum or buttocks due to and Platelet count doesn’t help
accumulation of melanocytes and detect RH antibodies.
they disappears in 1 year. They are
not linked to steroid use and 13. Hypothermia is common in newborn
pathologic conditions. because of their inability to control heat.
The following would be an appropriate
10. Signs of cold stress that the nurse nursing intervention to prevent heat loss
must be alert when caring for a Newborn except:
is:
Place the crib beside the wall
Hypothermia Doing Kangaroo care
Decreased activity level By using mechanical pressure
Shaking Drying and wrapping the baby
Increased RR
A. Place the crib beside the wall. Placing
D. Increased RR. Hypothermia is the crib beside the wall is
inaccurate cause normally, inappropriate because it can provide
temperature of a newborn drop, Also heat loss by radiation. Doing
a child under cold stress will kick and Kangaroo care or hugging the baby,
cry to increase the metabolic rate mechanical pressure or incubators
thereby increasing heat so B isn’t a and drying and wrapping the baby
good choice. A newborn doesn’t have will help conserve heat.
the ability to shiver, so letter B and C
is wrong. A newborn will increase its 14. The following conditions are caused
RR because the NB will need more by cold stress except
oxygen because of too much activity.
Hypoglycemia
Situation 3 Nursing care after Increase ICP
delivery has an important Metabolic acidosis
Cerebral palsy
aspect in every stages of
delivery B. Increase ICP. Hypoglycemia may
occur due to increase metabolic rate,
11. After the baby is delivered, the cord and because of newborns are born
was cut between two clamps using a slightly acidic, and they catabolize
sterile scissors and blade, then the baby brownfat which will produce ketones
is placed at the: which is an acid will cause metabolic
acidosis. Also a NB with severe
Mother’s breast hypothermia is in high risk for
Mother’s side kernicterus (too much bilirubin in the
Give it to the grandmother brain) can lead to Cerebral palsy.
Baby’s own mat or bed
There is no connection in the increase Atrial Septal defect
of ICP with hypothermia. (NOTE: Transposition of great arteries
pathognomonic sign of Kernicterus in Pulmunary Stenosis
adult- asterexis, or involuntary
flapping of the hand.) B. Atrial Septal defect. Foramen ovale is
the opening between two Atria so, if
15. During the feto-placental circulation, its will not close Atrial Septal defect
the shunt between two atria is called can occur.

Ductus venosous Situation 4 Children are


Foramen Magnum vulnerable to some minor
Ductus arteriosus
Foramen Ovale health problems or injuries
hence the nurse should be able
D. Foramen Ovale. Foramen ovale is to teach mothers to give
opening between two atria, Ductus appropriate home care.
venosus is the shunt from liver to the
inferior vena cava, and your Ductus 18. A mother brought her child to the
Arteriosus is the shunt from the clinic with nose bleeding. The nurse
pulmonary artery to the aorta. showed the mother the most appropriate
position for the child which is:
16. What would cause the closure of the
Foramen ovale after the baby had been Sitting up
delivered? With low back rest
With moderate back rest
Decreased blood flow Lying semi flat
Shifting of pressures from right
side to the left side of the heart A. Sitting up. The correct position is
Increased PO2 making the child having an upright
Increased in oxygen saturation sitting position with the head slightly
tilted forward. This position will
B. Shifting of pressures from right side to minimize the amount of blood
the left side of the heart. During feto- pressure in nasal vessels and keep
placental circulation, the pressure in blood moving forward not back into
the heart is much higher in the right the nasopharynx, which will have the
side, but once breathing/crying is choking sensation and increase risk
established, the pressure will shift of aspiration. Choices b, c, d, are
from the R to the L side, and will inappropriate cause they can cause
facilitate the closure of Foramen blood to enter the nasopharynx.
Ovale. (Note: that is why you should
position the NB in R side lying 19. A common problem in children is the
position to increase pressure in the L inflammation of the middle ear. This is
side of the heart.) related to the malfunctioning of the:
Review:
Increase PO2-> closure of ductus Tympanic membrane
arteriosus Eustachian tube
Decreased bloodflow -> closure Adenoid
of the ductus venosus Nasopharynx
Circulation in the lungs is
initiated by -> lung expansion B. Eustachian tube. This is because
and pulmonary ventilation children has short, horizontal
What will sustain 1st breath-> Eustachian tubes. The dysfunction in
decreased artery pressure the Eustachian tube enables bacterial
What will complete circulation-> invasion of the middle ear and
cutting of the cord obstructs drainage of secretions.

17. Failure of the Foramen Ovale to close 20. For acute otitis media, the treatment
will cause what Congenital Heart is prompt antibiotic therapy. Delayed
Disease? treatment may result in complications of:

Total anomalous Pulmunary Artery Tonsillitis


Eardrum Problems
Brain damage 24. Which of the following statements
Diabetes mellitus would the nurse expects a 5-year old boy
to say whose pet gerbil just died
C. Brain damage. One of the
complication of recurring acute otitis “The boogieman got him”
media is risk for having Meningitis, “He’s just a bit dead”
thereby causing possible brain “Ill be good from now own so I wont die
damage. That is why patient must like my gerbil”
follow a complete treatment regimen “Did you hear the joke about…”
and follow up care. A,B and D are not
complications of AOM. B. “He’s just a bit dead”. A 5 y/o views
death in “degrees”, so the child most
21. When assessing gross motor likely will say that “he is just a bit
development in a 3 year old, which of the dead”. Personification of death like
following activities would the nurse boogeyman occurs in ages 7 to 9 as
expect to finds? well as denying death can if they will
be good. Denying death using jokes
Riding a tricycle and attributing life qualities to death
Hopping on one foot occurs during age 3-5.
Catching a ball
Skipping on alternate foot. 25. When assessing the fluid and
electrolyte balance in an infant, which of
A. Riding a tricycle. Answer is A, riding a the following would be important to
tricycle is appropriate for a 3 y/o remember?
child. Hopping on one foot can be
done by a 4 y/o child, as well as Infant can concentrate urine at an adult
catching and throwing a ball over level
hand. Skipping can be done by a 5 The metabolic rate of an infant is slower
y/o. than in adults
Infants have more intracellular water
22. When assessing the weight of a 5- that adult do
month old, which of the following Infant have greater body surface
indicates healthy growth? area than adults

Doubling of birth weight D. Infant have greater body surface area


Tripling of birth weight than adults. Infants have greater
Quadrupling of birth weight body surface area than adult,
Stabilizing of birth weight increasing their risk to F&E
imbalances. Also infants cant
A. Doubling of birth weight. During the concentrate a urine at an adult level
first 6 months of life the weight from and their metabolic rate, also called
birth will be doubled and as soon as water turnover, is 2 to 3 times higher
the baby reaches 1 year, its birth than adult. Plus more fluids of the
weight is tripled. infants are at the ECF spaces not in
the ICF spaces.
23. An appropriate toy for a 4 year old
child is: 26. When assessing a child with aspirin
overdose, which of the following will be
Push-pull toys expected?
Card games
Doctor and nurse kits Metabolic alkalosis
Books and Crafts Respiratory alkalosis
Metabolic acidosis
C. Doctor and nurse kits. Letter C is Respiratory acidosis
appropriate because it will enhance
the creativity and imagination of a C. Metabolic acidosis. Remember that
pre-school child. Letter B and D are Aspirin is acid (Acetylsalicylic ACID).
inappropriate because they are too
complex for a 4 y/o. Push-pull toys
are recommended for infants.
27. Which of the following is not a
possible systemic clinical manifestation 30. Which of the following is an
of severe burns? inappropriate interventions when caring
for a child with HIV?
Growth retardation
Hypermetabolism Teaching family about disease
Sepsis transmission
Blisters and edema Offering large amount of fresh
fruits and vegetables
D. Blisters and edema. The question was Encouraging child to perform at optimal
asking for a SYSTEMIC clinical level
manifestation, Letters A,B and C are Teach proper hand washing technique
systemic manifestations while
Blisters and Edema weren’t. B. Offering large amount of fresh fruits
and vegetables. A child with HIV is
28. When assessing a family for potential immunocompromised. Fresh fruits
child abuse risks, the nurse would and vegetables, which may be
observe for which of the following? contaminated with organisms and
pesticides can be harmful, if not fatal
Periodic exposure to stress to the child, therefore these items
Low socio-economic status should be avoided.
High level of self esteem
Problematic pregnancies Situation 5 Agata, 2 years old
is rushed to the ER due to
D. Problematic pregnancies. Typical
factors that may be risk for Child cyanosis precipitated by
abuse are problematic pregnancies, crying. Her mother observed
chronic exposure to stress not that after playing she gets
periodic, low level of self esteem not tired. She was diagnosed with
high level. Also child abuse can
happen in all socio-economic status Tetralogy of Fallot.
not just on low socio-economic status 31. The goal of nursing care fro Agata is
to:
29. Which of the following is a possible
indicator of Munchausen syndrome by
Prevent infection
proxy type of child abuse?
Promote normal growth and
development
Bruises found at odd locations, with Decrease hypoxic spells
different stages of healing Hydrate adequately
STD’s and genital discharges
Unexplained symptoms of diarrhea, C. Decrease hypoxic spells. The correct
vomiting and apnea with no answer is letter C. Though letter B
organic basis would be a good answer too, this goal
Constant hunger and poor hygiene is too vague and not specific. Nursing
interventions will not solely promote
C. Unexplained symptoms of diarrhea, normal G&D unless he will undergo
vomiting and apnea with no organic surgical repair. So decreasing Hypoxic
basis. Munchausen syndrome by Spells is more SMART. Letter A and D
Proxy is the fabrication or are inappropriate.
inducement of an illness by one
person to another person, usually 32. The immediate nursing intervention
mother to child. It is characterized by for cyanosis of Agata is:
symptoms such as apnea and
siezures, which may be due to
Call up the pediatrician
suffocation, drugs or poisoning,
Place her in knee chest position
vomiting which can be induced with
Administer oxygen inhalation
poisons and diarrhea with the use of
Transfer her to the PICU
laxatives. Letter A can be seen in a
Physical abuse, Letter B for sexual
B. Place her in knee chest position. The
abuse and Letter C is for Physical
immediate intervention would be to
Neglect.
place her on knee-chest or
“squatting” position because it traps Separation anxiety begin at: 9
blood into the lower extremities. months
Though also letter C would be a good Peaks: 18 months
choice but the question is asking for
“Immediate” so letter B is more 35. When Agata was brought to the OR,
appropriate. Letter A and D are her parents where crying. What would be
incorrect because its normal for a the most appropriate nursing diagnosis?
child who have ToF to have hypoxic or
“tets” spells so there is no need to Infective family coping r/t situational
transfer her to the NICU or to alert crisis
the Pediatrician. Anxiety r/t powerlessness
Fear r/t uncertain prognosis
33. Agata was scheduled for a palliative Anticipatory grieving r/t gravity of
surgery, which creates anastomosis of child’s physical status
the subclavian artery to the pulmonary
artery. This procedure is: D. Anticipatory grieving r/t gravity of
child’s physical status. In this item
Waterston-Cooley letter A and be are inappropriate
Raskkind Procedure response so remove them. The
Coronary artery bypass possible answers are C and D. Fear
Blalock-Taussig defined as the perceived threat (real
or imagined) that is consciously
D. Blalock-Taussig. Blalock-Taussig recognized as danger (NANDA) is
procedure its just a temporary or applicable in the situation but its
palliative surgery which creates a defining characteristics are not
shunt between the aorta and applicable. Crying per se can not be a
pulmonary artery so that the blood subjective cue to signify fear, and
can leave the aorta and enter the most of the symptoms of fear in
pulmonary artery and thus NANDA are physiological. Anticipatory
oxygenating the lungs and return to grieving on the other hand are
the left side of the heart, then to the intellectual and EMOTIONAL
aorta then to the body. This responses based on a potential loss.
procedure also makes use of the And remember that procedures like
subclavian vein so pulse is not this cannot assure total recovery. So
palpable at the right arm. The full letter D is a more appropriate Nursing
repair for ToF is called the Brock diagnosis
procedure. Raskkind is a palliative 36. Which of the following respiratory
surgery for TOGA. condition is always considered a medical
emergency?
34. Which of the following is not an
indicator that Agata experiences Laryngeotracheobronchitis (LTB)
separation anxiety brought about her Epiglottitis
hospitalization? Asthma
Cystic Fibrosis
Friendly with the nurse
Prolonged loud crying, consoled only by B. Epiglottitis. Acute and sever
mother inflammation of the epiglottis can
Occasional temper tantrums and always cause life threatening airway
says NO obstruction, that is why its always
Repeatedly verbalizes desire to go home treated as a medical emergency. NSG
intervention : Prepare tracheostomy
A. Friendly with the nurse. Because set at bed side. LTB, can also cause
toddlers views hospitalization is airway obstruction but its not an
abandonment, separation anxiety is emergency. Asthma is also not an
common. Its has 3 phases: PDD emergency. CF is a chronic disease,
(parang c puff daddy LOL) 1. Protest so its not a medical emergency
2. despair 3. detachment (or denial).
Choices B, C, D are usually seen in a 37. Which of the following statements by
child with separation anxiety (usually the family of a child with asthma
in the protest stage). indicates a need for additional teaching?
REVIEW:
“We need to identify what things
triggers his attacks” C. Expanded program on immunization
“He is to use bronchodilator inhaler
before steroid inhaler” 40. One important principle of the
“We’ll make sure he avoids exercise immunization program is based on?
to prevent asthma attacks”
“he should increase his fluid intake Statistical occurrence
regularly to thin secretions” Epidemiologic situation
Cold chain management
C. “We’ll make sure he avoids exercise Surveillance study
to prevent asthma
attacks”. Asthmatic children don’t B. Epidemiologic situation. Letters A, C
have to avoid exercise. They can and D are not included in the
participate on physical activities as principles of EPI.
tolerated. Using a bronchodilator
before administering steroids is 41. The main element of immunization
correct because steroids are just anti- program is one of the following?
inflammatory and they don’t have
effects on the dilation Information, education and
of the bronchioles. OF course letters A communication
and B are obviously correct. Assessment and evaluation of the
program
38. Which of the following would require Research studies
careful monitoring in the child with ADHD Target setting
who is receiving Methylphenidate
(Ritalin)? 42. What does herd immunity means?

Dental health Interruption of transmission


Mouth dryness All to be vaccinated
Height and weight Selected group for vaccination
Excessive appetite Shorter incubation\

C. Height and weight. Dental problems 43. Measles vaccine can be given
are more likely to occur in children simultaneously. What is the combined
under going TCA therapy. Mouth vaccine to be given to children starting at
dryness is a expected side effects of 15 months?
Ritalin since it activates the SNS. Also
loss of appetite is more likely to MCG
happen, not increase in appetite. The MMR
correct answer is letter C, because BCG
Ritalin can affect the child’s G&D. BBR
Intervention: medication “holidays or
vacation”. (This means during B. MMR. MMR or Measles, Mumps,
weekends or holidays or school Rubella is a vaccine furnished in one
vacations, where the child wont be in vial and is routinely given in one
school, the drug can be withheld.) injection (Sub-Q). It can be given at
15 months but can also be given as
Situation 6 Laura is assigned early as 12th month.
as the Team Leader during the
immunization day at the RHU
39. What program for the DOH is
launched at 1976 in cooperation with
WHO and UNICEF to reduce morbidity
and mortality among infants caused by
immunizable disease?

Patak day
Immunization day on Wednesday
Expanded program on immunization
Bakuna ng kabtaan
Situation 7: Braguda brought B. PD no. 6 Presidential Proclamation no.
her 5-month old daughter in 6 (April 3, 1986) is the “Implementing
a United Nations goal on Universal
the nearest RHU because her Child Immunization by 1990”. PD 996
baby sleeps most of the time, (September 16, 1976) is “providing
with decreased appetite, has for compulsory basic immunization
for infants and children below 8 years
colds and fever for more than a
of age. PD no. 46 (September 16,
week. The physician diagnosed 1992) is the “Reaffirming the
pneumonia. commitment of the Philippines to the
universal Child and Mother goal of
44. Based on this data given by Braguda, the World Health Assembly. RA 9173
you can classify Braguda’s daughter to is of course the “Nursing act of 2002”
have:
47. Braguda asks you about Vitamin A
Pneumonia: cough and colds supplementation. You responded that
Severe pneumonia giving Vitamin A starts when the infant
Very severe pneumonia reaches 6 months and the first dose is”
Pneumonia moderate
200,000 “IU”
B. Severe pneumonia. For a child aging 100,000 “IU”
2months up to 5 years old can be 500,000 “IU”
classified to have sever pneumonia 10,000 “IU”
when he have any of the following
danger signs: B. 100,000 “IU”. An infant aging 6-11
Not able to drink months will be given Vitamin
Convulsions supplementation of 100, 000 IU and
Abnormally sleepy or difficult to for Preschoolers ages 12-83 months
wake 200,000 “IU” will be given.
Stridor in calm child or
Severe under-nutrition 48. As part of CARI program, assessment
of the child is your main responsibility.
45. For a 3-month old child to be You could ask the following question to
classified to have Pneumonia (not the mother except:
severe), you would expect to find RR of:
“How old is the child?”
60 bpm “IS the child coughing? For how long?”
40 bpm “Did the child have chest
70 bpm indrawing?”
50 pbm “Did the child have fever? For how
long?”
D. 50 pbm. A child can be classified to
have Pneumonia (not severe) if: C. “Did the child have chest
the young infant is less than 2 indrawing?”. The CARI program of the
months- 60 bpm or more DOH includes the “ASK” and “LOOK,
if the child is 2 months up to less LISTEN” as part of the assessment of
than 12 months- 50 bpm or more the child who has suspected
if the child is 12 months to 4 y/o- 40 Pneumonia. Choices A, B and D are
bpm or more included in the “ASK” assessment
while Chest indrawings is included in
46. You asked Braguda if her baby the “LOOK, LISTEN” and should not
received all vaccines under EPI. What be asked to the mother.
legal basis is used in implementing the
UN’s goal on Universal Child 49. A newborn’s failure to pass
Immunization? meconium within 24 hours after birth
may indicate which of the following?
PD no. 996
PD no. 6 Aganglionic Mega colon
PD no. 46 Celiac disease
RA 9173 Intussusception
Abdominal wall defect
abducted. The sound is produced
A. Aganglionic Mega colon. Failure to when the femoral head enters the
pass meconium of Newborn during acetabulum. Letter A is wrong
the first 24 hours of life may indicate because its should be “asymmetrical
Hirschsprung disease or Congenital gluteal fold”. Letter B and C are not
Aganglionic Megacolon, an anomaly applicable for newborns because they
resulting in mechanical obstruction are seen in older children.
due to inadequate motility in an
intestinal segment. B, C, and D are 53. While assessing a male neonate
not associated in the failure to pass whose mother desires him to be
meconium of the newborn circumcised, the nurse observes that the
50. The nurse understands that a good neonate’s urinary meatus appears to be
snack for a 2 year old with a diagnosis of located on the ventral surface of the
acute asthma would be: penis. The physician is notified because
the nurse would suspect which of the
Grapes following?
Apple slices
A glass of milk Phimosis
A glass of cola Hydrocele
Epispadias
B. Apple slices. Grapes is in appropriate Hypospadias
because of its “balat” that can cause
choking. A glass of milk is not a good D. Hypospadias. Hypospadias is a c
snack because it’s the most common condition in which the urethral
cause of Iron-deficiency anemia in opening is located below the glans
children (milk contains few iron), A penis or anywhere along the ventral
glass of cola is also not appropriate surface of the penile shaft.
cause it contains complex sugar. Epispadias, the urethral meatus is
(walang kinalaman ang asthma dahil located at the dorsal surface of the
ala naman itong diatery restricted penile shaft. (Para di ka malilto, I-
foods na nasa choices.) alphabetesize mo Dorsal, (Above) eh
mauuna sa Ventral (Below) , Epis
51. Which of the following immunizations mauuna sa Hypo.)
would the nurse expect to administer to
a child who is HIV (+) and severely 54. When teaching a group of parents
immunocomromised? about seat belt use, when would the
nurse state that the child be safely
Varicella restrained in a regular automobile
Rotavirus seatbelt?
MMR
IPV 30 lb and 30 in
35 lb and 3 y/o
D. IPV. IPV or Inactivated polio vaccine 40 lb and 40 in
does not contain live micro organisms 60 lb and 6 y/o
which can be harmful to an
immunocompromised child. Unlike C. 40 lb and 40 in. Basta tandaan ang
OPV, IPV is administered via IM route. rule of 4! 4 years old, 40 lbs and 40
in.
52. When assessing a newborn for
developmental dysplasia of the hip, the 55. When assessing a newborn with cleft
nurse would expect to assess which of lip, the nurse would be alert which of the
the following? following will most likely be
compromised?
Symmetrical gluteal folds
Trendelemburg sign Sucking ability
Ortolani’s sign Respiratory status
Characteristic limp Locomotion
GI function
C. Ortolani’s sign. Correct answer is
Ortolani’s sign; it is the abnormal A. Sucking ability. Because of the defect,
clicking sound when the hips are the child will be unable to form the
mouth adequately arounf the nipple 59. Dietary restriction in a child who has
thereby requiring special devices to Hemocystenuria will include which of the
allow feeding and sucking following amino acid?
gratification. Respiratory status may
be compromised when the child is fed Lysine
improperly or during post op period Methionine
Isolensine tryptophase
56. For a child with recurring nephritic Valine
syndrome, which of the following areas of
potential disturbances should be a prime B. Methionine. Hemocystenuria is the
consideration when planning ongoing elevated excretion of the amino acid
nursing care? hemocystiene, and there is inability
to convert the amino acid methionine
Muscle coordination or cystiene. So dietary restriction of
Sexual maturation this amino acids is advised. This
Intellectual development disease can lead to mental
Body image retardation.

D. Body image. Because of edema, 60. A milk formula that you can suggest for
associated with nephroitic syndrome, a child with Galactosemia:
potential self concept and body Lofenalac
image disturbance related to changes Lactum
in appearance and social isolation Neutramigen
should be considered. Sustagen
C. Neutramigen. Neutramien is
57. An inborn error of metabolism that suggested for a child with
causes premature destruction of RBC? Galactosemia. Lofenalac is suggested
for a child with PKU.
G6PD
Hemocystinuria
Phenylketonuria
PNLE Community
Celiac Disease

A. G6PD. G6PD is the premature


Health Nursing
Exam 1
destruction of RBC when the blood is
exposed to antioxidants, ASA (ano
un? Aspirin), legumes and flava
beans. 1. Which is the primary goal of
community health nursing?
58. Which of the following would be a
diagnostic test for Phenylketonuria which A. To support and supplement the
uses fresh urine mixed with ferric efforts of the medical profession in
chloride? the promotion of health and
prevention of illness
Guthrie Test B. To enhance the capacity of
Phenestix test individuals, families and
Beutler’s test communities to cope with their
Coomb’s test health needs
C. To increase the productivity of the
B. Phenestix test. Phenestix test is a people by providing them with
diagnostic test which uses a fresh services that will increase their level
urine sample (diapers) and mixed of health
with ferric chloride. If positive, there D. To contribute to national
will be a presence of green spots at development through promotion of
the diapers. Guthrie test is another family welfare, focusing particularly
test for PKU and is the one that on mothers and children.
mostly used. The specimen used is
the blood and it tests if CHON is Answer: (B) To enhance the capacity
converted to amino acid. of individuals, families and
communities to cope with their
health needs
2. CHN is a community-based practice.
Which best explains this statement? 5. A business firm must employ an
occupational health nurse when it has at
A. The service is provided in the least how many employees?
natural environment of people.
B. The nurse has to conduct A. 21
community diagnosis to B. 101
determine nursing needs and C. 201
problems. D. 301

C. The services are based on the Answer: (B) 101. Again, this is based on
available resources within the R.A. 1054
community.
D. Priority setting is based on the 6. When the occupational health nurse
magnitude of the health problems employs ergonomic principles, she is
identified. performing which of her roles?

Answer: (B) The nurse has to conduct A. Health care provider


community diagnosis to determine B. Health educator
nursing needs and problems C. Health care coordinator
D. Environmental manager
3. Population-focused nursing practice
requires which of the following Answer: (D) Environmental
processes? manager. Ergonomics is improving
efficiency of workers by improving the
A. Community organizing worker’s environment through
B. Nursing process appropriately designed furniture, for
C. Community diagnosis example
D. Epidemiologic process
7. A garment factory does not have an
Answer: (C) Community occupational nurse. Who shall provide
diagnosis. Population-focused nursing the occupational health needs of the
care means providing care based on the factory workers?
greater need of the majority of the
population. The greater need is identified A. Occupational health nurse at the
through community diagnosis. Provincial Health Office
B. Physician employed by the factory
4. R.A. 1054 is also known as the C. Public health nurse of the RHU
Occupational Health Act. Aside from of their municipality
number of employees, what other factor D. Rural sanitary inspector of the RHU
must be considered in determining the of their municipality
occupational health privileges to which Answer: (C) Public health nurse of
the workers will be entitled? the RHU of their municipality. You’re
right! This question is based on R.A.1054
A. Type of occupation: agricultural,
commercial, industrial 8. “Public health services are given free
B. Location of the workplace in of charge.” Is this statement true or
relation to health facilities false?
C. Classification of the business
enterprise based on net profit A. The statement is true; it is the
D. Sex and age composition of responsibility of government to
employees provide basic services.
B. The statement is false; people
Answer: (B) Location of the pay indirectly for public health
workplace in relation to health services.
facilities. Based on R.A. 1054, an C. The statement may be true or
occupational nurse must be false, depending on the specific
employed when there are 30 to 100 service required.
employees and the workplace is more D. The statement may be true or
than 1 km. away from the nearest false, depending on policies of the
health center government concerned.
D. Public health nursing focuses
Answer: (B) The statement is false; on preventive, not curative,
people pay indirectly for public services.
health services. Community health
services, including public health services, Answer: (D) Public health nursing
are pre-paid services, though taxation, focuses on preventive, not curative,
for example. services.The catchment area in PHN
consists of a residential community,
9. According to C.E.Winslow, which of the many of whom are well individuals who
following is the goal of Public Health? have greater need for preventive rather
than curative services.
A. For people to attain their
birthrights of health and 12. According to Margaret Shetland, the
longevity philosophy of public health nursing is
B. For promotion of health and based on which of the following?
prevention of disease
C. For people to have access to basic A. Health and longevity as birthrights
health services B. The mandate of the state to protect
D. For people to be organized in their the birthrights of its citizens
health efforts C. Public health nursing as a
specialized field of nursing
Answer: (A) For people to attain their D. The worth and dignity of man
birthrights of health and
longevity. According to Winslow, all Answer: (D) The worth and dignity of
public health efforts are for people to man. This is a direct quote from Dr.
realize their birthrights of health and Margaret Shetland’s statements on
longevity Public Health Nursing

10. We say that a Filipino has attained 13. Which of the following is the mission
longevity when he is able to reach the of the Department of Health?
average lifespan of Filipinos. What other
statistic may be used to determine A. Health for all Filipinos
attainment of longevity? B. Ensure the accessibility and
quality of health care
A. Age-specific mortality rate C. Improve the general health status
B. Proportionate mortality rate of the population
C. Swaroop’s index D. Health in the hands of the Filipino
D. Case fatality rate people by the year 2020

Answer: (C) Swaroop’s Answer: (B) Ensure the accessibility


index. Swaroop’s index is the and quality of health care
percentage of the deaths aged 50 years
or older. Its inverse represents the 14. Region IV Hospital is classified as
percentage of untimely deaths (those what level of facility?
who died younger than 50 years
A. Primary
11. Which of the following is the most B. Secondary
prominent feature of public health C. Intermediate
nursing? D. Tertiary

A. It involves providing home care to Answer: (D) Tertiary. Regional hospitals


sick people who are not confined in are tertiary facilities because they serve
the hospital. as training hospitals for the region.
B. Services are provided free of
charge to people within the 15. Which is true of primary facilities?
catchment area.
C. The public health nurse functions A. They are usually government-run.
as part of a team providing a public B. Their services are provided on
health nursing services. an out-patient basis.
C. They are training facilities for
health professionals.
D. A community hospital is an 19. R.A. 7160 mandates devolution of
example of this level of health basic services from the national
facilities. government to local government units.
Which of the following is the major goal
Answer: (B) Their services are of devolution?
provided on an out-patient
basis. Primary facilities government A. To strengthen local government
and non-government facilities that units
provide basic out-patient services B. To allow greater autonomy to local
government units
16. Which is an example of the school C. To empower the people and
nurse’s health care provider functions? promote their self-reliance
D. To make basic services more
A. Requesting for BCG from the RHU accessible to the people
for school entrant immunization Answer: (C) To empower the
B. Conducting random classroom people and promote their self-
inspection during a measles reliance. People empowerment is
epidemic the basic motivation behind
C. Taking remedial action on an devolution of basic services to
accident hazard in the school LGU’s.
playground
D. Observing places in the school 20. Who is the Chairman of the Municipal
where pupils spend their free time Health Board?

Answer: (B) Conducting random A. Mayor


classroom inspection during a B. Municipal Health Officer
measles epidemic. Random classroom C. Public Health Nurse
inspection is assessment of D. Any qualified physician
pupils/students and teachers for signs of
a health problem prevalent in the Answer: (A) Mayor. The local executive
community serves as the chairman of the Municipal
Health Board.
17. When the nurse determines whether
resources were maximized in 21. Which level of health facility is the
implementing Ligtas Tigdas, she is usual point of entry of a client into the
evaluating health care delivery system?

A. Effectiveness A. Primary
B. Efficiency B. Secondary
C. Adequacy C. Intermediate
D. Appropriateness D. Tertiary

Answer: (B) Efficiency. Efficiency is Answer: (A) Primary. The entry of a


determining whether the goals were person into the health care delivery
attained at the least possible cost. system is usually through a consultation
in out-patient services.
18. You are a new B.S.N. graduate. You
want to become a Public Health Nurse. 22. The public health nurse is the
Where will you apply? supervisor of rural health midwives.
Which of the following is a supervisory
A. Department of Health function of the public health nurse?
B. Provincial Health Office
C. Regional Health Office A. Referring cases or patients to the
D. Rural Health Unit midwife
B. Providing technical guidance to
Answer: (D) Rural Health Unit. R.A. the midwife
7160 devolved basic health services to C. Providing nursing care to cases
local government units (LGU’s ). The referred by the midwife
public health nurse is an employee of the D. Formulating and implementing
LGU. training programs for midwives
Answer: (B) Providing technical notifiable diseases. What law mandates
guidance to the midwife. The nurse reporting of cases of notifiable diseases?
provides technical guidance to the
midwife in the care of clients, particularly A. Act 3573
in the implementation of management B. R.A. 3753
guidelines, as in Integrated Management C. R.A. 1054
of Childhood Illness. D. R.A. 1082

23. One of the participants in a hilot Answer: (A) Act 3573. Act 3573, the
training class asked you to whom she Law on Reporting of Communicable
should refer a patient in labor who Diseases, enacted in 1929, mandated
develops a complication. You will answer, the reporting of diseases listed in the
to the law to the nearest health station

A. Public Health Nurse 27. According to Freeman and Heinrich,


B. Rural Health Midwife community health nursing is a
C. Municipal Health Officer developmental service. Which of the
D. Any of these health professionals following best illustrates this statement?

Answer: (C) Municipal Health Officer. A A. The community health nurse


public health nurse and rural health continuously develops himself
midwife can provide care during normal personally and professionally.
childbirth. A physician should attend to a B. Health education and
woman with a complication during labor. community organizing are
necessary in providing
24. You are the public health nurse in a community health services.
municipality with a total population of C. Community health nursing is
about 20,000. There are 3 rural health intended primarily for health
midwives among the RHU personnel. promotion and prevention and
How many more midwife items will the treatment of disease.
RHU need? D. The goal of community health
nursing is to provide nursing services
A. 1 to people in their own places of
B. 2 residence.
C. 3
D. The RHU does not need any more Answer: (B) Health education and
midwife item. community organizing are
necessary in providing community
Answer: (A) 1. Each rural health midwife health services. The community
is given a population assignment of health nurse develops the health
about 5,000. capability of people through health
education and community organizing
25. If the RHU needs additional midwife activities
items, you will submit the request for
additional midwife items for approval to 28. Which disease was declared through
the Presidential Proclamation No. 4 as a
target for eradication in the Philippines?
A. Rural Health Unit
B. District Health Office A. Poliomyelitis
C. Provincial Health Office B. Measles
D. Municipal Health Board C. Rabies
D. Neonatal tetanus
Answer: (D) Municipal Health
Board. As mandated by R.A. 7160, Answer: (B) Measles. Presidential
basic health services have been Proclamation No. 4 is on the Ligtas Tigdas
devolved from the national government Program.
to local government units
29. The public health nurse is responsible
26. As an epidemiologist, the nurse is for presenting the municipal health
responsible for reporting cases of statistics using graphs and tables. To
compare the frequency of the leading
causes of mortality in the municipality, A. To educate the people regarding
which graph will you prepare? community health problems
B. To mobilize the people to resolve
A. Line community health problems
B. Bar C. To maximize the community’s
C. Pie resources in dealing with health
D. Scatter diagram problems
D. To maximize the community’s
Answer: (B) Bar. A bar graph is used to resources in dealing with health
present comparison of values, a line problems
graph for trends over time or age, a pie
graph for population composition or Answer: (D) To maximize the
distribution, and a scatter diagram for community’s resources in dealing
correlation of two variables. with health problems. Community
organizing is a developmental service,
30. Which step in community organizing with the goal of developing the people’s
involves training of potential leaders in self-reliance in dealing with community
the community? health problems. A, B and C are
objectives of contributory objectives to
A. Integration this goal.
B. Community organization
C. Community study 3. An indicator of success in community
D. Core group formation organizing is when people are able to

Answer: (D) Core group formation. In A. Participate in community


core group formation, the nurse is able to activities for the solution of a
transfer the technology of community community problem
organizing to the potential or informal B. Implement activities for the solution
community leaders through a training of the community problem
program. C. Plan activities for the solution of the
community problem
PNLE Community D. Identify the health problem as a
common concern

Health Nursing Answer: (A) Participate in


community activities for the
solution of a community
Exam 2 problem. Participation in community
activities in resolving a community
1. In which step are plans formulated for problem may be in any of the
solving community problems? processes mentioned in the other
choices
A. Mobilization
B. Community organization
C. Follow-up/extension 4. Tertiary prevention is needed in which
D. Core group formation stage of the natural history of disease?

Answer: (B) Community A. Pre-pathogenesis


organization. Community organization B. Pathogenesis
is the step when community assemblies C. Prodromal
take place. During the community D. Terminal
assembly, the people may opt to
formalize the community organization Answer: (D) Terminal. Tertiary
and make plans for community action to prevention involves rehabilitation,
resolve a community health problem. prevention of permanent disability and
disability limitation appropriate for
2. The public health nurse takes an convalescents, the disabled,
active role in community participation. complicated cases and the terminally ill
What is the primary goal of community (those in the terminal stage of a
organizing? disease)
5. Isolation of a child with measles 9. The de los Reyes couple have a 6-year
belongs to what level of prevention? old child entering school for the first
time. The de los Reyes family has a:
A. Primary
B. Secondary A. Health threat
C. Intermediate B. Health deficit
D. Tertiary C. Foreseeable crisis
D. Stress point
Answer: (A) Primary. The purpose of
isolating a client with a communicable Answer: (C) Foreseeable crisis. Entry
disease is to protect those who are not of the 6-year old into school is an
sick (specific disease prevention). anticipated period of unusual demand
on the family.
6. On the other hand, Operation Timbang
is _____ prevention. 10. Which of the following is an
advantage of a home visit?
A. Primary
B. Secondary A. It allows the nurse to provide nursing
C. Intermediate care to a greater number of people.
D. Tertiary B. It provides an opportunity to do
first hand appraisal of the home
Answer: (B) Secondary. Operation situation.
Timbang is done to identify members of C. It allows sharing of experiences
the susceptible population who are among people with similar health
malnourished. Its purpose is early problems.
diagnosis and, subsequently, prompt D. It develops the family’s initiative in
treatment. providing for health needs of its
members.
7. Which type of family-nurse contact will
provide you with the best opportunity to Answer: (B) It provides an
observe family dynamics? opportunity to do first hand
appraisal of the home
A. Clinic consultation situation.. Choice A is not correct since
B. Group conference a home visit requires that the nurse
C. Home visit spend so much time with the family.
D. Written communication Choice C is an advantage of a group
conference, while choice D is true of a
Answer: (C) Home visit. Dynamics of clinic consultation.
family relationships can best be
observed in the family’s natural 11. Which is CONTRARY to the principles
environment, which is the home. in planning a home visit?

8. The typology of family nursing A. A home visit should have a purpose


problems is used in the statement of or objective.
nursing diagnosis in the care of families. B. The plan should revolve around
The youngest child of the de los Reyes family health needs.
family has been diagnosed as mentally C. A home visit should be conducted
retarded. This is classified as a: in the manner prescribed by the
RHU.
A. Health threat D. Planning of continuing care should
B. Health deficit involve a responsible family member.
C. Foreseeable crisis
D. Stress point Answer: (C) A home visit should be
conducted in the manner prescribed
Answer: (B) Health deficit. Failure of by the RHU.The home visit plan should
a family member to develop according be flexible and practical, depending on
to what is expected, as in mental factors, such as the family’s needs and
retardation, is a health deficit. the resources available to the nurse and
the family.
12. The PHN bag is an important tool in Answer: (B) Analytical. Analytical
providing nursing care during a home epidemiology is the study of factors or
visit. The most important principle of bag determinants affecting the patterns of
technique states that it occurrence and distribution of disease in
a community.
A. Should save time and effort.
B. Should minimize if not totally 15. Which of the following is a function of
prevent the spread of infection. epidemiology?
C. Should not overshadow concern for
the patient and his family. A. Identifying the disease condition
D. May be done in a variety of ways based on manifestations presented
depending on the home situation, by a client
etc. B. Determining factors that contributed
to the occurrence of pneumonia in a
Answer: (B) Should minimize if not 3 year old
totally prevent the spread of C. Determining the efficacy of the
infection. Bag technique is performed antibiotic used in the treatment of
before and after handling a client in the the 3 year old client with pneumonia
home to prevent transmission of D. Evaluating the effectiveness of
infection to and from the client. the implementation of the
Integrated Management of
13. To maintain the cleanliness of the bag Childhood Illness
and its contents, which of the following
must the nurse do? Answer: (D) Evaluating the
effectiveness of the implementation
A. Wash his/her hands before and of the Integrated Management of
after providing nursing care to Childhood Illness. Epidemiology is
the family members. used in the assessment of a community
B. In the care of family members, as or evaluation of interventions in
much as possible, use only articles community health practice.
taken from the bag.
C. Put on an apron to protect her 16. Which of the following is an
uniform and fold it with the right side epidemiologic function of the nurse
out before putting it back into the during an epidemic?
bag.
D. At the end of the visit, fold the lining A. Conducting assessment of suspected
on which the bag was placed, cases to detect the communicable
ensuring that the contaminated side disease
is on the outside. B. Monitoring the condition of the cases
affected by the communicable
Answer: (A) Wash his/her hands disease
before and after providing nursing C. Participating in the investigation
care to the family members. Choice to determine the source of the
B goes against the idea of utilizing the epidemic
family’s resources, which is encouraged D. Teaching the community on
in CHN. Choices C and D goes against preventive measures against the
the principle of asepsis of confining the disease
contaminated surface of objects.
Answer: (C) Participating in the
14. The public health nurse conducts a investigation to determine the
study on the factors contributing to the source of the epidemic. Epidemiology
high mortality rate due to heart disease is the study of patterns of occurrence
in the municipality where she works. and distribution of disease in the
Which branch of epidemiology does the community, as well as the factors that
nurse practice in this situation? affect disease patterns. The purpose of
an epidemiologic investigation is to
A. Descriptive identify the source of an epidemic, i.e.,
B. Analytical what brought about the epidemic.
C. Therapeutic
D. Evaluation 17. The primary purpose of conducting
an epidemiologic investigation is to
A. Delineate the etiology of the A. Epidemic occurrence
epidemic B. Cyclical variation
B. Encourage cooperation and support C. Sporadic occurrence
of the community D. Secular variation
C. Identify groups who are at risk of
contracting the disease Answer: (B) Cyclical variation. A
D. Identify geographical location of cyclical variation is a periodic fluctuation
cases of the disease in the in the number of cases of a disease in
community the community.

Answer: (A) Delineate the etiology 21. In the year 1980, the World Health
of the epidemic. Delineating the Organization declared the Philippines,
etiology of an epidemic is identifying its together with some other countries in the
source. Western Pacific Region, “free” of which
disease?
18. Which is a characteristic of person-to-
person propagated epidemics? A. Pneumonic plague
B. Poliomyelitis
A. There are more cases of the disease C. Small pox
than expected. D. Anthrax
B. The disease must necessarily be
transmitted through a vector. Answer: (C) Small pox. The last
C. The spread of the disease can be documented case of Small pox was in
attributed to a common vehicle. 1977 at Somalia.
D. There is a gradual build up of
cases before the epidemic 22. In the census of the Philippines in
becomes easily noticeable. 1995, there were about 35,299,000
males and about 34,968,000 females.
Answer: (D) There is a gradual build What is the sex ratio?
up of cases before the epidemic
becomes easily noticeable. A gradual A. 99.06:100
or insidious onset of the epidemic is B. 100.94:100
usually observable in person-to-person C. 50.23%
propagated epidemics. D. 49.76%

19. In the investigation of an epidemic, Answer: (B) 100.94:100. Sex ratio is


you compare the present frequency of the number of males for every 100
the disease with the usual frequency at females in the population
this time of the year in this community.
This is done during which stage of the 23. Primary health care is a total
investigation? approach to community development.
Which of the following is an indicator of
A. Establishing the epidemic success in the use of the primary health
B. Testing the hypothesis care approach?
C. Formulation of the hypothesis
D. Appraisal of facts A. Health services are provided free of
charge to individuals and families.
Answer: (A) Establishing the B. Local officials are empowered as the
epidemic. Establishing the epidemic is major decision makers in matters of
determining whether there is an health.
epidemic or not. This is done by C. Health workers are able to provide
comparing the present number of cases care based on identified health needs
with the usual number of cases of the of the people.
disease at the same time of the year, as D. Health programs are sustained
well as establishing the relatedness of according to the level of
the cases of the disease. development of the community.

20. The number of cases of Dengue fever Answer: (D) Health programs are
usually increases towards the end of the sustained according to the level
rainy season. This pattern of occurrence of development of the
of Dengue fever is best described as community. Primary health care is
essential health care that can be A. Alma Ata Declaration on PHC
sustained in all stages of development B. Letter of Instruction No. 949
of the community. C. Presidential Decree No. 147
D. Presidential Decree 996
24. Sputum examination is the major
screening tool for pulmonary Answer: (B) Letter of Instruction
tuberculosis. Clients would sometimes No. 949. Letter of Instruction 949 was
get false negative results in this exam. issued by then President Ferdinand
This means that the test is not perfect in Marcos, directing the formerly called
terms of which characteristic of a Ministry of Health, now the Department
diagnostic examination? of Health, to utilize Primary Health Care
approach in planning and implementing
A. Effectiveness health programs
B. Efficacy
C. Specificity 29. Which of the following demonstrates
D. Sensitivity intersectoral linkages?
Answer: (D) Sensitivity. Sensitivity is
the capacity of a diagnostic examination A. Two-way referral system
to detect cases of the disease. If a test is B. Team approach
100% sensitive, all the cases tested will C. Endorsement done by a midwife to
have a positive result, i.e., there will be another midwife
no false negative results. D. Cooperation between the PHN
and public school teacher
25. Use of appropriate technology
requires knowledge of indigenous Answer: (D) Cooperation
technology. Which medicinal herb is between the PHN and public
given for fever, headache and cough? school teacher. Intersectoral
linkages refer to working
A. Sambong relationships between the health
B. Tsaang gubat sector and other sectors involved in
C. Akapulko community development.
D. Lagundi
30. The municipality assigned to you has
Answer: (D) Lagundi. Sambong is a population of about 20,000. Estimate
used as a diuretic. Tsaang gubat is used the number of 1-4 year old children who
to relieve diarrhea. Akapulko is used for will be given Retinol capsule 200,000 I.U.
its antifungal property every 6 months.

26. What law created the Philippine A. 1,500


Institute of Traditional and Alternative B. 1,800
Health Care? C. 2,000
D. 2,300
A. R.A. 8423
B. R.A. 4823 Answer: (D) 2,300. Based on the
C. R.A. 2483 Philippine population composition, to
D. R.A. 3482 estimate the number of 1-4 year old
27. In traditional Chinese medicine, the children, multiply total population by
yielding, negative and feminine force is 11.5%.
termed

A. Yin
B. Yang
C. Qi
D. Chai
PNLE
Answer: (A) Yin. Yang is the male
dominating, positive and masculine
force.
Community
28. What is the legal basis for Primary
Health Care approach in the Philippines?
Health Nursing Answer: (B) 5.2/1,000. To compute
crude death rate divide total number of
deaths (94) by total population (18,000)
Exam 3 and multiply by 1,000

1. Estimate the number of pregnant 5. Knowing that malnutrition is a frequent


women who will be given tetanus toxoid community health problem, you decided
during an immunization outreach activity to conduct nutritional assessment. What
in a barangay with a population of about population is particularly susceptible to
1,500. protein energy malnutrition (PEM)?

A. 265 A. Pregnant women and the elderly


B. 300 B. Under-5 year old children
C. 375 C. 1-4 year old children
D. 400 D. School age children

Answer: (A) 265. To estimate the Answer: (C) 1-4 year old
number of pregnant women, multiply children. Preschoolers are the most
the total population by 3.5%. susceptible to PEM because they have
generally been weaned. Also, this is the
2. To describe the sex composition of the population who, unable to feed
population, which demographic tool may themselves, are often the victims of poor
be used? intrafamilial food distribution.

A. Sex ratio 6. Which statistic can give the most


B. Sex proportion accurate reflection of the health status of
C. Population pyramid a community?
D. Any of these may be used.
A. 1-4 year old age-specific mortality
Answer: (D) Any of these may be rate
used. Sex ratio and sex proportion are B. Infant mortality rate
used to determine the sex composition of C. Swaroop’s index
a population. A population pyramid is D. Crude death rate
used to present the composition of a
population by age and sex. Answer: (C) Swaroop’s
index. Swaroop’s index is the
3. Which of the following is a natality proportion of deaths aged 50 years
rate? and above. The higher the Swaroop’s
index of a population, the greater the
A. Crude birth rate proportion of the deaths who were
B. Neonatal mortality rate able to reach the age of at least 50
C. Infant mortality rate years, i.e., more people grew old
D. General fertility rate before they died.

Answer: (A) Crude birth rate. Natality 7. In the past year, Barangay A had an
means birth. A natality rate is a birth average population of 1655. 46 babies
rate. were born in that year, 2 of whom died
less than 4 weeks after they were born.
4. You are computing the crude death There were 4 recorded stillbirths. What is
rate of your municipality, with a total the neonatal mortality rate?
population of about 18,000, for last year.
There were 94 deaths. Among those who A. 27.8/1,000
died, 20 died because of diseases of the B. 43.5/1,000
heart and 32 were aged 50 years or C. 86.9/1,000
older. What is the crude death rate? D. 130.4/1,000

A. 4.2/1,000 Answer: (B) 43.5/1,000. To compute for


B. 5.2/1,000 neonatal mortality rate, divide the
C. 6.3/1,000 number of babies who died before
D. 7.3/1,000 reaching the age of 28 days by the total
number of live births, then multiply by
1,000. Answer: (C) De facto. The other method
of population assignment, de jure, is
8. Which statistic best reflects the based on the usual place of residence of
nutritional status of a population? the people.

A. 1-4 year old age-specific 12. The Field Health Services and
mortality rate Information System (FHSIS) is the
B. Proportionate mortality rate recording and reporting system in public
C. Infant mortality rate health care in the Philippines. The
D. Swaroop’s index Monthly Field Health Service Activity
Report is a form used in which of the
Answer: (A) 1-4 year old age-specific components of the FHSIS?
mortality rate. Since preschoolers are
the most susceptible to the effects of A. Tally report
malnutrition, a population with poor B. Output report
nutritional status will most likely have a C. Target/client list
high 1-4 year old age-specific mortality D. Individual health record
rate, also known as child mortality rate.
Answer: (A) Tally report. A tally report
9. What numerator is used in computing is prepared monthly or quarterly by the
general fertility rate? RHU personnel and transmitted to the
Provincial Health Office
A. Estimated midyear population
B. Number of registered live births 13. To monitor clients registered in long-
C. Number of pregnancies in the year term regimens, such as the Multi-Drug
D. Number of females of reproductive Therapy, which component will be most
age useful?

Answer: (B) Number of registered live A. Tally report


births. To compute for general or total B. Output report
fertility rate, divide the number of C. Target/client list
registered live births by the number of D. Individual health record
females of reproductive age (15-45
years), then multiply by 1,000. Answer: (C) Target/client list. The MDT
Client List is a record of clients enrolled
10. You will gather data for nutritional in MDT and other relevant data, such as
assessment of a purok. You will gather dates when clients collected their
information only from families with monthly supply of drugs
members who belong to the target
population for PEM. What method of data 14. Civil registries are important sources
gathering is best for this purpose? of data. Which law requires registration
of births within 30 days from the
A. Census occurrence of the birth?
B. Survey
C. Record review A. P.D. 651
D. Review of civil registry B. Act 3573
C. R.A. 3753
Answer: (B) Survey. A survey, also D. R.A. 3375
called sample survey, is data gathering Answer: (A) P.D. 651. P.D. 651 amended
about a sample of the population R.A. 3753, requiring the registry of births
within 30 days from their occurrence
11. In the conduct of a census, the
method of population assignment based 15. Which of the following professionals
on the actual physical location of the can sign the birth certificate?
people is termed
A. Public health nurse
A. De jure B. Rural health midwife
B. De locus C. Municipal health officer
C. De facto D. Any of these health professionals
D. De novo
Answer: (D) Any of these health Answer: (D) Those who just had a
professionals. D. R.A. 3753 states that delivery within the past 15
any birth attendant may sign the months. The ideal birth spacing is at
certificate of live birth. least two years. 15 months plus 9
months of pregnancy = 2 years
16. Which criterion in priority setting of
health problems is used only in 19. Freedom of choice is one of the
community health care? policies of the Family Planning Program of
the Philippines. Which of the following
A. Modifiability of the problem illustrates this principle?
B. Nature of the problem presented
C. Magnitude of the health problem A. Information dissemination about the
D. Preventive potential of the health need for family planning
problem B. Support of research and development
in family planning methods
Answer: (C) Magnitude of the health C. Adequate information for couples
problem. Magnitude of the problem regarding the different methods
refers to the percentage of the D. Encouragement of couples to take
population affected by a health problem. family planning as a joint
The other choices are criteria considered responsibility
in both family and community health
care Answer: (C) Adequate information
forcouples regarding the different
17. The Sentrong Sigla Movement has methods. To enable the couple to
been launched to improve health service choose freely among different methods
delivery. Which of the following is/are of family planning, they must be given
true of this movement? full information regarding the different
methods that are available to them,
A. This is a project spearheaded by local considering the availability of quality
government units. services that can support their choice.
B. It is a basis for increasing funding
from local government units. 20. A woman, 6 months pregnant, came
C. It encourages health centers to focus to the center for consultation. Which of
on disease prevention and control. the following substances is
D. Its main strategy is certification contraindicated?
of health centers able to comply
with standards. A. Tetanus toxoid
B. Retinol 200,000 IU
Answer: (D) Its main strategy is C. Ferrous sulfate 200 mg
certification of health centers D. Potassium iodate 200 mg. capsule
able to comply with
standards. Sentrong Sigla Movement Answer: (B) Retinol 200,000
is a joint project of the DOH and local IU. Retinol 200,000 IU is a form of
government units. Its main strategy is megadose Vitamin A. This may have a
certification of health centers that are teratogenic effect
able to comply with standards set by
the DOH. 21. During prenatal consultation, a client
asked you if she can have her delivery at
18. Which of the following women should home. After history taking and physical
be considered as special targets for examination, you advised her against a
family planning? home delivery. Which of the following
findings disqualifies her for a home
A. Those who have two children or more delivery?
B. Those with medical conditions such
as anemia A. Her OB score is G5P3.
C. Those younger than 20 years and B. She has some palmar pallor.
older than 35 years C. Her blood pressure is 130/80.
D. Those who just had a delivery D. Her baby is in cephalic presentation.
within the past 15 months
Answer: (A) Her OB score is
G5P3. Only women with less than 5
pregnancies are qualified for a home flat or inverted. Frequent washing dries
delivery. It is also advisable for a up the nipples, making them prone to the
primigravida to have delivery at a formation of fissures.
childbirth facility
22. Inadequate intake by the pregnant 25. A primigravida is instructed to offer
woman of which vitamin may cause her breast to the baby for the first time
neural tube defects? within 30 minutes after delivery. What is
the purpose of offering the breast this
A. Niacin early?
B. Riboflavin
C. Folic acid A. To initiate the occurrence of milk
D. Thiamine letdown
B. To stimulate milk production by
Answer: (C) Folic acid. It is estimated the mammary acini
that the incidence of neural tube C. To make sure that the baby is able to
defects can be reduced drastically if get the colostrum
pregnant women have an adequate D. To allow the woman to practice
intake of folic acid. breastfeeding in the presence of the
health worker
23. You are in a client’s home to attend
to a delivery. Which of the following will Answer: (B) To stimulate milk
you do first? production by the mammary
acini. Suckling of the nipple stimulates
A. Set up the sterile area. prolactin reflex (the release of prolactin
B. Put on a clean gown or apron. by the anterior pituitary gland), which
C. Cleanse the client’s vulva with soap initiates lactation
and water.
D. Note the interval, duration and 26. In a mothers’ class, you discuss
intensity of labor contractions. proper breastfeeding technique. Which is
of these is a sign that the baby has
Answer: (D) Note the interval, “latched on” to the breast properly?
duration and intensity of labor
contractions.. Assessment of the A. The baby takes shallow, rapid sucks.
woman should be done first to B. The mother does not feel nipple pain.
determine whether she is having true C. The baby’s mouth is only partly open.
labor and, if so, what stage of labor D. Only the mother’s nipple is inside the
she is in. baby’s mouth.

24. In preparing a primigravida for Answer: (B) The mother does not feel
breastfeeding, which of the following will nipple pain.. When the baby has
you do? properly latched on to the breast, he
takes deep, slow sucks; his mouth is wide
A. Tell her that lactation begins within a open; and much of the areola is inside his
day after delivery. mouth. And, you’re right! The mother
B. Teach her nipple stretching exercises does not feel nipple pain.
if her nipples are everted.
C. Instruct her to wash her nipples 27. You explain to a breastfeeding
before and after each breastfeeding. mother that breast milk is sufficient for
D. Explain to her that putting the baby all of the baby’s nutrient needs only up
to breast will lessen blood loss after to ____.
delivery.
A. 3 months
Answer: (D) Explain to her that B. 6 months
putting the baby to breast will C. 1 year
lessen blood loss after D. 2 years
delivery. Suckling of the nipple
stimulates the release of oxytocin by the Answer: (B) 6 months. After 6 months,
posterior pituitary gland, which causes the baby’s nutrient needs, especially the
uterine contraction. Lactation begins 1 to baby’s iron requirement, can no longer
3 days after delivery. Nipple stretching be provided by mother’s milk alone
exercises are done when the nipples are
28. What is given to a woman within a parental consent. This is because of
month after the delivery of a baby? which legal document?

A. Malunggay capsule A. P.D. 996


B. Ferrous sulfate 100 mg. OD B. R.A. 7846
C. Retinol 200,000 I.U., 1 capsule C. Presidential Proclamation No. 6
D. Potassium iodate 200 mg, 1 capsule D. Presidential Proclamation No. 46

Answer: (C) Retinol 200,000 I.U., 1 Answer: (A) P.D. 996. Presidential
capsule. A capsule of Retinol 200,000 Decree 996, enacted in 1976, made
IU is given within 1 month after immunization in the EPI compulsory for
delivery. Potassium iodate is given children under 8 years of age. Hepatitis B
during pregnancy; malunggay capsule vaccination was made compulsory for the
is not routinely administered after same age group by R.A. 7846.
delivery; and ferrous sulfate is taken
for two months after delivery. 2. Which immunization produces a
permanent scar?
29. Which biological used in Expanded
Program on Immunization (EPI) is stored A. DPT
in the freezer? B. BCG
C. Measles vaccination
A. DPT D. Hepatitis B vaccination
B. Tetanus toxoid
C. Measles vaccine Answer: (B) BCG. BCG causes the
D. Hepatitis B vaccine formation of a superficial abscess, which
begins 2 weeks after immunization. The
Answer: (C) Measles vaccine. Among abscess heals without treatment, with
the biologicals used in the Expanded the formation of a permanent scar
Program on Immunization, measles
vaccine and OPV are highly sensitive to 3. A 4-week old baby was brought to the
heat, requiring storage in the freezer health center for his first immunization.
Which can be given to him?
30. Unused BCG should be discarded how
many hours after reconstitution? A. DPT1
B. OPV1
A. 2 C. Infant BCG
B. 4 D. Hepatitis B vaccine 1
C. 6
D. At the end of the day Answer: (C) Infant BCG. Infant BCG
may be given at birth. All the other
Answer: (B) 4. While the unused immunizations mentioned can be given
portion of other biologicals in EPI may at 6 weeks of age
be given until the end of the day, only
BCG is discarded 4 hours after 4. You will not give DPT 2 if the mother
reconstitution. This is why BCG says that the infant had
immunization is scheduled only in the
morning. A. Seizures a day after DPT 1.
B. Fever for 3 days after DPT 1.
C. Abscess formation after DPT 1.
D. Local tenderness for 3 days after DPT
1.
PNLE Community Answer: (A) Seizures a day after DPT
1. Seizures within 3 days after
Health Nursing administration of DPT is an indication of
hypersensitivity to pertussis vaccine, a
component of DPT. This is considered a
Exam 4 specific contraindication to subsequent
doses of DPT.
1. In immunizing school entrants with
BCG, you are not obliged to secure
5. A 2-month old infant was brought to 8. Which of the following signs will
the health center for immunization. indicate that a young child is suffering
During assessment, the infant’s from severe pneumonia?
temperature registered at 38.1°C. Which
is the best course of action that you will A. Dyspnea
take? B. Wheezing
C. Fast breathing
A. Go on with the infant’s D. Chest indrawing
immunizations.
B. Give Paracetamol and wait for his Answer: (D) Chest indrawing. In IMCI,
fever to subside. chest indrawing is used as the positive
C. Refer the infant to the physician for sign of dyspnea, indicating severe
further assessment. pneumonia
D. Advise the infant’s mother to bring
him back for immunization when he 9. Using IMCI guidelines, you classify a
is well. child as having severe pneumonia. What
is the best management for the child?
Answer: (A) Go on with the infant’s
immunizations. In the EPI, fever up to A. Prescribe an antibiotic.
38.5°C is not a contraindication to B. Refer him urgently to the
immunization. Mild acute respiratory hospital.
tract infection, simple diarrhea and C. Instruct the mother to increase fluid
malnutrition are not contraindications intake.
either. D. Instruct the mother to continue
breastfeeding.
6. A pregnant woman had just received
her 4th dose of tetanus toxoid. Answer: (B) Refer him urgently to the
Subsequently, her baby will have hospital. Severe pneumonia requires
protection against tetanus for how long? urgent referral to a hospital. Answers A, C
and D are done for a client classified as
A. 1 year having pneumonia.
B. 3 years
C. 10 years 10. A 5-month old infant was brought by
D. Lifetime his mother to the health center because
Answer: (A) 1 year. The baby will have of diarrhea occurring 4 to 5 times a day.
passive natural immunity by placental His skin goes back slowly after a skin
transfer of antibodies. The mother will pinch and his eyes are sunken. Using the
have active artificial immunity lasting IMCI guidelines, you will classify this
for about 10 years. 5 doses will give the infant in which category?
mother lifetime protection
A. No signs of dehydration
7. A 4-month old infant was brought to B. Some dehydration
the health center because of cough. Her C. Severe dehydration
respiratory rate is 42/minute. Using the D. The data is insufficient.
Integrated Management of Child Illness
(IMCI) guidelines of assessment, her Answer: (B) Some dehydration. Using
breathing is considered the assessment guidelines of IMCI, a
child (2 months to 5 years old) with
A. Fast diarrhea is classified as having SOME
B. Slow DEHYDRATION if he shows 2 or more of
C. Normal the following signs: restless or irritable,
D. Insignificant sunken eyes, the skin goes back slow
after a skin pinch
Answer: (C) Normal. In IMCI, a
respiratory rate of 50/minute or more 11. Based on assessment, you classified
is fast breathing for an infant aged 2 a 3-month old infant with the chief
to 12 months. complaint of diarrhea in the category of
SOME DEHYDRATION. Based on IMCI
management guidelines, which of the
following will you do?
A. Bring the infant to the nearest facility decreased colloidal osmotic pressure of
where IV fluids can be given. the blood brought about by
B. Supervise the mother in giving hypoalbuminemia. Decreased blood
200 to 400 ml. of Oresol in 4 albumin level is due a protein-deficient
hours. diet.
C. Give the infant’s mother instructions
on home management. 14. Assessment of a 2-year old child
D. Keep the infant in your health center revealed “baggy pants”. Using the IMCI
for close observation. guidelines, how will you manage this
child?
Answer: (B) Supervise the mother in
giving 200 to 400 ml. of Oresol in 4 A. Refer the child urgently to a hospital
hours. In the IMCI management for confinement.
guidelines, SOME DEHYDRATION is B. Coordinate with the social worker to
treated with the administration of Oresol enroll the child in a feeding program.
within a period of 4 hours. The amount of C. Make a teaching plan for the mother,
Oresol is best computed on the basis of focusing on menu planning for her
the child’s weight (75 ml/kg body child.
weight). If the weight is unknown, the D. Assess and treat the child for health
amount of Oresol is based on the child’s problems like infections and intestinal
age. parasitism.

12. A mother is using Oresol in the Answer: (A) Refer the child
management of diarrhea of her 3-year urgently to a hospital for
old child. She asked you what to do if her confinement. “Baggy pants” is a sign
child vomits. You will tell her to of severe marasmus. The best
management is urgent referral to a
A. Bring the child to the nearest hospital hospital.
for further assessment.
B. Bring the child to the health center 15. During the physical examination of a
for intravenous fluid therapy. young child, what is the earliest sign of
C. Bring the child to the health center xerophthalmia that you may observe?
for assessment by the physician.
D. Let the child rest for 10 minutes A. Keratomalacia
then continue giving Oresol more B. Corneal opacity
slowly. C. Night blindness
D. Conjunctival xerosis
Answer: (D) Let the child rest for 10
minutes then continue giving Oresol Answer: (D) Conjunctival xerosis. The
more slowly. If the child vomits earliest sign of Vitamin A deficiency
persistently, that is, he vomits (xerophthalmia) is night blindness.
everything that he takes in, he has to be However, this is a functional change,
referred urgently to a hospital. which is not observable during physical
Otherwise, vomiting is managed by examination.The earliest visible lesion is
letting the child rest for 10 minutes and conjunctival xerosis or dullness of the
then continuing with Oresol conjunctiva due to inadequate tear
administration. Teach the mother to give production.
Oresol more slowly
13. A 1 ½ year old child was classified as 16. To prevent xerophthalmia, young
having 3rd degree protein energy children are given Retinol capsule every
malnutrition, kwashiorkor. Which of the 6 months. What is the dose given to
following signs will be most apparent in preschoolers?
this child?
A. 10,000 IU
A. Voracious appetite B. 20,000 IU
B. Wasting C. 100,000 IU
C. Apathy D. 200,000 IU
D. Edema Answer: (D) 200,000 IU. Preschoolers
are given Retinol 200,000 IU every 6
Answer: (D) Edema. Edema, a major months. 100,000 IU is given once to
sign of kwashiorkor, is caused by
infants aged 6 to 12 months. The dose sign that indicates the need for urgent
for pregnant women is 10,000 IU referral to a hospital?
17. The major sign of iron deficiency
anemia is pallor. What part is best A. Inability to drink
examined for pallor? B. High grade fever
C. Signs of severe dehydration
A. Palms D. Cough for more than 30 days
B. Nailbeds
C. Around the lips Answer: (A) Inability to drink. A sick
D. Lower conjunctival sac child aged 2 months to 5 years must be
referred urgently to a hospital if he/she
Answer: (A) Palms. The anatomic has one or more of the following signs:
characteristics of the palms allow a not able to feed or drink, vomits
reliable and convenient basis for everything, convulsions, abnormally
examination for pallor. sleepy or difficult to awaken

18. Food fortification is one of the 21. Management of a child with measles
strategies to prevent micronutrient includes the administration of which of
deficiency conditions. R.A. 8976 the following?
mandates fortification of certain food
items. Which of the following is among A. Gentian violet on mouth lesions
these food items? B. Antibiotics to prevent pneumonia
C. Tetracycline eye ointment for corneal
A. Sugar opacity
B. Bread D. Retinol capsule regardless of
C. Margarine when the last dose was given
D. Filled milk
Answer: (D) Retinol capsule
Answer: (A) Sugar. R.A. 8976 mandates regardless of when the last dose
fortification of rice, wheat flour, sugar was given. An infant 6 to 12 months
and cooking oil with Vitamin A, iron classified as a case of measles is given
and/or iodine Retinol 100,000 IU; a child is given
200,000 IU regardless of when the last
19. What is the best course of action dose was given
when there is a measles epidemic in a
nearby municipality? 22. A mother brought her 10 month old
infant for consultation because of fever,
A. Give measles vaccine to babies which started 4 days prior to
aged 6 to 8 months. consultation. To determine malaria risk,
B. Give babies aged 6 to 11 months one what will you do?
dose of 100,000 I.U. of Retinol
C. Instruct mothers to keep their babies A. Do a tourniquet test.
at home to prevent disease B. Ask where the family resides.
transmission. C. Get a specimen for blood smear.
D. Instruct mothers to feed their babies D. Ask if the fever is present everyday.
adequately to enhance their babies’
resistance. Answer: (B) Ask where the family
resides. Because malaria is endemic,
Answer: (A) Give measles vaccine to the first question to determine malaria
babies aged 6 to 8 months. Ordinarily, risk is where the client’s family resides. If
measles vaccine is given at 9 months of the area of residence is not a known
age. During an impending epidemic, endemic area, ask if the child had
however, one dose may be given to traveled within the past 6 months, where
babies aged 6 to 8 months. The mother is he/she was brought and whether he/she
instructed that the baby needs another stayed overnight in that area.
dose when the baby is 9 months old.
23. The following are strategies
20. A mother brought her daughter, 4 implemented by the Department of
years old, to the RHU because of cough Health to prevent mosquito-borne
and colds. Following the IMCI assessment diseases. Which of these is most
guide, which of the following is a danger effective in the control of Dengue fever?
A. Stream seeding with larva-eating fish unexplained weight loss; night sweats;
B. Destroying breeding places of and hemoptysis.
mosquitoes
C. Chemoprophylaxis of non-immune 27. Which clients are considered targets
persons going to endemic areas for DOTS Category I?
D. Teaching people in endemic areas to
use chemically treated mosquito nets A. Sputum negative cavitary cases
B. Clients returning after a default
Answer: (B) Destroying breeding C. Relapses and failures of previous PTB
places of mosquitoes. Aedes aegypti, treatment regimens
the vector of Dengue fever, breeds in D. Clients diagnosed for the first time
stagnant, clear water. Its feeding time is through a positive sputum exam
usually during the daytime. It has a \
cyclical pattern of occurrence, unlike Answer: (D) Clients diagnosed for the
malaria which is endemic in certain first time through a positive sputum
parts of the country exam. Category I is for new clients
24. Secondary prevention for malaria diagnosed by sputum examination and
includes clients diagnosed to have a serious form
of extrapulmonary tuberculosis, such as
A. Planting of neem or eucalyptus trees TB osteomyelitis.
B. Residual spraying of insecticides at
night 28. To improve compliance to treatment,
C. Determining whether a place is what innovation is being implemented in
endemic or not DOTS?
D. Growing larva-eating fish in mosquito
breeding places A. Having the health worker follow up
the client at home
Answer: (C) Determining whether a B. Having the health worker or a
place is endemic or not. This is responsible family member
diagnostic and therefore secondary level monitor drug intake
prevention. The other choices are for C. Having the patient come to the
primary prevention health center every month to get his
medications
25. Scotch tape swab is done to check for D. Having a target list to check on
which intestinal parasite? whether the patient has collected his
monthly supply of drugs
A. Ascaris
B. Pinworm Answer: (B) Having the health worker
C. Hookworm or a responsible family member
D. Schistosoma monitor drug intake. Directly
Observed Treatment Short Course is so-
Answer: (B) Pinworm. Pinworm ova are called because a treatment partner,
deposited around the anal orifice. preferably a health worker accessible to
the client, monitors the client’s
26. Which of the following signs indicates compliance to the treatment.
the need for sputum examination for
AFB? 29. Diagnosis of leprosy is highly
dependent on recognition of symptoms.
A. Hematemesis Which of the following is an early sign of
B. Fever for 1 week leprosy?
C. Cough for 3 weeks
D. Chest pain for 1 week A. Macular lesions
B. Inability to close eyelids
Answer: (C) Cough for 3 weeks. A C. Thickened painful nerves
client is considered a PTB suspect when D. Sinking of the nosebridge
he has cough for 2 weeks or more, plus
one or more of the following signs: fever Answer: (C) Thickened painful
for 1 month or more; chest pain lasting nerves. The lesion of leprosy is not
for 2 weeks or more not attributed to macular. It is characterized by a change
other conditions; progressive, in skin color (either reddish or whitish)
and loss of sensation, sweating and hair
growth over the lesion. Inability to close spread of the disease to susceptible
the eyelids (lagophthalmos) and sinking hosts.
of the nosebridge are late symptoms
3. When residents obtain water from an
30. Which of the following clients should artesian well in the neighborhood, the
be classified as a case of multibacillary level of this approved type of water
leprosy? facility is

A. 3 skin lesions, negative slit skin A. I


smear B. II
B. 3 skin lesions, positive slit skin smear C. III
C. 5 skin lesions, negative slit skin D. IV
smear
D. 5 skin lesions, positive slit skin Answer: (B) II. A communal faucet or
smear water standpost is classified as Level II.

Answer: (D) 5 skin lesions, positive 4. For prevention of hepatitis A, you


slit skin smear. A multibacillary decided to conduct health education
leprosy case is one who has a positive activities. Which of the following is
slit skin smear and at least 5 skin lesions IRRELEVANT?

PNLE Community A. Use of sterile syringes and


needles
B. Safe food preparation and food
Health Nursing handling by vendors
C. Proper disposal of human excreta and
personal hygiene
Exam 5 D. Immediate reporting of water pipe
leaks and illegal water connections
. In the Philippines, which condition is the
most frequent cause of death associated Answer: (A) Use of sterile syringes
with schistosomiasis? and needles. Hepatitis A is transmitted
through the fecal oral route. Hepatitis B
A. Liver cancer is transmitted through infected body
B. Liver cirrhosis secretions like blood and semen.
C. Bladder cancer
D. Intestinal perforation 5. Which biological used in Expanded
Program on Immunization (EPI) should
Answer: (B) Liver cirrhosis. The NOT be stored in the freezer?
etiologic agent of schistosomiasis in the
Philippines is Schistosoma japonicum, A. DPT
which affects the small intestine and the B. Oral polio vaccine
liver. Liver damage is a consequence of C. Measles vaccine
fibrotic reactions to schistosoma eggs in D. MMR
the liver
Answer: (A) DPT. DPT is sensitive to
2. What is the most effective way of freezing. The appropriate storage
controlling schistosomiasis in an endemic temperature of DPT is 2 to 8° C only. OPV
area? and measles vaccine are highly sensitive
to heat and require freezing. MMR is not
A. Use of molluscicides an immunization in the Expanded
B. Building of foot bridges Program on Immunization.
C. Proper use of sanitary toilets
D. Use of protective footwear, such as 6. You will conduct outreach
rubber boots immunization in a barangay with a
population of about 1500. Estimate the
Answer: (C) Proper use of sanitary number of infants in the barangay.
toilets. The ova of the parasite get out
of the human body together with feces. A. 45
Cutting the cycle at this stage is the B. 50
most effective way of preventing the C. 55
D. 60 Answer: (B) Instruct the mother to
give the child Oresol. Since the child
Answer: (A) 45. To estimate the does not manifest any other danger sign,
number of infants, multiply total maintenance of fluid balance and
population by 3% replacement of fluid loss may be done by
giving the client Oresol.
7. In Integrated Management of
Childhood Illness, severe conditions 10. The pathognomonic sign of measles
generally require urgent referral to a is Koplik’s spot. You may see Koplik’s spot
hospital. Which of the following severe by inspecting the _____.
conditions DOES NOT always require
urgent referral to a hospital? A. Nasal mucosa
B. Buccal mucosa
A. Mastoiditis C. Skin on the abdomen
B. Severe dehydration D. Skin on the antecubital surface
C. Severe pneumonia
D. Severe febrile disease Answer: (B) Buccal mucosa. Koplik’s
spot may be seen on the mucosa of the
Answer: (B) Severe dehydration. The mouth or the throat.
order of priority in the management of
severe dehydration is as follows: 11. Among the following diseases, which
intravenous fluid therapy, referral to a is airborne?
facility where IV fluids can be initiated
within 30 minutes, Oresol/nasogastric A. Viral conjunctivitis
tube, Oresol/orem. When the foregoing B. Acute poliomyelitis
measures are not possible or effective, C. Diphtheria
tehn urgent referral to the hospital is D. Measles
done.
Answer: (D) Measles. Viral
8. A client was diagnosed as having conjunctivitis is transmitted by direct or
Dengue fever. You will say that there is indirect contact with discharges from
slow capillary refill when the color of the infected eyes. Acute poliomyelitis is
nailbed that you pressed does not return spread through the fecal-oral route and
within how many seconds? contact with throat secretions, whereas
diphtheria is through direct and indirect
A. 3 contact with respiratory secretions.
B. 5
C. 8 12. Among children aged 2 months to 3
D. 10 years, the most prevalent form of
meningitis is caused by which
Answer: (A) 3. Adequate blood supply microorganism?
to the area allows the return of the color
of the nailbed within 3 seconds. A. Hemophilus influenzae
B. Morbillivirus
9. A 3-year old child was brought by his C. Steptococcus pneumoniae
mother to the health center because of D. Neisseria meningitides
fever of 4-day duration. The child had a
positive tourniquet test result. In the Answer: (A) Hemophilus
absence of other signs, which is the most influenzae. Hemophilus meningitis is
appropriate measure that the PHN may unusual over the age of 5 years. In
carry out to prevent Dengue shock developing countries, the peak
syndrome? incidence is in children less than 6
months of age. Morbillivirus is the
A. Insert an NGT and give fluids per NGT. etiology of measles. Streptococcus
B. Instruct the mother to give the pneumoniae and Neisseria meningitidis
child Oresol. may cause meningitis, but age
C. Start the patient on intravenous distribution is not specific in young
fluids STAT. children
D. Refer the client to the physician for
appropriate management. 13. Human beings are the major
reservoir of malaria. Which of the
following strategies in malaria control is B. Cholera
based on this fact? C. Amebiasis
D. Dysentery
A. Stream seeding
B. Stream clearing Answer: (B) Cholera. Passage of
C. Destruction of breeding places profuse watery stools is the major
D. Zooprophylaxis symptom of cholera. Both amebic and
bacillary dysentery are characterized by
Answer: (D) the presence of blood and/or mucus in
Zooprophylaxis. Zooprophylaxis is the stools. Giardiasis is characterized by
done by putting animals like cattle or fat malabsorption and, therefore,
dogs close to windows or doorways just steatorrhea.
before nightfall. The Anopheles
mosquito takes his blood meal from the 17. In the Philippines, which specie of
animal and goes back to its breeding schistosoma is endemic in certain
place, thereby preventing infection of regions?
humans
A. S. mansoni
14. The use of larvivorous fish in malaria A.
control is the basis for which strategy of B. S. malayensis
malaria control? C. S. haematobium

A. Stream seeding Answer: (B) S. japonicum. S. mansoni


B. Stream clearing is found mostly in Africa and South
C. Destruction of breeding places America; S. haematobium in Africa and
D. Zooprophylaxis the Middle East; and S. malayensis only
in peninsular Malaysia.
Answer: (A) Stream seeding. Stream
seeding is done by putting tilapia fry in 18. A 32-year old client came for
streams or other bodies of water consultation at the health center with the
identified as breeding places of the chief complaint of fever for a week.
Anopheles mosquito Accompanying symptoms were muscle
pains and body malaise. A week after the
15. Mosquito-borne diseases are start of fever, the client noted yellowish
prevented mostly with the use of discoloration of his sclera. History
mosquito control measures. Which of the showed that he waded in flood waters
following is NOT appropriate for malaria about 2 weeks before the onset of
control? symptoms. Based on his history, which
disease condition will you suspect?
A. Use of chemically treated mosquito
nets A. Hepatitis A
B. Seeding of breeding places with B. Hepatitis B
larva-eating fish C. Tetanus
C. Destruction of breeding places of D. Leptospirosis
the mosquito vector Answer: (D)
D. Use of mosquito-repelling soaps, such Leptospirosis. Leptospirosis is
as those with basil or citronella transmitted through contact with the skin
or mucous membrane with water or
Answer: (C) Destruction of breeding moist soil contaminated with urine of
places of the mosquito infected animals, like rats.
vector. Anopheles mosquitoes breed in
slow-moving, clear water, such as
mountain streams. 19. MWSS provides water to Manila and
other cities in Metro Manila. This is an
16. A 4-year old client was brought to the example of which level of water facility?
health center with the chief complaint of
severe diarrhea and the passage of “rice A. I
water” stools. The client is most probably B. II
suffering from which condition? C. III
D. IV
A. Giardiasis
Answer: (C) III. Waterworks systems, B. Infectious mononucleosis
such as MWSS, are classified as level III. C. Cytomegalovirus disease
D. Pneumocystis carinii pneumonia
20. You are the PHN in the city health Answer: (B) Infectious
center. A client underwent screening for mononucleosis. Cytomegalovirus
AIDS using ELISA. His result was positive. disease is an acute viral disease
What is the best course of action that characterized by fever, sore throat and
you may take? lymphadenopathy.

A. Get a thorough history of the client, 23. To determine possible sources of


focusing on the practice of high risk sexually transmitted infections, which is
behaviors. the BEST method that may be
B. Ask the client to be accompanied by undertaken by the public health nurse?
a significant person before revealing
the result. A. Contact tracing
C. Refer the client to the physician since B. Community survey
he is the best person to reveal the C. Mass screening tests
result to the client. D. Interview of suspects
D. Refer the client for a
supplementary test, such as Answer: (A) Contact tracing. Contact
Western blot, since the ELISA tracing is the most practical and reliable
result may be false. method of finding possible sources of
person-to-person transmitted infections,
Answer: (D) Refer the client for a such as sexually transmitted diseases
supplementary test, such as
Western blot, since the ELISA result 24. Antiretroviral agents, such as AZT,
may be false. A client having a reactive are used in the management of AIDS.
ELISA result must undergo a more Which of the following is NOT an action
specific test, such as Western blot. A expected of these drugs.
negative supplementary test result
means that the ELISA result was false A. They prolong the life of the client
and that, most probably, the client is not with AIDS.
infected. B. They reduce the risk of opportunistic
infections
21. Which is the BEST control measure C. They shorten the period of
for AIDS? communicability of the disease.
D. They are able to bring about a
A. Being faithful to a single sexual cure of the disease condition.
partner
B. Using a condom during each sexual Answer: (D) They are able to bring
contact about a cure of the disease
C. Avoiding sexual contact with condition. There is no known treatment
commercial sex workers for AIDS. Antiretroviral agents reduce
D. Making sure that one’s sexual partner the risk of opportunistic infections and
does not have signs of AIDS prolong life, but does not cure the
\ underlying immunodeficiency
Answer: (A) Being faithful to a
single sexual partner. Sexual fidelity 25. A barangay had an outbreak of
rules out the possibility of getting the German measles. To prevent congenital
disease by sexual contact with another rubella, what is the BEST advice that you
infected person. Transmission occurs can give to women in the first trimester
mostly through sexual intercourse and of pregnancy in the barangay?
exposure to blood or tissues
A. Advice them on the signs of German
22. The most frequent causes of death measles.
among clients with AIDS are B. Avoid crowded places, such as
opportunistic diseases. Which of the markets and moviehouses.
following opportunistic infections is C. Consult at the health center where
characterized by tonsillopharyngitis? rubella vaccine may be given.
D. Consult a physician who may give
A. Respiratory candidiasis them rubella immunoglobulin.
pneumonia, are higher in incidence in
Answer: (D) Consult a physician who adults.
may give them rubella
immunoglobulin. Rubella vaccine is 28. Complications to infectious parotitis
made up of attenuated German measles (mumps) may be serious in which type of
viruses. This is contraindicated in clients?
pregnancy. Immune globulin, a specific
prophylactic against German measles, A. Pregnant women
may be given to pregnant women. B. Elderly clients
C. Young adult males
26. You were invited to be the resource D. Young infants
person in a training class for food Answer: (C) Young adult
handlers. Which of the following would males. Epididymitis and orchitis are
you emphasize regarding prevention of possible complications of mumps. In
staphylococcal food poisoning? post-adolescent males, bilateral
inflammation of the testes and
A. All cooking and eating utensils must epididymis may cause sterility.
be thoroughly washed.
B. Food must be cooked properly to
destroy staphylococcal
microorganisms.
C. Food handlers and food servers must
have a negative stool examination
result.
D. Proper handwashing during food
preparation is the best way of
preventing the condition.

Answer: (D) Proper handwashing


during food preparation is the best
way of preventing the
condition. Symptoms of this food
poisoning are due to staphylococcal
enterotoxin, not the microorganisms
themselves. Contamination is by food
handling by persons with staphylococcal
skin or eye infections

27. In a mothers’ class, you discussed


childhood diseases such as chicken pox.
Which of the following statements about
chicken pox is correct?
PNLE Medical
A. The older one gets, the more
susceptible he becomes to the
Surgical
complications of chicken pox.
B. A single attack of chicken pox will Nursing Exam 1
prevent future episodes, including
conditions such as shingles.
SITUATION : Arthur, A registered nurse,
C. To prevent an outbreak in the
witnessed an old woman hit by a
community, quarantine may be
motorcycle while crossing a train railway.
imposed by health authorities.
The old woman fell at the railway. Arthur
D. Chicken pox vaccine is best given
rushed at the scene.
when there is an impending outbreak
1. As a registered nurse, Arthur knew
in the community.
that the first thing that he will do at the
scene is
Answer: (A) The older one gets, the
more susceptible he becomes to the
A. Stay with the person, Encourage
complications of chicken pox. Chicken
her to remain still and Immobilize the
pox is usually more severe in adults than
leg while While waiting for the
in children. Complications, such as
ambulance.
B. Leave the person for a few B. On his left hand, because of
moments to call for help. reciprocal motion.
C. Reduce the fracture manually. C. On his right hand, to support the
D. Move the person to a safer right leg.
place. D. On his right hand, because only his
right leg is weak.
2. Arthur suspects a hip fracture when he 7. You also told Mr. Rojas to hold the cane
noticed that the old woman’s leg is
A. 1 Inches in front of the foot.
A. A. Lengthened, Abducted and B. 3 Inches at the lateral side of the
Internally Rotated. foot.
B. Shortened, Abducted and C. 6 Inches at the lateral side of
Externally Rotated. the foot.
C. Shortened, Adducted and Internally D. 12 Inches at the lateral side of the
Rotated. foot.
D. Shortened, Adducted and
Externally Rotated. 8. Mr. Rojas was discharged and 6
months later, he came back to the
3. The old woman complains of pain. John emergency room of the hospital because
noticed that the knee is reddened, warm he suffered a mild stroke. The right side
to touch and swollen. John interprets that of the brain was affected. At the
this signs and symptoms are likely rehabilitative phase of your nursing care,
related to you observe Mr. Rojas use a cane and
you intervene if you see him
A. Infection
B. Thrombophlebitis A. Moves the cane when the right
C. Inflammation leg is moved.
D. Degenerative disease B. Leans on the cane when the right
4. The old woman told John that she has leg swings through.
osteoporosis; Arthur knew that all of the C. keeps the cane 6 Inches out to the
following factors would contribute to side of the right foot.
osteoporosis except D. Holds the cane on the right side.

A. Hypothyroidism SITUATION: Alfred, a 40 year old


B. End stage renal disease construction worker developed cough,
C. Cushing’s Disease night sweats and fever. He was brought
D. Taking Furosemide and Phenytoin. to the nursing unit for diagnostic studies.
5. Martha, The old woman was now He told the nurse he did not receive a
Immobilized and brought to the BCG vaccine during childhood
emergency room. The X-ray shows a 9. The nurse performs a Mantoux Test.
fractured femur and pelvis. The ER Nurse The nurse knows that Mantoux Test is
would carefully monitor Martha for which also known as
of the following sign and symptoms?
A. PPD
A. Tachycardia and Hypotension B. PDP
B. Fever and Bradycardia C. PDD
C. Bradycardia and Hypertension D. DPP
D. Fever and Hypertension
SITUATION: Mr. D. Rojas, An obese 35 10. The nurse would inject the solution in
year old MS Professor of OLFU Lagro is what route?
admitted due to pain in his weight
bearing joint. The diagnosis was A. IM
Osteoarthritis. B. IV
6. As a nurse, you instructed Mr. Rojas C. ID
how to use a cane. Mr. Rojas has a D. SC
weakness on his right leg due to self
immobilization and guarding. You plan to 11. The nurse notes that a positive result
teach Mr. Rojas to hold the cane for Alfred is

A. On his left hand, because his right A. 5 mm wheal


side is weak. B. 5 mm Induration
C. 10 mm Wheal D. Pyridoxine
D. 10 mm Induration
18. According to the DOH, the most
12. The nurse told Alfred to come back hazardous period for development of
after clinical disease is during the first

A. a week A. 6-12 months after


B. 48 hours B. 3-6 months after
C. 1 day C. 1-2 months after
D. 4 days D. 2-4 weeks after

13. Mang Alfred returns after the 19. This is the name of the program of
Mantoux Test. The test result read the DOH to control TB in the country
POSITIVE. What should be the nurse’s
next action? A. DOTS
B. National Tuberculosis Control
A. Call the Physician Program
B. Notify the radiology dept. for CXR C. Short Coursed Chemotherapy
evaluation D. Expanded Program for
C. Isolate the patient Immunization
D. Order for a sputum exam
20. Susceptibility for the disease [ TB ] is
14. Why is Mantoux test not routinely increased markedly in those with the
done in the Philippines? following condition except

A. It requires a highly skilled nurse to A. 23 Year old athlete with


perform a Mantoux test diabetes insipidus
B. The sputum culture is the gold B. 23 Year old athlete taking long
standard of PTB Diagnosis and it will term Decadron therapy and anabolic
definitively determine the extent of steroids
the cavitary lesions C. 23 Year old athlete taking illegal
C. Chest X Ray Can diagnose the drugs and abusing substances
specific microorganism responsible D. Undernourished and Underweight
for the lesions individual who undergone
D. Almost all Filipinos will test gastrectomy
positive for Mantoux Test
21. Direct sputum examination and Chest
15. Mang Alfred is now a new TB patient X ray of TB symptomatic is in what level
with an active disease. What is his of prevention?
category according to the DOH?
A. Primary
A. I B. Secondary
B. II C. Tertiary
C. III D. Quarterly
D. IV
SITUATION: Michiel, A male patient
16. How long is the duration of the diagnosed with colon cancer was newly
maintenance phase of his treatment? put in colostomy.
22. Michiel shows the BEST adaptation
A. 2 months with the new colostomy if he shows
B. 3 months which of the following?
C. 4 months
D. 5 months A. Look at the ostomy site
B. Participate with the nurse in
17. Which of the following drugs is his daily ostomy care
UNLIKELY given to Mang Alfred during the C. Ask for leaflets and contact
maintenance phase? numbers of ostomy support groups
D. Talk about his ostomy openly to the
A. Rifampicin nurse and friends
B. Isoniazid
C. Ethambutol
23. The nurse plans to teach Michiel D. Protruding stoma with swollen
about colostomy irrigation. As the nurse appearance
prepares the materials needed, which of
the following item indicates that the 29. Michiel asked the nurse, what foods
nurse needs further instruction? will help lessen the odor of his colostomy.
The nurse best response would be
A. Plain NSS / Normal Saline
B. K-Y Jelly A. Eat eggs
C. Tap water B. Eat cucumbers
D. Irrigation sleeve C. Eat beet greens and parsley
24. The nurse should insert the D. Eat broccoli and spinach
colostomy tube for irrigation at
approximately 30. The nurse will start to teach Michiel
about the techniques for colostomy
A. 1-2 inches irrigation. Which of the following should
B. 3-4 inches be included in the nurse’s teaching plan?
C. 6-8 inches
D. 12-18 inches A. Use 500 ml to 1,000 ml NSS
25. The maximum height of irrigation B. Suspend the irrigant 45 cm
solution for colostomy is above the stoma
C. Insert the cone 4 cm in the stoma
A. 5 inches D. If cramping occurs, slow the
B. 12 inches irrigation
C. 18 inches
D. 24 inches 31. The nurse knew that the normal color
of Michiel’s stoma should be
26. Which of the following behavior of the
client indicates the best initial step in A. Brick Red
learning to care for his colostomy? B. Gray
C. Blue
A. Ask to defer colostomy care to D. Pale Pink
another individual SITUATION: James, A 27 basketball player
B. Promises he will begin to listen the sustained inhalation burn that required
next day him to have tracheostomy due to
C. Agrees to look at the massive upper airway edema.
colostomy 32. Wilma, His sister and a nurse is
D. States that colostomy care is the suctioning the tracheostomy tube of
function of the nurse while he is in James. Which of the following, if made by
the hospital Wilma indicates that she is committing
an error?
27. While irrigating the client’s
colostomy, Michiel suddenly complains of A. Hyperventilating James with 100%
severe cramping. Initially, the nurse oxygen before and after suctioning
would B. Instilling 3 to 5 ml normal saline to
loosen up secretion
A. Stop the irrigation by clamping C. Applying suction during catheter
the tube withdrawal
B. Slow down the irrigation D. Suction the client every hour
C. Tell the client that cramping will
subside and is normal 33. What size of suction catheter would
D. Notify the physician Wilma use for James, who is 6 feet 5
inches in height and weighing
28. The next day, the nurse will assess approximately 145 lbs?
Michiel’s stoma. The nurse noticed that a
prolapsed stoma is evident if she sees A. Fr. 5
which of the following? B. Fr. 10
C. Fr. 12
A. A sunken and hidden stoma D. Fr. 18
B. A dusky and bluish stoma
C. A narrow and flattened stoma
34. Wilma is using a portable suction unit
at home, What is the amount of suction SITUATION : Juan Miguel Lopez Zobel
required by James using this unit? Ayala de Batumbakal was diagnosed with
Acute Close Angle Glaucoma. He is being
A. 2-5 mmHg seen by Nurse Jet.
B. 5-10 mmHg 40. What specific manifestation would
C. 10-15 mmHg nurse Jet see in Acute close angle
D. 20-25 mmHg glaucoma that she would not see in an
open angle glaucoma?
35. If a Wall unit is used, What should be
the suctioning pressure required by A. Loss of peripheral vision
James? B. Irreversible vision loss
C. There is an increase in IOP
A. 50-95 mmHg D. Pain
B. 95-110 mmHg
C. 100-120 mmHg 41. Nurse jet knew that Acute close angle
D. 155-175 mmHg glaucoma is caused by
36. Wilma was shocked to see that the
Tracheostomy was dislodged. Both the A. Sudden blockage of the
inner and outer cannulas was removed anterior angle by the base of the
and left hanging on James’ neck. What iris
are the 2 equipment’s at james’ bedside B. Obstruction in trabecular
that could help Wilma deal with this meshwork
situation? C. Gradual increase of IOP
D. An abrupt rise in IOP from 8 to 15
A. New set of tracheostomy tubes and mmHg
Oxygen tank
B. Theophylline and Epinephrine 42. Nurse jet performed a TONOMETRY
C. Obturator and Kelly clamp test to Mr. Batumbakal. What does this
D. Sterile saline dressing test measures
37. Which of the following method if used
by Wilma will best assure that the A. It measures the peripheral vision
tracheostomy ties are not too tightly remaining on the client
placed? B. Measures the Intra Ocular
Pressure
A. Wilma places 2 fingers between C. Measures the Client’s Visual Acuity
the tie and neck D. Determines the Tone of the eye in
B. The tracheotomy can be pulled response to the sudden increase in
slightly away from the neck IOP.
C. James’ neck veins are not engorged
D. Wilma measures the tie from the 43. The Nurse notices that Mr.
nose to the tip of the earlobe and to Batumbakal cannot anymore determine
the xiphoid process. RED from BLUE. The nurse knew that
38. Wilma knew that James have an which part of the eye is affected by this
adequate respiratory condition if she change?
notices that
A. IRIS
A. James’ respiratory rate is 18 B. PUPIL
B. James’ Oxygen saturation is 91% C. RODS [RETINA]
C. There are frank blood suction from D. CONES [RETINA]
the tube
D. There are moderate amount of 44. Nurse Jet knows that Aqueous Humor
tracheobronchial secretions is produce where?

39. Wilma knew that the maximum time A. In the sub arachnoid space of the
when suctioning James is meninges
B. In the Lateral ventricles
A. 10 seconds C. In the Choroids
B. 20 seconds D. In the Ciliary Body
C. 30 seconds
D. 45 seconds
45. Nurse Jet knows that the normal IOP A. Atropine Sulfate
is B. Pindolol [Visken]
C. Naloxone Hydrochloride [Narcan]
A. 8-21 mmHg D. Mesoridazine Besylate [Serentil]
B. 2-7 mmHg
C. 31-35 mmHg SITUATION : Wide knowledge about the
D. 15-30 mmHg human ear, it’s parts and it’s functions
will help a nurse assess and analyze
46. Nurse Jet wants to measure Mr. changes in the adult client’s health.
Batumbakal’s CN II Function. What test
would Nurse Jet implement to measure 52. Nurse Anna is doing a caloric testing
CN II’s Acuity? to his patient, Aida, a 55 year old
university professor who recently went
A. Slit lamp into coma after being mauled by her
B. Snellen’s Chart disgruntled 3rd year nursing students
C. Wood’s light whom she gave a failing mark. After
D. Gonioscopy instilling a warm water in the ear, Anna
noticed a rotary nystagmus towards the
47. The Doctor orders pilocarpine. Nurse irrigated ear. What does this means?
jet knows that the action of this drug is to A. Indicates a CN VIII Dysfunction
B. Abnormal
A. Contract the Ciliary muscle C. Normal
B. Relax the Ciliary muscle D. Inconclusive
C. Dilate the pupils
D. Decrease production of Aqueous 53. Ear drops are prescribed to an infant,
Humor The most appropriate method to
administer the ear drops is
48. The doctor orders timolol [timoptic].
Nurse jet knows that the action of this A. Pull the pinna up and back and
drug is direct the solution towards the
eardrum
A. Reduce production of CSF B. Pull the pinna down and back
B. Reduce production of Aquesous and direct the solution onto the
Humor wall of the canal
C. Constrict the pupil C. Pull the pinna down and back and
D. Relaxes the Ciliary muscle direct the solution towards the
eardrum
49. When caring for Mr. Batumbakal, Jet D. Pull the pinna up and back and
teaches the client to avoid direct the solution onto the wall of
the canal
A. Watching large screen TVs
B. Bending at the waist 54. Nurse Jenny is developing a plan of
C. Reading books care for a patient with Menieres disease.
D. Going out in the sun What is the priority nursing intervention
in the plan of care for this particular
50. Mr. Batumbakal has undergone eye patient?
angiography using an Intravenous dye
and fluoroscopy. What activity is A. Air, Breathing, Circulation
contraindicated immediately after B. Love and Belongingness
procedure? C. Food, Diet and Nutrition
D. Safety
A. Reading newsprint
B. Lying down 55. After mastoidectomy, Nurse John
C. Watching TV should be aware that the cranial nerve
D. Listening to the music that is usually damage after this
procedure is
51. If Mr. Batumbakal is receiving
pilocarpine, what drug should always be A. CN I
available in any case systemic toxicity B. CN II
occurs? C. CN VII
D. CN VI
nystagmus occurred towards the left ear.
56. The physician orders the following for What does this finding indicates?
the client with Menieres disease. Which
of the following should the nurse A. Indicating a Cranial Nerve VIII
question? Dysfunction
B. The test should be repeated again
A. Dipenhydramine [Benadryl] because the result is vague
B. Atropine sulfate C. This is Grossly abnormal and
C. Out of bed activities and should be reported to the
ambulation neurosurgeon
D. Diazepam [Valium] D. This indicates an intact and
57. Nurse Anna is giving dietary working vestibular branch of CN
instruction to a client with Menieres VIII
disease. Which statement if made by the
client indicates that the teaching has 61. A client with Cataract is about to
been successful? undergo surgery. Nurse Oca is preparing
plan of care. Which of the following
A. I will try to eat foods that are nursing diagnosis is most appropriate to
low in sodium and limit my fluid address the long term need of this type
intake of patient?
B. I must drink atleast 3,000 ml of
fluids per day A. Anxiety R/T to the operation and its
C. I will try to follow a 50% outcome
carbohydrate, 30% fat and 20% B. Sensory perceptual alteration
protein diet R/T Lens extraction and
D. I will not eat turnips, red meat and replacement
raddish C. Knowledge deficit R/T the pre
operative and post operative self care
58. Peachy was rushed by his father, D. Body Image disturbance R/T the
Steven into the hospital admission. eye packing after surgery
Peachy is complaining of something 62. Nurse Joseph is performing a WEBERS
buzzing into her ears. Nurse Joemar TEST. He placed the tuning fork in the
assessed peachy and found out It was an patients forehead after tapping it onto his
insect. What should be the first thing that knee. The client states that the fork is
Nurse Joemar should try to remove the louder in the LEFT EAR. Which of the
insect out from peachy’s ear? following is a correct conclusion for nurse
Josph to make?
A. Use a flashlight to coax the
insect out of peachy’s ear A. He might have a sensory hearing
B. Instill an antibiotic ear drops loss in the left ear
C. Irrigate the ear B. Conductive hearing loss is possible
D. Pick out the insect using a sterile in the right ear
clean forceps C. He might have a sensory
hearing loss in the right hear,
59. Following an ear surgery, which and/or a conductive hearing loss
statement if heard by Nurse Oca from the in the left ear.
patient indicates a correct understanding D. He might have a conductive
of the post operative instructions? hearing loss in the right ear, and/or a
sensory hearing loss in the left ear.
A. Activities are resumed within 5
days 63. Aling myrna has Menieres disease.
B. I will make sure that I will clean my What typical dietary prescription would
hair and face to prevent infection nurse Oca expect the doctor to
C. I will use straw for drinking prescribe?
D. I should avoid air travel for a
while A. A low sodium , high fluid intake
B. A high calorie, high protein dietary
60. Nurse Oca will do a caloric testing to intake
a client who sustained a blunt injury in C. low fat, low sodium and high
the head. He instilled a cold water in the calorie intake
client’s right ear and he noticed that
D. low sodium and restricted fluid 68. After nursing intervention, you will
intake expect the patient to have

SITUATION : [ From DEC 1991 NLE ] A 45 1. Maintain body temperature at 36.5


year old male construction worker was C
admitted to a tertiary hospital for 2. Exhibit return of BP and Pulse to
incessant vomiting. Assessment normal
disclosed: weak rapid pulse, acute 3. Manifest normal skin turgor of skin
weight loss of .5kg, furrows in his and tongue
tongue, slow flattening of the skin was 4. Drinks fluids as prescribed
noted when the nurse released her A. 1,3
pinch. B. 2,4
Temperature: 35.8 C , BUN Creatinine C. 1,3,4
ratio : 10 : 1, He also complains for D. 2,3,4
postural hypotension. There was no
infection. SITUATION: A 65 year old woman was
admitted for Parkinson’s Disease. The
64. Which of the following is the charge nurse is going to make an initial
appropriate nursing diagnosis? assessment.
A. Fluid volume deficit R/T furrow 69. Which of the following is a
tongue characteristic of a patient with advanced
B. Fluid volume deficit R/T Parkinson’s disease?
uncontrolled vomiting
C. Dehydration R/T subnormal body A. Disturbed vision
temperature B. Forgetfulness
D. Dehydration R/T incessant vomiting C. Mask like facial expression
D. Muscle atrophy
65. Approximately how much fluid is lost
in acute weight loss of .5kg? 70. The onset of Parkinson’s disease is
between 50-60 years old. This disorder is
A. 50 ml caused by
B. 750 ml
C. 500 ml A. Injurious chemical substances
D. 75 ml B. Hereditary factors
C. Death of brain cells due to old age
66. Postural Hypotension is D. Impairment of dopamine
producing cells in the brain
A. A drop in systolic pressure less
than 10 mmHg when patient changes 71. The patient was prescribed with
position from lying to sitting. levodopa. What is the action of this drug?
B. A drop in systolic pressure
greater than 10 mmHg when A. Increase dopamine availability
patient changes position from B. Activates dopaminergic receptors
lying to sitting in the basal ganglia
C. A drop in diastolic pressure less C. Decrease acetylcholine availability
than 10 mmHg when patient changes D. Release dopamine and other
position from lying to sitting catecholamine from neurological
D. A drop in diastolic pressure greater storage sites
than 10 mmHg when patient changes 72. You are discussing with the dietician
position from lying to sitting what food to avoid with patients taking
levodopa?
67. Which of the following measures will
not help correct the patient’s condition A. Vitamin C rich food
B. Vitamin E rich food
A. Offer large amount of oral fluid C. Thiamine rich food
intake to replace fluid lost D. Vitamin B6 rich food
B. Give enteral or parenteral fluid
C. Frequent oral care 73. One day, the patient complained of
D. Give small volumes of fluid at difficulty in walking. Your response would
frequent interval be
A. You will need a cane for SITUATION : Knowledge of the drug
support PROPANTHELINE BROMIDE [Probanthine]
B. Walk erect with eyes on horizon Is necessary in treatment of various
C. I’ll get you a wheelchair disorders.
D. Don’t force yourself to walk 79. What is the action of this drug?

SITUATION: Mr. Dela Isla, a client with A. Increases glandular secretion for
early Dementia exhibits thought process clients affected with cystic fibrosis
disturbances. B. Dissolve blockage of the urinary
74. The nurse will assess a loss of ability tract due to obstruction of cystine
in which of the following areas? stones
C. Reduces secretion of the
A. Balance glandular organ of the body
B. Judgment D. Stimulate peristalsis for treatment
C. Speech of constipation and obstruction
D. Endurance
80. What should the nurse caution the
75. Mr. Dela Isla said he cannot client when using this medication
comprehend what the nurse was saying.
He suffers from: A. Avoid hazardous activities like
driving, operating machineries
A. Insomnia etc.
B. Aphraxia B. Take the drug on empty stomach
C. Agnosia C. Take with a full glass of water in
D. Aphasia treatment of Ulcerative colitis
D. I must take double dose if I missed
76. The nurse is aware that in the previous dose
communicating with an elderly client, the
nurse will 81. Which of the following drugs are not
compatible when taking Probanthine?
A. Lean and shout at the ear of the
client A. Caffeine
B. Open mouth wide while talking to B. NSAID
the client C. Acetaminophen
C. Use a low-pitched voice D. Alcohol
D. Use a medium-pitched voice
82. What should the nurse tell clients
77. As the nurse talks to the daughter of when taking Probanthine?
Mr. Dela Isla, which of the following
statement of the daughter will require A. Avoid hot weathers to prevent
the nurse to give further teaching? heat strokes
B. Never swim on a chlorinated pool
A. I know the hallucinations are parts C. Make sure you limit your fluid
of the disease intake to 1L a day
B. I told her she is wrong and I D. Avoid cold weathers to prevent
explained to her what is right hypothermia
C. I help her do some tasks he cannot
do for himself 83. Which of the following disease would
D. Ill turn off the TV when we go to Probanthine exert the much needed
another room action for control or treatment of the
disorder?
78. Which of the following is most
important discharge teaching for Mr. Dela A. Urinary retention
Isla B. Peptic Ulcer Disease
C. Ulcerative Colitis
A. Emergency Numbers D. Glaucoma
B. Drug Compliance
C. Relaxation technique SITUATION : Mr. Franco, 70 years old,
D. Dietary prescription suddenly could not lift his spoons nor
speak at breakfast. He was rushed to the
hospital unconscious. His diagnosis was D. Clients orientation to time and
CVA. space will be much affected
84. Which of the following is the most
important assessment during the acute SITUATION : a 20 year old college
stage of an unconscious patient like Mr. student was rushed to the ER of PGH
Franco? after he fainted during their ROTC drill.
Complained of severe right iliac pain.
A. Level of awareness and response to Upon palpation of his abdomen, Ernie
pain jerks even on slight pressure. Blood test
B. Papillary reflexes and response to was ordered. Diagnosis is acute
sensory stimuli appendicitis.
C. Coherence and sense of hearing
D. Patency of airway and 89. Which result of the lab test will be
adequacy of respiration significant to the diagnosis?
A. RBC : 4.5 TO 5 Million / cu. mm.
85. Considering Mr. Franco’s conditions, B. Hgb : 13 to 14 gm/dl.
which of the following is most important C. Platelets : 250,000 to 500,000
to include in preparing Franco’s bedside cu.mm.
equipment? D. WBC : 12,000 to 13,000/cu.mm

A. Hand bell and extra bed linen 90. Stat appendectomy was indicated.
B. Sandbag and trochanter rolls Pre op care would include all of the
C. Footboard and splint following except?
D. Suction machine and gloves
A. Consent signed by the father
86. What is the rationale for giving Mr. B. Enema STAT
Franco frequent mouth care? C. Skin prep of the area including the
pubis
A. He will be thirsty considering that D. Remove the jewelries
he is doesn’t drink enough fluids
B. To remove dried blood when 91. Pre-anesthetic med of Demerol and
tongue is bitten during a seizure atrophine sulfate were ordered to :
C. The tactile stimulation during
mouth care will hasten return to A. Allay anxiety and apprehension
consciousness B. Reduce pain
D. Mouth breathing is used by C. Prevent vomiting
comatose patient and it’ll cause D. Relax abdominal muscle
oral mucosa dying and cracking. 92. Common anesthesia for
appendectomy is
87. One of the complications of
prolonged bed rest is decubitus ulcer. A. Spinal
Which of the following can best prevent B. General
its occurrence? C. Caudal
D. Hypnosis
A. Massage reddened areas with
lotion or oils 93. Post op care for appendectomy
B. Turn frequently every 2 hours include the following except
C. Use special water mattress
D. Keep skin clean and dry A. Early ambulation
B. Diet as tolerated after fully
88. If Mr. Franco’s Right side is weak, conscious
What should be the most accurate C. Nasogastric tube connect to
analysis by the nurse? suction
D. Deep breathing and leg exercise
A. Expressive aphasia is
prominent on clients with right 94. Peritonitis may occur in ruptured
sided weakness appendix and may cause serious
B. The affected lobe in the patient is problems which are
the Right lobe
C. The client will have problems in 1. Hypovolemia, electrolyte
judging distance and proprioception imbalance
2. Elevated temperature, weakness A. Occurs suddenly and reversible
and diaphoresis B. Is progressive and reversible
3. Nausea and vomiting, rigidity of C. tends to be progressive and
the abdominal wall irreversible
4. Pallor and eventually shock D. Occurs suddenly and irreversible
A. 1 and 2 100. Which behavior results from organic
B. 2 and 3 psychoses?
C. 1,2,3
D. All of the above A. Memory deficit
B. Disorientation
95. If after surgery the patient’s C. Impaired Judgement
abdomen becomes distended and no D. Inappropriate affect
bowel sounds appreciated, what would
be the most suspected complication?

A. Intussusception
B. Paralytic Ileus
C. Hemorrhage
D. Ruptured colon

96. NGT was connected to suction. In


caring for the patient with NGT, the nurse
must

A. Irrigate the tube with saline as


ordered
B. Use sterile technique in irrigating
the tube
C. advance the tube every hour to
avoid kinks
D. Offer some ice chips to wet lips

97. When do you think the NGT tube be


removed?

A. When patient requests for it


B. Abdomen is soft and patient asks
for water
C. Abdomen is soft and flatus has PNLE Medical
been expelled
D. B and C only
Surgical Nursing
Situation: Amanda is suffering from
chronic arteriosclerosis Brain syndrome
she fell while getting out of the bed one
Exam 2
morning and was brought to the hospital, 1. After a cerebrovascular accident, a 75
and she was diagnosed to have yr old client is admitted to the health
cerebrovascular thrombosis thus care facility. The client has left-sided
transferred to a nursing home. weakness and an absent gag reflex. He’s
98. What do you call a STROKE that incontinent and has a tarry stool. His
manifests a bizarre behavior? blood pressure is 90/50 mm Hg, and his
hemoglobin is 10 g/dl. Which of the
A. Inorganic Stroke following is a priority for this client?
B. Inorganic Psychoses
C. Organic Stroke A. checking stools for occult blood
D. Organic Psychoses B. performing range-of-motion
exercises to the left side
99. The main difference between chronic C. keeping skin clean and dry
and organic brain syndrome is that the D. elevating the head of the bed
former to 30 degrees
ANS: D C. reducing sodium increases
Because the client’s gag reflex is potassium absorption
absent, elevating the head of the bed D. reducing sodium decreases
to 30 degrees helps minimize the edema
client’s risk of aspiration. Checking
the stools, performing ROM exercises, ANS: D
and keeping the skin clean and dry Reducing sodium intake reduces fluid
are important, but preventing retention. Fluid retention increases blood
aspiration through positioning is the volume, which changes blood vessel
priority permeability and allows plasma to move
into interstitial tissue, causing edema.
2. The nurse is caring for a client with a Urea nitrogen excretion can be increased
colostomy. The client tells the nurse that only by improved renal function. Sodium
he makes small pin holes in the drainage intake doesn’t affect the glomerular
bag to help relieve gas. The nurse should filtration rate. Potassium absorption is
teach him that this action: improved only by increasing the
glomerular filtration rate; it isn’t affected
A. destroys the odor-proof seal by sodium intake.
B. wont affect the colostomy system
C. is appropriate for relieving the gas 5. The nurse is caring for a client with a
in a colostomy system cerebral injury that impaired his speech
D. destroys the moisture barrier seal and hearing. Most likely, the client has
experienced damage to the:
ANS: A
Any hole, no matter how small, will A. frontal lobe
destroy the odor-proof seal of a B. parietal lobe
drainage bag. Removing the bag or C. occipital lobe
unclamping it is the only appropriate D. temporal lobe
method for relieving gas.
ANS: D
3. When assessing the client with celiac The portion of the cerebrum that
disease, the nurse can expect to find controls speech and hearing is the
which of the following? temporal lobe. Injury to the frontal lobe
causes personality changes, difficulty
A. steatorrhea speaking, and disturbance in memory,
B. jaundiced sclerae reasoning, and concentration. Injury to
C. clay-colored stools the parietal lobe causes sensory
D. widened pulse pressure alterations and problems with spatial
relationships. Damage to the occipital
ANS: A lobe causes vision disturbances
because celiac disease destroys the
absorbing surface of the intestine, fat 6. The nurse is assessing a
isn’t absorbed but is passed in the postcraniotomy client and finds the urine
stool. Steatorrhea is bulky, fatty stools output from a catheter is 1500 ml for the
that have a foul odor. Jaundiced 1st hour and the same for the 2nd hour.
sclerae result from elevated bilirubin The nurse should suspect:
levels. Clay-colored stools are seen
with biliary disease when bile flow is A. Cushing’s syndrome
blocked. Celiac disease doesn’t cause B. Diabetes mellitus
a widened pulse pressure. C. Adrenal crisis
D. Diabetes insipidus
4. A client is hospitalized with a diagnosis
of chronic glomerulonephritis. The client ANS: D
mentions that she likes salty foods. The Diabetes insipidus is an abrupt onset
nurse should warn her to avoid foods of extreme polyuria that commonly
containing sodium because: occurs in clients after brain surgery.
Cushing’s syndrome is excessive
A. reducing sodium promotes urea glucocorticoid secretion resulting in
nitrogen excretion sodium and water retention. Diabetes
B. reducing sodium improves her mellitus is a hyperglycemic state
glomerular filtration rate marked by polyuria, polydipsia, and
polyphagia. Adrenal crisis is from decreased cardiac output because
undersecretion of glucocorticoids his blood pressure is normal. Although
resulting in profound hypoglycemia, the client’s serum glucose is elevated,
hypovolemia, and hypotension food isn’t a priority because fluids and
insulin should be administered to lower
7. The nurse is providing postprocedure the serum glucose level. Therefore, a
care for a client who underwent diagnosis of Imbalanced Nutrition: Less
percutaneous lithotripsy. In this then body requirements isn’t appropriate.
procedure, an ultrasonic probe inserted A temperature of 100.6º F isn’t life
through a nephrostomy tube into the threatening, eliminating ineffective
renal pelvis generates ultra-high- thermoregulation as the top priority.
frequency sound waves to shatter renal
calculi. The nurse should instruct the 9. Capillary glucose monitoring is being
client to: performed every 4 hours for a client
diagnosed with diabetic ketoacidosis.
A. limit oral fluid intake for 1 to 2 Insulin is administered using a scale of
weeks regular insulin according to glucose
B. report the presence of fine, results. At 2 p.m., the client has a
sandlike particles through the capillary glucose level of 250 mg/dl for
nephrostomy tube. which he receives 8 U of regular insulin.
C. Notify the physician about The
cloudy or foul smelling urine nurse should expect the dose’s:
D. Report bright pink urine within 24
hours after the procedure A. onset to be at 2 p.m. and its peak
at 3 p.m.
ANS: C B. onset to be at 2:15 p.m. and its
The client should report the presence of peak at 3 p.m.
foulsmelling or cloudy urine. Unless C. onset to be at 2:30 p.m. and its
contraindicated, the client should be peak at 4 p.m.
instructed to drink large quantities of D. onset to be at 4 p.m. and its peak
fluid each day to flush the kidneys. Sand- at 6 p.m.
like debris is normal because of residual
stone products. Hematuria is common ANS: C
after lithotripsy. Regular insulin, which is a short-acting
insulin, has an onset of 15 to 30 minutes
8. A client with a serum glucose level of and a peak of 2 to 4 hours. Because the
618 mg/dl is admitted to the facility. He’s nurse gave the insulin at 2 p.m., the
awake and oriented, has hot dry skin, expected onset would be from 2:15 to
and has the following vital signs: 2:30 p.m. and the peak from 4 p.m. to 6
temperature of 100.6º F (38.1º C), heart p.m.
rate of 116 beats/minute, and blood
pressure of 108/70 mm Hg. Based on 10. A client with a head injury is being
these assessment findings, which nursing monitored for increased intracranial
diagnosis takes the highest priority? pressure (ICP). His blood pressure is
90/60 mmHG and the ICP is 18 mmHg;
A. deficient fluid volume related therefore his cerebral perfusion pressure
to osmotic diuresis (CPP) is:
B. decreased cardiac output related to
elevated heart rate A. 52 mm Hg
C. imbalanced nutrition: Less than B. 88 mm Hg
body requirements related to insulin C. 48 mm Hg
deficiency D. 68 mm Hg
D. ineffective thermoregulation
related to dehydration ANS: A
CPP is derived by subtracting the ICP
ANS: A from the mean arterial pressure (MAP).
A serum glucose level of 618 mg/dl For adequate cerebral perfusion to take
indicates hyperglycemia, which causes place, the minimum goal is 70 mmHg.
polyuria and deficient fluid volume. In The MAP is derived using the following
this client, tachycardia is more likely to formula:
result from deficient fluid volume than MAP = ((diastolic blood pressure x 2) +
systolic blood pressure) / 3 A. basilar
MAP = ((60 x2) + 90) / 3 B. temporal
MAP = 70 mmHg C. occipital
To find the CPP, subtract the client’s ICP D. parietal
from the MAP; in this case , 70 mmHg –
18 mmHg = 52 mmHg. ANS: A
Ottorrhea and rhinorrhea are classic
11. A 52 yr-old female tells the nurse that signs of basilar skull fracture. Injury to
she has found a painless lump in her the dura commonly occurs with this
right breast during her monthly self- fracture, resulting in cerebrospinal fluid
examination. Which assessment finding (CSF) leaking through the ears and nose.
would strongly suggest that this client’s Any fluid suspected of being CSF should
lump is cancerous? be checked for glucose or have a halo
test done
A. eversion of the right nipple and a
mobile mass 14. A male client should be taught about
B. nonmobile mass with irregular testicular examinations:
edges
C. mobile mass that is oft and easily A. when sexual activity starts
delineated B. after age 60
D. nonpalpable right axillary lymph C. after age 40
nodes D. before age 20

ANS: B ANS: D
Breast cancer tumors are fixed, hard, and Testicular cancer commonly occurs in
poorly delineated with irregular edges. men between ages 20 and 30. A male
Nipple retraction —not eversion—may be client should be taught how to perform
a sign of cancer. A mobile mass that is testicular self-examination before age 20,
soft and easily delineated is most often a preferably when he enters his teens.
fluid-filled benigned cyst. Axillary lymph
nodes may or may not be palpable on 15. Before weaning a client from a
initial detection of a cancerous mass. ventilator, which assessment parameter
is most important for the nurse to
review?
12. A Client is scheduled to have a
descending colostomy. He’s very anxious A. fluid intake for the last 24 hours
and has many questions regarding the B. baseline arterial blood gas
surgical procedure, care of stoma, and (ABG) levels
lifestyle changes. It would be most C. prior outcomes of weaning
appropriate for the nurse to make a D. electrocardiogram (ECG) results
referral to which member of the health
care team? ANS: B
Before weaning a client from mechanical
A. Social worker ventilation, it’s most important to have a
B. registered dietician baseline ABG levels. During the weaning
C. occupational therapist process, ABG levels will be checked to
D. enterostomal nurse therapist assess how the client is tolerating the
procedure. Other assessment parameters
ANS: D are less critical. Measuring fluid volume
An enterostomal nurse therapist is a intake and output is always important
registered nurse who has received when a client is being mechanically
advance education in an accredited ventilated. Prior attempts at weaning and
program to care for clients with stomas. ECG results are documented on the
The enterostomal nurse therapist can client’s record, and the nurse can refer to
assist with selection of an appropriate them before the weaning process begins.
stoma site, teach about stoma care, and
provide emotional support.
16. The nurse is speaking to a group of
13. Ottorrhea and rhinorrhea are most women about early detection of breast
commonly seen with which type of skull cancer. The average age of the women in
fracture? the group is 47. Following the American
Cancer Society (ACS) guidelines, the pin care and assess for development of
nurse should recommend that the neurovascular complications.
women:
19. A client is hospitalized with a
A. perform breast self-examination diagnosis of chronic renal failure. An
annually arteriovenous fistula was created in his
B. have a mammogram annually left arm for hemodialysis. When
C. have a hormonal receptor assay preparing the client for discharge, the
annually nurse should reinforce which dietary
D. have a physician conduct a clinical instruction?
evaluation every 2 years
A. “Be sure to eat meat at every
ANS: B meal.”
According to the ACS guidelines, “Women B. “Monitor your fruit intake and eat
older than age 40 should perform breast plenty of bananas.”
selfexamination monthly (not annually).” C. “Restrict your salt intake.”
The hormonal receptor assay is done on D. “Drink plenty of fluids.”
a known breast tumor to determine 20. The nurse is caring for a client who
whether the tumor is estrogen- or has just had a modified radical
progesterone-dependent. mastectomy with immediate
reconstruction. She’s in her 30s and has
17. When caring for a client with tow children. Although she’s worried
esophageal varices, the nurse knows that about her future, she seems to be
bleeding in this disorder usually stems adjusting well to her diagnosis. What
from: should the nurse do to support
her coping?
A. esophageal perforation
B. pulmonary hypertension A. Tell the client’s spouse or partner
C. portal hypertension to be supportive while she recovers.
D. peptic ulcers B. Encourage the client to proceed
with the next phase of treatment.
ANS: C C. Recommend that the client remain
Increased pressure within the portal cheerful for the sake of her children.
veins causes them to bulge, leading to D. Refer the client to the American
rupture and bleeding into the lower Cancer Society’s Reach for Recovery
esophagus. Bleeding associated with program or another support program.
esophageal varices doesn’t stem from 21. A 21 year-old male has been seen in
esophageal perforation, pulmonary the clinic for a thickening in his right
hypertension, or peptic ulcers testicle. The physician ordered a human
chorionic gonadotropin (HCG) level. The
18. A 49-yer-old client was admitted for nurse’s explanation to the client should
surgical repair of a Colles’ fracture. An include the fact that:
external fixator was placed during
surgery. The surgeon explains that this A. The test will evaluate prostatic
method of repair: function.
B. The test was ordered to identify the
A. has very low complication rate site of a possible infection.
B. maintains reduction and C. The test was ordered because
overall hand function clients who have testicular cancer
C. is less bothersome than a cast has elevated levels of HCG.
D. is best for older people D. The test was ordered to evaluate
the testosterone level.
ANS: B 22. A client is receiving captopril
Complex intra-articular fractures are (Capoten) for heart failure. The nurse
repaired with external fixators because should notify the physician that the
they have a better long-term outcome medication therapy is ineffective if an
than those treated with casting. This is assessment reveals:
especially true in a young client. The
incidence of complications, such as pin A. A skin rash.
tract infections and neuritis, is 20% to B. Peripheral edema.
60%. Clients must be taught how to do C. A dry cough.
D. Postural hypotension. diabetes mellitus. Which technique
23. Which assessment finding indicates demonstrates surgical asepsis?
dehydration?
A. Putting on sterile gloves then
A. Tenting of chest skin when pinched. opening a container of sterile saline.
B. Rapid filling of hand veins. B. Cleaning the wound with a circular
C. A pulse that isn’t easily obliterated. motion, moving from outer circles
D. Neck vein distention toward the center.
24. The nurse is teaching a client with a C. Changing the sterile field after
history of atherosclerosis. To decrease sterile water is spilled on it.
the risk of atherosclerosis, the nurse D. Placing a sterile dressing ½” (1.3
should encourage the client to: cm) from the edge of the sterile field.
30. A client with a forceful, pounding
A. Avoid focusing on his weight. heartbeat is diagnosed with mitral valve
B. Increase his activity level. prolapse. This client should avoid which
C. Follow a regular diet. of the following?
D. Continue leading a high-stress
lifestyle. A. high volumes of fluid intake
25. For a client newly diagnosed with B. aerobic exercise programs
radiationinduced thrombocytopenia, the C. caffeine-containing products
nurse should include which intervention D. foods rich in protein
in the plan of care? 31. A client with a history of hypertension
is diagnosed with primary
A. Administer aspirin if the hyperaldosteronism. This diagnosis
temperature exceeds 38.8º C. indicates that the client’s hypertension is
B. Inspect the skin for petechiae once caused by excessive hormone secretion
every shift. from which organ?
C. Provide for frequent periods of rest.
D. Place the client in strict isolation. A. adrenal cortex
26. A client is chronically short of breath B. pancreas
and yet has normal lung ventilation, clear C. adrenal medulla
lungs, and an arterial oxygen saturation D. parathyroid
(SaO2) 96% or better. The client most 32. A client has a medical history of
likely has: rheumatic fever, type 1 (insulin
dependent) diabetes mellitus,
A. poor peripheral perfusion hypertension, pernicious anemia, and
B. a possible Hematologic problem appendectomy. She’s admitted to the
C. a psychosomatic disorder hospital and undergoes mitral valve
D. left-sided heart failure replacement surgery. After discharge, the
27. For a client in addisonian crisis, it client is scheduled for a tooth extraction.
would be very risky for a nurse to Which history finding is a major risk
administer: factor for infective endocarditis?

A. potassium chloride A. appendectomy


B. normal saline solution B. pernicious anemia
C. hydrocortisone C. diabetes mellitus
D. fludrocortisone D. valve replacement
28. The nurse is reviewing the laboratory 33. A 62 yr-old client diagnosed with
report of a client who underwent a bone pyelonephritis and possible septicemia
marrow biopsy. The finding that would has had five urinary tract infections over
most strongly support a diagnosis of the past two years. She’s fatigued from
acute leukemia is the existence of a large lack of sleep; urinates frequently, even
number of immature: during the night; and has lost weight
recently. Test reveal the following:
A. lymphocytes sodium level 152 mEq/L, osmolarity 340
B. thrombocytes mOsm/L, glucose level 125 mg/dl, and
C. reticulocytes potassium level 3.8 mEq/L. which of the
D. leukocytes following nursing diagnoses is most
29. The nurse is performing wound care appropriate for this client?
on a foot ulcer in a client with type 1
A. Deficient fluid volume related to 37. A client is admitted to the health care
inability to conserve water facility with active tuberculosis. The
B. Imbalanced nutrition: less than nurse should include which intervention
body requirements related to in the plan of care?
hypermetabolic state
C. Deficient fluid volume related to A. Putting on a mask when entering
osmotic diuresis induced by the client’s room.
hypernatremia B. Instructing the client to wear a
D. Imbalanced nutrition: less than mask at all times
body requirements related to C. Wearing a gown and gloves when
catabolic effects of insulin deficiency providing direct care
34. A 20 yr-old woman has just been D. Keeping the door to the client’s
diagnosed with Crohn’s disease. She has room open to observe the client
lost 10 lb (4.5 kg) and has cramps and 38. The nurse is caring for a client who
occasional diarrhea. The nurse should underwent a subtotal gastrectomy 24
include which of the following when hours earlier. The client has a nasogastric
doing a nutritional assessment? (NG) tube. The nurse should:

A. Let the client eat as desired during A. Apply suction to the NG tube every
the hospitalization. hour.
B. Weight the client daily. B. Clamp the NG tube if the client
C. Ask the client to list what she eats complains of nausea.
during a typical day. C. Irrigate the NG tube gently with
D. Place the client on I & O status and normal saline solution.
draw blood for electrolyte levels. D. Reposition the NG tube if pulled
35. When instructions should be included out.
in the discharge teaching plan for a client 39. Which statement about fluid
after thyroidectomy for Grave’s disease? replacement is accurate for a client with
hyperosmolar hyperglycemic nonketotic
A. Keep an accurate record of intake syndrome (HHNS)?
and output.
B. Use nasal desmopressin acetate A. administer 2 to 3 L of IV fluid
DDAVP). rapidly
C. Be sure to get regulate follow-up B. administer 6 L of IV fluid over the
care. first 24 hours
D. Be sure to exercise to improve C. administer a dextrose solution
cardiovascular fitness. containing normal saline solution
36. A client comes to the emergency D. administer IV fluid slowly to
department with chest pain, dyspnea, prevent circulatory overload and
and an irregular heartbeat. An collapse
electrocardiogram shows a heart rate of 40. Which of the following is an adverse
110 beats/minute (sinus tachycardia) reaction to glipizide (Glucotrol)?
with frequent premature ventricular
contractions. Shortly after admission, the A. headache
client has ventricular tachycardia and B. constipation
becomes unresponsive. After successful C. hypotension
resuscitation, the client is taken to the D. photosensitivity
intensive care unit. Which nursing 41. The nurse is caring for four clients on
diagnosis is appropriate at this time? a stepdown intensive care unit. The client
at the highest risk for developing
A. Deficient knowledge related to nosocomial pneumonia is the one who:
interventions used to treat acute
illness A. has a respiratory infection
B. Impaired physical mobility related B. is intubated and on a ventilator
to complete bed rest C. has pleural chest tubes
C. Social isolation related to restricted D. is receiving feedings through a
visiting hours in the intensive care jejunostomy tube
unit 42. The nurse is teaching a client with
D. Anxiety related to the threat of chronic bronchitis about breathing
death exercises. Which of the following should
the nurse include in the teaching?
A. Make inhalation longer than C. Increased absorption of vit D and
exhalation. excretion of vit E
B. Exhale through an open mouth. D. Increased absorption of vit E and
C. Use diaphragmatic breathing. excretion of Vit D
D. Use chest breathing. 48. A visiting nurse is performing home
43. A client is admitted to the hospital assessment for a 59-yr old man recently
with an exacerbation of her chronic discharged after hip replacement surgery.
systemic lupus erythematosus (SLE). She Which home assessment finding warrants
gets angry when her call bell isn’t health promotion teaching from the
answered immediately. The most nurse?
appropriate response to her would be:
A. A bathroom with grab bars for the
A. “You seem angry. Would you like to tub and toilet
talk about it?” B. Items stored in the kitchen so that
B. “Calm down. You know that stress reaching up and bending down aren’t
will make your symptoms worse.” necessary
C. “Would you like to talk about the C. Many small, unsecured area rugs
problem with the nursing D. Sufficient stairwell lighting, with
supervisor?” switches to the top and bottom of the
D. “I can see you’re angry. I’ll come stairs
back when you’ve calmed down.” 49. A client with autoimmune
44. On a routine visit to the physician, a thrombocytopenia and a platelet count of
client with chronic arterial occlusive 800/uL develops epistaxis and melena.
disease reports stopping smoking after Treatment with corticosteroids and
34 years. To relive symptoms of immunoglobulins has been unsuccessful,
intermittent claudication, a condition and the physician recommends a
associated with chronic arterial occlusive splenectomy. The client states, “I don’t
disease, the nurse should recommend need surgery—this will go away on its
which additional measure? own.” In considering her response to the
client, the nurse must depend on the
A. Taking daily walks. ethical principle of:
B. Engaging in anaerobic exercise.
C. Reducing daily fat intake to less A. beneficence
than 45% of total calories B. autonomy
D. Avoiding foods that increase levels C. advocacy
of highdensity lipoproteins (HDLs) D. justice
45. A physician orders gastric 50. Which of the following is t he most
decompression for a client with small critical intervention needed for a client
bowel obstruction. The nurse should plan with myxedema coma?
for the suction to be:
A. Administering and oral dose of
A. low pressure and intermittent levothyroxine (Synthroid)
B. low pressure and continuous B. Warming the client with a warming
C. high pressure and continuous blanket
D. high pressure and intermittent C. Measuring and recording accurate
46. Which nursing diagnosis is most intake and output
appropriate for an elderly client with D. Maintaining a patent airway
osteoarthritis? 51. Because diet and exercise have failed
to control a 63 yr-old client’s blood
A. Risk for injury glucose level, the client is prescribed
B. Impaired urinary elimination glipizide (Glucotrol). After oral
C. Ineffective breathing pattern administration, the onset of action is:
D. Imbalanced nutrition: less than
body requirements A. 15 to 30 minutes
47. Parathyroid hormone (PTH) has which B. 30 to 60 minutes
effects on the kidney? C. 1 to 1 ½ hours
D. 2 to 3 hours
A. Stimulation of calcium reabsorption 52. A client with pneumonia is receiving
and phosphate excretion supplemental oxygen, 2 L/min via nasal
B. Stimulation of phosphate cannula. The client’s history includes
reabsorption and calcium excretion chronic obstructive pulmonary disease
(COPD) and coronary artery disease. D. order soft restraints from the
Because of these findings, the nurse storeroom
closely monitors the oxygen flow and the 57. For the first 72 hours after
client’s respiratory status. Which thyroidectomy surgery, the nurse would
complication may arise if the client assess the client for Chvostek’s sign and
receives a high oxygen concentration? Trousseau’s sign because they indicate
which of the following?
A. Apnea
B. Anginal pain A. hypocalcemia
C. Respiratory alkalosis B. hypercalcemia
D. Metabolic acidosis C. hypokalemia
53. A client with type 1 diabetes mellitus D. Hyperkalemia
has been on a regimen of multiple daily 58. In a client with enteritis and frequent
injection therapy. He’s being converted to diarrhea, the nurse should anticipate an
continuous subcutaneous insulin therapy. acidbase imbalance of:
While teaching the client bout continuous
subcutaneous insulin therapy, the nurse A. respiratory acidosis
would be accurate in telling him the B. respiratory alkalosis
regimen includes the use of: C. metabolic acidosis
D. metabolic alkalosis
A. intermediate and long-acting 59. When caring for a client with the
insulins nursing diagnosis Impaired swallowing
B. short and long-acting insulins related to neuromuscular impairment,
C. short-acting only the nurse should:
D. short and intermediate-acting
insulins A. position the client in a supine
54. a client who recently had a position
cerebrovascular accident requires a cane B. elevate the head of the bed 90
to ambulate. When teaching about cane degrees during meals
use, the rationale for holding a cane on C. encourage the client to remove
the uninvolved side is to: dentures
D. encourage thin liquids for dietary
A. prevent leaning intake
B. distribute weight away from the 60. A nurse is caring for a client who has
involved side a tracheostomy and temperature of 39º
C. maintain stride length C. which intervention will most likely
D. prevent edema lower the client’s arterial blood oxygen
55. A client with a history of an anterior saturation?
wall myocardial infarction is being
transferred from the coronary care unit A. Endotracheal suctioning
(CCU) to the cardiac stepdown unit B. Encouragement of coughing
(CSU). While giving report to the CSU C. Use of cooling blanket
nurse, the CCU nurse says, “His D. Incentive spirometry
pulmonary artery wedge pressures have 61. A client with a solar burn of the chest,
been in the high normal range.” The CSU back, face, and arms is seen in urgent
nurse should be especially observant for: care. The nurse’s primary concern should
be:
A. hypertension
B. high urine output A. fluid resuscitation
C. dry mucous membranes B. infection
D. pulmonary crackles C. body image
56. The nurse is caring for a client with a D. pain management
fractures hip. The client is combative, 62. Which statement is true about
confused, and trying to get out of bed. crackles?
The nurse should:
A. They’re grating sounds.
A. leave the client and get help B. They’re high-pitched, musical
B. obtain a physician’s order to squeaks.
restrain the client C. They’re low-pitched noises that
C. read the facility’s policy on sound like snoring.
restraints
D. They may be fine, medium, or to the hair shafts close to the scalp.
course. These findings suggest that the client
63. A woman whose husband was suffers from:
recently diagnosed with active
pulmonary tuberculosis (TB) is a A. scabies
tuberculin skin test converter. B. head lice
Management of her care would include: C. tinea capitis
D. impetigo
A. scheduling her for annual 68. Following a small-bowel resection, a
tuberculin skin testing client develops fever and anemia. The
B. placing her in quarantine until surface surrounding the surgical wound is
sputum cultures are negative warm to touch and necrotizing fasciitis is
C. gathering a list of persons with suspected. Another manifestation that
whom she has had recent contact would most suggest necrotizing fasciitis
D. advising her to begin prophylactic is:
therapy with isoniazid (INH)
64. The nurse is caring for a client who A. erythema
ahs had an above the knee amputation. B. leukocytosis
The client refuses to look at the stump. C. pressure-like pain
When the nurse attempts to speak with D. swelling
the client about his surgery, he tells the 69. A 28 yr-old nurse has complaints of
nurse that he doesn’t wish to discuss it. itching and a rash of both hands. Contact
The client also refuses to have his family dermatitis is initially suspected. The
visit. The nursing diagnosis that best diagnosis is confirmed if the rash
describes the client’s problem is: appears:

A. Hopelessness A. erythematous with raised papules


B. Powerlessness B. dry and scaly with flaking skin
C. Disturbed body image C. inflamed with weeping and crusting
D. Fear lesions
65. A client with three children who is still D. excoriated with multiple fissures
I the child bearing years is admitted for 70. When assessing a client with partial
surgical repair of a prolapsed bladder. thickness burns over 60% of the body,
The nurse would find that the client which of the following should the nurse
understood the surgeon’s preoperative report immediately?
teaching when the client states:
A. Complaints of intense thirst
A. “If I should become pregnant B. Moderate to severe pain
again, the child would be delivered C. Urine output of 70 ml the 1st hour
by cesarean delivery.” D. Hoarseness of the voice
B. “If I have another child, the 71. A client is admitted to the hospital
procedure may need to be repeated.” following a burn injury to the left hand
C. “This surgery may render me and arm. The client’s burn is described as
incapable of conceiving another white and leathery with no blisters.
child.” Which degree of severity is this burn?
D. “This procedure is accomplished in
two separate surgeries.” A. first-degree burn
66. A client experiences problems in B. second-degree burn
body temperature regulation associated C. third-degree burn
with a skin impairment. Which gland is D. fourth-degree burn
most likely involved? 72. The nurse is caring for client with a
new donor site that was harvested to
A. Eccrine treat a new burn. The nurse position the
B. Sebaceous client to:
C. Apocrine
D. Endocrine A. allow ventilation of the site
67. A school cafeteria worker comes to B. make the site dependent
the physician’s office complaining of C. avoid pressure on the site
severe scalp itching. On inspection, the D. keep the site fully covered
nurse finds nail marks on the scalp and 73. a 45-yr-old auto mechanic comes to
small light-colored round specks attached the physician’s office because an
exacerbation of his psoriasis is making it 78. The nurse is caring for a bedridden,
difficult to work. He tells the nurse that elderly adult. To prevent pressure ulcers,
his finger joints are stiff and sore in the which intervention should the nurse
morning. The nurse should respond by: include in the plan of care?

A. Inquiring further about this A. Turn and reposition the client a


problem because psoriatic arthritis minimum of every 8 hours.
can accompany psoriasis vulgaris B. Vigorously massage lotion into
B. Suggesting he take aspirin for relief bony prominences.
because it’s probably early C. Post a turning schedule at the
rheumatoid arthritis client’s bedside.
C. Validating his complaint but D. Slide the client, rather than lifting
assuming it’s an adverse effect of his when turning.
vocation 79. Following a full-thickeness (3rd
D. Asking him if he has been degree) burn of his left arm, a client is
diagnosed or treated for carpal treated with artificial skin. The client
tunnel syndrome understands postoperative care of the
74. The nurse is providing home care artificial skin when he states that during
instructions to a client who has recently the first 7 days after the procedure, he’ll
had a skin graft. Which instruction is restrict:
most important for the client to
remember? A. range of motion
B. protein intake
A. Use cosmetic camouflage C. going outdoors
techniques. D. fluid ingestion
B. Protect the graft from direct 80. A client received burns to his entire
sunlight. back and left arm. Using the Rule of
C. Continue physical therapy. Nines, the nurse can calculate that he
D. Apply lubricating lotion to the graft has sustained burns on what percentage
site. of his body?
75. a 28 yr-old female nurse is seen in
the employee health department for mild A. 9%
itching and rash of both hands. Which of B. 18%
the following could be causing this C. 27%
reaction? D. 36%
81. The nurse is providing care for a
A. possible medication allergies client who has a sacral pressure ulcer
B. current life stressors she may be with wet-to-dry dressing. Which guideline
experiencing is appropriate for a wet-to-dry dressing?
C. chemicals she may be using and
use of latex gloves A. The wound should remain moist
D. recent changes made in laundry form the dressing.
detergent or bath soap. B. The wet-to-dry dressing should be
76. The nurse assesses a client with tightly packed into the wound.
urticaria. The nurse understands that C. The dressing should be allowed to
urticaria is another name for: dry out before removal.
D. A plastic sheet-type dressing
A. hives should cover the wet dressing.
B. a toxin 82. While in skilled nursing facility, a
C. a tubercle client contracted scabies, which is
D. a virus diagnosed the day after discharge. The
77. A client with psoriasis visits the client is living at her daughter’s home
dermatology clinic. When inspecting the with six other persons. During her visit to
affected areas, the nurse expects to see the clinic, she asks a staff nurse, “What
which type of secondary lesion? should my family do?” the most accurate
response from the nurse is:
A. scale
B. crust A. “All family members will need to be
C. ulcer treated.”
D. scar
B. “If someone develops symptoms, for the presence of blood in the
tell him to see a physician right cerebrospinal fluid (CSF) and to measure
away.” CSF pressure. Which result would
C. “Just be careful not to share linens indicate n abnormality?
and towels with family members.”
D. “After you’re treated, family A. The presence of glucose in the CSF.
members won’t be at risk for B. A pressure of 70 to 200 mm H2O
contracting scabies.” C. The presence of red blood cells
83. In an industrial accident, client who (RBCs) in the first specimen tube
weighs 155 lb (70.3 kg) sustained full- D. A pressure of 00 to 250 mmH2O
thickness burns over 40% of his body. 88. The nurse is administering eyedrops
He’s in the burn unit receiving fluid to a client with glaucoma. To achieve
resuscitation. Which observation shows maximum absorption, the nurse should
that the fluid resuscitation is benefiting instill the eyedrop into the:
the client?
A. conjunctival sac
A. A urine output consistently above B. pupil
100 ml/hour. C. sclera
B. A weight gain of 4 lb (1.8 kg) in 24 D. vitreous humor
hours. 89. A 52 yr-old married man with two
C. Body temperature readings all adolescent children is beginning
within normal limits rehabilitation following a cerebrovascular
D. An electrocardiogram (ECG) accident. As the nurse is planning the
showing no arrhythmias. client’s care, the nurse should recognize
84. The nurse is reviewing the laboratory that his condition will affect:
results of a client with rheumatoid
arthritis. Which of the following A. only himself
laboratory results should the nurse B. only his wife and children
expect to find? C. him and his entire family
D. no one, if he has complete
A. Increased platelet count recovery
B. Elevated erythrocyte sedimentation 90. Which action should take the highest
rate (ESR) priority when caring for a client with
C. Electrolyte imbalance hemiparesis caused by a cerebrovascular
D. Altered blood urea nitrogen (BUN) accident (CVA)?
and creatinine levels
85. Which nursing diagnosis takes the A. Perform passive range-of-motion
highest priority for a client with (ROM) exercises.
Parkinson’s crisis? B. Place the client on the affected
side.
A. Imbalanced nutrition: less than C. Use hand rolls or pillows for
body requirements support.
B. Ineffective airway clearance D. Apply antiembolism stockings
C. Impaired urinary elimination 91. The nurse is formulating a teaching
D. Risk for injury plan for a client who has just experienced
86. A client with a spinal cord injury and a transient ischemic attack (TIA). Which
subsequent urine retention receives fact should the nurse include in the
intermittent catheterization every 4 teaching plan?
hours. The average catheterized urine
volume has been 550 ml. The nurse A. TIA symptoms may last 24 to 48
should plan to: hours.
B. Most clients have residual effects
A. Increase the frequency of the after having a TIA.
catheterizations. C. TIA may be a warning that the
B. Insert an indwelling urinary client may have cerebrovascular
catheter accident (CVA)
C. Place the client on fluid restrictions D. The most common symptom of TIA
D. Use a condom catheter instead of is the inability to speak.
an invasive one. 92. The nurse has just completed
87.The nurse is caring for a client who is teaching about postoperative activity to
to undergo a lumbar puncture to assess a client who is going to have a cataract
surgery. The nurse knows the teaching of the
has been effective if the client: following signs first?

A. coughs and deep breathes A. pupillary asymmetry


postoperatively B. irregular breathing pattern
B. ties his own shoes C. involuntary posturing
C. asks his wife to pick up his shirt D. declining level of consciousness
from the floor after he drops it. 98. Emergency medical technicians
D. States that he doesn’t need to transport a 28 yr-old iron worker to the
wear an eyepatch or guard to bed emergency department. They tell the
93. The least serious form of brain nurse, “He fell from a two-story building.
trauma, characterized by a brief loss of He has a large contusion on his left chest
consciousness and period of confusion, is and a hematoma in the left parietal area.
called: He has compound fracture of his left
femur and he’s comatose. We intubated
A. contusion him and he’s maintaining an arterial
B. concussion oxygen saturation of 92% by pulse
C. coup oximeter with a manual-resuscitation
D. contrecoup bag.” Which intervention by the nurse
94. When the nurse performs a has the highest priority?
neurologic assessment on Anne Jones,
her pupils are dilated and don’t respond A. Assessing the left leg
to light. B. Assessing the pupils
C. Placing the client in
A. glaucoma Trendelenburg’s position
B. damage to the third cranial nerve D. Assessing the level of
C. damage to the lumbar spine consciousness
D. Bell’s palsy 99. Alzheimer’s disease is the secondary
95. A 70 yr-old client with a diagnosis of diagnosis of a client admitted with
leftsided cerebrovascular accident is myocardial infarction. Which nursing
admitted to the facility. To prevent the intervention should appear on this
development of diffuse osteoporosis, client’s plan of care?
which of the following objectives is most
appropriate? A. Perform activities of daily living for
the client to decease frustration.
A. Maintaining protein levels. B. Provide a stimulating environment.
B. Maintaining vitamin levels. C. Establish and maintain a routine.
C. Promoting weight-bearing exercises D. Try to reason with the client as
D. Promoting range-of-motion (ROM) much as possible.
exercises 100. For a client with a head injury whose
96. A client is admitted with a diagnosis neck has been stabilized, the preferred
of meningitis caused by Neisseria bed position is:
meningitides. The nurse should institute
which type of isolation precautions? A. Trendelenburg’s
B. 30-degree head elevation
A. Contact precautions C. flat
B. Droplet precautions D. side-lying
C. Airborne precautions Answers and Rationales
D. Standard precautions
97. A young man was running along an 1. ANS: D
ocean pier, tripped on an elevated area Because the client’s gag reflex is
of the decking, and struck his head on absent, elevating the head of the bed
the pier railing. According to his friends, to 30 degrees helps minimize the
“He was unconscious briefly and then client’s risk of aspiration. Checking
became alert and behaved as though the stools, performing ROM exercises,
nothing had happened.” Shortly and keeping the skin clean and dry
afterward, he began complaining of a are important, but preventing
headache and asked to be taken to the aspiration through positioning is the
emergency department. If the client’s priority.
intracranial pressure (ICP) is increasing, 2. ANS: A
the nurse would expect to observe which Any hole, no matter how small, will
destroy the odor-proof seal of a the kidneys. Sand-like debris is
drainage bag. Removing the bag or normal because of residual stone
unclamping it is the only appropriate products. Hematuria is common after
method for relieving gas. lithotripsy.
3. ANS: A 8. ANS: A
because celiac disease destroys the A serum glucose level of 618 mg/dl
absorbing surface of the intestine, fat indicates hyperglycemia, which
isn’t absorbed but is passed in the causes polyuria and deficient fluid
stool. Steatorrhea is bulky, fatty volume. In this client, tachycardia is
stools that have a foul odor. more likely to result from deficient
Jaundiced sclerae result from fluid volume than from decreased
elevated bilirubin levels. Clay-colored cardiac output because his blood
stools are seen with biliary disease pressure is normal. Although the
when bile flow is blocked. Celiac client’s serum glucose is elevated,
disease doesn’t cause a widened food isn’t a priority because fluids
pulse pressure. and insulin should be administered to
4. ANS: D lower the serum glucose level.
Reducing sodium intake reduces fluid Therefore, a diagnosis of Imbalanced
retention. Fluid retention increases Nutrition: Less then body
blood volume, which changes blood requirements isn’t appropriate. A
vessel permeability and allows temperature of 100.6º F isn’t life
plasma to move into interstitial threatening, eliminating ineffective
tissue, causing edema. Urea nitrogen thermoregulation as the top priority.
excretion can be increased only by 9. ANS: C
improved renal function. Sodium Regular insulin, which is a short-
intake doesn’t affect the glomerular acting insulin, has an onset of 15 to
filtration rate. Potassium absorption 30 minutes and a peak of 2 to 4
is improved only by increasing the hours. Because the nurse gave the
glomerular filtration rate; it isn’t insulin at 2 p.m., the expected onset
affected by sodium intake. would be from 2:15 to 2:30 p.m. and
5. ANS: D the peak from 4 p.m. to 6 p.m.
The portion of the cerebrum that 10. ANS: A
controls speech and hearing is the CPP is derived by subtracting the ICP
temporal lobe. Injury to the frontal from the mean arterial pressure
lobe causes personality changes, (MAP). For adequate cerebral
difficulty speaking, and disturbance perfusion to take place, the minimum
in memory, reasoning, and goal is 70 mmHg. The MAP is derived
concentration. Injury to the parietal using the following formula:
lobe causes sensory alterations and MAP = ((diastolic blood pressure x 2)
problems with spatial relationships. + systolic blood pressure) / 3
Damage to the occipital lobe causes MAP = ((60 x2) + 90) / 3
vision disturbances. MAP = 70 mmHg
6. ANS: D To find the CPP, subtract the client’s
Diabetes insipidus is an abrupt onset ICP from the MAP; in this case , 70
of extreme polyuria that commonly mmHg – 18 mmHg = 52 mmHg.
occurs in clients after brain surgery. 11. ANS: B
Cushing’s syndrome is excessive Breast cancer tumors are fixed, hard,
glucocorticoid secretion resulting in and poorly delineated with irregular
sodium and water retention. Diabetes edges. Nipple retraction —not
mellitus is a hyperglycemic state eversion—may be a sign of cancer. A
marked by polyuria, polydipsia, and mobile mass that is soft and easily
polyphagia. Adrenal crisis is delineated is most often a fluid-filled
undersecretion of glucocorticoids benigned cyst. Axillary lymph nodes
resulting in profound hypoglycemia, may or may not be palpable on initial
hypovolemia, and hypotension. detection of a cancerous mass.
7. ANS: C 12. ANS: D
The client should report the presence An enterostomal nurse therapist is a
of foulsmelling or cloudy urine. registered nurse who has received
Unless contraindicated, the client advance education in an accredited
should be instructed to drink large program to care for clients with
quantities of fluid each day to flush stomas. The enterostomal nurse
therapist can assist with selection of casting. This is especially true in a
an appropriate stoma site, teach young client. The incidence of
about stoma care, and provide complications, such as pin tract
emotional support. infections and neuritis, is 20% to
13. ANS: A 60%. Clients must be taught how to
Ottorrhea and rhinorrhea are classic do pin care and assess for
signs of basilar skull fracture. Injury development of neurovascular
to the dura commonly occurs with complications.
this fracture, resulting in 19. ANS: C
cerebrospinal fluid (CSF) leaking In a client with chronic renal failure,
through the ears and nose. Any fluid unrestricted intake of sodium,
suspected of being CSF should be protein, potassium, and fluids may
checked for glucose or have a halo lead to a dangerous accumulation of
test done. electrolytes and protein metabolic
14. ANS: D products, such as amino acids and
Testicular cancer commonly occurs in ammonia. Therefore, the client must
men between ages 20 and 30. A male limit his intake of sodium, meat (high
client should be taught how to in Protein), bananas (high in
perform testicular self-examination potassium), and fluid because the
before age 20, preferably when he kidneys can’t secrete adequate urine.
enters his teens. 20. ANS: D
15. ANS: B The client isn’t withdrawn or showing
Before weaning a client from other signs of anxiety or depression.
mechanical ventilation, it’s most Therefore, the nurse can probably
important to have a baseline ABG safely approach her about talking
levels. During the weaning process, with others who have had similar
ABG levels will be checked to assess experiences, either through Reach for
how the client is tolerating the Recovery or another formal support
procedure. Other assessment group. The nurse may educate the
parameters are less critical. client’s spouse or partner to listen to
Measuring fluid volume intake and concerns, but the nurse shouldn’t tell
output is always important when a the client’s spouse what to do. The
client is being mechanically client must consult with her physician
ventilated. Prior attempts at weaning and make her own decisions
and ECG results are documented on about further treatment. The client
the client’s record, and the nurse can needs to express her sadness,
refer to them before the weaning frustration, and fear. She can’t be
process begins. expected to be cheerful at all times.
16. ANS: B 21. ANS: C
According to the ACS guidelines, HCG is one of the tumor markers for
“Women older than age 40 should testicular cancer. The HCG level
perform breast selfexamination won’t identify the site of an infection
monthly (not annually).” The or evaluate prostatic function or
hormonal receptor assay is done on a testosterone level.
known breast tumor to determine 22. ANS: B
whether the tumor is estrogen- or Peripheral edema is a sign of fluid
progesterone-dependent. volume overload and worsening heart
17. ANS: C failure. A skin rash, dry cough, and
Increased pressure within the portal postural hypotension are adverse
veins causes them to bulge, leading reactions to captopril, but the don’t
to rupture and bleeding into the indicate that therapy isn’t effective.
lower esophagus. Bleeding 23. ANS: A
associated with esophageal varices Tenting of chest skin when pinched
doesn’t stem from esophageal indicates decreased skin elasticity
perforation, pulmonary hypertension, due to dehydration. Hand veins fill
or peptic ulcers. slowly with dehydration, not rapidly.
18. ANS: B A pulse that isn’t easily obliterated
Complex intra-articular fractures are and neck vein distention indicate
repaired with external fixators fluid overload, not dehydration.
because they have a better long-term 24. ANS: B
outcome than those treated with The client should be encouraged to
increase his activity level. dressings placed in a sterile basin.
Maintaining an ideal weight; following Wounds should be cleaned from the
a low-cholesterol, low-sodium diet; most contaminated area to the least
and avoiding stress are all important contaminated area—for example,
factors in decreasing the risk of from the center outward. The outer
atherosclerosis. inch of a sterile field shouldn’t be
25. ANS: B considered sterile.
Because thrombocytopenia impairs 30. ANS: C
blood clotting, the nurse should Caffeine is a stimulant, which can
assess the client regularly for signs of exacerbate palpitations and should
bleeding, such as petechiae, purpura, be avoided by a client with
epistaxis, and bleeding gums. The symptomatic mitral valve prolapse.
nurse should avoid administering High fluid intake helps maintain
aspirin because it can increase the adequate preload and cardiac output.
risk of bleeding. Frequent rest periods Aerobic exercise helps in increase
are indicated for clients with anemia, cardiac output and decrease heart
not thrombocytopenia. Strict isolation rate. Protein-rich foods aren’t
is indicated only for clients who have restricted but high calorie foods are.
highly contagious or virulent 31. ANS: A
infections that are spread by air or Excessive of aldosterone in the
physical contact. adrenal cortex is responsible for the
26. ANS: B client’s hypertension. This hormone
SaO2 is the degree to which acts on the renal tubule, where it
hemoglobin is saturated with oxygen. promotes reabsorption of sodium and
It doesn’t indicate the client’s overall excretion of potassium and hydrogen
Hgb adequacy. Thus, an individual ions. The pancreas mainly secretes
with a subnormal Hgb level could hormones involved in fuel
have normal SaO2 and still be short metabolism. The adrenal medulla
of breath. In this case, the nurse secretes the cathecolamines—
could assume that the client has a epinephrine and norepinephrine. The
Hematologic problem. Poor peripheral parathyroids secrete parathyroid
perfusion would cause subnormal hormone.
SaO2. There isn’t enough data to 32. ANS: D
assume that the client’s problem is A heart valve prosthesis, such as a
psychosomatic. If the problem were mitral valve replacement, is a major
left-sided heart failure, the client risk factor for infective endocarditis.
would exhibit pulmonary crackles. Other risk factors include a history of
27. ANS: A heart disease (especially mitral valve
Addisonian crisis results in prolapse), chronic debilitating
Hyperkalemia; therefore, disease, IV drug abuse, and
administering potassium chloride is immunosuppression. Although
contraindicated. Because the client diabetes mellitus may predispose a
will be hyponatremic, normal saline person to cardiovascular disease, it
solution is indicated. Hydrocortisone isn’t a major risk factor for infective
and fludrocortisone are both useful in endocarditis, nor is an appendectomy
replacing deficient adrenal cortex or pernicious anemia.
hormones. 33. ANS: A
28. ANS: D The client has signs and symptoms of
Leukemia is manifested by an diabetes insipidus, probably caused
abnormal overpopulation of immature by the failure of her renal tubules to
leukocytes in the bone marrow. respond to antidiuretic hormone as a
29. ANS: C consequence of pyelonephritis. The
A sterile field is considered hypernatremia is secondary to her
contaminated when it becomes wet. water loss. Imbalanced nutrition
Moisture can act as a wick, allowing related to hypermetabolic state or
microorganisms to contaminate the catabolic effect of insulin deficiency is
field. The outside of containers, such an inappropriate nursing diagnosis
as sterile saline bottles, aren’t sterile. for the client.
The containers should be opened 34. ANS: C
before sterile gloves are put on and When performing a nutritional
the solution poured over the sterile assessment, one of the first things
the nurse should do is to assess what tuberculosis should be in a room with
the client typically eats. The client laminar air flow, and the door should
shouldn’t be permitted to eat as be closed at all times.
desired. Weighing the client daily, 38. ANS: C
placing her on I & O status, and The nurse can gently irrigate the tube
drawing blood to determine but must take care not to reposition
electrolyte level aren’t part of a it. Repositioning can cause bleeding.
nutritional assessment. Suction should be applied
35. Ans. C continuously, not every hour. The NG
Regular follow-up care for the client tube shouldn’t be clamped
with Grave’s disease is critical postoperatively because secretions
because most cases eventually result and gas will accumulate, stressing
in hypothyroidism. Annual thyroid- the suture line.
stimulating hormone tests and the 39. ANS: A
client’s ability to recognize signs and Regardless of the client’s medical
symptoms of thyroid dysfunction will history, rapid fluid resuscitation is
help detect thyroid abnormalities critical for maintaining cardiovascular
early. Intake and output is important integrity. Profound intravascular
for clients with fluid and electrolyte depletion requires aggressive fluid
imbalances but not thyroid disorders. replacement. A typical fluid
DDAVP is used to treat diabetes resuscitation protocol is 6 L of fluid
insipidus. While exercise to improve over the first 12 hours, with more
cardiovascular fitness is important, fluid to follow over the next 24 hours.
for this client the importance of Various fluids can be used, depending
regular follow-up is most critical. on the degree of hypovolemia.
36. ANS: D Commonly prescribed fluids include
Anxiety related to the threat of death dextran (in case of hypovolemic
is an appropriate nursing diagnosis shock), isotonic normal saline
because the client’s anxiety can solution and, when the client is
adversely affect hear rate and stabilized, hypotonic half-normal
rhythm by stimulating the autonomic saline solution.
nervous system. Also, because the 40. ANS: D
client required resuscitation, the Glipizide may cause adverse skin
threat of death is a real and reactions, such as pruritus, and
immediate concern. Unless anxiety is photosensitivity. It doesn’t cause
dealt with first, the client’s emotional headache, constipation, or
state will impede learning. Client hypotension.
teaching should be limited to clear 41. ANS: B
concise explanations that reduce When clients are on mechanical
anxiety and promote cooperation. An ventilation, the artificial airway
anxious client has difficulty learning, impairs the gag and cough reflexes
so the deficient knowledge would that help keep organisms out of the
continue despite attempts teaching. lower respiratory tract. The artificial
Impaired physical mobility and social airway also prevents the upper
isolation are necessitated by the respiratory system from humidifying
client’s critical condition; therefore, and heating air to enhance
they aren’t considered problems mucociliary clearance. Manipulations
warranting nursing diagnoses. of the artificial airway sometimes
37. ANS: A allow secretions into the lower
Because tuberculosis is transmitted airways. Whit standard procedures
by droplet nuclei from the respiratory the other choices wouldn’t be at high
tract, the nurse should put on a mask risk.
when entering the client’s room. 42. ANS: C
Having the client wear a mask at all In chronic bronchitis, the
the times would hinder sputum diaphragmatic is flat and weak.
expectoration and make the mask Diaphragmatic breathing helps to
moist from respirations. If no contact strengthen the diaphragm and
with the client’s blood or body fluids maximizes ventilation. Exhalation
is anticipated, the nurse need not should longer than inhalation to
wear a gown or gloves when prevent collapse of the bronchioles.
providing direct care. A client with The client with chronic bronchitis
should exhale through pursed lips to D. PTH doesn’t have a role in the
prolong exhalation, keep the metabolism of Vit E.
bronchioles from collapsing, and 48. ANS: C
prevent air trapping. Diaphragmatic The presence of unsecured area rugs
breathing—not chest breathing— poses a hazard in all homes,
increases lung expansion. particularly in one with a resident at
43. ANS: A high risk for falls.
Verbalizing the observed behavior is 49. ANS: B
a therapeutic communication Autonomy ascribes the right of the
technique in which the nurse individual to make his own decisions.
acknowledges what the client is In this case, the client is capable of
feeling. Offering to listen to the client making his own decision and the
express her anger can help the nurse nurse should support his autonomy.
and the client understand its cause Beneficence and justice aren’t the
and begin to deal with it. Although principles that directly relate to the
stress can exacerbate the symptoms situation. Advocacy is the nurse’s role
of SLE, telling the client to calm down in supporting the principle of
doesn’t acknowledge her feelings. autonomy.
Offering to get the nursing supervisor 50. ANS: D
also doesn’t acknowledge the client’s Because respirations are depressed
feelings. Ignoring the client’s feelings in myxedema coma, maintaining a
suggest that the nurse has no patent airway is the most critical
interest in what the client has said. nursing intervention. Ventilatory
44. ANS: A support is usually needed. Thyroid
Daily walks relieve symptoms of replacement will be administered IV.
intermittent claudication, although Although myxedema coma is
the exact mechanism is unclear. associated with severe hypothermia,
Anaerobic exercise may exacerbate a warming blanket shouldn’t be used
these symptoms. Clients with chronic because it may cause vasodilation
arterial occlusive disease must and shock. Gradual warming blankets
reduce daily fat intake to 30% or less would be appropriate. Intake and
of total calories. The client should output are very important but aren’t
limit dietary cholesterol because critical
hyperlipidemia is associated with interventions at this time.
atherosclerosis, a known cause of 51. ANS: A
arterial occlusive disease. However, Glipizide begins to act in 15 to 30
HDLs have the lowest cholesterol minutes. The other options are
concentration, incorrect.
so this client should eat foods that 52. ANS: A
raise HDL levels. Hypoxia is the main breathing
45. ANS: A stimulus for a client with COPD.
Gastric decompression is typically Excessive oxygen administration may
low pressure and intermittent. High lead to apnea by removing that
pressure and continuous gastric stimulus. Anginal pain results from a
suctioning predisposes the gastric reduced myocardial oxygen supply. A
mucosa to injury and ulceration. client with COPD may have anginal
46. ANS: A pain from generalized
In osteoarthritis, stiffness is common vasoconstriction secondary to
in large, weight bearing joints such as hypoxia; however, administering
the hips. This joint stiffness alters oxygen at any concentration dilates
functional ability and range of blood vessels, easing anginal pain.
motion, placing the client at risk for Respiratory alkalosis results from
falling and injury. Therefore, client alveolar hyperventilation, not
safety is in jeopardy. Osteoporosis excessive oxygen administration. In a
doesn’t affect urinary elimination, client with COPD, high oxygen
breathing, or nutrition. concentrations decrease the
47. ANS: A ventilatory drive, leading to
PTH stimulates the kidneys to respiratory acidosis, not alkalosis.
reabsorb calcium and excrete High oxygen concentrations don’t
phosphate and converts vit D to its cause metabolic acidosis.
active form: 1 , 25 dihydroxy vitamin
53. ANS: C with severe vomiting, may lead to
Continuous subcutaneous insulin metabolic alkalosis.
regimen uses a basal rate and 59. ANS: B
boluses of short-acting insulin. The head of the bed must be
Multiple daily injection therapy uses a elevated while the client is eating.
combination of short-acting and The client should be placed in a
intermediate or long-acting insulins. recumbent position—not a supine
54. ANS: B position— when lying down to reduce
Holding a cane on the uninvolved the risk of aspiration. Encourage the
side distributes weight away from the client to wear properly fitted dentures
involved side. Holding the cane close to enhance his chewing ability.
to the body prevents leaning. Use of Thickened liquids, not thin liquids,
a cane won’t maintain stride length decrease aspiration risk.
or prevent edema. 60. ANS: A
55. ANS: D Endotracheal suctioning secretions as
High pulmonary artery wedge well as gases from the airway and
pressures are diagnostic for left-sided lowers the arterial oxygen saturation
heart failure. With leftsided heart (SaO2) level. Coughing and incentive
failure, pulmonary edema can spirometry improve oxygenation and
develop causing pulmonary crackles. should raise or maintain oxygen
In leftsided heart failure, hypotension saturation. Because of superficial
may result and urine output will vasoconstriction, using a cooling
decline. Dry mucous membranes blanket can lower peripheral oxygen
aren’t directly associated with saturation readings, but SaO2 levels
elevated pulmonary artery wedge wouldn’t be affected.
pressures. 61. ANS: D
56. ANS: B With a superficial partial thickness
It’s mandatory in most settings to burn such as a solar burn (sunburn),
have a physician’s order before the nurse’s main concern is pain
restraining a client. A client should management. Fluid resuscitation and
never be left alone while the nurse infection become concerns if the burn
summons assistance. All staff extends to the dermal and
members require annual instruction subcutaneous skin layers. Body
on the use of restraints, and the image disturbance is a concern that
nurse should be familiar with the has a lower priority than pain
facility’s policy. management.
57. ANS: A 62. ANS: D
The client who has undergone a Crackles result from air moving
thyroidectomy is t risk for developing through airways that contain fluid.
hypocalcemia from inadvertent Heard during inspiration and
removal or damage to the expiration, crackles are discrete
parathyroid gland. The client with sounds that vary in pitch and
hypocalcemia will exhibit a positive intensity. They’re classified as fine,
Chvostek’s sign (facial muscle medium, or coarse. Pleural friction
contraction when the facial nerve in rubs have a distinctive grating sound.
front of the ear is tapped) and a As the name indicates, these breath
positive Trousseau’s sign (carpal sounds result when inflamed pleurae
spasm when a blood pressure cuff is rub together. Continuous,
inflated for few minutes). These signs highpitched, musical squeaks, called
aren’t present with hypercalcemia, wheezes, result when air moves
hypokalemia, or Hyperkalemia. rapidly through airways narrowed by
58. ANS: C asthma or infection or when an
Diarrhea causes a bicarbonate deficit. airway is partially obstructed by a
With loss of the relative alkalinity of tumor or foreign body. Wheezes, like
the lower GI tract, the relative acidity gurgles, occur on expiration and
of the upper GI tract predominates sometimes on inspiration. Loud,
leading to metabolic acidosis. coarse, low-pitched sounds
Diarrhea doesn’t lead to respiratory resembling snoring are called
acid-base imbalances, such as gurgles. These sounds develop when
respiratory acidosis and respiratory thick secretions partially obstruct
alkalosis. Loss of acid, which occurs
airflow through the large upper dermatitis caused by the itch mite,
airways. Sacoptes scabiei, which lives just
63. Ans. D beneath the skin. Tinea capitis, or
Individuals who are tuberculin skin ringworm, causes patchy hair loss
test converters should begin a 6- and circular lesions with healing
month regimen of an antitubercular centers. Impetigo is an infection
drug such as INH, and they should caused by Staphylococcus or
never have another skin test. After an Sterptococcus, manifested by
individual has a positive tuberculin vesicles or pustules that form a thick,
skin test, subsequent skin tests will honey-colored crust.
cause severe skin reactions but won’t 68. ANS: C
provide new information about the Severe pressure-like pain out of
client’s TB status. The client doesn’t proportion to visible signs
have active TB, so can’t transmit, or distinguishes necrotizing fasciitis
spread, the bacteria. Therefore, she from cellulites. Erythema,
shouldn’t be quarantined or asked for leukocytosis, and swelling are present
information about recent contacts. in both cellulites and necrotizing
64. ANS: C fasciitis.
Disturbed body image is a negative 69. ANS: A
perception of the self that makes Contact dermatitis is caused by
healthful functioning more difficult. exposure to a physical or chemical
The defining characteristics for this allergen, such as cleaning products,
nursing diagnosis include undergoing skin care products, and latex gloves.
a change in body structure or Initial symptoms of itching,
function, hiding or overexposing a erythema, and raised papules occur
body part, not looking at a body part, at the site of the exposure and can
and responding verbally or begin within 1 hour of exposure.
nonverbally to the actual or Allergic reactions tend to be red and
perceived change in structure or not scaly or flaky. Weeping, crusting
function. This client may have any of lesions are also uncommon unless
the other diagnoses, but the signs the reaction is quite severe or has
and symptoms described in he case been present for a long time.
most closely match the defining Excoriation is more common in skin
characteristics for disturbed body disorders associated with a moist
image. environment.
65. ANS: B 70. ANS: D
Because the pregnant uterus exerts a Hoarseness indicate injury to the
lot of pressure on the urinary bladder, respiratory system and could indicate
the bladder repair may need to be the need for immediate intubation.
repeated. These clients don’t Thirst following burns is expected
necessarily have to have a cesarean because of the massive fluid shifts
delivery if they become pregnant, and resultant loss leading to
and this procedure doesn’t render dehydration. Pain, either severe or
them sterile. This procedure is moderate, is expected with a burn
completed in one surgery. injury. The client’s output is
66. ANS: A adequate.
Eccrine glands are associated with 71. ANS: C
body temperature regulation. Third-degree burn may appear white,
Sebaceous glands lubricate the skin red, or black and are dry and leathery
and hairs, and apocrine glands are with no blisters. There may be little
involved in bacteria decomposition. pain because nerve endings have
Endocrine glands secrete hormones been destroyed. First-degree burns
responsible for the regulation of body are superficial and involve the
processes, such as metabolism and epidermis only. There is local pain
glucose regulation. and redness but no blistering.
67. ANS: B Second-degree burn appear red and
The light-colored spots attached to moist with blister formation and are
the hair shafts are nits, which are the painful. Fourth-degree burns involve
eggs of head lice. They can’t be underlying muscle and bone tissue.
brushed off the hair shaft like 72. ANS: C
dandruff. Scabies is a contagious A universal concern I the care of
donor sites for burn care is to keep gets turned and thus, help prevent
the site away from sources of pressure ulcers. Turning should occur
pressure. Ventilation of the site and every 1-2 hours—not every 8 hours—
keeping the site fully covered are for clients who are in bed for
practices in some institutions but prolonged periods. The nurse should
aren’t hallmarks of donor site care. apply lotion to keep the skin moist
Placing the site in a position of but should avoid vigorous massage,
dependence isn’t a justified aspect of which could damage capillaries.
donor site care. When moving the client, the nurse
73. ANS: A should lift rather than slide the client
Anyone with psoriasis vulgaris who to void shearing.
reports joint pain should be evaluated 79. ANS: A
for psoriaic arthritis. Approximately To prevent disruption of the artificial
15% to 20% of individuals with skin’s adherence to the wound bed,
psoriasis will also develop psoriatic the client should restrict range of
arthritis, which can be painful and motion of the involved limb. Protein
cause deformity. It would be incorrect intake and fluid intake are important
to assume that his pain is caused by for healing and regeneration and
early rheumatoid arthritis or his shouldn’t be restricted. Going
vocation without asking more outdoors is acceptable as long as the
questions or performing diagnostic left arm is protected from direct
studies. Carpal tunnel syndrome sunlight.
causes sensory and motor changes in 80. ANS: C
the fingers rather than localized pain According to the Rule of Nines, the
in the joints. posterior and anterior trunk, and legs
74. ANS: B each make up 18% of the total body
To avoid burning and sloughing, the surface. The head, neck, and arms
client must protect the graft from each make up 9% of total body
sunlight. The other three durface, and the perineum makes up
interventions are all helpful to the 1%. In this case, the client received
client and his recovery but are less burns to his back (18%) and one arm
important. (9%), totaling 27%.
75. ANS: C 81. ANS: A
Because the itching and rash are A wet-to-dry saline dressing should
localized, an environmental cause in always keep the wound moist. Tight
the workplace should be suspected. packing or dry packing can cause
With the advent of universal tissue damage and pain. A dry gauze
precautions, many nurses are —not a plastic-sheet-type dressing—
experiencing allergies to latex gloves. should cover the wet dressing.
Allergies to medications, laundry 82. ANS: A
detergents, or bath soaps or a When someone in a group of persons
dermatologic reaction to stress sharing a home contracts scabies,
usually elicit a more generalized or each individual in the same home
widespread rash. needs prompt treatment whether
76. ANS: A he’s symptomatic or not. Towels and
Hives and urticaria are two names for linens should be washed in hot water.
the same skin lesion. Toxin is a Scabies can be transmitted from one
poison. A tubercle is a tiny round person to another before symptoms
nodule produced by the tuberculosis develop
bacillus. A virus is an infectious 83. ANS: A
parasite. In a client with burns, the goal of fluid
77. ANS: A resuscitation is to maintain a mean
A scale is the characteristic arterial blood pressure that provides
secondary lesion occurring in adequate perfusion of vital
psoriasis. Although crusts, ulcers, and structures. If the kidneys are
scars also are secondary lesions in adequately perfused, they will
skin disorders, they don’t accompany produce an acceptable urine output
psoriasis. of at least 0.5 ml/kg/hour. Thus, the
78. ANS: C expected urine output of a 155-lb
A turning schedule with a signing client is 35 ml/hour, and a urine
sheet will help ensure that the client output consistently above 100
ml/hour is more than adequate. 88. ANS: A
Weight gain from fluid resuscitation The nurse should instill the eyedrop
isn’t a goal. In fact, a 4 lb weight gain into the conjunctival sac where
in 24 hours suggests third spacing. absorption can best take place. The
Body temperature readings and ECG pupil permits light to enter the eye.
interpretations may demonstrate The sclera maintains the eye’s shape
secondary benefits of fluid and size. The vitreous humor
resuscitation but aren’t primary maintains the retina’s placement and
indicators. the shape of the eye.
84. ANS: B 89. ANS: C
The ESR test is performed to detect According to family theory, any
inflammatory processes in the body. change in a family member, such as
It’s a nonspecific test, so the health illness, produces role changes in all
care professional must view results in family members and affects the
conjunction with physical signs and entire family, even if the client
symptoms. Platelet count, eventually recovers completely.
electrolytes, BUN, and creatinine 90. ANS: B
levels aren’t usually affected by the To help prevent airway obstruction
inflammatory process. and reduce the risk of aspiration, the
85. ANS: B nurse should position a client with
In Parkinson’s crisis, dopamine- hemiparesis on the affected side.
related symptoms are severely Although performing ROM exercises,
exacerbated, virtually immobilizing providing pillows for support, and
the client. A client who is confined to applying antiembolism stockings can
bed during a crisis is at risk for be appropriate for a client with CVA,
aspiration and pneumonia. Also, the first concern is to maintain a
excessive drooling increases the risk patent airway.
of airway obstruction. Because of 91. ANS: C
these concerns, ineffective airway TIA may be a warning that the client
clearance is the priority diagnosis for will experience a CVA, or stroke, in
this client. Although imbalanced the near future. TIA aymptoms last
nutrition:less than body no longer than 24 hours and clients
requirements, impaired urinary usually have complete recovery after
elimination and risk for injury also are TIA. The most common symptom of
appropriate diagnoses for this client, TIA is sudden, painless loss of vision
they aren’t immediately lasting up to 24 hours.
lifethreatening and thus are less 92. ANS: C
urgent. Bending to pick up something from
86. ANS: A the floor would increase intraocular
As a rule of practice, if intermittent pressure, as would bending to tie his
catheterization for urine retention shoes. The client needs to wear eye
typically yields 500 ml or more, the protection to bed to prevent
frequency of catheterization should accidental injury during sleep.
be increased. Indwelling 93. ANS: B
catheterization is less preferred Concussions are considered minor
because of the risk of urinary tract with no structural signs of injury. A
infection and the loss of bladder tone. contusion is bruising of the brain
Fluid restrictions aren’t indicated for tissue with small hemorrhages in the
this case; the problem isn’t tissue. Coup and contrecoup are type
overhydration, rather it’s urine of injuries in which the damaged area
retention. A condom catheter doesn’t on the brain forms directly below that
help empty the bladder of a client site of impact (coup) or at the
with urine retention. site opposite the injury (contrecoup)
87. ANS: D due to movement of the brain within
The normal pressure is 70 to 200 mm the skull.
H2O are considered abnormal. The 94. ANS: B
presence of glucose is an expected The third cranial nerve (oculomotor)
finding in CSF, and RBCs typically is responsible for pupil constriction.
occur in the first specimen tube from When there is damage to the nerve,
the trauma caused by the procedure. the pupils remain dilated and don’t
respond to light. Glaucoma, lumbar
spine injury, and Bell’s palsy won’t they can’t participate in abstract
affect pupil constriction. thinking.
95. ANS: C 100. ANS: B
When the mechanical stressors of For clients with increased intracranial
weight bearing are absent, diffuse pressure (ICP), the head of the bed is
osteoporosis can occur. Therefore, if elevated to promote venous outflow.
the client does weight-bearing Trendelenburg’s position is
exercises, disuse complications can contraindicated because it can raise
be prevented. Maintaining protein ICP. Flat or neutral positioning is
and vitamins levels is important, but indicated when elevating the head of
neither will prevent osteoporosis. the bed would increase the risk of
ROM exercises will help prevent neck injury or airway obstruction.
muscle atrophy and contractures. Sidelying isn’t specifically a
96. ANS: B therapeutic treatment for increased
This client requires droplet ICP.
precautions because the organism
can be transmitted through airborne
droplets when the client coughs,
sneezes, or doesn’t cover his mouth.
Airborne precautions would be
instituted for a client infected with PNLE Medical
tuberculosis. Standard precautions
would be instituted for a client when
contact with body substances is
Surgical Nursing
likely. Contact precautions would be
instituted for a client infected with an
organism that is transmitted through
Exam 3
skin-to-skin 1. Lisa is newly diagnosed with asthma
contact. and is being discharged from the hospital
97. ANS: D after an episode of status asthmaticus.
With a brain injury such as an Discharge teaching should include which
epidural hematoma (a diagnosis that of the following:
is most likely based on this client’s
symptoms), the initial sign of A. Limitations in sports that will be
increasing ICP is a change in the level imposed by the illness
of consciousness. As neurologic B. Specific instructions on staying cal
deterioration progresses, during an attack
manifestations involving pupillary C. The relationship of symptoms and
symmetry, breathing patterns, and a specific trigger such as physical
posturing will exercise
occur. D. Incidence of status asthmaticus in
98. ANS: A children and teens
In the scenario, airway and breathing 2. Which of the following symptoms is
are established so the nurse’s next most characteristic of a client with a
priority should be circulation. With a cancer of the lung?
compound fracture of the femur,
there is a high risk of profuse A. air hunger
bleeding; therefore, the nurse should B. exertional dyspnea
assess the site. Neurologic C. cough with night sweats
assessment is a secondary concern D. persistent changing cough
to airway, breathing and circulation. 3. The client has ST segment depression
The nurse doesn’t have enough data on his 12-lead ECG. The nurse
to warrant putting the client in determines that this would indicate the
Trendelenburg’s position. following:
99. ANS: C
Establishing and maintaining a A. necrosis
routine is essential to decreasing B. injury
extraneous stimuli. The client should C. ischemia
participate in daily care as much as D. nothing significant
possible. Attempting to reason with 4. Red has just returned from the
such clients isn’t successful, because postanesthesia care unit (PACU) from a
hemorrhidectomy. His postoperative D. Avoiding lesions on the feet
orders include sitz baths every morning. 10. Mr. Roberto Robles complains of a
The nurse understands that sitz bath is severe headache and is extremely
use for: anxious. The nurse checks his vital signs
and finds him to have a heart rate of 57
A. promote healing bpm and a blood pressure of 230/110
B. relive tension mmHg. The nurse should also assess for?
C. lower body temperature
D. cause swelling A. presence of bowel sounds
5. Trousseau’s sign is associated with B. presence of babinski reflex
which electrolyte imbalance? C. fecal incontinence
D. urinary catheter patency
A. hyponatremia 11. A 40n year old male patient is
B. hypocalcemia complaining of chronic progressive and
C. hypernatremia mental deterioration is admitted to the
D. hypercalcemia unit. The nurse recognizes that these
6. A 36 year old female complains of characteristics indicate a disease that
headache and neck pain. The nurse’s results in degeneration of the basal
assessments reveal painful flexion of the ganglia and cerebral cortex. The disease
neck to the chest. The nurse understands is called:
that nuchal rigidity is associated with:
A. multiple sclerosis
A. brain tumor B. myasthenia gravis
B. CVA C. Huntington’s disease
C. meningitis D. Guillain-Barre syndrome
D. subdural hematoma 12. Dianne Hizon is a 27 year old woman
7. The nurse teaching the client about who has been admitted to the ER due to
behavioral changes, which can affect severe vomiting. Her ABG values are pH=
development of atherosclerosis, should 7.50, PaCO2= 85, HCO3= 31, and SaO2=
discuss which of the following as a non- 93%. The nurse interpretation of this ABG
modifiable risk factor for atherosclerosis? analysis is:

A. cigarette smoking A. respiratory acidosis


B. hyperlipidemia B. respiratory alkalosis
C. female over 55 years of age C. metabolic acidosis
D. sedentary lifestyle D. metabolic alkalosis
8. A 76 year old man enters the ER with 13. Mr. Perkson has a parkinson’s disease
complaints of back pain and feeling and he finds the resting tremor he is
fatigued. Upon examination, his blood experiencing in his right hand very
pressure is 190/100, pulse is 118, and frustrating. The nurse advises him to:
hematocrit and hemoglobin are both low.
The nurse palpates the abdomen which is A. take a warm bath
soft, non-tender and auscultates an B. hold an object
abdominal pulse. The most likely C. practice deep breathing
diagnosis is: D. take diazepam as needed
14. A shuffling gait is typically associated
A. Buerger’s disease with the patient who has:
B. CHF
C. Secondary hypertension A. Parkinson’s disease
D. Aneurysm B. Multiple sclerosis
9. Nurse Fiona is caring a patient with C. Raynaud’s disease
Raynaud’s disease. Which of the D. Myasthenia gravis
following outcomes concerning 15. The priority in preparing the room for
medication regimen is of highest priority? a client with a C7 spinal cord injury is
having:
A. Controlling the pain once
vasospasm occur A. the halo brace device
B. Relaxing smooth muscle to avoid B. a catheterization tray
vasospasms C. a ventilator on stand by
C. Preventing major disabilities that D. the spinal kinetic bed
may occur
16. A 47 year old man with liver failure explain why there is so much emphasis
who has developed ascites. The nurse on bleeding precautions. Which of the
understands that ascites is due to: following provides the most appropriate
response?
A. dehydration
B. protein deficiency A. “The low protein diet will result in
C. bleeding disorders reduced clotting.”
D. vitamin deficiency B. “The increased production of bile
17. A client with rheumatoid arthritis may decreases clotting factors.”
reveal which of the following assessment C. “The liver affected by cirrhosis is
data: unable to produce clotting factors.”
D. “The required medications reduce
A. Heberden’s nodes clotting factors.”
B. Morning stiffness no longer than 30 22. Betty Lee is a 58 year old woman
minutes who is being admitted to the medical
C. Asymmetric joint swelling ward with trigeminal neuralgia. The nurse
D. Swan neck deformities anticipates that Mr. Lee will demonstrate
18. Elsa Santos is a 18 year old student which of the following major complaints?
admitted to the ward with a diagnosis of
epilepsy. She tells the nurse that she is A. excruciating, intermittent,
experiencing a generalized tingling paroxysmal facial pain
sensation and is “smelling roses”. The B. unilateral facial droop
nurse understands that Esla is probably C. painless eye spasm
experiencing: D. mildly painful unilateral eye
twitching
A. an acute alcohol withdrawal 23. A 38 year old woman returns from a
B. an acute CVA subtotal thryroidectomy for the
C. an aura treatment of hyperthyroidism. Upon
D. an olfactory hallucination assessment, the immediate priority that
19. Mr. Lucas, a 63 year old, went to the the nurse would include is:
clinic complaining of hoarseness of voice
and a cough. His wife states that his A. Assess for pain
voice has changed in the last few B. Assess for neurological status
months. The nurse interprets that Mr. C. Assess fluid volume status
Lucas’s symptoms are consistent with D. Assess for respiratory distress
which of the following disorders: 24. Nurse Shiela is teaching self-care to a
client with psoriasis. The nurse should
A. chronic sinusitis encourage which of the following for his
B. laryngeal cancer scaled lesion?
C. gastroesophageal reflux disease
D. coronary artery disease A. Importance of follow-up
20. Sarah complains of a nursing appointments
sensation, cramping pain in the top part B. Emollients and moisturizers to
of her abdomen that becomes worse in soften scales
the afternoon and sometimes awakes her C. Keep occlusive dressings on the
at night. She reports that when she eats, lesions 24 hours a day
it helps the pain go away but that pain is D. Use of a clean razor blade each
now becoming more intense. Which of time he shaves
the following is the best condition for the 25. A 48 year old woman presents to the
nurse to draw: hospital complaining of chest pain,
tachycardia and dyspnea. On exam,
A. these symptoms are consistent heart sounds are muffled. Which of the
with an ulcer following assessment findings would
B. The client probably has indigestion support a diagnosis of cardiac
C. A snack before going to bed should tamponade?
be advised
D. The client probably developing A. A deviated trachea
cholelithiasis B. Absent breath sounds to the lower
21. Nurse Cynthia is providing a lobes
discharge teaching to a client with C. Pulse 40 with inspiration
chronic cirrhosis. His wife asks her to D. Blood pressure 140/80
Answers and Rationales decrease the vasospasm and
increase the arterial flow to the
1. C. The relationship of symptoms affected part. The drugs used are
and a specific trigger such as calcium antagonists.
physical exercise. COPD clients have 10. D. urinary catheter patency. The
low oxygen and high carbon dioxide patient is complaining of symptoms
levels. Therefore, hypoxia is the main of autonomic dysreflexia, which
stimulus for ventilation is persons consists of the triad of hypertension,
with chronic hypercapnea. Increasing bradycardia and a headache. Major
the level of oxygen would decrease causes of autonomic dysreflexia
the stimulus to breathe. include urinary bladder distention
2. D. persistent changing cough. The and fecal impaction. Checking the
most common sign of lung cancer is a patency of the urinary catheter will
persistent cough that changes. Other check for bladder distention.
signs are dyspnea, bloody sputum 11. C. Huntington’s
and long term pulmonary infection. disease. Huntington’s disease is a
Option A is common with asthma, hereditary disease in which
option B is common with COPD and degeneration of the basal ganglia
option C is common with TB. and cerebral cortex causes chronic
3. C. ischemia. Depressed ST progressive chorea (muscle
segment and inverted T-waves twitching) and mental deterioration,
represent myocardial ischemia. Injury ending in dementia. Huntington’s
has a ST segment elevation. disease usually strikes people ages
4. A. promote healing. Sitz bath 25 to 55.
provides moist heat to the perineal 12. D. metabolic alkalosis. Ms. Hizon’s
and anal area to clean, promote pH is above 7.45, which makes it
healing and drainage and reduce alkalatic, and her bicarbonate is high
soreness to the area. Sitz bath helps which is also makes it basic. Thus,
healing with cleaning action and the diagnosis is metabolic alkalosis.
promotion of circulation, thereby 13. B. hold an object. The resting or
reducing swelling. Sitz bath usually non-intentional tremor may be
has no therapeutic value in lowering controlled with purposeful movement
body temperature. Although relief of such as holding an object. A warm
tension can occur, this effect is bath, deep breathing and diazepam
secondary to the promotion of will promote relaxation but are not
healing. specific interventions for tremor.
5. B. hypocalcemia. Trousseau’s sign 14. A. Parkinson’s disease. A shuffling
is a carpal pedal spasm elicited when gait from the musculoskeletal rigidity
a blood pressure cuff is inflated on of the patient with Parkinson’s
the arm of a patient with disease is common. Patients
hypocalcemia. experiencing a stroke usually exhibit
6. C. meningitis. A patient with loss of voluntary control over motor
meningitis will exhibit signs that movements associated with
include photophobia and nuchal generalized weakness; a shuffling
rigidity, which is pain on the flexion gait is usually not observed in stroke
of the chin to chest. patient.
7. C. female over 55 years of 15. C. a ventilator on stand
age. Lifestyle, cigarette smoking and by. Although a ventilator is not
hyperlipidemia can be changed by required for injury below C3, the
changing behaviors. innervation of intercostal muscles is
8. D. Aneurysm. The symptoms affected. Hemorrhage and cord
exhibited by the client are typical of swelling extends the level of injury
an abdominal aortic aneurysm. The making it likely that this client will
most significant sign is the audible need a ventilator.
pulse in the abdominal area. If 16. B. protein deficiency. Protein
hemorrhage were present, the deficiency allows fluid to leak out of
abdomen would be tender and firm. the vascular system and third space
9. B. Relaxing smooth muscle to into the tissues and spaces in the
avoid vasospasms. The major task of body such as the peritoneal space.
the health care team is to medicate Bleeding tendencies, dehydration and
the client drugs that produce smooth
muscle relaxation, which will
vitamin deficiency can occur but skin that increases pruritus and
don’t cause ascites. causes psoriasis to be worse.
17. D. Swan neck deformities. Swan Washing and drying the skin with
neck deformities of the hand are rough linens or pressure may cause
classic deformities associated with excoriation. Constant occlusion may
rheumatoid arthritis secondary to the increase the effects of the medication
presence of fibrous connective tissue and increase the risk of infection.
within the joint space. Clients with RA 25. C. Pulse 40 with
do experience morning stiffness, but inspiration. Paradoxical pulse is a
it can last from 30 minutes up to hallmark symptom of cardiac
several hours. RA is characterized by tamponade. As pressure is exerted on
symmetrical joint movement, and the left ventricle from fluid, the
heberden’s nodes are characteristic natural increase in pressure from the
of osteoarthritis. right ventricle during inspiration
18. C. an aura. An aura frequently creates even more pressure,
precedes an epileptic seizure and diminishing cardiac output.
may manifest as vague physic
discomfort or specific aromas.
Patients experiencing auras aren’t PNLE Medical
having a CVA, substance withdrawal
or hallucination.
19. B. laryngeal cancer. These
Surgical Nursing
symptoms, along with dysphagia,
foul-smelling breath, and pain when
drinking hot or acidic, are common
Exam 4
signs of laryngeal cancer.
20. A. these symptoms are consistent
with an ulcer. The description of pain 1. A patient is admitted to the medical
is consistent with ulcer pain. The pain surgical unit following surgery. Four days
is epigastric and is worse when the after surgery, the patient spikes a 38.9
stomach is empty and is relived by degrees C oral temperature and exhibits
food. a wet, productive cough. The nurse
21. C. “The liver affected by cirrhosis is assesses the patient with understanding
unable to produce clotting that an infection that is acquired during
factors.” When bile production is hospitalization is known as:
reduced, the body has reduced ability
to absorb fat-soluble vitamins. A. a community acquired infection
Without adequate Vitamin K B. an iatrogenic infection
absorption, clotting factors II, VII, IX, C. a nosocomial infection
and X are not produced in sufficient D. an opportunistic infection
amounts. 2. A client with anemia has a hemoglobin
22. A. excruciating, intermittent, of 6.5 g/dL. The client is experiencing
paroxysmal facial pain. Trigeminal symptoms of cerebral tissue hypoxia.
neuralgia is a syndrome of Which of the following nursing
excruciating, intermittent, interventions would be most important in
paroxysmal facial pain. It manifests providing care?
as intense, periodic pain in the lips,
gums, teeth or chin. The other A. Providing rest periods throughout
symptoms aren’t characteristic of the day
trigeminal neuralgia. B. Instituting energy conservation
23. D. Assess for respiratory techniques
distress. Though fluid volume status, C. Assisting in ambulation to the
neurological status and pain are all bathroom
important assessment, the D. Checking temperature of water
immediate priority for postoperative prior to bathing
is the airway management. 3. A client was involved in a motor
Respiratory distress may result from vehicular accident in which the seat belt
hemorrhage, edema, laryngeal was not worn. The client is exhibiting
damage or tetany. crepitus, decrease breath sounds on the
24. B. Emollients and moisturizers to left, complains of shortness of breath,
soften scales. Emollients will ease dry and has a respiratory rate of 34 breaths
per minute. Which of the following A. Hold the Lanoxin
assessment findings would concern the B. Give the half dose now, wait an
nurse most? hour and give the other half
C. Call the physician
A. Temperature of 102 degrees F and D. Give the Lanoxin as ordered
productive cough 9. Nurse Marian is caring for a client with
B. ABG with PaO2 of 92 and PaCO2 of haital hernia, which of the following
40 mmHg should be included in her teaching plan
C. Trachea deviating to the right regarding causes:
D. Barrel-chested appearance
4. The proper way to open an envelop- A. To avoid heavy lifting
wrapped sterile package after removing B. A dietary plan based on soft foods
the outer package or tape is to open the C. Its prevalence in young adults
first position of the wrapper: D. Its prevalence in fair-skinned
individuals
A. away from the body 10. Joseph has been diagnosed with
B. to the left of the body hepatic encephalopathy. The nurse
C. to the right of the body observes flapping tremors. The nurse
D. toward the body understands that flapping tremors
5. Assessment of a client with possible associated with hepatic encephalopathy
thrombophlebitis to the left leg and a are also known as:
deep vein thrombosis is done by pulling
up on the toes while gently holding down A. aphasia
on the knee. The client complains of B. ascites
extreme pain in the calf. This should be C. astacia
documented as: D. asterixis
11. Hyperkalemia can be treated with
A. positive tourniquet test administration of 50% dextrose and
B. positive homan’s sign insulin. The 50% dextrose:
C. negative homan’s sign
D. negative tourniquet test A. causes potassium to be excreted
6. Thomas Elison is a 79 year old man B. causes potassium to move into the
who is admitted with diagnosis of cell
dementia. The doctor orders a series of C. causes potassium to move into the
laboratory tests to determine whether Mr. serum
Elison’s dementia is treatable. The nurse D. counteracts the effects of insulin
understands that the most common 12. Which of the following findings would
cause of dementia in this population is: strongly indicate the possibility of
cirrhosis?
A. AIDS
B. Alzheimer’s disease A. dry skin
C. Brain tumors B. hepatomegaly
D. Vascular disease C. peripheral edema
7. Which of the following nursing D. pruritus
interventions is contraindicated in the 13. Aling Puring has just been diagnosed
care of a client with acute osteomyelitis? with close-angle (narrow-angle)
glaucoma. The nurse assesses the client
A. Apply heat compress to the for which of the following common
affected area presenting symptoms of the disorder?
B. Immobilize the affected area
C. Administer narcotic analgesics for A. halo vision
pain B. dull eye pain
D. Administer OTC analgesics for pain C. severe eye and face pain
8. A client with congestive heart failure D. impaired night vision
has digoxin (Lanoxin) ordered everyday. 14. Chvostek’s sign is associated with
Prior to giving the medication, the nurse which electrolyte impabalnce?
checks the digoxin level which is
therapeutic and ausculates an apical A. hypoclacemia
pulse. The apical pulse is 63 bpm for 1 B. hypokalemia
full minute. The nurse should: C. hyponatremia
D. hypophosphatenia
15. What laboratory test is a common 21. Hazel Murray, 32 years old complains
measure of the renal function? of abrupt onset of chest and back pain
and loss of radial pulses. The nurse
A. CBC suspects that Mrs. Murray may have:
B. BUN/Crea
C. Glucose A. Acute MI
D. Alanine amino transferase (ALT) B. CVA
16. Nurse Edward is performing C. Dissecting abdominal aorta
discharge teaching for a newly diagnosed D. Dissecting thoracic aneurysm
diabetic patient scheduled for a fasting 22. Nurse Alexandra is establishing a
blood glucose test. The nurse explains to plan of care for a client newly admitted
the patient that hyperglycemia is defined with SIADH. The priority diagnosis for this
as a blood glucose level above: client would be which of the following?

A. 100 mg/dl A. Fluid volume deficit


B. 120 mg/dl B. Anxiety related to disease process
C. 130 mg/dl C. Fluid volume excess
D. 150 mg/dl D. Risk for injury
17. Mang Edison is on bed rest has 23. Nursing management of the client
developed an ulcer that is full thickness with a UTI should include:
and is penetrating the subcutaneous
tissue. The nurse documents that this A. Taking medication until feeling
ulcer is in which of the following stages? better
B. Restricting fluids
A. Stage 1 C. Decreasing caffeine drinks and
B. Stage 2 alcohol
C. Stage 3 D. Douching daily
D. Stage 4 24. Felicia Gomez is 1 day postoperative
18. A 24 year old male patient comes to from coronary artery bypass surgery. The
the clinic after contracting genital nurse understands that a postoperative
herpes. Which of the following patient who’s maintained on bed rest is
intervention would be most appropriate? at high risk for developing:

A. Encourage him to maintain bed A. angina


rest for several days B. arterial bleeding
B. Monitor temperature every 4 hours C. deep vein thrombosis (DVT)
C. Instruct him to avoid sexual D. dehiscence of the wound
contact during acute phases of illness 25. Which of the following statement is
D. Encourage him to use antifungal true regarding the visual changes
agents regularly associated with cataracts?
19. An 8 year old boy is brought to the
trauma unit with a chemical burn to the A. Both eyes typically cataracts at the
face. Priority assessment would include same time
which of the following? B. The loss of vision is experienced as
a painless, gradual blurring
A. Skin integrity C. The patient is suddenly blind
B. BP and pulse D. The patient is typically experiences
C. Patency of airway a painful, sudden blurring of vision.
D. Amount of pain Answers and Rationales
20. A client with anemia due to
chemotherapy has a hemoglobin of 7.0 1. C. a nosocomial
g/dL. Which of the following complaints infection. Nosocomial, or hospital-
would be indicative of tissue hypoxia acquired are infections acquired
related to anemia? during hospitalization for which the
patient isn’t being primarily treated.
A. dizziness Community acquired or opportunistic
B. fatigue relieved by rest infections may not be acquired during
C. skin that is warm and dry to the hospitalization. An iatrogenic
touch infection is caused by the doctor or
D. apathy by medical therapy. And an
opportunistic infection affects a the affected area and spread bacteria
compromised host. through vasodilation.
2. C. Assisting in ambulation to the 8. D. Give the Lanoxin as ordered. The
bathroom. Cerebral tissue hypoxia is Lanoxin should be held for a pulse of
commonly associated with dizziness. 60 bpm. Nurses cannot arbitrarily
The greatest potential risk to the give half of a dose without a
client with dizziness is injury, physician’s order. Unless specific
especially with changes in position. parameters are given concerning
Planning for periods of rest and pulse rate, most resources identify 60
conserving energy are important with as the reference pulse.
someone with anemia because of his 9. A. To avoid heavy lifting. Heavy
or her fatigue level but most lifting is one factor that leads to
important is safety. development of a hiatal hernia.
3. C. Trachea deviating to the right. A Dietary factors involve limiting fat
mediastinal shift is indicative of a intake, not restricting client to soft
tension pneumothorax along with the foods. It is more prevalent in
other symptoms in the question. individuals who are middle-aged or
Since the individual was involved in a older. Fair-skinned individuals are not
MVA, assessment would be targeted prone to this condition.
at acute traumatic injuries to the 10. D. asterixis. Flapping tremors
lungs, heart or chest wall rather than associated with hepatic
other conditions indicated in the encephalophaty are asterixis.
other answers. Option A is common Aphasia is the inability to speak.
with pneumonia; values in option B Ascites is an accumulation of fluid in
are not alarming; and option D is the peritoneal cavity. Astacia is the
typical of someone with COPD. inability to stand or sit still.
4. A. away from the body. When 11. D. counteracts the effects of
opening an envelop-wrapped sterile insulin. The 50% dextrose is given to
package, reaching across the counteract the effects of insulin.
package and using the first motion to Insulin drives the potassium into the
open the top cover away from the cell, thereby lowering the serum
body eliminates the need to later potassium levels. The dextrose
reach across the steri9le field while doesn’t directly cause potassium
opening the package. To remove excretion or any movement of
equipment from the package, potassium.
opening the first portion of the 12. B. hepatomegaly. Although option
package toward, to the left, or to the D is correct, it is not a strong
right of the body would require indicator of cirrhosis. Pruritus can
reaching across a sterile field. occur for many reasons. Options A
5. B. positive homan’s sign. Pain in and C are incorrect, fluid
the calf while pulling up on the toes is accumulations is usually in the form
abnormal and indicates a positive of ascites in the abdomen.
test. If the client feels nothing or just Hepatomegaly is an enlarged liver,
feels like the calf muscle is which is correct. The spleen may also
stretching, it is considered negative. be enlarged.
A tourniquet test is used to measure 13. C. severe eye and face
for varicose veins. pain. Narrow-angle glaucoma
6. B. Alzheimer’s disease. Alzheimer’s develops abruptly and manifests with
disease is the most common cause of acute face and eye pain and is a
dementia in the elderly population. medial emergency. Halo vision, dull
AIDS, brain tumors and vascular eye pain and impaired night vision
disease are all less common causes are symptoms associated with open-
of progressive loss of mental function angle glaucoma.
in elderly patients. 14. A. hypoclacemia. Chvostek’s sign is
7. A. Apply heat compress to the a spasm of the facial muscles elicited
affected area. Options B, C and D are by tapping the facial nerve and is
appropriate nursing interventions associated with hypocalcemia.
when caring for a client diagnosed Clinical signs of hypokalemia are
with osteomyelitis. The application of muscle weakness, leg cramps,
heat can increase edema and pain in fatigue, nausea and vomiting. Muscle
cramps, anorexia, nausea and
vomiting are clinical signs of dissecting abdominal aneurysm are
hyponatremia. Clinical manifestations incorrect responses.
associated with hypophosphatemia 22. C. Fluid volume excess. SIADH
include muscle pain, confusion, results in fluid retention and
seizures and coma. hyponatremia. Correction is aimed at
15. B. BUN/Crea. The BUN is primarily restoring fluid and electrolyte
used as indicator of kidney function balance. Anxiety and risk for injury
because most renal diseases should be addressed following fluid
interfere with its excretion and cause volume excess.
blood vessels to rise. Creatinine is 23. C. Decreasing caffeine drinks and
produced in relatively constant alcohol. Caffeine and alcohol can
amounts, according to the amount of increase bladder spasms and
muscle mass and is excreted entirely mucosal irritation, thus increase the
by the kidneys making it a good signs and symptoms of UTI. All
indicator of renal function. antibiotics should be taken
16. B. 120 mg/dl. Hyperglycemia is completely to prevent resistant
defined as a blood glucose level strains of organisms.
greater than 120 mg/dl. Blood 24. C. deep vein thrombosis
glucose levels of 120 mg/dl, 130 (DVT). DVT, is the most probable
mg/dl and 150 mg/dl are considered complication for postoperative
hyperglycemic. A blood glucose of patients on bed rest. Options A, B and
100 mg/dl is normal. D aren’t likely complications of the
17. C. Stage 3. A stage 3 ulcer is full post operative period.
thickness involving the subcutaneous 25. B. The loss of vision is experienced
tissue. A stage 1 ulcer has a defined as a painless, gradual
area of persistent redness in lightly blurring. Typically, a patient with
pigmented skin. A stage 2 ulcer cataracts experiences painless,
involves partial thickness skin loss. gradual loss of vision. Although both
Stage 4 ulcers extend through the eyes may develop at different rates.
skin and exhibit tissue necrosis and
muscle or bone involvement.
18. C. Instruct him to avoid sexual
contact during acute phases of
illness. Herpes is a virus and is
spread through direct contact. An
antifungal would not be useful; bed
rest and temperature measurement
are usually not necessary.
19. C. Patency of airway. A burn face,
neck or chest may cause airway
closure because of the edema that
occurs within hours. Remember the
ABC’s: airway, breathing and
circulation. Airway always comes
first, even before pain. The nurse will
also assess options B and D, but
these are not the highest priority
assessments.
20. A. dizziness. Central tissue hypoxia
is commonly associated with
dizziness. Recognition of cerebral
hypoxia is critical since the body will
attempt to shunt oxygenated blood
to vital organs.
21. D. Dissecting thoracic aneurysm. A
dissecting thoracic aneurysm may
cause loss of radical pulses and
severe chest and back pain. An MI
typically doesn’t cause loss of radial
pulses or severe back pain. CVA and
troubled by he child’s emotional illness.
The nurse’s most therapeutic initial
response would be:

A. “You may be able to lessen your


feelings of guilt by seeking
counseling”
B. “It would be helpful if you become
involved in volunteer work at this
time”
C. “I recognize it’s hard to deal with
this, but try to remember that this
too shall pass”

PNLE Exam 1 D. “Joining a support group of parents


who are coping with this problem can
be quite helpful.
1. A 10 year old who has sustained a
5. To check for wound hemorrhage after a
head injury is brought to the emergency
client has had a surgery for the removal
department by his mother. A diagnosis of
of a tumor in the neck, nurse grace
a mild concussion is made. At the time of
should:
discharge, nurse Ron should instruct the
mother to:
A. Loosen an edge of the dressing and
lift it to see the wound
A. Withhold food and fluids for 24
B. Observe the dressing at the back of
hours.
the neck for the presence of blood
B. Allow him to play outdoors with his
C. Outline the blood as it appears on
friends.
the dressing to observe any
C. Arrange for a follow up visit with
progression
the child’s primary care provider in
D. Press gently around the incision to
one week.
express accumulated blood from the
D. Check for any change in
wound
responsiveness every two hours until
6. A 16-year-old primigravida arrives at
the follow-up visit.
the labor and birthing unit in her 38th
2. A male client has suffered a motor
week of gestation and states that she is
accident and is now suffering from
labor. To verify that the client is in true
hypovolemic shock. Nurse Helen should
labor nurse Trina should:
frequency assess the client’s vital signs
during the compensatory stage of shock,
A. Obtain sides for a fern test
because:
B. Time any uterine contractions
C. Prepare her for a pelvic
A. Arteriolar constriction occurs
examination
B. The cardiac workload decreases
D. Apply nitrazine paper to moist
C. Decreased contractility of the heart
vaginal tissue
occurs
7. As part of the diagnostic workup for
D. The parasympathetic nervous
pulmonic stenosis, a child has cardiac
system is triggered
catheterization. Nurse Julius is aware that
3. A paranoid male client with
children with pulmonic stenosis have
schizophrenia is losing weight, reluctant
increased pressure:
to eat, and voicing concerns about being
poisoned. The best intervention by nurse
A. In the pulmonary vein
Dina would be to:
B. In the pulmonary artery
C. On the left side of the heart
A. Allow the client to open canned or
D. On the right side of the heart
pre-packaged food
8. An obese client asks nurse Julius how
B. Restrict the client to his room until
to lose weight. Before answering, the
2 lbs are gained
nurse should remember that long-term
C. Have a staff member personally
weight loss occurs best when:
taste all of the client’s food
D. Tell the client the food has been x-
A. Eating patterns are altered
rayed by the staff and is safe
B. Fats are limited in the diet
4. One day the mother of a young adult
C. Carbohydrates are regulated
confides to nurse Frida that she is very
D. Exercise is a major component When the client has contractions, the
9. As a very anxious female client is nurse notes a 15 beat per minute
talking to the nurse May, she starts deceleration of the fetal heart rate below
crying. She appears to be upset that she the baseline lasting 15 seconds. Nurse
cannot control her crying. The most Cathy should:
appropriate response by the nurse would
be: A. Change the maternal position
B. Prepare for an immediate birth
A. “Is talking about your problem C. Call the physician immediately
upsetting you?” D. Obtain the client’s blood pressure
B. “It is Ok to cry; I’ll just stay with 15. A male client receiving prolonged
you for now” steroid therapy complains of always
C. “You look upset; lets talk about why being thirsty and urinating frequently.
you are crying.” The best initial action by the nurse would
D. “Sometimes it helps to get it out of be to:
your system.”
10. A patient has partial-thickness burns A. Perform a finger stick to test the
to both legs and portions of his trunk. client’s blood glucose level
Which of the following I.V. fluids is given B. Have the physician assess the
first? client for an enlarged prostate
C. Obtain a urine specimen from the
A. Albumin client for screening purposes
B. D5W D. Assess the client’s lower
C. Lactated Ringer’s solution extremities for the presence of pitting
D. 0.9% sodium chloride solution with edema
2 mEq of potassium per 100 ml 16. Nurse Bea recognizes that a
11. During the first 48 hours after a pacemaker is indicated when a client is
severe burn of 40% of the clients body experiencing:
surface, the nurse’s assessment should
include observations for water A. Angina
intoxication. Associated adaptations B. Chest pain
include: C. Heart block
D. Tachycardia
A. Sooty-colored sputum 17. When administering pancrelipase
B. Frothy pink-tinged sputum (Pancreases capsules) to child with cystic
C. Twitching and disorientation fibrosis, nurse Faith knows they should be
D. Urine output below 30ml per hour given:
12. After a muscle biopsy, nurse Willy
should teach the client to: A. With meals and snacks
B. Every three hours while awake
A. Change the dressing as needed C. On awakening, following meals,
B. Resume the usual diet as soon as and at bedtime
desired D. After each bowel movement and
C. Bathe or shower according to after postural draianage
preference 18. A preterm neonate is receiving
D. Expect a rise in body temperature oxygen by an overhead hood. During the
for 48 hours time the infant is under the hood, it
13. Before a client whose left hand has would be appropriate for nurse Gian to:
been amputated can be fitted for a
prosthesis, nurse Joy is aware that: A. Hydrate the infant q15 min
B. Put a hat on the infant’s head
A. Arm and shoulder muscles must be C. Keep the oxygen concentration
developed consistent
B. Shrinkage of the residual limb must D. Remove the infant q15 min for
be completed stimulation
C. Dexterity in the other extremity 19. A client’s sputum smears for acid fast
must be achieved bacilli (AFB) are positive, and
D. Full adjustment to the altered body transmission-based airborne precautions
image must have occurred are ordered. Nurse Kyle should instruct
14. Nurse Cathy applies a fetal monitor visitors to:
to the abdomen of a client in active labor.
A. Limit contact with non-exposed C. A nonelectric call bell
family members D. A cold-stream vaporizer
B. Avoid contact with any objects 25. Nurse Oliver interviews a young
present in the client’s room female client with anorexia nervosa to
C. Wear an Ultra-Filter mask when obtain information for the nursing history.
they are in the client’s room The client’s history is likely to reveal a:
D. Put on a gown and gloves before
going into the client’s room A. Strong desire to improve her body
20. A client with a head injury has a image
fixed, dilated right pupil; responds only to B. Close, supportive mother-daughter
painful stimuli; and exhibits decorticate relationship
posturing. Nurse Kate should recognize C. Satisfaction with and desire to
that these are signs of: maintain her present weight
D. Low level of achievement in school,
A. Meningeal irritation with little concerns for grades
B. Subdural hemorrhage 26. Nurse Bea should plan to assist a
C. Medullary compression client with an obsessive-compulsive
D. Cerebral cortex compression disorder to control the use of ritualistic
21. After a lateral crushing chest injury, behavior by:
obvious right-sided paradoxic motion of A. Providing repetitive activities that
the client’s chest demonstrates multiple require little thought
rib fraactures, resulting in a flail chest. B. Attempting to reduce or limit
The complication the nurse should situations that increase anxiety
carefully observe for would be: C. Getting the client involved with
activities that will provide distraction
A. Mediastinal shift D. Suggesting that the client perform
B. Tracheal laceration menial tasks to expiate feelings of
C. Open pneumothorax guilt
D. Pericardial tamponade 27. A 2 ½ year old child undergoes a
22. When planning care for a client at 30- ventriculoperitoneal shunt revision.
weeks gestation, admitted to the hospital Before discharge, nurse John, knowing
after vaginal bleeding secondary to the expected developmental behaviors
placenta previa, the nurse’s primary for this age group, should tell the parents
objective would be: to call the physician if the child:

A. Provide a calm, quiet environment A. Tries to copy all the father’s


B. Prepare the client for an immediate mannerisms
cesarean birth B. Talks incessantly regardless of the
C. Prevent situations that may presence of others
stimulate the cervix or uterus C. Becomes fussy when frustrated
D. Ensure that the client has regular and displays a shortened attention
cervical examinations assess for span
labor D. Frequently starts arguments with
23. When planning discharge teaching for playmates by claiming all toys are
a young female client who has had a “mine”
pneumothorax, it is important that the 28. A urinary tract infection is a potential
nurse include the signs and symptoms of danger with an indwelling catheter. Nurse
a pneumothorax and teach the client to Gina can best plan to avoid this
seek medical assistance if she complication by:
experiences:
A. Assessing urine specific gravity
A. Substernal chest pain B. Maintaining the ordered hydration
B. Episodes of palpitation C. Collecting a weekly urine specimen
C. Severe shortness of breath D. Emptying the drainage bag
D. Dizziness when standing up frequently
24. After a laryngectomy, the most 29. A client has sustained a fractured
important equipment to place at the right femur in a fall on stairs. Nurse Troy
client’s bedside would be: with the emergency response team
assess for signs of circulatory impairment
A. Suction equipment by:
B. Humidified oxygen
A. Turning the client to side lying 35. The laboratory calls to state that a
position client’s lithium level is 1.9 mEq/L after 10
B. Asking the client to cough and days of lithium therapy. Nurse Reese
deep breathe should:
C. Taking the client’s pedal pulse in
the affected limb A. Notify the physician of the findings
D. Instructing the client to wiggle the because the level is dangerously high
toes of the right foot B. Monitor the client closely because
30. To assess orientation to place in a the level of lithium in the blood is
client suspected of having dementia of slightly elevated
the alzheimers type, nurse Chris should C. Continue to administer the
ask: medication as ordered because the
level is within the therapeutic range
A. “Where are you?” D. Report the findings to the physician
B. “Who brought you here?” so the dosage can be increased
C. “Do you know where you are?” because the level is below
D. “How long have you been there?” therapeutic range
31. Nurse Mary assesses a postpartum 36. A client has a regular 30-day
client who had an abruption placentae menstrual cycles. When teaching about
and suspects that disseminated the rhythm method, Which the client and
intravascular coagulation (DIC) is her husband have chosen to use for
occurring when assessments family planning, nurse Dianne should
demonstrate: emphasize that the client’s most fertile
days are:
A. A boggy uterus
B. Multiple vaginal clots A. Days 9 to 11
C. Hypotension and tachycardia B. Days 12 to 14
D. Bleeding from the venipuncture C. Days 15 to 17
site D. Days 18 to 20
32. When a client on labor experiences 37. Before an amniocentesis, nurse
the urge to push a 9cm dilation, the Alexandra should:
breathing pattern that nurse Rhea should
instruct the client to use is the: A. Initiate the intravenous therapy as
ordered by the physiscian
A. Expulsion pattern B. Inform the client that the procedure
B. Slow paced pattern could precipitate an infection
C. Shallow chest pattern C. Assure that informed consent has
D. blowing pattern been obtained from the client
33. Nurse Ronald should explain that the D. Perform a vaginal examination on
most beneficial between-meal snack for a the client to assess cervical dilation
client who is recovering from the full- 38. While a client is on intravenous
thickness burns would be a: magnesium sulfate therapy for
preeclampsia, it is essential for nurse
A. Cheeseburger and a malted Amy to monitor the client’s deep tendon
B. Piece of blueberry pie and milk reflexes to:
C. Bacon and tomato sandwich and
tea A. Determine her level of
D. Chicken salad sandwich and soft consciousness
drink B. Evaluate the mobility of the
34. Nurse Wilma recognizes that failure extremities
of a newborn to make the appropriate C. Determine her response to painful
adaptation to extrauterine life would be stimuli
indicated by: D. Prevent development of respiratory
distress
A. flexed extremities 39. A preschooler is admitted to the
B. Cyanotic lips and face hospital with a diagnosis of acute
C. A heart rate of 130 beats per glomerulonephritis. The child’s history
minute reveals a 5-pound weight gain in one
D. A respiratory rate of 40 breath per week and peritoneal edema. For the most
minute accurate information on the status of the
child’s edema, nursing intervention The statement that should alert nurse
should include: Gina to this feeling would be:

A. Obtaining the child’s daily weight A. “I can’t wait to see all my friends
B. Doing a visual inspection of the again”
child B. “I feel washed out; there isn’t much
C. Measuring the child’s intake and left”
output C. “I can’t wait to get home to see my
D. Monitoring the child’s electrolyte grandchild”
values D. “My husband plans for me to
40. Nurse Mickey is administering recuperate at our daughter’s home”
dexamethasome (Decadron) for the early 45. A client with obstruction of the
management of a client’s cerebral common bile duct may show a prolonged
edema. This treatment is effective bleeding and clotting time because:
because:
A. Vitamin K is not absorbed
A. Acts as hyperosmotic diuretic B. The ionized calcium levels falls
B. Increases tissue resistance to C. The extrinsic factor is not absorbed
infection D. Bilirubin accumulates in the plasma
C. Reduces the inflammatory 46. Realizing that the hypokalemia is a
response of tissues side effect of steroid therapy, nurse
D. Decreases the information of Monette should monitor a client taking
cerebrospinal fluid steroid medication for:
41. During newborn nursing assessment,
a positive Ortolani’s sign would be A. Hyperactive reflexes
indicated by: B. An increased pulse rate
C. Nausea, vomiting, and diarrhea
A. A unilateral droop of hip D. Leg weakness with muscle cramps
B. A broadening of the perineum 47. When assessing a newborn suspected
C. An apparent shortening of one leg of having Down syndrome, nurse Rey
D. An audible click on hip would expect to observe:
manipulation
42. When caring for a dying client who is A. long thin fingers
in the denial stage of grief, the best B. Large, protruding ears
nursing approach would be to: C. Hypertonic neck muscles
D. Simian lines on the hands
A. Agree and encourage the client’s 48. A 10 year old girl is admitted to the
denial pediatric unit for recurrent pain and
B. Allow the denial but be available to swelling of her joints, particularly her
discuss death knees and ankles. Her diagnosis is
C. Reassure the client that everything juvenile rheumatoid arthritis. Nurse Janah
will be OK recognizes that besides joint
D. Leave the client alone to confront inflammation, a unique manifestation of
the feelings of impending loss the rheumatoid process involves the:
43. To decrease the symptoms of
gastroesophageal reflux disease (GERD), A. Ears
the physician orders dietary and B. Eyes
medication management. Nurse Helen C. Liver
should teach the client that the meal D. Brain
alteration that would be most 49. A disturbed client is scheduled to
appropriate would be: begin group therapy. The client refuses to
attend. Nurse Lolit should:
A. Ingest foods while they are hot
B. Divide food into four to six meals a A. Accept the client’s decision without
day discussion
C. Eat the last of three meals daily by B. Have another client to ask the
8pm client to consider
D. Suck a peppermint candy after C. Tell the client that attendance at
each meal the meeting is required
44. After a mastectomy or hysterectomy, D. Insist that the client join the group
clients may feel incomplete as women. to help the socialization process
50. Because a severely depressed client A. “I need a lot of help with my
has not responded to any of the troubles”
antidepressant medications, the B. “Society makes people react in old
psychiatrist decides to try ways”
electroconvulsive therapy (ECT). Before C. “I decided that it’s time I own up to
the treatment the nurse should: my problems”
D. “My life needs straightening out
A. Have the client speak with other and this might help”
clients receiving ECT 55. A child visits the clinic for a 6-week
B. Give the client a detailed checkup after a tonsillectomy and
explanation of the entire procedure adenoidectomy. In addition to assessing
C. Limit the client’s intake to a light hearing, the nurse should include an
breakfast on the days of the assessment of the child’s:
treatment
D. Provide a simple explanation of the A. Taste and smell
procedure and continue to reassure B. Taste and speech
the client C. Swallowing and smell
51. Nurse Vicky is aware that teaching D. Swallowing and speech
about colostomy care is understood when 56. A client is diagnosed with cancer of
the client states, “I will contact my the jaw. A course of radiation therapy is
physician and report ____”: to be followed by surgery. The client is
concerned about the side effects related
A. If I notice a loss of sensation to to the radiation treaments. Nurse Ria
touch in the stoma tissue” should explain that the major side effects
B. When mucus is passed from the that will experienced is:
stoma between irrigations”
C. The expulsion of flatus while the A. Fatigue
irrigating fluid is running out” B. Alopecia
D. If I have difficulty in inserting the C. Vomiting
irrigating tube into the stoma” D. Leucopenia
52. The client’s history that alerts nurse 57. Nurse Katrina prepares an older-adult
Henry to assess closely for signs of client for sleep, actions are taken to help
postpartum infection would be: reduce the likelihood of a fall during the
night. Targeting the most frequent cause
A. Three spontaneous abortions of falls, the nurse should:
B. negative maternal blood type
C. Blood loss of 850 ml after a vaginal A. Offer the client assistance to the
birth bathroom
D. Maternal temperature of 99.9° F 12 B. Move the bedside table closer to
hours after delivery the client’s bed
53. A client is experiencing stomatitis as C. Encourage the client to take an
a result of chemotherapy. An appropriate available sedative
nursing intervention related to this D. Assist the client to telephone the
condition would be to: spouse to say “goodnight”
58. When evaluating a growth and
A. Provide frequent saline development of a 6 month old infant,
mouthwashes nurse Patty would expect the infant to be
B. Use karaya powder to decrease able to:
irritation
C. Increase fluid intake to compensate A. Sit alone, display pincer grasp,
for the diarrhea wave bye bye
D. Provide meticulous skin care of the B. Pull self to a standing position,
abdomen with Betadine release a toy by choice, play peek-a-
54. During a group therapy session, one boo
of the clients ask a male client with the C. Crawl, transfer toy from one hand
diagnosis of antisocial personality to the other, display of fear of
disorder why he is in the hospital. strangers
Considering this client’s type of D. Turn completely over, sit
personality disorder, the nurse might momentarily without support, reach
expect him to respond: to be picked up
59. A breastfeeding mother asks the C. Weight gain is expected, and
nurse what she can do to ease the dietary plan are needed
discomfort caused by a cracked nipple. D. Depression is normal and should be
Nurse Tina should instruct the client to: expected
64. An adolescent client with anorexia
A. Manually express milk and feed it nervosa refuses to eat, stating, “I’ll get
to the baby in a bottle too fat.” Nurse Andrea can best respond
B. Stop breastfeeding for two days to to this behavior initially by:
allow the nipple to heal
C. Use a breast shield to keep the A. Not talking about the fact that the
baby from direct contact with the client is not eating
nipple B. Stopping all of the client’s
D. Feed the baby on the unaffected priviledges until food is eaten
breast first until the affected breast C. Telling the client that tube feeding
heals will eventually be necessary
60. Nurse Sandy observes that there is D. Pointing out to the client that death
blood coming from the client’s ear after can occur with malnutrition.
head injury. Nurse Sandy should: 65. A pain scale is used to assess the
degree of pain. The client rates the pain
A. Turn the client to the unaffected as an 8 on a scale of 10 before
side medication and a 7 on a scale of 10 after
B. Cleanse the client’s ear with sterile being medicated. Nurse Glenda
gauze determines that the:
C. Test the drainage from the client’s
ear with Dextrostix A. Client has a low pain tolerance
D. Place sterile cotton loosely in the B. Medication is not adequately
external ear of the client effective
61. Nurse Gio plans a long term care for C. Medication has sufficiently
parents of children with sickle-cell decreased the pain level
anemia, which includes periodic group D. Client needs more education about
conferences. Some of the discussions the use of the pain scale
should be directed towards: 66. To enhance a neonate’s behavioral
development, therapeutic nursing
A. Finding special school facilities for measures should include:
the child
B. Making plans for moving to a more A. Keeping the baby awake for longer
therapeutic climate periods of time before each feeding
C. Choosing a means of birth control B. Assisting the parents to stimulate
to avoid future pregnancies their baby through touch, sound, and
D. Airing their feelings regarding the sight.
transmission of the disease to the C. Encouraging parental contact for at
child least one 15-minute period every four
62. The central problem the nurse might hours.
face with a disturbed schizophrenic client D. Touching and talking to the baby at
is the client’s: least hourly, beginning within two to
four hours after birth
A. Suspicious feelings 67. Before formulating a plan of care for
B. Continuous pacing a 6 year old boy with attention deficit
C. Relationship with the family hyperactivity disorder (ADHD), nurse Kyla
D. Concern about working with others is aware that the initial aim of therapy is
63. When planning care with a client to help the client to:
during the postoperative recovery period
following an abdominal hysterectomy A. Develop language skills
and bilateral salpingo-oophorectomy, B. Avoid his own regressive behavior
nurse Frida should include the C. Mainstream into a regular class in
explanation that: school
D. Recognize himself as an
A. Surgical menopause will occur independent person of worth
B. Urinary retention is a common 68. Nurse Wally knows that the most
problem important aspect of the preoperative
care for a child with Wilms’ tumor would A. It involves providing home care to
be: sick people who are not confined in
the hospital
A. Checking the size of the child’s B. Services are provided free of
liver charge to people within the
B. Monitoring the child’s blood catchment area.
pressure C. The public health nurse functions
C. Maintaining the child in a prone as part of a team providing a public
position health nursing services.
D. Collecting the child’s urine for D. Public health nursing focuses on
culture and sensitivity preventive, not curative, services.
69. At 11:00 pm the count of 74. Which of the following is the mission
hydrocodone (Vicodin) is incorrect. After of the Department of Health?
several minutes of searching the
medication cart and medication A. Health for all Filipinos
administration records, no explanation B. Ensure the accessibility and quality
can be found. The primary nurse should of health care
notify the: C. Improve the general health status
of the population
A. Nursing unit manager D. Health in the hands of the Filipino
B. Hospital administrator people by the year 2020
C. Quality control manager 75. Nurse Pauline determines whether
D. Physician ordering the medication resources were maximized in
70. When caring for the a client with a implementing Ligtas Tigdas, she is
pneumothorax, who has a chest tube in evaluating:
place, nurse Kate should plan to:
A. Effectiveness
A. Administer cough suppressants at B. Efficiency
appropriate intervals as ordered C. Adequacy
B. Empty and measure the drainage D. Appropriateness
in the collection chamber each shift 76. Lissa is a B.S.N. graduate. She want
C. Apply clamps below the insertion to become a Public Health Nurse. Where
site when ever getting the client out will she apply?
of bed
D. Encourage coughing, deep A. Department of Health
breathing, and range of motion to the B. Provincial Health Office
arm on the affected side C. Regional Health Office
71. According to C.E.Winslow, which of D. Rural Health Unit
the following is the goal of Public Health? 77. As an epidemiologist, Nurse Celeste
is responsible for reporting cases of
A. For people to attain their notifiable diseases. What law mandates
birthrights of health and longevity reporting of cases of notifiable diseases?
B. For promotion of health and
prevention of disease A. Act 3573
C. For people to have access to basic B. R.A. 3753
health services C. R.A. 1054
D. For people to be organized in their D. R.A. 1082
health efforts 78. Nurse Fay is aware that isolation of a
72. What other statistic may be used to child with measles belongs to what level
determine attainment of longevity? of prevention?

A. Age-specific mortality rate A. Primary


B. Proportionate mortality rate B. Secondary
C. Swaroop’s index C. Intermediate
D. Case fatality rate D. Tertiary
73. Which of the following is the most 79. Nurse Gina is aware that the
prominent feature of public health following is an advantage of a home
nursing? visit?
A. It allows the nurse to provide A. Checking with the relative of the
nursing care to a greater number of patient
people. B. Preparing a nursing care plan in
B. It provides an opportunity to do collaboration with the patient
first hand appraisal of the home C. Consulting with the physician
situation. D. Coordinating with other members
C. It allows sharing of experiences of the team
among people with similar health 85. Nurse Tony stresses the need for all
problems. the employees to follow orders and
D. It develops the family’s initiative in instructions from him and not from
providing for health needs of its anyone else. Which of the following
members. principles does he refer to?
80. The PHN bag is an important tool in
providing nursing care during a home A. Scalar chain
visit. The most important principle of bag B. Discipline
technique states that it: C. Unity of command
D. Order
A. Should save time and effort. 86. Nurse Joey discusses the goal of the
B. Should minimize if not totally department. Which of the following
prevent the spread of infection. statements is a goal?
C. Should not overshadow concern for
the patient and his family. A. Increase the patient satisfaction
D. May be done in a variety of ways rate
depending on the home situation, B. Eliminate the incidence of delayed
etc. administration of medications
81. Nurse Willy reads about Path Goal C. Establish rapport with patients
theory. Which of the following behaviors D. Reduce response time to two
is manifested by the leader who uses this minutes
theory? 87. Nurse Lou considers shifting to
transformational leadership. Which of the
A. Recognizes staff for going beyond following statements best describes this
expectations by giving them citations type of leadership?
B. Challenges the staff to take
individual accountability for their own A. Uses visioning as the essence of
practice leadership
C. Admonishes staff for being B. Serves the followers rather than
laggards being served
D. Reminds staff about the sanctions C. Maintains full trust and confidence
for non performance in the subordinates
82. Nurse Cathy learns that some leaders D. Possesses innate charisma that
are transactional leaders. Which of the makes others feel good in his
following does NOT characterize a presence.
transactional leader? 88. Nurse Mae tells one of the staff, “I
don’t have time to discuss the matter
A. Focuses on management tasks with you now. See me in my office later”
B. Is a caretaker when the latter asks if they can talk
C. Uses trade-offs to meet goals about an issue. Which of the following
D. Inspires others with vision conflict resolution strategies did she use?
83. Functional nursing has some
advantages, which one is an EXCEPTION? A. Smoothing
B. Compromise
A. Psychological and sociological C. Avoidance
needs are emphasized. D. Restriction
B. Great control of work activities. 89. Nurse Bea plans of assigning
C. Most economical way of delivering competent people to fill the roles
nursing services. designed in the hierarchy. Which process
D. Workers feel secure in dependent refers to this?
role
84. Which of the following is the best A. Staffing
guarantee that the patient’s priority B. Scheduling
needs are met? C. Recruitment
D. Induction 96. The use of larvivorous fish in malaria
90. Nurse Linda tries to design an control is the basis for which strategy of
organizational structure that allows malaria control?
communication to flow in all directions
and involve workers in decision making. A. Stream seeding
Which form of organizational structure is B. Stream clearing
this? C. Destruction of breeding places
D. Zooprophylaxis
A. Centralized 97. In Integrated Management of
B. Decentralized Childhood Illness, severe conditions
C. Matrix generally require urgent referral to a
D. Informal hospital. Which of the following severe
91. When documenting information in a conditions DOES NOT always require
client’s medical record, the nurse should: urgent referral to a hospital?

A. erase any errors. A. Mastoiditis


B. use a #2 pencil. B. Severe dehydration
C. leave one line blank before each C. Severe pneumonia
new entry. D. Severe febrile disease
D. end each entry with the nurse’s 98. A mother brought her daughter, 4
signature and title. years old, to the RHU because of cough
92. Which of the following factors are and colds. Following the IMCI assessment
major components of a client’s general guide, which of the following is a danger
background drug history? sign that indicates the need for urgent
referral to a hospital?
A. Allergies and socioeconomic status
B. Urine output and allergies A. Inability to drink
C. Gastric reflex and age B. High grade fever
D. Bowel habits and allergies C. Signs of severe dehydration
93. Which procedure or practice requires D. Cough for more than 30 days
surgical asepsis? 99. Food fortification is one of the
strategies to prevent micronutrient
A. Hand washing deficiency conditions. R.A. 8976
B. Nasogastric tube irrigation mandates fortification of certain food
C. I.V. cannula insertion items. Which of the following is among
D. Colostomy irrigation these food items?
94. The nurse is performing wound care
using surgical asepsis. Which of the A. Sugar
following practices violates surgical B. Bread
asepsis? C. Margarine
D. Filled milk
A. Holding sterile objects above the 100. The major sign of iron deficiency
waist anemia is pallor. What part is best
B. Pouring solution onto a sterile field examined for pallor?
cloth
C. Considering a 1″ (2.5-cm) edge A. Palms
around the sterile field contaminated B. Nailbeds
D. Opening the outermost flap of a C. Around the lips
sterile package away from the body D. Lower conjunctival sac
95. On admission, a client has the Answers and Rationales
following arterial blood gas (ABG) values:
PaO2, 50 mm Hg; PaCO2, 70 mm Hg; pH, 1. C. Check for any change in
7.20; HCO3–, 28 mEq/L. Based on these responsiveness every two hours until
values, the follow-up visit. Signs of an
the nurse should formulate which nursing epidural hematoma in children
diagnosis for this client? usually do not appear for 24 hours or
more hours; a follow-up visit usually
A. Risk for deficient fluid volume is arranged for one to two days after
B. Deficient fluid volume the injury.
C. Impaired gas exchange 2. A. Arteriolar constriction occurs.The
D. Metabolic acidosis early compensation of shock is
cardiovascular and is seen in changes twitching, sleepiness, and
in pulse, BP, and pulse pressure; convulsions.
blood is shunted to vital centers, 12. B. Resume the usual diet as soon
particularly heart and brain. as desired. As long as the client has
3. A. Allow the client to open canned no nausea or vomiting, there are no
or pre-packaged food. The client’s dietary restriction.
comfort, safety, and nutritional status 13. B. Shrinkage of the residual limb
are the priorities; the client may feel must be completed. Shrinkage of the
comfortable to eat if the food has residual limb, resulting from
been sealed before reaching the reduction of subcutaneous fat and
mental health facility. interstitial fluid, must occur for an
4. D. “Joining a support group of adequate fit between the limb and
parents who are coping with this the prosthesis.
problem can be quite helpful. Taking 14. A. Change the maternal
with others in similar circumstances position. Stimulation of the
provides support and allows for sympathetic nervous system is an
sharing of experiences. initial response to mild hypoxia that
5. B. Observe the dressing at the back accompanies partial cord
of the neck for the presence of compression (umbilical vein) during
blood. Drainage flows by gravity. contractions; changing the maternal
6. C. Prepare her for a pelvic position can alleviate the
examination. Pelvic examination compression.
would reveal dilation and effacement 15. A. Perform a finger stick to test the
7. D. On the right side of the client’s blood glucose level. The client
heart. Pulmonic stenosis increases has signs of diabetes, which may
resistance to blood flow, causing right result from steroid therapy, testing
ventricular hyperthropy; with right the blood glucose level is a method of
ventricular failure there is an increase screening for diabetes, thus
in pressure on the right side of the gathering more data.
heart. 16. C. Heart block. This is the primary
8. A. Eating patterns are altered. A indication for a pacemaker because
new dietary regimen, with a balance there is an interfere with the
of foods from the food pyramid, must electrical conduction system of the
be established and continued for heart.
weight reduction to occur and be 17. A. With meals and
maintained. snacks. Pancreases capsules must be
9. B. “It is Ok to cry; I’ll just stay with taken with food and snacks because
you for now”. This portrays a it acts on the nutrients and readies
nonjudgmental attitude that them for absorption.
recognizes the client’s needs. 18. B. Put a hat on the infant’s
10. C. Lactated Ringer’s head. Oxygen has cooling effect, and
solution. Lactated Ringer’s solution the baby should be kept warm so that
replaces lost sodium and corrects metabolic activity and oxygen
metabolic acidosis, both of which demands are not increased.
commonly occur following a burn. 19. C. Wear an Ultra-Filter mask when
Albumin is used as adjunct therapy, they are in the client’s room. Tubercle
not primary fluid replacement. bacilli are transmitted through air
Dextrose isn’t given to burn patients currents; therefore personal
during the first 24 hours because it protective equipment such as an
can cause pseudodiabetes. The Ultra-Filter mask is necessary.
patient is hyperkalemic from the 20. D. Cerebral cortex
potassium shift from the intracellular compression. Cerebral compression
space to the plasma, so potassium affects pyramidal tracts, resulting in
would be detrimental. decorticate rigidity and cranial nerve
11. C. Twitching and injury, which cause pupil dilation.
disorientation. Excess extracellular 21. A.Mediastinal shift. Mediastinal
fluid moves into cells (water structures move toward the uninjured
intoxication); intracellular fluid excess lung, reducing oxygenation and
in sensitive brain cells causes altered venous return.
mental status; other signs include 22. C. Prevent situations that may
anorexia nervosa, nausea, vomiting, stimulate the cervix or
uterus. Stimulation of the cervix or 34. B. Cyanotic lips and face. Central
uterus may cause bleeding or cyanosis (blue lips and face) indicates
hemorrhage and should be avoided. lowered oxygenation of the blood,
23. C. Severe shortness of breath. This caused by either decreased lung
could indicate a recurrence of the expansion or right to left shunting of
pneumothorax as one side of the lung blood.
is inadequate to meet the oxygen 35. A. Notify the physician of the
demands of the body. findings because the level is
24. A. Suction equipment. Respiratory dangerously high. Levels close to 2
complications can occur because of mEq/L are dangerously close to the
edema of the glottis or injury to the toxic level; immediate action must be
recurrent laryngeal nerve. taken.
25. A. Strong desire to improve her 36. C. Days 15 to 17. Ovulation occurs
body image. Clients with anorexia approximately 14 days before the
nervosa have a disturbed self image next menses, about the 16th day in
and always see themselves as fat 30 day cycle; the 15th to 17th days
and needing further reducing. would be the best time to avoid
26. B. Attempting to reduce or limit sexual intercourse.
situations that increase 37. C. Assure that informed consent
anxiety. Persons with high anxiety has been obtained from the client. An
levels develop various behaviors to invasive procedure such as
relieve their anxiety; by reducing amniocentesis requires informed
anxiety, the need for these consent.
obsessive-compulsive action is 38. D. Prevent development of
reduced. respiratory distress. Respiratory
27. C. Becomes fussy when frustrated distress or arrest may occur when the
and displays a shortened attention serum level of magnesium sulfate
span. Shortened attention span and reaches 12 to 15 mg/dl; deep tendon
fussy behavior may indicate a change reflexes disappear when the serum
in intracranial pressure and/or shunt level is 10 to 12 mg/dl; the drug is
malfunction. withheld in the absence of deep
28. B. Maintaining the ordered tendon reflexes; the therapeutic
hydration. Promoting hydration serum level is 5 to 8 mg/dl.
maintains urine production at a 39. A. Obtaining the child’s daily
higher rate, which flushes the bladder weight. Weight monitoring is the
and prevents urinary stasis and most useful means of assessing fluid
possible infection. balance and changes in the
29. C. Taking the client’s pedal pulse in edematous state; 1 liter of fluid
the affected limb. Monitoring a pedal weighs about 2.2 pounds.
pulse will assess circulation to the 40. C. Reduces the inflammatory
foot. response of tissues. Corticosteroids
30. A. “Where are you?”. “Where are act to decrease inflammation which
you?” is the best question to elicit decreases edema.
information about the client’s 41. D. An audible click on hip
orientation to place because it manipulation. With specific
encourages a response that can be manipulation, an audible click may be
assessed. heard of felt as he femoral head slips
31. D. Bleeding from the venipuncture into the acetabulum.
site. This indicates a fibrinogenemia; 42. B. Allow the denial but be available
massive clotting in the area of the to discuss death. This does not
separation has resulted in a lowered remove client’s only way of coping,
circulating fibrinogen. and it permits future movement
32. D. blowing pattern. Clients should through the grieving process when
use a blowing pattern to overcome the client is ready.
the premature urge to push. 43. B. Divide food into four to six meals
33. A. Cheeseburger and a malted. Of a day. The volume of food in the
the selections offered, this is the stomach should be kept small to limit
highest in calories and protein, which pressure on the cardiac sphincter.
are needed for increased basal 44. B. “I feel washed out; there isn’t
metabolic rate and for tissue repair. much left”. The client’s statement
infers an emptiness with an hospitalized clients is getting up or
associated loss. attempting to get up to the bathroom
45. A. Vitamin K is not unassisted.
absorbed. Vitamin K, a fat soluble 58. D. Turn completely over, sit
vitamin, is not absorbed from the GI momentarily without support, reach
tract in the absence of bile; bile to be picked up. These abilities are
enters the duodenum via the age-appropriate for the 6 month old
common bile duct. child.
46. D. Leg weakness with muscle 59. D. Feed the baby on the unaffected
cramps. Impulse conduction of breast first until the affected breast
skeletal muscle is impaired with heals. The most vigorous sucking will
decreased potassium levels, occur during the first few minutes of
muscular weakness and cramps may breastfeeding when the infant would
occur with hypokalemia. be on the unaffected breast; later
47. D. Simian lines on the hands. This suckling is less traumatic.
is characteristic finding in newborns 60. D. Place sterile cotton loosely in
with Down syndrome. the external ear of the client. This
48. B. Eyes. Rheumatoid arthritis can would absorb the drainage without
cause inflammation of the iris and causing further trauma.
ciliary body of the eyes which may 61. D. Airing their feelings regarding
lead to blindness. the transmission of the disease to the
49. A. Accept the client’s decision child. Discussion with parents who
without discussion. This is all the have children with similar problems
nurse can do until trust is helps to reduce some of their
established; facing the client to discomfort and guilt.
attend will disrupt the group. 62. A. Suspicious feelings. The nurse
50. D. Provide a simple explanation of must deal with these feelings and
the procedure and continue to establish basic trust to promote a
reassure the client. The nurse should therapeutic milieu.
offer support and use clear, simple 63. A. Surgical menopause will
terms to allay client’s anxiety. occur. When a bilateral oophorectomy
51. D. If I have difficulty in inserting the is performed, both ovaries are
irrigating tube into the stoma”. This excised, eliminating ovarian
occurs with stenosis of the stoma; hormones and initiating response.
forcing insertion of the tube could 64. D. Pointing out to the client that
cause injury. death can occur with
52. C. Blood loss of 850 ml after a malnutrition. The client expects the
vaginal birth. Excessive blood loss nurse to focus on eating, but the
predisposes the client to an increased emphasis should be placed on
risk of infection because of decreased feelings rather than actions.
maternal resistance; they expected 65. B. Medication is not adequately
blood loss is 350 to 500 ml. effective. The expected effect should
53. A. Provide frequent saline be more than a one point decrease in
mouthwashes. This is soothing to the the pain level.
oral mucosa and helps prevent 66. B. Assisting the parents to
infection. stimulate their baby through touch,
54. B. “Society makes people react in sound, and sight. Stimuli are provided
old ways”. The client is incapable of via all the senses; since the infant’s
accepting responsibility for self- behavioral development is enhanced
created problems and blames society through parent-infant interactions,
for the behavior. these interactions should be
55. A. Taste and smell. Swelling can encouraged.
obstruct nasal breathing, interfering 67. D. Recognize himself as an
with the senses of taste and smell. independent person of
56. A. Fatigue. Fatigue is a major worth. Academic deficits, an inability
problem caused by an increase in to function within constraints
waste products because of catabolic required of certain settings, and
processes. negative peer attitudes often lead to
57. A. Offer the client assistance to the low self-esteem.
bathroom. Statistics indicate that the 68. B. Monitoring the child’s blood
most frequent cause of falls by pressure. Because the tumor is of
renal origin, the rennin angiotensin group conference, while choice D is
mechanism can be involved, and true of a clinic consultation.
blood pressure monitoring is 80. B. Should minimize if not totally
important. prevent the spread of infection. Bag
69. A. Nursing unit manager. Controlled technique is performed before and
substance issues for a particular after handling a client in the home to
nursing unit are the responsibility of prevent transmission of infection to
that unit’s nurse manager. and from the client.
70. D. Encourage coughing, deep 81. A. Recognizes staff for going
breathing, and range of motion to the beyond expectations by giving them
arm on the affected side. All these citations. Path Goal theory according
interventions promote aeration of the to House and associates rewards
re-expanding lung and maintenance good performance so that others
of function in the arm and shoulder would do the same.
on the affected side. 82. D. Inspires others with
71. A. For people to attain their vision. Inspires others with a vision is
birthrights of health and characteristic of a transformational
longevity. According to Winslow, all leader. He is focused more on the
public health efforts are for people to day-to-day operations of the
realize their birthrights of health and department/unit.
longevity. 83. A. Psychological and sociological
72. C. Swaroop’s index. Swaroop’s needs are emphasized. When the
index is the percentage of the deaths functional method is used, the
aged 50 years or older. Its inverse psychological and sociological needs
represents the percentage of of the patients are neglected; the
untimely deaths (those who died patients are regarded as ‘tasks to be
younger than 50 years). done”
73. D. Public health nursing focuses on 84. B. Preparing a nursing care plan in
preventive, not curative, collaboration with the patient. The
services.. The catchment area in PHN best source of information about the
consists of a residential community, priority needs of the patient is the
many of whom are well individuals patient himself. Hence using a
who have greater need for preventive nursing care plan based on his
rather than curative services. expressed priority needs would
74. B. Ensure the accessibility and ensure meeting his needs effectively.
quality of health care. Ensuring the 85. C. Unity of command. The principle
accessibility and quality of health of unity of command means that
care is the primary mission of DOH. employees should receive orders
75. B. Efficiency. Efficiency is coming from only one manager and
determining whether the goals were not from two managers. This averts
attained at the least possible cost. the possibility of sowing confusion
76. D. Rural Health Unit. R.A. 7160 among the members of the
devolved basic health services to organization.
local government units (LGU’s ). The 86. A. Increase the patient satisfaction
public health nurse is an employee of rate. Goal is a desired result towards
the LGU. which efforts are directed. Options
77. A. Act 3573. Act 3573, the Law on AB, C and D are all objectives which
Reporting of Communicable Diseases, are aimed at specific end.
enacted in 1929, mandated the 87. A. Uses visioning as the essence of
reporting of diseases listed in the law leadership. Transformational
to the nearest health station. leadership relies heavily on visioning
78. A. Primary. The purpose of isolating as the core of leadership.
a client with a communicable disease 88. C. Avoidance. This strategy shuns
is to protect those who are not sick discussing the issue head-on and
(specific disease prevention). prefers to postpone it to a later time.
79. B. It provides an opportunity to do In effect the problem remains
first hand appraisal of the home unsolved and both parties are in a
situation. Choice A is not correct lose-lose situation.
since a home visit requires that the 89. A. Staffing. Staffing is a
nurse spend so much time with the management function involving
family. Choice C is an advantage of a putting the best people to accomplish
tasks and activities to attain the 95. C. Impaired gas exchange. The
goals of the organization. client has a below-normal value for
90. B. Decentralized. Decentralized the partial pressure of arterial oxygen
structures allow the staff to make (PaO2) and an above-normal value
decisions on matters pertaining to for the partial pressure of arterial
their practice and communicate in carbon dioxide (PaCO2), supporting
downward, upward, lateral and the nursing diagnosis of Impaired gas
diagonal flow. exchange. ABG values can’t indicate
91. D. end each entry with the nurse’s a diagnosis of Fluid volume deficit (or
signature and title. The end of each excess) or Risk for deficient fluid
entry should include the nurse’s volume. Metabolic acidosis is a
signature and title; the signature medical, not nursing, diagnosis; in
holds the nurse accountable for the any event, these ABG values indicate
recorded information. Erasing errors respiratory, not metabolic, acidosis.
in documentation on a legal 96. A. Stream seeding. Stream seeding
document such as a client’s chart is done by putting tilapia fry in
isn’t permitted by law. Because a streams or other bodies of water
client’s medical record is considered identified as breeding places of the
a legal document, the nurse should Anopheles mosquito.
make all entries in ink. The nurse is 97. B. Severe dehydration. The order of
accountable for the information priority in the management of severe
recorded and therefore shouldn’t dehydration is as follows: intravenous
leave any blank lines in which fluid therapy, referral to a facility
another health care worker could where IV fluids can be initiated within
make additions. 30 minutes, Oresol/nasogastric tube,
92. A. Allergies and socioeconomic Oresol/orem. When the foregoing
status. General background data measures are not possible or
consist of such components as effective, tehn urgent referral to the
allergies, medical history, habits, hospital is done.
socioeconomic status, lifestyle, 98. A. Inability to drink. A sick child
beliefs, and sensory deficits. Urine aged 2 months to 5 years must be
output, gastric reflex, and bowel referred urgently to a hospital if
habits are significant only if a disease he/she has one or more of the
affecting these functions is present. following signs: not able to feed or
93. C. I.V. cannula insertion. Caregivers drink, vomits everything, convulsions,
must use surgical asepsis when abnormally sleepy or difficult to
performing wound care or any awaken.
procedure in which a sterile body 99. A. Sugar. R.A. 8976 mandates
cavity is entered or skin integrity is fortification of rice, wheat flour, sugar
broken. To achieve surgical asepsis, and cooking oil with Vitamin A, iron
objects must be rendered or kept free and/or iodine.
of all pathogens. Inserting an I.V. 100. A. Palms. The anatomic
cannula requires surgical asepsis characteristics of the palms allow a
because it disrupts skin integrity and reliable and convenient basis for
involves entry into a sterile cavity (a examination for pallor.
vein). The other options are used to
ensure medical asepsis or clean
technique to prevent the spread of
infection. The GI tract isn’t sterile;
therefore, irrigating a nasogastric
tube or a colostomy requires only
clean technique.
94. B. Pouring solution onto a sterile
field cloth. Pouring solution onto a
sterile field cloth violates surgical
asepsis because moisture
penetrating the cloth can carry
microorganisms to the sterile field via
capillary action. The other options are
practices that help ensure surgical
asepsis.
B. Incubation period is 1 week
C. Incubation period is 1 month
D. Incubation period varies
depending on the site of the bite

4. Which of the following should Nurse


Cherry do first in taking care of a male
client with rabies?

A. Encourage the patient to take a


bath
B. Cover IV bottle with brown
paper bag
C. Place the patient near the comfort
room
D. Place the patient near the door

5. Which of the following is the screening


test for dengue hemorrhagic fever?

A. Complete blood count


B. ELISA
C. Rumpel-leede test
D. Sedimentation rate

PNLE Exam 2 6. Mr. Dela Rosa is suspected to have


1. A woman in a child bearing age malaria after a business trip in Palawan.
receives a rubella vaccination. Nurse Joy The most important diagnostic test in
would give her which of the following malaria is:
instructions?
A. WBC count
A. Refrain from eating eggs or egg B. Urinalysis
products for 24 hours C. ELISA
B. Avoid having sexual intercourse D. Peripheral blood smear
C. Don’t get pregnant at least 3
months
D. Avoid exposure to sun 7. The Nurse supervisor is planning for
patient’s assignment for the AM shift.
The nurse supervisor avoids assigning
2. Jonas who is diagnosed with which of the following staff members to a
encephalitis is under the treatment of client with herpes zoster?
Mannitol. Which of the following patient
outcomes indicate to Nurse Ronald that A. Nurse who never had chicken
the treatment of Mannitol has been pox
effective for a patient that has increased B. Nurse who never had roseola
intracranial pressure? C. Nurse who never had german
measles
A. Increased urinary output D. Nurse who never had mumps
B. Decreased RR
C. Slowed papillary response
D. Decreased level of consciousness 8. Clarissa is 7 weeks pregnant. Further
examination revealed that she is
susceptible to rubella. When would be
3. Mary asked Nurse Maureen about the the most appropriate for her to receive
incubation period of rabies. Which rubella immunization?
statement by the Nurse Maureen is
appropriate? A. At once
B. During 2nd trimester
A. Incubation period is 6 months C. During 3rd trimester
D. After the delivery of the baby 15. Which is the primary characteristic
lesion of syphilis?

9. A female child with rubella should be A. Sore eyes


isolated from a: B. Sore throat
C. Chancroid
A. 21 year old male cousin living in D. Chancre
the same house 16. What is the fast breathing of Jana
B. 18 year old sister who recently who is 3 weeks old?
got married
C. 11 year old sister who had rubeola A. 60 breaths per minute
during childhood B. 40 breaths per minute
D. 4 year old girl who lives next door C. 10 breaths per minute
D. 20 breaths per minute
10. What is the primary prevention of 17. Which of the following signs and
leprosy? symptoms indicate some dehydration?

A. Nutrition A. Drinks eagerly


B. Vitamins B. Restless and irritable
C. BCG vaccination C. Unconscious
D. DPT vaccination D. A and B
11. A bacteria which causes diphtheria is 18. What is the first line for dysentery?
also known as?
A. Amoxicillin
A. Amoeba B. Tetracycline
B. Cholera C. Cefalexin
C. Klebs-loeffler bacillus D. Cotrimoxazole
D. Spirochete 19. In home made oresol, what is the
12. Nurse Ron performed mantoux skin ratio of salt and sugar if you want to
test today (Monday) to a male adult prepare with 1 liter of water?
client. Which statement by the client
indicates that he understood the A. 1 tbsp. salt and 8 tbsp. sugar
instruction well? B. 1 tbsp. salt and 8 tsp. sugar
C. 1 tsp. salt and 8 tsp. sugar
A. I will come back later D. 8 tsp. salt and 8 tsp. sugar
B. I will come back next month 20. Gentian Violet is used for:
C. I will come back on Friday
D. I will come back on Wednesday, A. Wound
same time, to read the result B. Umbilical infections
13. A male client had undergone C. Ear infections
Mantoux skin test. Nurse Ronald notes an D. Burn
8mm area of indurations at the site of 21. Which of the following is a live
the skin test. The nurse interprets the attenuated bacterial vaccine?
result as:
A. BCG
A. Negative B. OPV
B. Uncertain and needs to be C. Measles
repeated D. None of the above
C. Positive 22. EPI is based on?
D. Inconclusive
14. Tony will start a 6 month therapy with A. Basic health services
Isoniazid (INH). Nurse Trish plans to teach B. Scope of community affected
the client to: C. Epidemiological situation
D. Research studies
A. Use alcohol moderately 23. TT? provides how many percentage
B. Avoid vitamin supplements while o of protection against tetanus?
therapy
C. Incomplete intake of dairy products A. 100
D. May be discontinued if symptoms B. 99
subsides C. 80
D. 90
24. Temperature of refrigerator to A. Survey
maintain potency of measles and OPV B. Close ended
vaccine is: C. Questionnaire
D. Demographic
A. -3c to -8c 33. A researcher that makes a
B. -15c to -25c generalization based on observations of
C. +15c to +25c an individuals behavior is said to be
D. +3c to +8c which type of reasoning:
25. Diptheria is a:
A. Inductive
A. Bacterial toxin B. Logical
B. Killed bacteria C. Illogical
C. Live attenuated D. Deductive
D. Plasma derivatives 34. The balance of a research’s benefit
26. Budgeting is under in which part of vs. its risks to the subject is:
management process?
A. Analysis
A. Directing B. Risk-benefit ratio
B. Controlling C. Percentile
C. Organizing D. Maximum risk
D. Planning 35. An individual/object that belongs to a
27. Time table showing planned work general population is a/an:
days and shifts of nursing personnel is:
A. Element
A. Staffing B. Subject
B. Schedule C. Respondent
C. Scheduling D. Author
D. Planning 36. An illustration that shows how the
28. A force within an individual that members of an organization are
influences the strength of behavior? connected:

A. Motivation A. Flowchart
B. Envy B. Bar graph
C. Reward C. Organizational chart
D. Self-esteem D. Line graph
29. “To be the leading hospital in the 37. The first college of nursing that was
Philippines” is best illustrate in: established in the Philippines is:

A. Mission A. Fatima University


B. Philosophy B. Far Eastern University
C. Vision C. University of the East
D. Objective D. University of Sto. Tomas
30. It is the professionally desired norms 38. Florence nightingale is born on:
against which a staff performance will be
compared? A. France
B. Britain
A. Job descriptions C. U.S
B. Survey D. Italy
C. Flow chart 39. Objective data is also called:
D. Standards
31. Reprimanding a staff nurse for work A. Covert
that is done incorrectly is an example of B. Overt
what type of reinforcement? C. Inference
D. Evaluation
A. Feedback 40. An example of subjective data is:
B. Positive reinforcement
C. Performance appraisal A. Size of wounds
D. Negative reinforcement B. VS
32. Questions that are answerable only C. Lethargy
by choosing an option from a set of given D. The statement of patient “My hand
alternatives are known as? is painful”
41. What is the best position in palpating C. Impaired urinary elimination
the breast? related to fluid loss
D. Risk for infection related to
A. Trendelenburg epidermal disruption
B. Side lying 49. In assessing a client’s incision 1 day
C. Supine after the surgery, Nurse Betty expect to
D. Lithotomy see which of the following as signs of a
42. When is the best time in performing local inflammatory response?
breast self examination?
A. Greenish discharge
A. 7 days after menstrual period B. Brown exudates at incision edges
B. 7 days before menstrual period C. Pallor around sutures
C. 5 days after menstrual period D. Redness and warmth
D. 5 days before menstrual period 50. Nurse Ronald is aware that the
43. Which of the following should be amiotic fluid in the third trimester weighs
given the highest priority before approximately:
performing physical examination to a
patient? A. 2 kilograms
B. 1 kilograms
A. Preparation of the room C. 100 grams
B. Preparation of the patient D. 1.5 kilograms
C. Preparation of the nurse 51. After delivery of a baby girl. Nurse
D. Preparation of environment Gina examines the umbilical cord and
44. It is a flip over card usually kept in expects to find a cord to:
portable file at nursing station.
A. Two arteries and two veins
A. Nursing care plan B. One artery and one vein
B. Medicine and treatment record C. Two arteries and one vein
C. Kardex D. One artery and two veins
D. TPR sheet 52. Myrna a pregnant client reports that
45. Jose has undergone thoracentesis. her last menstrual cycle is July 11, her
The nurse in charge is aware that the expected date of birth is
best position for Jose is:
A. November 4
A. Semi fowlers B. November 11
B. Low fowlers C. April 4
C. Side lying, unaffected side D. April 18
D. Side lying, affected side 53. Which of the following is not a good
46. The degree of patients abdominal source of iron?
distension may be determined by: A. Butter
B. Pechay
A. Auscultation C. Grains
B. Palpation D. Beef
C. Inspection 54. Maureen is admitted with a diagnosis
D. Percussion of ectopic pregnancy. Which of the
47. A male client is addicted with following would you anticipate?
hallucinogen. Which physiologic effect
should the nurse expect? A. NPO
B. Bed rest
A. Bradyprea C. Immediate surgery
B. Bradycardia D. Enema
C. Constricted pupils 55. Gina a postpartum client is diagnosed
D. Dilated pupils with endometritis. Which position would
48. Tristan a 4 year old boy has suffered you expect to place her based on this
from full thickness burns of the face, diagnosis?
chest and neck. What will be the priority
nursing diagnosis? A. Supine
B. Left side lying
A. Ineffective airway clearance related C. Trendelinburg
to edema D. Semi-fowlers
B. Impaired mobility related to pain
56. Nurse Hazel knows that Myrna 2 fingers above umbilicus
understands her condition well when she 2 fingers below umbilicus
remarks that urinary frequency is caused 1 finger below umbilicus
by: 64. Which of the following criteria allows
Nurse Kris to perform home deliveries?
A. Pressure caused by the ascending
uterus Normal findings during assessment
B. Water intake of 3L a day Previous CS
C. Effect of cold weather Diabetes history
D. Increase intake of fruits and Hypertensive history
vegetables 65. Nurse Carla is aware that one of the
57. How many ml of blood is loss during following vaccines is done by
the first 24 hours post delivery of Myrna? intramuscular (IM) injection?

100 Measles
500 OPV
200 BCG
400 Tetanus toxoid
58. Which of the following hormones 66. Asin law is on which legal basis:
stimulates the secretion of milk?
RA 8860
Progesterone RA 2777
Prolactin RI 8172
Oxytocin RR 6610
Estrogen 67. Nurse John is aware that the herbal
59. Nurse Carla is aware that Myla’s medicine appropriate for urolithiasis is:
second stage of labor is beginning when
the following assessment is noted: Akapulco
Sambong
Bay of water is broken Tsaang gubat
Contractions are regular Bayabas
Cervix is completely dilated 68. Community/Public health bag is
Presence of bloody show defined as:
60. The leaking fluid is tested with
nitrazine paper. Nurse Kelly confirms that An essential and indispensable
the client’s membrane have ruptures equipment of the community health
when the paper turns into a: nurse during home visit
It contains drugs and equipment used
Pink by the community health nurse
Violet Is a requirement in the health center
Green and for home visit
Blue It is a tool used by the community
61. After amniotomy, the priority nursing health nurse in rendering effective
action is: procedures during home visit
69. TT4 provides how many percentage
Document the color and consistency of protection against tetanus?
of amniotic fluid
Listen the fetal heart tone 70
Position the mother in her left side 80
Let the mother rest 90
62. Which is the most frequent reason for 99
postpartum hemorrhage? 70. Third postpartum visit must be done
by public health nurse:
Perineal lacerations
Frequent internal examination (IE) Within 24 hours after delivery
CS After 2-4 weeks
Uterine atomy Within 1 week
63. On 2nd postpartum day, which height After 2 months
would you expect to find the fundus in a 71. Nurse Candy is aware that the family
woman who has had a caesarian birth? planning method that may give 98%
1 finger above umbilicus
protection to another pregnancy to Sheltered workshop
women Custodial
Educational
Pills 79. Nurse Judy is aware that following
Tubal ligation condition would reflect presence of
Lactational Amenorrhea method congenital G.I anomaly?
(LAM)
IUD Cord prolapse
72. Which of the following is not a part of Polyhydramios
IMCI case management process Placenta previa
Oligohydramios
Counsel the mother 80. Nurse Christine provides health
Identify the illness teaching for the parents of a child
Assess the child diagnosed with celiac disease. Nurse
Treat the child Christine teaches the parents to include
73. If a young child has pneumonia when which of the following food items in the
should the mother bring him back for child’s diet:
follow up?
Rye toast
After 2 days Oatmeal
In the afternoon White bread
After 4 days Rice
After 5 days 81. Nurse Randy is planning to
74. It is the certification recognition administer oral medication to a 3 year
program that develop and promotes old child. Nurse Randy is aware that the
standard for health facilities: best way to proceed is by:

Formula “Would you like to drink your


Tutok gamutan medicine?”
Sentrong program movement “If you take your medicine now, I’ll
Sentrong sigla movement give you lollipop”
75. Baby Marie was born May 23, 1984. “See the other boy took his
Nurse John will expect finger thumb medicine? Now it’s your turn.”
opposition on: “Here’s your medicine. Would you like
a mango or orange juice?”
April 1985 82. At what age a child can brush her
February 1985 teeth without help?
March 1985
June 1985 6 years
76. Baby Reese is a 12 month old child. 7 years
Nurse Oliver would anticipate how many 5 years
teeth? 8 years
83. Ribivarin (Virazole) is prescribed for a
9 female hospitalized child with RSV. Nurse
7 Judy prepare this medication via which
8 route?
6
77. Which of the following is the primary Intra venous
antidote for Tylenol poisoning? Oral
Oxygen tent
Narcan Subcutaneous
Digoxin 84. The present chairman of the Board of
Acetylcysteine Nursing in the Philippines is:
Flumazenil
78. A male child has an intelligence Maria Joanna Cervantes
quotient of approximately 40. Which kind Carmencita Abaquin
of environment and interdisciplinary Leonor Rosero
program most likely to benefit this child Primitiva Paquic
would be best described as: 85. The obligation to maintain efficient
ethical standards in the practice of
Habit training nursing belong to this body:
BON 93. Which of the following is formal
ANSAP continuing education?
PNA
RN Conference
86. A male nurse was found guilty of Enrollment in graduate school
negligence. His license was revoked. Re- Refresher course
issuance of revoked certificates is after Seminar
how many years? 94. The BSN curriculum prepares the
graduates to become?
1 year
2 years Nurse generalist
3 years Nurse specialist
4 years Primary health nurse
87. Which of the following information Clinical instructor
cannot be seen in the PRC identification 95. Disposal of medical records in
card? government hospital/institutions must be
done in close coordination with what
Registration Date agency?
License Number
Date of Application Department of Health
Signature of PRC chairperson Records Management Archives Office
88. Breastfeeding is being enforced by Metro Manila Development Authority
milk code or: Bureau of Internal Revenue
96. Nurse Jolina must see to it that the
EO 51 written consent of mentally ill patients
R.A. 7600 must be taken from:
R.A. 6700
P.D. 996 Nurse
89. Self governance, ability to choose or Priest
carry out decision without undue Family lawyer
pressure or coercion from anyone: Parents/legal guardians
97. When Nurse Clarence respects the
Veracity client’s self-disclosure, this is a gauge for
Autonomy the nurses’
Fidelity
Beneficence Respectfulness
90. A male patient complained because Loyalty
his scheduled surgery was cancelled Trustworthiness
because of earthquake. The hospital Professionalism
personnel may be excused because of: 98. The Nurse is aware that the following
tasks can be safely delegated by the
Governance nurse to a non-nurse health worker
Respondent superior except:
Force majeure
Res ipsa loquitor Taking vital signs
91. Being on time, meeting deadlines Change IV infusions
and completing all scheduled duties is Transferring the client from bed to
what virtue? chair
Irrigation of NGT
Fidelity 99. During the evening round Nurse Tina
Autonomy saw Mr. Toralba meditating and
Veracity afterwards started singing prayerful
Confidentiality hymns. What would be the best response
92. This quality is being demonstrated by of Nurse Tina?
Nurse Ron who raises the side rails of a
confused and disoriented patient? Call the attention of the client and
encourage to sleep
Responsibility Report the incidence to head nurse
Resourcefulness Respect the client’s action
Autonomy Document the situation
Prudence
100. In caring for a dying client, you Oral
should perform which of the following Anal
activities Phallic
Latency
Do not resuscitate 108. The nurse is aware that ego
Assist client to perform ADL development begins during:
Encourage to exercise
Assist client towards a peaceful death Toddler period
101. The Nurse is aware that the ability Preschool age
to enter into the life of another person School age
and perceive his current feelings and Infancy
their meaning is known: 109. Situation: A 19 year old nursing
student has lost 36 lbs for 4 weeks. Her
Belongingness parents brought her to the hospital for
Genuineness medical evaluation. The diagnosis was
Empathy ANOREXIA NERVOSA. The Primary gain of
Respect a client with anorexia nervosa is:
102. The termination phase of the NPR is
best described one of the following: Weight loss
Weight gain
Review progress of therapy and Reduce anxiety
attainment of goals Attractive appearance
Exploring the client’s thoughts, 110. The nurse is aware that the primary
feelings and concerns nursing diagnosis for the client is:
Identifying and solving patients
problem Altered nutrition : less than body
Establishing rapport requirement
103. During the process of cocaine Altered nutrition : more than body
withdrawal, the physician orders which of requirement
the following: Impaired tissue integrity
Risk for malnutrition
Haloperidol (Haldol) 111. After 14 days in the hospital, which
Imipramine (Tofranil) finding indicates that her condition in
Benztropine (Cogentin) improving?
Diazepam (Valium)
104. The nurse is aware that cocaine is She tells the nurse that she had no
classified as: idea that she is thin
She arrives earlier than scheduled
Hallucinogen time of group therapy
Psycho stimulant She tells the nurse that she eat 3
Anxiolytic times or more in a day
Narcotic She gained 4 lbs in two weeks
105. In community health nursing, it is 112. The nurse is aware that ataractics or
the most important risk factor in the psychic energizers are also known as:
development of mental illness?
Anti manic
Separation of parents Anti depressants
Political problems Antipsychotics
Poverty Anti anxiety
Sexual abuse 113. Known as mood elevators:
106. All of the following are
characteristics of crisis except Anti depressants
Antipsychotics
The client may become resistive and Anti manic
active in stopping the crisis Anti anxiety
It is self-limiting for 4-6 weeks 114. The priority of care for a client with
It is unique in every individual Alzheimer’s disease is
It may also affect the family of the
client Help client develop coping
107. Freud states that temper tantrums mechanism
is observed in which of the following:
Encourage to learn new hobbies and Repression
interest Projection
Provide him stimulating environment 123. Nurse John is aware that self
Simplify the environment to eliminate mutilation among Bipolar disorder
the need to make chores patients is a means of:
115. Autism is diagnosed at:
Overcoming fear of failure
Infancy Overcoming feeling of insecurity
3 years old Relieving depression
5 years old Relieving anxiety
School age 124. Which of the following may cause an
116. The common characteristic of increase in the cystitis symptoms?
autism child is:
Water
Impulsitivity Orange juice
Self destructiveness Coffee
Hostility Mango juice
Withdrawal 125. In caring for clients with renal
117. The nurse is aware that the most calculi, which is the priority nursing
common indication in using ECT is: intervention?

Schizophrenia Record vital signs


Bipolar Strain urine
Anorexia Nervosa Limit fluids
Depression Administer analgesics as prescribed
118. A therapy that focuses on here and 126. In patient with renal failure, the diet
now principle to promote self- should be:
acceptance?
Low protein, low sodium, low
Gestalt therapy potassium
Cognitive therapy Low protein, high potassium
Behavior therapy High carbohydrate, low protein
Personality therapy High calcium, high protein
119. A client has many irrational 127. Which of the following cannot be
thoughts. The goal of therapy is to corrected by dialysis?
change her:
Hypernatremia
Personality Hyperkalemia
Communication Elevated creatinine
Behavior Decreased hemoglobin
Cognition 128. Tony with infection is receiving
120. The appropriate nutrition for Bipolar antibiotic therapy. Later the client
I disorder, in manic phase is: complaints of ringing in the ears. This
ototoxicity is damage to:
Low fat, low sodium
Low calorie, high fat 4th CN
Finger foods, high in calorie 8th CN
Small frequent feedings 7th CN
121. Which of the following activity would 9th CN
be best for a depressed client? 129. Nurse Emma provides teaching to a
patient with recurrent urinary tract
Chess infection includes the following:
Basketball
Swimming Increase intake of tea, coffee and
Finger painting colas
122. The nurse is aware that clients with Void every 6 hours per day
severe depression, possess which Void immediately after intercourse
defense mechanism: Take tub bath everyday
130. Which assessment finding indicates
Introjection circulatory constriction in a male client
Suppression with a newly applied long leg cast?
Blanching or cyanosis of legs 138. The ANA recognized nursing
Complaints of pressure or tightness informatics heralding its establishment
Inability to move toes as a new field in nursing during what
Numbness of toes year?
131. During acute gout attack, the nurse
administer which of the following drug: 1994
1992
Prednisone (Deltasone) 2000
Colchicines 2001
Aspirin 139. When is the first certification of
Allopurinol (Zyloprim) nursing informatics given?
132. Information in the patients chart is
inadmissible in court as evidence when: 1990-1993
2001-2002
The client objects to its use 1994-1996
Handwriting is not legible 2005-2008
It has too many unofficial 140. The nurse is assessing a female
abbreviations client with possible diagnosis of
The clients parents refuses to use it osteoarthritis. The most significant risk
133. Nurse Karen is revising a client plan factor for osteoarthritis is:
of care. During which step of the nursing
process does such revision take place? Obesity
Race
Planning Job
Implementation Age
Diagnosing 141. A male client complains of vertigo.
Evaluation Nurse Bea anticipates that the client may
134. When examining a client with have a problem with which portion of the
abdominal pain, Nurse Hazel should ear?
assess:
Tymphanic membranes
Symptomatic quadrant either second Inner ear
or first Auricle
The symptomatic quadrant last External ear
The symptomatic quadrant first 142. When performing Weber’s test,
Any quadrant Nurse Rosean expects that this client will
135. How long will nurse John obtain an hear
accurate reading of temperature via oral
route? On unaffected side
Longer through bone than air
3 minutes conduction
1 minute On affected side by bone conduction
8 minutes By neither bone or air conduction
15 minutes 143. Toy with a tentative diagnosis of
136. The one filing the criminal care myasthenia gravis is admitted for
against an accused party is said to be diagnostic make up. Myasthenia gravis
the? can confirmed by:

Guilty Kernigs sign


Accused Brudzinski’s sign
Plaintiff A positive sweat chloride test
Witness A positive edrophonium (Tensilon)
137. A male client has a standing DNR test
order. He then suddenly stopped 144. A male client is hospitalized with
breathing and you are at his bedside. You Guillain-Barre Syndrome. Which
would: assessment finding is the most
significant?
Call the physician
Stay with the client and do nothing Even, unlabored respirations
Call another nurse Soft, non distended abdomen
Call the family Urine output of 50 ml/hr
Warm skin D. Incubation period varies
145. For a female client with suspected depending on the site of the bite
intracranial pressure (ICP), a most B. Cover IV bottle with brown paper
appropriate respiratory goal is: bag
C. Rumpel-leede test
Maintain partial pressure of arterial D. Peripheral blood smear
oxygen (Pa O2) above 80mmHg A. Nurse who never had chicken pox
Promote elimination of carbon dioxide D. After the delivery of the baby
Lower the PH B. 18 year old sister who recently got
Prevent respiratory alkalosis married
146. Which nursing assessment would C. BCG vaccination
identify the earliest sign of ICP? C. Klebs-loeffler bacillus
D. I will come back on Wednesday,
Change in level of consciousness same time, to read the result
Temperature of over 103°F C. Positive
Widening pulse pressure B. Avoid vitamin supplements while o
Unequal pupils therapy
147. The greatest danger of an D. Chancre
uncorrected atrial fibrillation for a male A. 60 breaths per minute
patient will be which of the following: D. A and B
D. Cotrimoxazole
Pulmonary embolism C. 1 tsp. salt and 8 tsp. sugar
Cardiac arrest B. Umbilical infections
Thrombus formation A. BCG
Myocardial infarction C. Epidemiological situation
148. Linda, A 30 year old post D. 90
hysterectomy client has visited the B. -15c to -25c
health center. She inquired about BSE A. Bacterial toxin
and asked the nurse when BSE should be D. Planning
performed. You answered that the BSE is B. Schedule
best performed: A. Motivation
C. Vision
7 days after menstruation D. Standards
At the same day each month D. Negative reinforcement
During menstruation B. Close ended
Before menstruation A. Inductive
149. An infant is ordered to recive 500 ml B. Risk-benefit ratio
of D5NSS for 24 hours. The Intravenous A. Element
drip is running at 60 gtts/min. How many C. Organizational chart
drops per minute should the flow rate D. University of Sto. Tomas
be? D. Italy
B. Overt
60 gtts/min. D. The statement of patient “My hand
21 gtts/min is painful”
30 gtts/min C. Supine
15 gtts/min A. 7 days after menstrual period
150. Mr. Gutierrez is to receive 1 liter of B. Preparation of the patient
D5RL to run for 12 hours. The drop factor C. Kardex
of the IV infusion set is 10 drops per C. Side lying, unaffected side
minute. Approximately how many drops D. Percussion
per minutes should the IV be regulated? D. Dilated pupils
A. Ineffective airway clearance
related to edema
13-14 drops
D. Redness and warmth
17-18 drops
B. 1 kilograms
10-12 drops
C. Two arteries and one vein
15-16 drops
D. April 18
Answers A. Butter
C. Don’t get pregnant at least 3 C. Immediate surgery
months D. Semi-fowlers
A. Increased urinary output
A. Pressure caused by the ascending C. Reduce anxiety
uterus A. Altered nutrition : less than body
B. 500 requirement
D. Estrogen D. She gained 4 lbs in two weeks
C. Cervix is completely dilated C. Antipsychotics
D. Blue A. Anti depressants
B. Listen the fetal heart tone D. Simplify the environment to
D. Uterine atomy eliminate the need to make chores
C. 2 fingers below umbilicus B. 3 years old
A. Normal findings during assessment D. Withdrawal
D. Tetanus toxoid D. Depression
C. RI 8172 A. Gestalt therapy
B. Sambong D. Cognition
A. An essential and indispensable C. Finger foods, high in calorie
equipment of the community health D. Finger painting
nurse during home visit A. Introjection
D. 99 B. Overcoming feeling of insecurity
B. After 2-4 weeks C. Coffee
C. Lactational Amenorrhea method D. Administer analgesics as
(LAM) prescribed
B. Identify the illness A. Low protein, low sodium, low
A. After 2 days potassium
D. Sentrong sigla movement D. Decreased hemoglobin
B. February 1985 B. 8th CN
D. 6 C. Void immediately after intercourse
C. Acetylcysteine A. Blanching or cyanosis of legs
A. Habit training B. Colchicines
B. Polyhydramios A. The client objects to its use
D. Rice D. Evaluation
D. “Here’s your medicine. Would you B. The symptomatic quadrant last
like a mango or orange juice?” A. 3 minutes
A. 6 years C. Plaintiff
C. Oxygen tent B. Stay with the client and do nothing
B. Carmencita Abaquin A. 1994
A. BON B. 2001-2002
D. 4 years D. Age
C. Date of Application B. Inner ear
A. EO 51 C. On affected side by bone
B. Autonomy conduction
C. Force majeure D. A positive edrophonium (Tensilon)
A. Fidelity test
D. Prudence A. Even, unlabored respirations
B. Enrollment in graduate school B. Promote elimination of carbon
C. Primary health nurse dioxide
A. Department of Health A. Change in level of consciousness
D. Parents/legal guardians C. Thrombus formation
C. Trustworthiness B. At the same day each month
B. Change IV infusions B. 21 gtts/min
C. Respect the client’s action A. 13-14 drops
D. Assist client towards a peaceful
death
C. Empathy
A. Review progress of therapy and
attainment of goals
D. Diazepam (Valium)
B. Psycho stimulant
C. Poverty
A. The client may become resistive
and active in stopping the crisis
B. Anal
D. Infancy

You might also like